Sei sulla pagina 1di 409

Vincenzo De Filippis

Algebra Lineare e Geometria


per i corsi di Laurea in Ingegneria
dell’Universitá di Messina

Anno Accademico 2018-2019


Indice

1 Richiami di Algebra. . . . . . . . . . . . . . . . . . . . . . . . . . . . . . . . . . . . . . . . . . . . 1
1.1 Strutture algebriche. . . . . . . . . . . . . . . . . . . . . . . . . . . . . . . . . . . . . . . . . 1
1.2 Relazioni d’equivalenza. . . . . . . . . . . . . . . . . . . . . . . . . . . . . . . . . . . . . . 3

2 Le Matrici . . . . . . . . . . . . . . . . . . . . . . . . . . . . . . . . . . . . . . . . . . . . . . . . . . . . . 7
2.1 Introduzione. . . . . . . . . . . . . . . . . . . . . . . . . . . . . . . . . . . . . . . . . . . . . . . 7
2.2 Prodotto righe per colonne. . . . . . . . . . . . . . . . . . . . . . . . . . . . . . . . . . . 8
2.3 Determinante di una matrice quadrata. . . . . . . . . . . . . . . . . . . . . . . . . . 9
2.4 Trasposta ed Aggiunta di una matrice. . . . . . . . . . . . . . . . . . . . . . . . . . 13
2.5 Dipendenza ed indipendenza tra n-uple. . . . . . . . . . . . . . . . . . . . . . . . . 14
2.6 Insiemi equivalenti di n-uple. . . . . . . . . . . . . . . . . . . . . . . . . . . . . . . . . . 16
2.7 Matrici equivalenti. . . . . . . . . . . . . . . . . . . . . . . . . . . . . . . . . . . . . . . . . . 18
2.8 Matrici in forma ridotta a gradini. . . . . . . . . . . . . . . . . . . . . . . . . . . . . . 21
2.9 Rango di una matrice. . . . . . . . . . . . . . . . . . . . . . . . . . . . . . . . . . . . . . . . 23
2.10 Matrici invertibili. . . . . . . . . . . . . . . . . . . . . . . . . . . . . . . . . . . . . . . . . . . 26
2.11 Le matrici elementari. . . . . . . . . . . . . . . . . . . . . . . . . . . . . . . . . . . . . . . . 28
2.12 Matrici ortogonali. . . . . . . . . . . . . . . . . . . . . . . . . . . . . . . . . . . . . . . . . . . 31
2.13 Matrici simili. . . . . . . . . . . . . . . . . . . . . . . . . . . . . . . . . . . . . . . . . . . . . . 32
2.14 Fattorizzazione A = LU. . . . . . . . . . . . . . . . . . . . . . . . . . . . . . . . . . . . . . 33
2.15 Fattorizzazione PA = LU. . . . . . . . . . . . . . . . . . . . . . . . . . . . . . . . . . . . . 38
2.16 Un algoritmo di inversione. . . . . . . . . . . . . . . . . . . . . . . . . . . . . . . . . . . 43
2.17 Esercizi svolti. . . . . . . . . . . . . . . . . . . . . . . . . . . . . . . . . . . . . . . . . . . . . . 44

3 Sistemi Lineari . . . . . . . . . . . . . . . . . . . . . . . . . . . . . . . . . . . . . . . . . . . . . . . . 57
3.1 Sistemi lineari equivalenti. . . . . . . . . . . . . . . . . . . . . . . . . . . . . . . . . . . . 57
3.2 Sistemi omogenei. . . . . . . . . . . . . . . . . . . . . . . . . . . . . . . . . . . . . . . . . . . 61
3.3 Metodo di Cramer per la risoluzione di un sistema lineare. . . . . . . . . 64
3.4 Metodo di eliminazione di Gauss. . . . . . . . . . . . . . . . . . . . . . . . . . . . . . 66
3.5 Esercizi svolti. . . . . . . . . . . . . . . . . . . . . . . . . . . . . . . . . . . . . . . . . . . . . . 69

v
vi Indice

4 Gli Spazi Vettoriali. . . . . . . . . . . . . . . . . . . . . . . . . . . . . . . . . . . . . . . . . . . . . 89


4.1 Introduzione . . . . . . . . . . . . . . . . . . . . . . . . . . . . . . . . . . . . . . . . . . . . . . . 89
4.2 Intersezione, unione e somma di sottospazi. . . . . . . . . . . . . . . . . . . . . . 90
4.3 Basi e dimensione di uno spazio vettoriale. . . . . . . . . . . . . . . . . . . . . . 94
4.4 Una nota sull’intersezione di sottospazi vettoriali. . . . . . . . . . . . . . . . 99
4.5 Formula di Grassmann. . . . . . . . . . . . . . . . . . . . . . . . . . . . . . . . . . . . . . . 100
4.6 Cambiamento di base in uno spazio vettoriale. . . . . . . . . . . . . . . . . . . 104
4.7 Esercizi svolti. . . . . . . . . . . . . . . . . . . . . . . . . . . . . . . . . . . . . . . . . . . . . . 107

5 Prodotti scalari, basi ortonormali e proiezioni ortogonali. . . . . . . . . . . . 127


5.1 Prodotto interno. . . . . . . . . . . . . . . . . . . . . . . . . . . . . . . . . . . . . . . . . . . . 127
5.2 Costruzione di basi ortonormali. . . . . . . . . . . . . . . . . . . . . . . . . . . . . . . 129
5.3 Complemento ortogonale e proiezione ortogonale. . . . . . . . . . . . . . . . 130
5.4 Esercizi svolti. . . . . . . . . . . . . . . . . . . . . . . . . . . . . . . . . . . . . . . . . . . . . . 132

6 Equazioni cartesiane di un sottospazio vettoriale. . . . . . . . . . . . . . . . . . . 139


6.1 Equazioni cartesiane ottenute tramite il complemento ortogonale
del sottospazio. . . . . . . . . . . . . . . . . . . . . . . . . . . . . . . . . . . . . . . . . . . . . 139
6.1.1 Esercizi svolti. . . . . . . . . . . . . . . . . . . . . . . . . . . . . . . . . . . . . . . 140
6.2 Equazioni cartesiane ottenute tramite lo studio del rango di una
matrice. . . . . . . . . . . . . . . . . . . . . . . . . . . . . . . . . . . . . . . . . . . . . . . . . . . . 141
6.2.1 Esercizi svolti. . . . . . . . . . . . . . . . . . . . . . . . . . . . . . . . . . . . . . . 142

7 Le applicazioni lineari. . . . . . . . . . . . . . . . . . . . . . . . . . . . . . . . . . . . . . . . . . 145


7.1 Introduzione. . . . . . . . . . . . . . . . . . . . . . . . . . . . . . . . . . . . . . . . . . . . . . . 145
7.2 Applicazioni lineari e matrici. . . . . . . . . . . . . . . . . . . . . . . . . . . . . . . . . 148
7.3 Applicazioni lineari e cambiamento di base. . . . . . . . . . . . . . . . . . . . . 161
7.3.1 Restrizione di un endomorfismo. . . . . . . . . . . . . . . . . . . . . . . . 165
7.4 Esercizi svolti. . . . . . . . . . . . . . . . . . . . . . . . . . . . . . . . . . . . . . . . . . . . . . 167

8 La forma canonica di un operatore lineare. . . . . . . . . . . . . . . . . . . . . . . . . 177


8.1 Autovalori ed autovettori di un endomorfismo. . . . . . . . . . . . . . . . . . . 177
8.2 Endomorfismi diagonalizzabili. . . . . . . . . . . . . . . . . . . . . . . . . . . . . . . . 183
8.2.1 Matrici simmetriche reali. . . . . . . . . . . . . . . . . . . . . . . . . . . . . . 190
8.3 La forma canonica di Jordan. . . . . . . . . . . . . . . . . . . . . . . . . . . . . . . . . . 194
8.3.1 Forma canonica a blocchi. . . . . . . . . . . . . . . . . . . . . . . . . . . . . . 194
8.3.2 Autospazi generalizzati. . . . . . . . . . . . . . . . . . . . . . . . . . . . . . . . 197
8.3.3 Il polinomio minimo di una matrice (o di un endomorfismo). 202
8.4 Esercizi svolti. . . . . . . . . . . . . . . . . . . . . . . . . . . . . . . . . . . . . . . . . . . . . . 212

9 Le forme bilineari e le forme quadratiche reali. . . . . . . . . . . . . . . . . . . . . 225


9.1 Introduzione. . . . . . . . . . . . . . . . . . . . . . . . . . . . . . . . . . . . . . . . . . . . . . . 225
9.2 Forme bilineari e matrici. . . . . . . . . . . . . . . . . . . . . . . . . . . . . . . . . . . . . 226
9.3 Cambiamento di basi. . . . . . . . . . . . . . . . . . . . . . . . . . . . . . . . . . . . . . . . 228
9.4 Forme bilineari simmetriche. . . . . . . . . . . . . . . . . . . . . . . . . . . . . . . . . . 231
9.4.1 La restrizione di una forma bilineare simmetrica . . . . . . . . . . 233
Indice vii

9.4.2 Forme bilineari simmetriche degeneri e non degeneri . . . . . . 234


9.5 Diagonalizzazione delle forme simmetriche reali. . . . . . . . . . . . . . . . . 236
9.6 Metodo per la diagonalizzazione delle forme simmetriche reali. . . . . 237
9.7 Segnatura di una forma bilineare simmetrica . . . . . . . . . . . . . . . . . . . . 245
9.8 Forme quadratiche reali. . . . . . . . . . . . . . . . . . . . . . . . . . . . . . . . . . . . . . 247
9.9 Criteri di positivitá. . . . . . . . . . . . . . . . . . . . . . . . . . . . . . . . . . . . . . . . . . 252
9.10 Esercizi svolti. . . . . . . . . . . . . . . . . . . . . . . . . . . . . . . . . . . . . . . . . . . . . . 253

10 I sistemi di riferimento in uno spazio affine ed euclideo. . . . . . . . . . . . . 267


10.1 Sistemi di riferimento nel piano e nello spazio. . . . . . . . . . . . . . . . . . . 267
10.2 Lo spazio affine e lo spazio euclideo. . . . . . . . . . . . . . . . . . . . . . . . . . . 269

11 I vettori geometrici. . . . . . . . . . . . . . . . . . . . . . . . . . . . . . . . . . . . . . . . . . . . . 271


11.1 Definizione di vettore geometrico. . . . . . . . . . . . . . . . . . . . . . . . . . . . . 271
11.2 Somma e prodotto per uno scalare. . . . . . . . . . . . . . . . . . . . . . . . . . . . . 272
11.3 Combinazioni lineari e componenti in un riferimento ortogonale. . . . 273
11.4 Prodotto scalare e proiezione su una retta. . . . . . . . . . . . . . . . . . . . . . . 274
11.5 Prodotto vettoriale e componente rispetto ad un piano. . . . . . . . . . . . . 276
11.6 Prodotto misto. . . . . . . . . . . . . . . . . . . . . . . . . . . . . . . . . . . . . . . . . . . . . 278
11.7 Esercizi svolti. . . . . . . . . . . . . . . . . . . . . . . . . . . . . . . . . . . . . . . . . . . . . . 279

12 Geometria nel piano affine e nel piano euclideo. . . . . . . . . . . . . . . . . . . . 283


12.1 Equazioni di una retta. . . . . . . . . . . . . . . . . . . . . . . . . . . . . . . . . . . . . . . 283
12.2 Reciproca posizione di due rette. . . . . . . . . . . . . . . . . . . . . . . . . . . . . . . 285
12.3 Fascio di rette. . . . . . . . . . . . . . . . . . . . . . . . . . . . . . . . . . . . . . . . . . . . . . 286
12.4 Angoli nel piano euclideo. . . . . . . . . . . . . . . . . . . . . . . . . . . . . . . . . . . . 287
12.5 Distanze nel piano euclideo. . . . . . . . . . . . . . . . . . . . . . . . . . . . . . . . . . . 288
12.6 Simmetrie. . . . . . . . . . . . . . . . . . . . . . . . . . . . . . . . . . . . . . . . . . . . . . . . . 289
12.7 Coordinate omogenee nel piano. . . . . . . . . . . . . . . . . . . . . . . . . . . . . . . 290
12.8 La circonferenza. . . . . . . . . . . . . . . . . . . . . . . . . . . . . . . . . . . . . . . . . . . . 292
12.9 Trasformazioni nel piano euclideo. . . . . . . . . . . . . . . . . . . . . . . . . . . . . 293
12.10Un breve cenno sulle curve algebriche piane. . . . . . . . . . . . . . . . . . . . 295
12.10.1Intersezione di due curve algebriche. . . . . . . . . . . . . . . . . . . . . 296
12.10.2Molteplicitá di un punto. . . . . . . . . . . . . . . . . . . . . . . . . . . . . . . 297
12.10.3Calcolo dei punti doppi di una curva. . . . . . . . . . . . . . . . . . . . . 298
12.11Esercizi svolti. . . . . . . . . . . . . . . . . . . . . . . . . . . . . . . . . . . . . . . . . . . . . . 299

13 Le Coniche. . . . . . . . . . . . . . . . . . . . . . . . . . . . . . . . . . . . . . . . . . . . . . . . . . . . 307
13.1 Intersezione di una conica ed una retta. . . . . . . . . . . . . . . . . . . . . . . . . 308
13.2 Polaritá rispetto ad una conica. . . . . . . . . . . . . . . . . . . . . . . . . . . . . . . . 310
13.3 Classificazione di una conica. . . . . . . . . . . . . . . . . . . . . . . . . . . . . . . . . 311
13.4 Intersezione fra due coniche. . . . . . . . . . . . . . . . . . . . . . . . . . . . . . . . . . 312
13.5 Diametri e centro di una conica. . . . . . . . . . . . . . . . . . . . . . . . . . . . . . . 316
13.6 Riduzione delle coniche in forma canonica. . . . . . . . . . . . . . . . . . . . . . 320
13.7 Fuochi di una conica. . . . . . . . . . . . . . . . . . . . . . . . . . . . . . . . . . . . . . . . 329
13.8 Esercizi svolti. . . . . . . . . . . . . . . . . . . . . . . . . . . . . . . . . . . . . . . . . . . . . . 329
viii Indice

14 Rette e piani nello spazio affine ed euclideo. . . . . . . . . . . . . . . . . . . . . . . . 341


14.1 Equazioni di un piano. . . . . . . . . . . . . . . . . . . . . . . . . . . . . . . . . . . . . . . 341
14.2 Reciproca posizione di due piani. . . . . . . . . . . . . . . . . . . . . . . . . . . . . . 343
14.3 Fascio di piani. . . . . . . . . . . . . . . . . . . . . . . . . . . . . . . . . . . . . . . . . . . . . . 343
14.4 Stella di piani. . . . . . . . . . . . . . . . . . . . . . . . . . . . . . . . . . . . . . . . . . . . . . 345
14.5 Equazioni di una retta. . . . . . . . . . . . . . . . . . . . . . . . . . . . . . . . . . . . . . . 347
14.6 Rette complanari. . . . . . . . . . . . . . . . . . . . . . . . . . . . . . . . . . . . . . . . . . . 348
14.7 Reciproca posizione tra una retta ed un piano. . . . . . . . . . . . . . . . . . . . 350
14.8 Calcolo dei parametri direttori di una retta. . . . . . . . . . . . . . . . . . . . . . 351
14.9 Coordinate omogenee nello spazio. . . . . . . . . . . . . . . . . . . . . . . . . . . . . 352
14.10Angoli nello spazio euclideo. . . . . . . . . . . . . . . . . . . . . . . . . . . . . . . . . . 353
14.11Distanze nello spazio euclideo. . . . . . . . . . . . . . . . . . . . . . . . . . . . . . . . 355
14.12Simmetrie nello spazio. . . . . . . . . . . . . . . . . . . . . . . . . . . . . . . . . . . . . . 361
14.13Cambiamento di riferimento cartesiano ortonormale. . . . . . . . . . . . . . 362
14.14Esercizi svolti. . . . . . . . . . . . . . . . . . . . . . . . . . . . . . . . . . . . . . . . . . . . . . 366

15 Le superfici quadriche. . . . . . . . . . . . . . . . . . . . . . . . . . . . . . . . . . . . . . . . . . 379


15.1 Definizione. . . . . . . . . . . . . . . . . . . . . . . . . . . . . . . . . . . . . . . . . . . . . . . . 379
15.2 Quadriche generali. . . . . . . . . . . . . . . . . . . . . . . . . . . . . . . . . . . . . . . . . . 380
15.3 Quadriche specializzate. . . . . . . . . . . . . . . . . . . . . . . . . . . . . . . . . . . . . . 383
15.3.1 Cono. . . . . . . . . . . . . . . . . . . . . . . . . . . . . . . . . . . . . . . . . . . . . . . 383
15.3.2 Cilindro. . . . . . . . . . . . . . . . . . . . . . . . . . . . . . . . . . . . . . . . . . . . 384
15.4 Quadriche riducibili. . . . . . . . . . . . . . . . . . . . . . . . . . . . . . . . . . . . . . . . . 385
15.5 Piani ed assi di simmetria delle quadriche generali. . . . . . . . . . . . . . . 386
15.6 Forma canonica di una quadrica generale con centro di simmetria. . . 388
15.7 Forma canonica di un paraboloide. . . . . . . . . . . . . . . . . . . . . . . . . . . . . 389
15.8 Forma canonica di un cono. . . . . . . . . . . . . . . . . . . . . . . . . . . . . . . . . . . 390
15.9 La sfera. . . . . . . . . . . . . . . . . . . . . . . . . . . . . . . . . . . . . . . . . . . . . . . . . . . 391
15.10Esercizi svolti. . . . . . . . . . . . . . . . . . . . . . . . . . . . . . . . . . . . . . . . . . . . . . 395

Riferimenti Bibliografici . . . . . . . . . . . . . . . . . . . . . . . . . . . . . . . . . . . . . . . . . . . . 403


Capitolo 1
Richiami di Algebra.

Lo studio dell’Algebra lineare e della Geometria richiede un’introduzione prelimi-


nare delle strutture algebriche piú generali. In particolare daremo nel presente capi-
tolo introduttivo le definizioni di gruppo, anello e campo. Tali strutture saranno in
seguito utili per caratterizzare la maggior parte degli oggetti utilizzati, tra i quali, per
esempio, i vettori, le matrici, gli spazi vettoriali, gli omomorfismi, le trasformazioni
piane.

1.1 Strutture algebriche.

Sia G un insieme non vuoto ed ⊕ una operazione binaria chiusa rispetto agli
elementi di G, cioé:
∀x, y ∈ G x ⊕ y = z ∈ G.
La coppia (G, ⊕) é detta Gruppoide. Data la generalitá delle proprietá soddisfatte
da un gruppoide, é utile arricchire la sua struttura algebrica con ulteriori assiomi.
Diciamo Semigruppo, un gruppoide (G, ⊕) che sia associativo, cioé in cui valga la
seguente proprietá (associativa):

∀x, y, z ∈ G x ⊕ (y ⊕ z) = (x ⊕ y) ⊕ z.

Un Monoide (G, ⊕) é un semigruppo che sia dotato di un elemento neutro e ∈ G


rispetto all’operazione ⊕:

∃e ∈ G tale che ∀x ∈ G e ⊕ x = x ⊕ e = x.

Definizione 1.1. Un Gruppo (G, ⊕) é una struttura algebrica definita dagli assiomi
del monoide ai quali si aggiunge quello dell’esistenza dell’elemento inverso di ogni
elemento in G. Piú precisamente vale il seguente:

1
2 1 Richiami di Algebra.

∀x ∈ G ∃y ∈ G tale che x ⊕ y = y ⊕ x = e

dove e é l’elemento neutro in G.


Un Gruppo (G, ⊕) é detto commutativo (o abeliano, in onore del matematico
norvegese N. Abel) se vale l’ulteriore seguente proprietá:

∀x, y ∈ G x ⊕ y = y ⊕ x.

Esempio 1. Gli insiemi Z, Q, R e C, rispettivamente dei numeri interi, razionali,


reali e complessi, formano un gruppo commutativo rispetto all’operazione di som-
ma, cioé nel caso ⊕ = +. L’insieme N dei numeri naturali non é un gruppo rispetto
alla somma, basti pensare al fatto che ogni numero 0 6= n ∈ N non ammette un
inverso.
Esempio 2. Gli insiemi Q∗ , R∗ e C∗ , rispettivamente dei razionali senza lo ’zero’,
reali senza lo ’zero’ e complessi senza lo ’zero’, formano un gruppo commutativo
rispetto all’operazione di prodotto, cioé nel caso ⊕ = ×. L’eliminazione dello ’zero’
é essenziale in quanto tale elemento non avrebbe l’inverso rispetto al prodotto.
Esempio 3. L’insieme di tutte le applicazioni biettive di un insieme in sé stesso
forma un gruppo non commutativo rispetto all’operazione di composizione tra ap-
plicazioni, cioé nel caso in cui l’operazione tra due applicazioni f , g é definita come
segue: ( f ⊕ g)(x) = f (g(x)).
Definizione 1.2. Sia A un insieme dotato di due operazioni interne ⊕ e ⊗. Il sistema
(A, ⊕, ⊗) é detto Anello se valgono i seguenti assiomi:
1. (A, ⊕) é un gruppo commutativo;
2. (A, ⊗) é un semigruppo;
3. vale la proprietá distributiva dell’operazione ⊗ rispetto a ⊕:

∀x, y, z, ∈ A x ⊗ (y ⊕ z) = (x ⊗ y) ⊕ (x ⊗ z).

Osserviamo che non é richiesto che un anello possegga l’elemento neutro rispet-
to alla seconda operazione ⊗ (se tale elemento esiste l’anello é detto unitario).
Inoltre non é detto che valga la proprietá commutativa rispetto a ⊗, se essa vale
l’anello é detto commutativo.
Ancora rispetto all’operazione ⊗, non é richiesta la validitá della legge di annulla-
mento del prodotto. In altre parole, il fatto che x⊗y = 0 non implica necessariamente
che x = 0 oppure y = 0.
Infine non é richiesta l’esistenza di un elemento inverso per ciascun x ∈ A, rispetto
all’operazione ⊗.
Esempio 4. L’insieme Z rispetto alle usuali operazioni di somma e prodotto, cioé
nel caso ⊕ = + e ⊗ = ×, é un anello unitario in cui vale la legge di annullamento del
prodotto, ma in cui non esiste l’elemento inverso rispetto alla seconda operazione.
Esempio 5. L’insieme mZ = {n ∈ Z : n sia multiplo di m} é un anello rispetto
alle usuali operazioni di somma e prodotto, ma non é unitario eccetto nel caso in cui
m ∈ {−1, +1}.
1.2 Relazioni d’equivalenza. 3

Esempio 6. Sia R[X] l’insieme di tutti i polinomi a coefficienti reali in una variabile
e si introducano in R[X] le usuali operazioni di somma e prodotto tra polinomi.
Rispetto a tali operazioni R[X] é un anello commutativo ed unitario, nel quale vale
la legge di annullamento del prodotto.
Esempio 7. In seguito vedremo un esempio di anello in cui non vale la legge di
annullamento del prodotto rispetto alla seconda operazione: é l’insieme delle matrici
quadrate di un dato ordine, in cui vengano definite le opportune operazioni. Per la
comprensione di tale esempio, rimandiamo al Capitolo relativo alle Matrici.
Definizione 1.3. Sia (A, ⊕, ⊗) un anello. Indichamo con e ∈ A l’elemento neutro
rispetto alla prima operazione ⊕ e sia A∗ = A − {e}. Nel caso in cui (A∗ , ⊗) sia un
gruppo, diremo che (A, ⊕, ⊗) é un Corpo. Inoltre, se in (A∗ , ⊗) vale la proprietá
commutativa (cioé se (A∗ , ⊗) é un gruppo commutativo) allora (A, ⊕, ⊗) é detto
’corpo commutativo’ o equivalentemente ’campo’.
Esempio 8. Se definiamo le usuali operazioni di somma e prodotto in Q, R e C,
essi hanno la struttura di corpi commutativi. Sono anche detti campi ’scalari’, ed
ogni loro elemento x puó essere chiamato ’scalare’.

1.2 Relazioni d’equivalenza.

Siano A e B due insiemi non vuoti. Si definisce prodotto cartesiano A × B, l’insieme


di tutte le coppie ordinate di tipo (a, b), tali che a ∈ A e b ∈ B:

A × B = {(a, b) : a ∈ A, b ∈ B}.

Nel caso in cui i due insiemi A e B posseggano un numero finito di elementi, la


cardinalitá (o potenza) dell’insieme A × B é data dal prodotto delle cardinalitá dei
due insiemi che intervengono nel prodotto cartesiano.
Definizione 1.4. Una relazione é una terna (A, B, R) in cui A e B siano due insiemi
non vuoti e R sia un sottoinsieme del prodotto cartesiano A × B. Usualmente per in-
dicare una relazione tra gli insiemi A e B si preferisce utilizzare il semplice simbolo
R (sottointendendo la terna (A, B, R)). Per indicare che due elementi a ∈ A e b ∈ B
sono in relazione tra loro, si scrive aRb oppure (a, b) ∈ R.
Esempio 9. Siano A = {2, 3, 7}, B = {1, 5, 7, 9}. Si definisca R nel seguente modo:

(a, b) ∈ R ⇐⇒ a > b.

Le coppie ordinate di elementi che soddisfano la relazione sono le seguenti:

R = {(2, 1), (3, 1), (7, 1), (7, 5)}.

Esempio 10. Siano A = B = R. Si definisca R nel seguente modo:


4 1 Richiami di Algebra.

(a, b) ∈ R ⇐⇒ a + b − 1 = 0.

Le coppie ordinate che soddisfano la relazione sono chiaramente tutte le coppie di


coordinate del piano (x, y) che appartengano alla retta di equazione x + y − 1 = 0.
Ad esempio (2, −1) ∈ R, (−3, 4) ∈ R etc. etc.
Poniamoci nel caso in cui A = B e consideriamo una qualsiasi relazione (A, A, R):
1. la relazione R si dice riflessiva se, per ogni elemento a ∈ A, la coppia (a, a) ∈ R;
si noti ad esempio che nessuna delle due relazioni introdotte negli Esempi 9 e 10
soddisfano alla proprietá di riflessivitá.
2. R é detta simmetrica se (a, b) ∈ R =⇒ (b, a) ∈ R. la relazione definita nel-
l’esempio 9 non é simmetrica, mentre quella introdotta nell’esempio 10 lo
é.
3. R é detta transitiva se, per ogni a, b, c ∈ A tali che (a, b) ∈ R ed anche (b, c) ∈ R
allora ne segue che (a, c) ∈ R; la relazione all’esempio 9 é transitiva, mentre la
relazione all’esempio 10 non lo é.
Definizione 1.5. Una relazione R su un insieme non vuoto A é detta equivalenza (o
relazione d’equivalenza) se essa soddisfa alle proprietá di riflessivitá, simmetricitá
e transitivitá.
Nessuna delle relazioni introdotte negli esempi 9 e 10 é una relazione d’equiva-
lenza.
Ecco un classico esempio di relazione d’equivalenza:
Esempio 11. Sia A l’insieme di tutte le rette del piano. Definiamo la seguente rela-
zione R in A: due rette r, s sono in relazione tra loro se esse sono parallele. É evidente
che ciascuna retta r é parallela a se stessa (R é riflessiva). Inoltre se (r, s) ∈ R allora
anche (s, r) ∈ R, poiché nel parallelismo tra rette non é vincolante l’ordine in cui
esse si considerino. Infine se (r, s) ∈ R ed anche (s,t) ∈ R allora le rette r e t so-
no ancora parallele, cioé (r,t) ∈ R. Si puó allora pensare di ripartire l’insieme A in
sottoinsiemi, ciascuno dei quali contenga esclusivamente elementi (rette) tra loro
in relazione. Tali sottoinsiemi sono dette classi di equivalenza. Ciascuna classe puó
essere rappresentata da un qualsiasi elemento ad essa appartenente, inoltre due di-
stinte classi non hanno alcun elemento in comune. Infine l’unione di tutte le classi
di equivalenza é esattamente l’intero insieme A.
Quanto detto per il precedente esempio puó essere esteso ad ogni insieme in cui
venga introdotta una relazione di equivalenza. L’utilitá della partizione risiede nel
fatto che lo studio di un rappresentante di una qualsiasi classe, permette di ottenere
informazioni algebriche su tutti gli elementi di tale classe.

Siano A un insieme non vuoto e R una relazione di equivalenza definita su A. Ogni


classe di equivalenza [x] puó essere rappresentata da un suo qualsiasi elemento x,
[x] = {y ∈ A : xRy}. L’insieme di tutte le classi di equivalenza {[x] : x ∈ A} é una
partizione di A ed é detto insieme quoziente, denotato RA .
Nell’esempio delle rette parallele, ogni classe é individuata da un angolo α ∈ [0, π],
1.2 Relazioni d’equivalenza. 5

ovvero quello formato, nel primo o secondo quadrante, dalla retta e dall’asse delle
ascisse X. Le rette appartenenti ad una stessa classe, cioé tra loro parallele, hanno
chiaramente lo stesso coefficiente angolare e quindi lo stesso rappresentante α.

Concludiamo con un ulteriore esempio di relazione di equivalenza:


Esempio 12. Sia A = Z, e fissiano un n ∈ N. Definiamo R come segue: due ele-
menti a, b ∈ Z sono in relazione se esiste un opportuno k ∈ Z tale che a − b = kn.
Verifichiamo che R é una equivalenza:
1. a − a = 0 · n, quindi aRa (riflessiva);
2. se aRb, allora esiste k ∈ Z tale che a − b = kn; quindi b − a = (−k)n, cioé bRa
(simmetrica);
3. se aRb e bRc, allora esistono k1 , k2 ∈ Z tali che a − b = k1 n e b − c = k2 n; quindi
a − c = a − b + b − c = (k1 + k2 )n, cioé aRc (transitiva).
Le classi di equivalenza sono esattamente n ed hanno come rappresentanti i
numeri naturali {0, 1, 2, . . . , n − 1}:
• [0] = {x ∈ Z : ∃k ∈ Z, x − 0 = kn}, cioé sono tutti i multipli di n;
• [1] = {x ∈ Z : ∃k ∈ Z, x − 1 = kn}, cioé sono i numeri interi che danno resto 1
dopo la divisione per n;
• [2] = {x ∈ Z : ∃k ∈ Z, x − 2 = kn}, cioé sono i numeri interi che danno resto 2
dopo la divisione per n;
• ......
• ......
• [n − 1] = {x ∈ Z : ∃k ∈ Z, x − (n − 1) = kn}, cioé sono i numeri interi che danno
resto n − 1 dopo la divisione per n;
• [n] = [0];
• [n + 1] = [1], in generale [n + k] = [k];
• [kn] = [0], per ogni k ∈ Z;
• [tn + k] = [k], per ogni t ∈ Z.
Ad esempio sia n = 3, allora aRb se a − b = 3k, per un opportuno k ∈ Z. Le classi
sono
• [0] = {x ∈ Z : ∃k ∈ Z, x − 0 = 3k} = {0, 3, 6, 9, . . . , −3, −6, −9, . . .}, cioé sono
tutti i multipli di 3;
• [1] = {x ∈ Z : ∃k ∈ Z, x = 3k + 1} = {1, 4, 7, 10, . . . , −2, −5, −8, −11, . . .}, cioé
sono i numeri interi che danno resto 1 dopo la divisione per 3;
• [2] = {x ∈ Z : ∃k ∈ Z, x = 3k + 2} = {2, 5, 8, 11, . . . , −1, −4, −7, −10, . . .}, cioé
sono i numeri interi che danno resto 2 dopo la divisione per 3;
• [3] = [0].
In questo caso scriviamo AR = {[0], [1], [2]}. Verfifchiamo infine a quali classi appar-
tengono i numeri 26 e −23:
• 26 = (8 × 3) + 2 ∈ [2]
• −23 = (−8 × 3) + 1 ∈ [1].
Capitolo 2
Le Matrici

2.1 Introduzione.

Una matrice é una tabella nella quale si possono individuare righe e colonne. Se la
matrice possiede m righe e n colonne, queste possono essere riempite utilizzando
m · n elementi di un dato insieme. Nel nostro caso, sceglieremo di utilizzare R come
insieme dal quale prelevare gli elementi tramite i quali riempire una matrice. Per
semplicitá, disporremo gli elementi in modo ordinato nelle righe e nelle colonne nel
seguente modo:
 
a11 a12 . . . a1n
 a21 a22 . . . a2n 
 ... ... ... ... .
 

am1 am2 . . . amn


In altre parole, indicheremo col simbolo ai j l’elemento che occupa il posto indi-
viduato dall’incrocio della riga i e dalla colonna j.
L’elemento ai j di una matrice é talvolta detto ingresso di posto (i, j) della matrice.
Piú sinteticamente scriveremo A = ai j m×n , per indicare una matrice che consti di
m righe e n colonne.
Le matrici quadrate sono quelle in cui m = n e le chiameremo matrici di ordine n. In
esse individueremo un particolare insieme di elementi: {a11 , a22 , a33 , . . . , ann } detta
diagonale principale della matrice. Denoteremo Mmn (R) l’insieme di tutte le matrici
con m righe e n colonne, i cui elementi sono scelti in R.
Introduciamo in Mmn (R) una prima operazione  interna, detta somma tra matrici, nel
modo seguente: siano A = ai j e B = bi j matrici in Mmn (R), la loro somma é
ancora una matrice C = ci j di Mmn (R) definita da
     
c11 c12 . . . c1n a11 a12 . . . a1n b11 b12 . . . b1n
 c21 c22 . . . c2n   a21 a22 . . . a2n   b21 b22 . . . b2n 
 ... ... ... ...  =  ... ... ... ... + ... ... ... ...  =
     

cm1 cm2 . . . cmn am1 am2 . . . amn bm1 bm2 . . . bmn

7
8 2 Le Matrici
 
a11 + b11 a12 + b12 . . . a1n + b1n
 a21 + b21 a22 + b22 . . . a2n + b2n 
 
 ... ... ... ... 
am1 + bm1 am2 + bm2 . . . amn + bmn
quindi ogni elemento ci j di C é la somma degli elementi omologhi in A e B.
Siano A, B,C matrici di Mmn (R), rispetto all’operazione di somma sono soddi-
sfatte le seguenti proprietá:
1. (A + B) +C = A + (B +C), proprietá associativa;
2. Esiste l’elemento neutro rispetto alla somma, ovvero la matrice
 
0 0 ... 0
 0 0 ... 0 
0= 
 ... ... ... ... 
0 0 ... 0

tale che A + 0 = 0 + A = A; 
3. ogni matrice A = ai j m×n possiede l’opposta B = −A = −ai j m×n , tale che
A + B = 0;
4. A + B = B + A, cioé vale la proprietá commutativa.
Quindi Mmn (R) é un gruppo commutativo rispetto all’operazione di somma tra
matrici. 
Siano ora α ∈ R e A = ai j m×n ∈ Mmn (R). Moltiplicare lo scalare α per la
matrice A vuol
 dire moltiplicarlo per ogni elemento di A ottenendo cosı́ la matrice
αA = αai j m×n .

2.2 Prodotto righe per colonne.


 
Consideriamo ora due matrici A = ai j m×n ∈ Mmn (R) e B = bi j n×q ∈ Mnq (R).
Si definisce prodotto righe per
 colonne di A e B quella operazione che abbia come
risultato la matrice C = ci j m×q ∈ Mmq (R), tale che l’elemento di posto (r, s) di C
sia uguale a
n
crs = ∑ ark bks = ar1 b1s + ar2 b2s + ar3 b3s + . . . + arn bns
k=1

In sostanza l’elemento crs é la somma di tutti i prodotti degli elementi della riga r
della prima matrice, ciascuno con il corrispondente elemento della colonna s della
seconda matrice.
Per esempio:  
  1 −1 0  
123  3 −2 5
· 1 1 1 = .
101 1 −2 1
0 −1 1
2.3 Determinante di una matrice quadrata. 9

Appare chiaro che tale prodotto non puó essere definito se A ∈ Mmn e B ∈ Mtq
con n 6= t.
Proprietá 2.1. Date le matrici A ∈ Mmn (R), B ∈ Mnn (R),C ∈ Mnt (R) e dato lo
scalare α ∈ R, valgono le seguenti proprietá:
1. A(BC) = (AB)C, proprietá associativa;
2. A(B +C) = AB + AC, (A + B)C = AC + BC, proprietá distributiva del prodotto tra
matrici rispetto alla somma tra matrici;
3. α(AB) = A(αB).
Consideriamo adesso Mn (R), l’insieme di tutte le matrici quadrate di ordine n.
In tale insieme scegliamo la matrice
 
1 0 0 ... 0
. .
 0 1 0 .. .. 

 
I =  ... ... . . . ... ... 
 
 
. . . . .
. .
. . . . .. . .
0 0 ... 0 1

i cui ingressi sono pari ad 1 su tutta la diagonale principale e zero altrove. Tale
matrice I ∈ Mn (R) si comporta come elemento neutro rispetto al prodotto righe per
colonne, cioé per ogni A ∈ Mn (R) si ha AI = IA = A. Viene in generale detta matrice
identitá.
Osservazione 2.2. Si noti infine che non vale la proprietá commutativa per il
prodotto righe per colonne, cioé NON é regola generale che i prodotti AB e BA
coincidano.

2.3 Determinante di una matrice quadrata.



Sia A = ai j n×n
∈ Mn (R) una matrice quadrata di ordine n, al solito rappresentata
da  
a11 a12 . . . a1n
 a21 a22 . . . a2n 
A=  ... ... ... ... .
 (2.1)
an1 an2 . . . ann
Essa é una tabella contenente n2 numeri reali, i quali potrebbero essere i dati rilevati
nell’osservare un fenomeno, oppure quelli relativi agli esiti di un esperimento. Fon-
damentalmente, fino ad ora non abbiamo in nessun caso assegnato ad una matrice
un unico valore reale. Questo é quello che ci proponiamo di fare in questa Sezione,
ovvero intendiamo definire una corrispondenza ϕ : Mn (R) −→ R che associ ad ogni
matrice A ∈ Mn (R) uno ed un solo valore in R, che denomineremo determinante di
A.
10 2 Le Matrici

Usualmente si indica il determinante di una matrice quadrata A con il termine det(A)


oppure con |A|.
Iniziamo col definire il determinante di matrici di ordini bassi:
n=1
A = [a], a ∈ R ed in questo caso det(A) = a.
n=2
 
a11 a12
A= . Il determinante é det(A) = a11 · a22 − a12 · a21 . Quindi dapprima
a21 a22
moltiplichiamo tra loro gli elementi della diagonale principale. In seguito moltipli-
chiamo tra loro gli elementi della diagonale secondaria cambiando il segno di questo
ultimo prodotto. Infine sommiamo i due prodotti ottenuti.
n=3
 
a11 a12 a13
A =  a21 a22 a23 . In tal caso il determinante di A é:
a31 a32 a33

det(A) = a11 a22 a33 + a12 a23 a31 + a13 a21 a32 +
(2.2)
− a13 a22 a31 − a11 a23 a32 − a12 a21 a33 .

Per calcolare tale determinante possiamo utilizzare il seguente metodo, detto ’di
Sarrus’. Ricopiamo le prime due colonne a destra della terza:
 
a11 a12 a13 a11 a12
 a21 a22 a23 a21 a22  .
a31 a32 a33 a31 a32

Individuiamo su questa tabella le 6 diagonali composte da 3 elementi ciascuna. Tre


diagonali sono dette principali e condotte dalla sinistra in alto della tabella alla de-
stra in basso. Tre sono dette secondarie e condotte dalla destra in alto alla sinistra
in basso. Eseguiamo i prodotti degli elementi di ciascuna diagonale, cambiando il
segno al prodotto degli elementi di ogni diagonale secondaria. Infine sommiamo i
prodotti ottenuti ed otteniamo esattamente la forma (2.2).
A questo punto, nel passaggio a matrici di ordine n ≥ 4, dobbiamo introdurre un al-
goritmo, che in realtá ci consente di ottenere il determinante di una matrice quadrata
qualsiasi, indipendentemente dal suo ordine. Per poterne discutere, diamo dapprima
alcune definizioni.
Definizione 2.3. Sia A ∈ Mmn (R). Fissiamo arbitrariamente p righe e q colonne di
A. Gli elementi che si trovano negli incroci delle righe e colonne scelte individua-
no una matrice B ∈ M pq (R). Tale matrice é detta sottomatrice di A. Quindi ogni
sottomatrice di una matrice A é ottenuta utilizzando gli elementi che appartengano
contemporaneamente ad un numero p di righe di A ed ad un numero q di colonne di
A.
2.3 Determinante di una matrice quadrata. 11

Definizione 2.4. Sia A ∈ Mmn (R) e sia B ∈ M p (R) (p ≤ m, n) una sottomatrice


quadrata di A. Il determinante di B é detto minore di ordine p di A. Quindi un
minore di A é il determinante di una qualsiasi sottomatrice quadrata B di A ottenuta
intersecando p righe e colonne di A.
Definizione 2.5. Sia A una matrice quadrata di ordine n e sia B la sottomatrice
quadrata di A, derivante dalla cancellazione della riga r e della colonna s in A. Per
quanto precedentemente detto, mrs = det(B) é un minore di ordine n − 1 di A. Tale
scalare mrs é detto minore complementare dell’elemento ars di A.
Definizione 2.6. Sia mi j il minore di ordine n−1 dell’elemento ai j della matrice A ∈
Mn (R). Diremo complemento algebrico dell’elemento ai j ∈ A, il seguente scalare:
Ai j = (−1)i+ j · mi j .
 
1010
Esempio 13. Sia A =  2 1 2 0 . Le seguenti sono sottomatrici di A:
3111
 
101
212

ottenuta dalle prime 2 righe e dalle prime 3 colonne di A,


 
10
20

ottenuta dalle prime due righe e dalla terza e quarta colonna di A,


 
10
11

ottenuta dalla prima e terza riga e dalla terza e quarta colonna di A.


Inoltre sono minori di ordine 2 i seguenti

1 0
= 0, 1 0 = 1.


2 0 1 1
 
123
Esempio 14. Sia A =  0 1 0 . Il complemento algebrico di a13 = 3 é
221

0 1
A13 = (−1)4 = −2.
2 2

Il complemento algebrico di a21 = 0 é



3 2 3

A21 = (−1) = 4.
2 1
12 2 Le Matrici

Teorema 2.7. (Primo Teorema di Laplace). Sia A ∈ Mn (R) una matrice quadrata
di ordine n. Il determinante di A é pari alla somma dei prodotti degli elementi di
una qualsiasi riga (o colonna) per i rispettivi complementi algebrici.
Esempio 15. Sia  
1 2 1 0
3 1 0 1
A=
1
.
1 1 1
3 2 1 0
Applicando il teorema di Laplace, rispetto alla prima riga, otteniamo:

1 0 1 3 0 1 3 1 1 3 1 0

det(A) = (+1) 1 1 1 + (−2) 1 1 1 + (+1) 1 1 1 + (0) 1 1
1 =
2 1 0 3 1 0 3 2 0 3 2 1

−2 + 10 − 4 + 0 = 4.
Definizione 2.8. Una matrice quadrata A ∈ Mn (R) é detta triangolare superiore se
si presenta nella seguente forma
 
a11 a12 a13 . . . . . . . . . a1n
 0 a22 a23 . . . . . . . . . a2n 
 
 0 0 a33 . . . . . . . . . a3n 
 
A=  ... ... ... ... ... ... ... 
 ... ... ... ... ... ... ... 
 
 . . . . . . . . . . . . . . . an−1,n−1 an−1,n 
0 0 0 0 ... ... ann

ovvero se ai j = 0 per ogni i > j.


Definizione 2.9. Una matrice quadrata A ∈ Mn (R) é detta triangolare inferiore se
si presenta nella seguente forma

0 ... ... ...


 
a11 0 0
 .. 
 . a22 0 ... ... ... 0 
 .. ..
 

 .
 . a33 ... ... ... 0 
A =  ... .. ..
 
 . . ... ... ... ... 

 . .. ..
 ..

 . . ... ... ... ... 

 . .. ..
 ..

. . . . . . . an−1,n−1
. 0 
an1 an2 an3 . . . . . . . . . ann

ovvero se ai j = 0 per ogni i < j.


Definizione 2.10. Una matrice triangolare superiore (od inferiore) é detta diagona-
le se ai j = 0 per ogni i 6= j, ovvero se gli unici elementi eventualmente non nulli
2.4 Trasposta ed Aggiunta di una matrice. 13

della matrice si trovano sulla sua diagonale principale. Essa quindi si presenta nella
seguente forma:  
a11 0 0 . . . . . . . . . 0
 0 a22 0 . . . . . . . . . 0 
 .. ..
 

 . . a33 . . . . . . . . . 0 
 
 .. .. .. . . 
A=  . . . . . . . . . . . . . 

 . . .
 .. .. .. . . . . . .

 ... ...  
 . . .
. . .

 . . . . . . . . . an−1,n−1 0 
0 0 0 ... ... ... ann
Come immediata conseguenza del primo Teorema di Laplace, otteniamo che:
Corollario 2.11. Se una matrice quadrata A ∈ Mn (R) é triangolare superiore (od
inferiore), allora il suo determinante é pari al prodotto degli elementi della sua
diagonale principale.
Corollario 2.12. Una matrice quadrata che abbia una riga o una colonna composta
da tutti 0, avrá determinante nullo.
Definizione 2.13. Una matrice con determinante nullo é detta matrice singolare (al
contrario é detta non singolare).

2.4 Trasposta ed Aggiunta di una matrice.

Sia A una matrice con m righe e n colonne, A ∈ Mmn (R). Diciamo trasposta di A, e
la indichiamo AT , la matrice definita come segue:
1. AT ∈ Mnm (R);
2. l’ingresso ( j, i) in AT é occupato dall’elemento ai j ∈ A.
In altre parole, possiamo scrivere le righe di AT ricopiando le colonne di A;
analogamente possiamo scrivere le colonne di AT ricopiando le righe di A.
 
1 1 2 0 −1
Esempio 16. Se A =  0 2 1 2 2 , allora
1332 1
 
1 01
 1 2 3
AT = 
 
 2 .
1 3
 0 2 2
−1 21

Osservazione 2.14. Se A ∈ Mn (R) é una matrice quadrata di ordine n, allora |A| =


|AT |.
14 2 Le Matrici

Osservazione 2.15. Siano A, B matrici quadrate. Valgono le seguenti:


1. (A · B)T = BT · AT .
2. (A + B)T = AT + BT .
Definizione 2.16. Una matrice A quadrata é detta simmetrica se A = AT .
Definizione 2.17. Sia A ∈ Mn (R) é una matrice quadrata di ordine n e sia Ars
il complemento algebrico dell’elemento ars ∈ A. Diciamo matrice aggiunta di A,
e la indichiamo Agg(A), quella matrice in cui l’elemento di posto (i, j) é pari al
complemento algebrico A ji .
 
121
Esempio 17. Se A =  3 1 0 , allora
011
 
130
AT =  2 1 1 
101
e  
1 −1 −1
Agg(A) =  −3 1 3  .
3 −1 −5

2.5 Dipendenza ed indipendenza tra n-uple.

Consideriamo il seguente insieme

Rn = {(r1 , . . . , rn ) | r1 , . . . , rn ∈ R}

ovvero l’insieme delle n-uple ordinate nel prodotto cartesiano

Rn = R × · · · × R .
| {z }
n−volte

Introduciamo ora le seguenti due operazioni:


1. Una operazione interna in Rn , ovvero la somma di due n-uple, che ha come ri-
sultante una n-upla nella quale ciascun elemento é la somma dei due elementi
corrispondenti e presenti nelle n-uple addende

(r1 , . . . , rn ) + (s1 , . . . , sn ) = (r1 + s1 , . . . , rn + sn ), ∀ r1 , . . . , rn , s1 , . . . , sn ∈ R.

Una prima importante, seppur banale, osservazione é che la n-upla (0, . . . , 0) ∈


Rn composta da tutti 0, detta n-upla nulla, si comporta come l’elemento neutro
rispetto alla somma tra n-uple appena definita. Nel seguito la indicheremo con il
simbolo 0.
2.5 Dipendenza ed indipendenza tra n-uple. 15

2. Una operazione esterna, ovvero il prodotto di una n-upla per uno scalare α ∈ R,
che ha come risultante una n-upla nella quale ogni elemento é il prodotto di α
per ciascun elemento della n-upla iniziale:

α(r1 , . . . , rn ) = (αr1 , . . . , αrn ) ∀ r1 , . . . , rn , α ∈ R.

Consideriamo adesso u1 , . . . , uk ∈ Rn , cioé un insieme di n-uple ordinate di numeri


reali e scegliamo ad arbitrio α1 , . . . , αk ∈ R. Alla luce delle operazioni precedente-
mente introdotte avremo che

α1 u1 + α2 u2 + · · · + αk uk (2.3)

é ancora una n-upla ordinata e viene detta combinazione lineare delle n-uple
{u1 , . . . , uk } tramite i coefficienti α1 , . . . , αk ∈ R.
Esempio 18. Consideriamo il prodotto cartesiano R4 ed i suoi elementi

v1 = (1, 1, 4, 0), v2 = (−1, −1, 0, 2), v3 = (0, −4, 3, 1).

Scegliamo α1 = 2, α2 = −3, α3 = 4 ed effettuiamo la combinazione lineare di


{v1 , v2 , v3 } tramite gli scalari α1 , α2 , α3 :

2 · (1, 1, 4, 0) + (−3) · (−1, −1, 0, 2) + 4 · (0, −4, 3, 1) = (5, −11, 20, −2) ∈ R4 .

Definizione 2.18. Siano u1 , . . . , ur n-uple reali. Diremo che u1 , . . . , ur sono linear-


mente dipendenti se esistono α1 , . . . , αr ∈ R non tutti nulli, tali che

α1 u1 + α2 u2 + . . . + αr ur = 0.

Diremo invece che u1 , . . . , ur sono linearmente indipendenti se vale la seguente:

α1 u1 + α2 u2 + . . . + αr ur = 0 se e solo se α1 = α2 = α3 = . . . = αr = 0.

Osservazione 2.19. Supponiamo che u1 , . . . , ur siano n-uple reali, tra loro linear-
mente dipendenti e che esistano α1 , . . . , αr ∈ R non tutti nulli, tali che

α1 u1 + α2 u2 + . . . + αr ur = 0.

Se αi 6= 0, allora

αi ui = −α1 u1 − α2 u2 . . . − αi−1 ui−1 − αi+1 ui+1 − . . . − αr ur

da cui
α1 α2 αi−1 αi+1 αr
ui = − u1 − u2 − . . . − ui−1 − ui+1 − . . . − ur .
αi αi αi αi αi
Quindi, in presenza n-uple linearmente dipendenti tra loro, almeno una di esse si
puó esprimere come combinazione lineare delle altre.
16 2 Le Matrici

Esempio 19. Siano u1 = (1, 2, 0), u2 = (3, 1, 0), u3 = (2, −1, 0), u4 = (0, 0, 1) ele-
menti di R4 .
Le 4-uple u1 , u2 , u3 sono linearmente dipendenti infatti:

(−1)u1 + (1)u2 + (−1)u3 = (0, 0, 0)

da cui u3 = u2 − u1 , cioé u3 é combinazione lineare di u2 e u1 .


Le 4-uple u2 , u3 , u4 sono linearmente indipendenti, infatti se supponiamo α2 u2 +
α3 u3 +α4 u4 = (0, 0, 0), otteniamo (3α2 +2α3 , α2 −α3 , α4 ) = (0, 0, 0) e quindi α2 =
α3 = α4 = 0.
Osservazione 2.20. Sia {u1 , . . . , uk } un insieme di n-uple. Se una di esse é la n-upla
nulla 0, allora u1 , . . . , uk sono linearmente dipendenti tra loro.
Infatti, posto ad esempio uk = 0, é sufficiente scegliere gli scalari

α1 = . . . = αk−1 = 0, e 0 6= αk ∈ R in modo arbitrario

per ottenere una combinazione lineare che si annulli.

2.6 Insiemi equivalenti di n-uple.

Per semplicitá diremo anche che se u1 , . . . , uk sono n-uple linearmente dipendenti


(indipendenti), allora l’insieme {u1 , . . . , uk } é un insieme linearmente dipendente
(rispettivamente indipendente).
Occupiamoci di come riconoscere se un dato insieme di n-uple sia costituito da
elementi linearmente dipendenti od indipendenti, nei casi in cui ció non sia evidente
dalla semplice applicazione della definizione di dipendenza lineare. Per rispondere
a tale quesito premettiamo alcune considerazioni. Supponiamo che le n-uple

{u1 , . . . , uk } (2.4)

siano tra loro linearmente dipendenti e che quindi esistano α1 , . . . , αk ∈ R non tutti
nulli, tali che
α1 u1 + α2 u2 + . . . + αk uk = 0 (2.5)
La prima osservazione é che, indipendentemente dall’ordine con cui scegliamo le
n-uple, esse risultano ovviamente sempre linearmente dipendenti. Si tratta solo di
cambiare l’ordine degli addendi che compaiono nella combinazione lineare (2.5).
Scegliamo adesso 0 6= β ∈ R e fissiamo una n-upla ui ∈ {u1 , . . . , uk }. Costruiamo la
seguente sequenza di n-uple

{u1 , . . . , ui−1 , β ui , ui+1 , . . . , uk }.

É immediato il fatto che



β α1 u1 +β α2 u2 +. . .+β αi ui +. . .+β αk uk = β α1 u1 +α2 u2 +. . .+αi ui +. . .+αk uk = 0.
2.6 Insiemi equivalenti di n-uple. 17

ovvero
γ1 u1 + γ2 u2 + . . . + γi (β ui ) + . . . + γk uk = 0
per γh = β αh , per ogni h 6= i, e γi = αi . Quindi {u1 , . . . , ui−1 , β ui , ui+1 , . . . , uk } co-
stituiscono un insieme linearmente dipendente di n-uple, al pari del precedente.
Siano ora 0 6= β ∈ R e fissiamo due distinte n-uple ui , u j ∈ {u1 , . . . , uk }. Consideria-
mo la seguente sequenza di n-uple

{u1 , . . . , ui−1 , ui + β u j , ui+1 , . . . , uk }. (2.6)

Osserviamo che,

α1 u1 + . . . + αi−1 ui−1 + αi (ui + β u j ) + αi+1 ui+1 + . . .


. . . + α j−1 u j−1 + (α j − β αi )u j + α j+1 u j+1 + . . . + αk uk = 0.
ovvero
γ1 u1 + γ2 u2 + . . . + γi (ui + β u j ) + . . . + γk uk = 0
per γh = αh , per ogni h 6= j, e γ j = α j −β αi . Concludiamo allora che (2.6) costituisce
un insieme linearmente dipendente di n-uple, al pari di quello iniziale.
A partire dall’insieme di n-uple (2.4) abbiamo quindi ottenuto 3 differenti insiemi
di n-uple tramite alcune operazioni che di seguito riepiloghiamo:
1. Lo scambio di posizione tra n-uple, rispetto all’ordine con cui compaiono nel-
l’insieme iniziale.
2. Il prodotto di una n-upla per uno scalare non nullo.
3. La somma di una n-upla con un multiplo di un’altra.
Le operazioni appena descritte sono dette operazioni elementari sulle n-uple di un
insieme.
Abbiamo inoltre dimostrato che tali operazioni, seppur generino insiemi differenti
da quello iniziale, non influiscono sulla eventuale dipendenza lineare dell’insieme
stesso, lasciandola invariata.
Definizione 2.21. Due insiemi di n-uple {u1 , . . . , uk } e {v1 , . . . , vk } sono detti equi-
valenti se uno di essi si ottiene dopo un numero finito di operazioni elementari sulle
n-uple dell’altro.
Quindi, se volessimo determinare la dipendenza od indipendenza lineare di un
insieme di n-uple, oppure anche il numero massimo di n-uple linearmente indipen-
denti in un insieme, potrebbe essere utile una sua trasformazione in un insieme
equivalente, il quale, seppur composto da n-uple differenti dall’insieme di partenza,
conserverebbe l’eventuale lineare dipendenza del primo.
Sia allora S = {u1 , . . . , uk } un insieme di n-uple. Vediamo cosa accade se esso é
linearmente dipendente. In tal caso, seguendo quanto detto nell’Osservazione 2.19,
esistono alcune n-uple in S che sono combinazione lineare delle altre. Se ui é una di
tali n-uple, allora esistono α1 , . . . , αk ∈ R tali che
α1 α2 αi−1 αi+1 αk
ui = − u1 − u2 − . . . − ui−1 − ui+1 − . . . − uk .
αi αi αi αi αi
18 2 Le Matrici

Tramite l’operazione elementare che prevede la somma di ui con


α1 α2 αi−1 αi+1 αk
u1 + u2 + . . . + ui−1 + ui+1 + . . . + uk
αi αi αi αi αi
otteniamo un insieme equivalente a S, nel quale la n-upla che occupa la posizione
i é identicamente nulla. In un certo senso, abbiamo dato inizio ad un processo di
semplificazione della sequenza di n-uple di S.
A margine della descrizione di tale processo, ci preme sottolineare che a breve ve-
dremo come determinare tali scalari α1 , . . . , αk . Allo stato attuale é rilevante il solo
fatto che essi esistano.
Ipotizzando quindi che vi siano almeno k − t elementi (t ≥ 1) di S che siano com-
binazione lineare degli altri e ripetendo il precedente procedimento per ciascuna di
tali n-uple, alla fine avremo ottenuto un insieme equivalente a S che avrá la seguente
forma:
S0 = {u1 , . . . , ut , 0, . . . , 0 }.
| {z }
(k−t)−volte

É ovvio che se qualcuna delle t n-uple restanti dovesse essere linearmente dipenden-
te dalle altre potremmo continuare con le stesse precedenti argomentazioni e passare
ad un insieme equivalente, nel quale la posizione di tale n-upla si sarebbe annullata.
Concluderemo il processo di semplificazione nel momento in cui le n-uple restanti
saranno tra loro linearmente indipendenti. In tal caso infatti non é possibile annullar-
ne alcuna, tramite le operazioni elementari. Appare chiaro che il numero di n-uple
linearmente indipendenti nella sequenza finale (quella semplificata) é pari a quello
delle n-uple linearmente indipendenti della sequenza iniziale.

2.7 Matrici equivalenti.

Consideriamo adesso una matrice


 
a11 a12 . . . . . . a1n
A =  . . . . . . . . . . . . . . .  ∈ Mmn (R).
am1 am2 . . . . . . amn

Appare evidente che ciascuna riga della matrice A é un elemento di Rn , quindi é una
n-upla ordinata di scalari. Analogamente ogni colonna di A é una m-upla, ovvero un
elemento di Rm .
Per cui, tanto per le righe quanto per le colonne possiamo introdurre le operazioni
di somma e di prodotto per uno scalare, precedentemente introdotti per le n-uple.
Poiché le definizioni e le proprietá concernenti alle righe ed alle colonne sono ana-
loghe, scegliamo di proseguire puntando l’attenzione esattamente sulle righe della
matrice. Tutto quello che vedremo in relazione alle righe della matrice vale quindi
anche per le sue colonne.
2.7 Matrici equivalenti. 19

Come per le n-uple ordinate reali, introduciamo le seguenti due operazioni:


1. la somma di righe (due n-uple) Ri , R j é una n-upla ottenuta sommando ciascun
elemento di una con il corrispondente dell’altra, ad esempio

R1 + R2 = (a11 + a21 , a12 + a22 , . . . , a1n + a2n ).

2. il prodotto di una riga per uno scalare α ∈ R é una n-upla ottenuta moltiplicando
ciascun elemento della riga per α, ad esempio

αR1 = (αa11 , αa12 , . . . , αa1n ).

Ed in modo del tutto analogo al precedente, se R1 , . . . , Rk , sono righe della ma-


trice, e scegliendo ad arbitrio α1 , . . . , αk ∈ R, possiamo introdurre la definizione di
combinazione lineare tra righe. Essa é

α1 R1 + α2 R2 + · · · + αk Rk (2.7)

che é ancora una n-upla ordinata.


Inoltre, le righe {R1 , . . . , Rk } della matrice A costituiscono un insieme linearmente
dipendente (indipendente) se esse sono linearmente dipendenti (rispettivamente in-
dipendenti) come n-uple.
Infine, consideriamo la sottomatrice B di A ottenuta estraendo un certo numero di
righe {R1 , . . . , Rk } da A. A partire dalle righe di B possiamo ottenere un insieme
equivalente di n-uple, tramite le operazioni elementari giá descritte:
1. Lo scambio di posizione tra 2 righe, rispetto all’ordine con cui compaiono in B.
2. Il prodotto di una riga per uno scalare non nullo.
3. La somma di una riga con un’altra riga precedentemente moltiplicata per uno
scalare non nullo.
Esempio 20. Vediamo, nella pratica, cosa vuol dire operare sulle righe di una
matrice:
1. Scambio diposizione
 tra due righe:  
12 01
la matrice  3 2  si trasforma nella  3 2  dopo lo scambio tra la prima e la
01 12
terza riga.
2. Moltiplicazione
 di una riga per uno scalare
  α 6= 0:
12 36
la matrice  3 2  si trasforma nella  3 2  dopo aver moltiplicato la prima riga
01 01
per 3.
3. Somma di una riga con un’altra riga, precedentemente moltiplicata per uno sca-
lare non nullo:
20 2 Le Matrici
   
12 1 2
la matrice  3 2  si trasforma nella  5 6  dopo aver sommato la seconda riga
01 0 1
con la prima riga moltiplicata per 2.
Per operare sulle righe R1 , . . . , Rm di una matrice A ∈ Mmn (R), nel seguito
useremo le seguenti notazioni:
• Ri ↔ R j per indicare lo scambio tra la riga di posto i e la riga di posto j della
matrice.
• Ri → αRi per indicare la moltiplicazione di una riga i per lo scalare α.
• R j → R j + αRi per indicare la somma della riga j con la riga i, precedentemente
moltiplicata per uno scalare α.
Le operazioni appena descritte modificano gli ingressi nelle righe, quindi gene-
rano una matrice A0 differente da A. Ma, alla luce di quanto detto precedentemente,
tali operazioni non influiscono sulla eventuale dipendenza lineare delle righe, per
cui il numero di righe linearmente dipendenti (indipendenti) in A é pari al numero
di righe lineramente dipendenti (indipendenti) in A0 .
Definizione 2.22. Due matrici A, A0 ∈ Mmn (R) si dicono equivalenti se l’una si ot-
tiene dall’altra tramite un numero finito di operazioni elementeri sulle righe (o sulle
colonne).
In particolare, analizziamo cosa accade nel caso in cui la matrice sulla quale ope-
riamo, tramite le trasformazioni elementari, sia quadrata.
A tal proposito, indichiamo con R1 , . . . , Rn le righe di una matrice quadrata A ∈
Mn (R), elencate in base all’ordine con cui compaiono nella descrizione della
matrice stessa. In simboli potremmo indicare A = (R1 , . . . , Rn ). Osserviamo che:
1. Dopo lo scambio fra due righe della matrice, il valore assoluto del determinante
della matrice finale é pari a quello della matrice iniziale, ma i segni dei due
determinanti sono discordi.
2. Una matrice con 2 righe uguali ha determinante nullo;
3. Dal Primo Teorema di Laplace si ha che

det(R1 , . . . , λ Ri , . . . , Rn ) = λ det(R1 , . . . , Ri , . . . , Rn ).

Ció significa che, dopo aver moltiplicato una riga per uno scalare λ 6= 0, il de-
terminante della matrice finale é pari a quello della matrice iniziale, moltiplicato
per λ .
4. Infine, dopo aver sommato un riga con un’altra riga precedentemente moltiplicata
per uno scalare λ 6= 0, i determinanti della matrice finale e di quella iniziale sono
esattamente identici.
Queste ultime considerazioni ci fanno comprendere come, nel caso di matrici
quadrate, in talune occasioni si possa riconoscere immediatamente se il determinan-
te della matrice é nullo oppure diverso da zero. In particolare, due matrici equivalenti
sono simultaneamente singolari o non singolari.
Da ció deriva la seguente:
2.8 Matrici in forma ridotta a gradini. 21

Osservazione 2.23. Se una riga di A ∈ Mn (R) é proporzionale ad un’altra il de-


terminante é nullo. Infatti una matrice equivalente ad A, ottenuta tramite opportune
operazioni elementari, avrebbe una riga nulla.
Piú in generale se una riga di una matrice quadrata é combinazione lineare di altre
righe, il determinante della matrice é nullo.
É opportuno concludere con la seguente osservazione, sebbene, alla luce di
quanto fin’ora detto, possa sembrare banale:
Osservazione 2.24. Se A e A0 sono due qualsiasi matrici tra loro equivalenti, allora
ogni sottomatrice di A é equivalente ad una sottomatrice di A0 (e viceversa). In par-
ticolare, ogni sottomatrice quadrata di A é singolare (non singolare) se e solo se la
sottomatrice di A0 ad essa equivalente é singolare (rispettivamente non singolare).

2.8 Matrici in forma ridotta a gradini.

Abbiamo giá sottolineato come l’utilizzo delle operazioni elementari ci consenta


di facilitare l’analisi della eventuale dipendenza od indipendenza lineare tra n-uple.
Vogliamo adesso descrivere un metodo per la corretta applicazione di tali operazio-
ni, scegliendo di agire direttamente sulle righe di una matrice.
Inizialmente é bene fornire alcune definizioni:
Definizione 2.25. Un elemento ai j ∈ A ∈ Mmn (R) é detto pivot della i-esima riga se
é il primo elemento non nullo di tale riga.
Definizione 2.26. Una matrice Mmn (R) si presenta in forma ridotta a gradini se
valgono contemporaneamente le seguenti:
1. tutte le eventuali righe nulle si trovano in fondo alla matrice;
2. ogni riga non nulla contiene un pivot;
3. se j > i ed aih é il pivot della riga Ri , allora il pivot delle riga R j deve essere un
elemento a jk , per un certo k > h, ovvero il pivot di R j si trova in una colonna a
destra rispetto a quella su cui si trova il pivot di Ri .
Una matrice A ∈ Mmn (R) in forma ridotta a gradini si presenta in genere nel
modo seguente:
 
0 · · · 0 a1, j1 · · · ··· ··· ··· ··· ··· ··· ··· ··· ··· a1,n
 0 ··· ··· 0 ··· 0 a2, j2 ··· ··· ··· ··· ··· ··· ··· a2,n 
 
 0 ··· 0 0 0 ··· 0 ··· 0 a3, j3 ··· ··· ··· ··· a3,n 
 
 ··· ··· ··· ··· ··· ··· ··· ··· ··· ··· ··· ··· ··· ··· ··· 
 
 ··· ··· ··· ··· ··· ··· ··· ··· ··· ··· ··· ··· ··· ··· ··· 
 
 ··· ··· ··· ··· ··· ··· ··· ··· ··· ··· ··· ··· ··· ··· ··· 
0 0 ··· ··· ··· ··· ··· 0 ··· 0 ··· 0 am, jm ··· am,n

ma potrebbe anche avere un certo numero di righe nulle in fondo, ovvero


22 2 Le Matrici
 
0 ··· 0 a1, j1 ··· ··· ··· ··· ··· ··· ··· ··· ··· ··· a1,n
 0
 ··· ··· 0 ··· 0 a2, j2 ··· ··· ··· ··· ··· ··· ··· a2,n 

 0
 ··· 0 0 0 ··· 0 ··· 0 a3, j3 ··· ··· ··· ··· a3,n 

 ··· ··· ··· ··· ··· ··· ··· ··· ··· ··· ··· ··· ··· ··· ··· 
 
 0
 0 ··· ··· ··· ··· ··· 0 ··· 0 ··· 0 ak, jk ··· ak,n 

 0
 0 ··· ··· ··· ··· ··· ··· ··· ··· ··· ··· 0 0  
 ··· ··· ··· ··· ··· ··· ··· ··· ··· ··· ··· ··· ··· ··· ··· 
0 0 ··· ··· ··· ··· ··· ··· ··· ··· ··· ··· 0 0
Esempio 21. La matrice  
1 1 2 0 −1
 0 9 2 −5 5 
040 8 7
non é in forma ridotta a gradini, poiché i pivot della seconda e terza riga apparten-
gono alla stessa colonna.
Esempio 22. La matrice  
000 0 0
 0 9 2 −5 5 
006 8 7
non é in forma ridotta a gradini, poiché la prima riga é nulla.
Esempio 23. Le matrici
     
1 1 2 0 −1 4 1 2 0 −1 0 0 2 0 −1
 0 5 0 −5 5  ,  0 0 0 −5 5   0 0 0 −5 5 
000 8 7 000 0 0 000 0 3

sono tutte espresse in forma ridotta a gradini.


Teorema 2.27. (Algoritmo di Gauss) Una qualsiasi matrice A ∈ Mmn (R) si puó
ridurre in forma a gradini tramite un numero finito di operazioni elementari sulle
righe.
Esempio 24. La matrice  
1 1 2 0 −1
A = 0 2 1 2 2 
1332 1
non é ridotta per righe. Determiniamone una forma ridotta.

Svolg. Operiamo nel modo seguente:


1. R3 → R3 − R1 in A. La matrice si trasforma nella
 
1 1 2 0 −1
A0 =  0 2 1 2 2 
0212 2
2.9 Rango di una matrice. 23

2. R3 → R3 − R2 in A0 . La matrice si trasforma nella


 
1 1 2 0 −1
A00 =  0 2 1 2 2  .
0000 0

Quest’ultima presenta il pivot a11 sulla prima riga e quello a22 sulla seconda.
Quindi A00 si presenta in forma ridotta a gradini.
Le matrici A, A0 , A00 sono equivalenti fra loro. t
u
Osservazione 2.28. Supponiamo di aver completato il processo di riduzione di una
matrice e di avere ottenuto una matrice ad essa equivalente e che si presenti in forma
a gradini:
 
0 · · · 0 a1, j1 · · · · · · · · · · · · · · · · · · · · · · · · · · · · · · a1,n
 0 · · · · · · 0 · · · 0 a2, j · · · · · · · · · · · · · · · · · · · · · a2,n 
 2 
 0 · · · 0 0 0 · · · 0 · · · 0 a3, j · · · · · · · · · · · · a3,n 
 3 
 ··· ··· ··· ··· ··· ··· ··· ··· ··· ··· ··· ··· ··· ··· ··· 
A=  .
 0 0 · · · · · · · · · · · · · · · 0 · · · 0 · · · 0 ak, jk · · · ak,n 

 0 0 ··· ··· ··· ··· ··· ··· ··· ··· ··· ··· 0 0 
 
 ··· ··· ··· ··· ··· ··· ··· ··· ··· ··· ··· ··· ··· ··· ··· 
0 0 ··· ··· ··· ··· ··· ··· ··· ··· ··· ··· 0 0

Se adesso estraiamo da A tutte le righe e le colonne contenenti i pivot, determiniamo


una sottomatrice quadrata B di A (in base a come abbiamo descritto A, l’ordine di B
é precisamente k), la quale si presenta in forma triangolare superiore:
 
a1, j1 · · · · · · · · · · · · a1, jk
 0 a2, j2 · · · · · · · · · a2, jk 
 
 0 · · · a3, j3 · · · · · · a3, jk 
 
B=  ··· ··· ··· ...
.

 
 ··· ··· ··· ··· ..
 . 

0 0 ··· ··· ak, jk

Inoltre, poiché ai, ji 6= 0 per ogni i = 1, . . . , k, avremo che det(B) = ∏ki=1 ai, ji 6= 0.
In particolare:
Teorema 2.29. Sia A ∈ Mn (R) é una matrice quadrata di ordine n. L’insieme
{R1 , . . . , Rn } di tutte le righe di A é linearmente indipendente se e solo se A é non
singolare.

2.9 Rango di una matrice.

Sia A ∈ Mmn (R), una matrice con m righe e n colonne.


24 2 Le Matrici

Definizione 2.30. Il rango (o caratteristica) di A é l’ordine massimo di un minore


non nullo di A. In altre parole, diciamo che la matrice A ha rango p, e scriviamo
rank(A) = p, se esiste almeno un minore non nullo di ordine p in A, ed inoltre ogni
minore di ordine p + 1 in A é nullo.
Ovviamente rank(A) ≤ min{m, n}.
Osservazione 2.31. Il rango di una matrice A ∈ Mmn (R) é pari al rango della sua tra-
sposta AT . Infatti ogni sottomatrice di AT é la trasposta di una sottomatrice di A (e vi-
ceversa). In particolare ció vale per le sottomatrici quadrate. Poiché il determinante
di ogni matrice quadrata é pari a quello della sua trasposta, allora:
• esiste almeno un minore non nullo di ordine p in A se e solo se esiste almeno un
minore non nullo di ordine p in AT ;
• ogni minore di ordine p + 1 in A é nullo se e solo se ogni minore di ordine p + 1
in AT é nullo.
Da qui la conclusione.
Esempio 25.  
2 10
 3 1 1  ha rango 3
−1 0 1
 
210
 3 1 1  ha rango 2
101
 
2110
ha rango 2
3210
 
2110
ha rango 1.
4220
Talvolta calcolare il rango di una matrice puó non essere facile come negli esempi
precedenti. Ci proponiamo ora di dimostrare come le operazioni elementari sulle
righe di una matrice, pur modificando la matrice di partenza, ne lasciano invariato il
rango. Lo scopo é quello di provare che, volendo calcolare il rango di una matrice,
si possa trasformarla in una ad essa equivalente e di piú facile lettura, essendo certi
che il rango della trasformata sia pari a quello della matrice di partenza. Per farlo, é
quindi sufficiente dimostrare il seguente:
Teorema 2.32. Due matrici equivalenti hanno la stesso rango.

Dim. In realtá é sufficiente mettere insieme alcuni dei risultati giá esposti prece-
dentemente. Infatti, se A, A0 ∈ Mmn (R) sono due matrici equivalenti, allora ciascuna
di esse deriva dall’altra in seguito ad un numero finito di operazioni elementari sulle
righe. Richiamando l’Osservazione 2.24, concludiamo che la matrice A possiede un
minore non nullo di ordine p se e solo se anche A0 ne possiede uno equivalente.
Inoltre ogni minore di ordine p + 1 in A é equivalente ad un minore di ordine p + 1
2.9 Rango di una matrice. 25

di A0 , e viceversa. Quindi, tutti i minori di ordine p + 1 in A sono nulli se e solo se


tali sono tutti i minori di ordine p + 1 di A0 . t
u
Teorema 2.33. Sia A ∈ Mmn (R). Il rango di A é pari al numero massimo di righe
(e di colonne) di A che siano linearmente indipendenti fra loro.
Quindi per calcolare il rango di una matrice potremmo operare nel modo se-
guente: se la matrice é giá ridotta per righe, calcoliamo semplicemente il numero di
righe non nulle (esso é il rango della matrice); altrimenti, per prima cosa riportiamo
la matrice ad una forma ridotta a gradini, tramite le operazioni elementari sulle ri-
ghe, quindi calcoliamo il numero di righe non nulle della forma ridotta ottenuta. É
chiaro che, al termine del processo di riduzione, il numero di righe non nulle é pari
al numero di pivot presenti nella matrice finale. Quindi potremmo definire il rango
della matrice come il numero di pivot presenti in una qualsiasi matrice ridotta per
righe ed equivalente a quella data.
 
1 1 2 0 −1
Esempio 26. La matrice  −3 −3 −6 0 3  ha rango 1, poiché due qualsiasi ri-
2 2 4 0 −2
ghe, scelte tra
 quelle della
 matrice, sono tra loro linearmente dipendenti.
13600
La matrice  0 2 1 0 2  ha rango 3 poiché le tre righe sono tutte tra loro indipen-
13331
denti.
Quanto appena detto viene anche utilizzato per dimostrare il seguente
Teorema 2.34. (Secondo teorema di Laplace) In una matrice quadrata, la somma
dei prodotti degli elementi di una linea (riga o colonna) per i complementi algebrici
degli elementi corrispondenti di un’altra linea parallela é zero.
Completiamo la discussione relativa al rango di una matrice, esponendo un ulte-
riore metodo per il calcolo dello stesso. Per farlo, introduciamo alcune definizioni:
Definizione 2.35. Sia B una sottomatrice quadrata di ordine k di A ∈ Mmn (R). Dicia-
mo matrice orlata di B, una sottomatrice di ordine k + 1 di A ottenuta aggiungendo
a B gli elementi di una riga e di una colonna di A.
Definizione 2.36. Sia M un minore di ordine k di A ∈ Mmn (R), ovvero il determi-
nante di una sottomatrice quadrata B di A di ordine k. Diciamo minore orlato di M,
il determinante della matrice orlata di B, ottenuta aggiungendo a B gli elementi di
una riga e di una colonna di A.
Vale il seguente:
Teorema 2.37. (Teorema degli orlati) Sia A ∈ Mmn (R). Il rango di A é pari a r
se e solo se esiste un minore non nullo di A di ordine r, tale che tutti i suoi minori
orlati siano nulli.
Osservazione 2.38. Attenendoci alla iniziale definizione di rango di una matrice,
dovremmo calcolare tutti i minori della matrice ed individuare l’ordine massimo
26 2 Le Matrici

di quelli non nulli. Ma se la matrice ha m righe e n colonne, allora il numero di


sottomatrici quadrate di ordine k é pari a
   
m n
·
k k

ovvero il prodotto tra le combinazioni di m elementi a gruppi di k e le combinazioni


di n elementi a gruppi di k. Per determinare il rango di A dovremmo allora conside-
rare tutte le possibili sottomatrici quadrate, per k = 1, . . . , min{m, n} e riconoscere
quelle non singolari e di ordine massimo.
Il Teorema degli orlati permette quindi di calcolare il rango di una matrice A piú fa-
cilmente rispetto al calcolo del determinante di tutte le sottomatrici quadrate estrai-
bili da A. Non occorre infatti controllare tutti i minori contenuti in una matrice, ma
solo i minori orlati di un qualsiasi minore non nullo.

2.10 Matrici invertibili.

Diremo che una matrice A ∈ Mn (R), quadrata di ordine n, é invertibile se esiste una
matrice quadrata B ∈ Mn (R) tale che A · B = I, dove I é la matrice identica di ordine
n, avente 1 su tutta la diagonale principale e zero in ogni altra posizione.
Per esempio:  
2 0 −1
A =  7 3 23 
13 4
é invertibile, infatti esiste
− 25 1 −1
 

B =  53
6 −3 3
10 

−6 2 −2
tale che  
100
A·B =  0 1 0 .
001
Se A, B ∈ Mn (R) sono tali che A · B = I, allora diremo che la matrice B é l’inversa
di A, e la indicheremo B = A−1 . Analogamente A é l’inversa di B ed é indicata
A = B−1 .
Osservazione 2.39. Siano A, B matrici quadrate invertibili. Allora

(A · B)−1 = (B)−1 · (A)−1 .

Lemma 2.40. (Esistenza dell’inversa) Una matrice quadrata non singolare (cioé
con determinante non nullo) é sempre invertibile.
2.10 Matrici invertibili. 27

Dim. Dimostriamo l’esistenza dell’inversa di A, costruendola tramite un opportu-


no algoritmo che sfrutti l’ipotesi det(A) 6= 0.
Si costruisca dapprima la matrice aggiunta di A. Quindi si divida ciascun elemento
dell’aggiunta per il determinante di A. Vedremo che la matrice ottenuta é esattamen-
te A−1 .
Costruiamo quindi la matrice Agg(A) −1
det(A) e verifichiamo che essa coincide con A . Indi-
chiamo con Ai j il complemento algebrico dell’elemento ai j in A. Allora l’aggiunta
di A é:  
A11 A21 . . . . . . An1
 A12 A22 . . . . . . An2 
 
 ... ... ... ... ... 
 
 ... ... ... ... ... 
A1n A2n . . . . . . Ann
e sfruttando entrambi i teoremi di Laplace abbiamo che
   
A11 A21 . . . . . . An1 a11 a12 . . . . . . a1n
 A12 A22 . . . . . . An2   a21 a22 . . . . . . a2n 
   
 ... ... ... ... ... · ... ... ... ... ...  =
   
 ... ... ... ... ...   ... ... ... ... ... 
A1n A2n . . . . . . Ann an1 an2 . . . . . . ann
 
det(A) 0 0 ... 0
 0 det(A) 0 . . . 0 
 
0 det(A) . . . 0   = det(A) · I
 0

 ... ... ... ... ... 
0 0 0 . . . det(A)

da cui A · Agg(A)
det(A) = I. t
u
 
110
Esempio 27. Sia A =  2 0 1 . Poiché det(A) = −2, essa é invertibile. Si ha che
111
   
121 −1 −1 1
AT =  1 0 1  , Agg(A) =  −1 1 −1  .
011 2 0 −2

Quindi
1 1
− 12
 
Agg(A)  2 2
A−1 = = 1
2 − 12 12 
−2
−1 0 1
ed infatti   1 1 1
   
110 2 2 −2 100
A · A−1 =  2 0 1  ·  12 − 21 12  =  0 1 0  .
111 −1 0 1 001
28 2 Le Matrici

2.11 Le matrici elementari.

Una matrice quadrata di ordine n é detta elementare se essa é ottenuta dalla matrice
identica di ordine n tramite una sola operazione elementare. Per esempio la matrice
elementare  
100
E = 0 0 1
010
é ottenuta dalla matrice identica I di ordine 3, scambiando tra di loro la seconda e la
terza riga.
Per quanto detto sulle operazioni elementari su matrici quadrate, segue facilmente
che ogni matrice elementare ha determinante non nullo, quindi é invertibile. Ini-
zialmente osserviamo che l’inversa di una matrice elementare é ancora una matrice
elementare. Infatti, si verifica facilmente che:
• se E é una matrice elementare ottenuta dalla matrice identica I, tramite l’opera-
zione di scambio tra righe, ovvero Ri ↔ R j , allora E −1 = E;
• se E é una matrice elementare ottenuta dalla matrice identica I, tramite l’ope-
razione di prodotto di una riga Ri per uno scalare β 6= 0, ovvero Ri → β Ri ,
allora E −1 é la matrice elementare ottenuta dalla matrice identica I, tramite
l’operazione Ri → β −1 Ri ;
• se E é una matrice elementare ottenuta dalla matrice identica I, tramite l’opera-
zione di somma di una riga Ri con il multiplo di una riga R j , ovvero Rii + β R j ,
allora E −1 é la matrice elementare ottenuta dalla matrice identica I, tramite
l’operazione Rii − β R j .
Le matrici elementari sono utilizzate per determinare in modo immediato le tra-
sformazioni per righe su qualsiasi matrice (non necessariamente quadrata). Piú pre-
cisamente, se E é una matrice elementare di ordine n, ottenuta con una qualche
trasformazione elementare, e se A é una matrice con n righe, allora la matrice E · A
é la trasformata di A tramite la medesima operazione con la quale si é ottenuta E.
Per tornare all’esempio precedente, sia
 
1201
A = 1 1 1 2
0111

allora la matrice
     
100 1201 1201
E ·A = 0 0 1·1 1 1 2 = 0 1 1 1
010 0111 1112

si ottiene dalla A scambiando tra loro la seconda e la terza riga (R2 ←→ R3 ).


 
400
Esempio 28. La matrice elementare E =  0 1 0  si ottiene dalla matrice iden-
001
2.11 Le matrici elementari. 29
 
1201
titá moltiplicando la prima riga per 4. Se A =  1 1 1 2  allora la matrice E · A =
0111
     
400 1201 4804
 0 1 0  ·  1 1 1 2  =  1 1 1 2  si ottiene dalla A moltiplicando la prima riga
001 0111 0111
per 4 (R1 → 4R1 ).
 
104
Esempio 29. La matrice elementare E =  0 1 0  si ottiene dalla matrice iden-
001  
1201
titá tramite l’operazione R1 → R1 + 4R3 . Se A =  1 1 1 2  allora la matrice
      0111
104 1201 1645
E · A =  0 1 0  ·  1 1 1 2  =  1 1 1 2  che si ottiene direttamente dalla A
001 0111 0111
tramite l’operazione R1 → R1 + 4R3 .
Osservazione 2.41. Grazie alle proprietá delle operazioni elementari si puó facil-
mente notare che se A é una matrice quadrata ed E é una matrice elementare dello
stesso ordine, allora det(EA) = det(E)det(A).
Inoltre se E1 , E2 , . . . , Ek sono matrici elementari, allora (E1 · E2 · . . . · Ek )A é una ma-
trice che si ottiene dopo aver effettuato sulla A ordinatamente tutte le trasformazioni
individuate dalle matrici Ek , Ek−1 ,. . . ,E1 :

A → Ek A −→ Ek−1 Ek A −→ . . . −→ E1 E2 . . . Ek A.

Osservazione 2.42. Sia A una qualsiasi matrice quadrata non singolare. Sappia-
mo che operando sulle righe si puó ottenere la sua forma a gradini, che nel caso
particolare di matrici quadrate é esattamente una forma triangolare superiore:
 0 0 0
a01n

a11 a12 a13 . . . . . .
 0 a0 . . . . . . . . . a02n 
22
 0 0 a0 . . . . . . a03n 
 
 33 .
 ... ... ... ... ... ... 
 0 0 . . . . . . a0n−1n−1 a0n−1n 
 

0 0 0 ... ... a0nn

In generale non lo riteniamo utile (per i nostri fini), ma potremmo operare ul-
teriori trasformazioni con lo stesso procedimento, con lo scopo di annullare an-
che gli elementi sopra la diagonale, ottenendo una forma diagonale per la matrice
trasformata:
30 2 Le Matrici

a0011
 
0 0 ... ... 0
 0 a0022 0 ... ... 0 
a0033
 
 0
 0 0 ... 0  .
 ... ... ... ... ... ... 
. . . a00n−1n−1
 
 0 0 ... 0 
0 0 0 ... ... a00nn
Poiché A é non singolare, segue che ogni aii é non nullo. Infine moltiplicando ogni
riga i per a−1
ii , otteniamo la matrice identitá come la trasformata della matrice in-
vertibile iniziale, in seguito ad un opportuno numero di operazioni elementari sul-
le righe. In altre parole, esistono opportune matrici elementari E1 , E2 , . . . , Ek tali
che (E1 · E2 · . . . · Ek )A = I. Quindi A = (E1 · E2 · . . . · Ek )−1 = Ek−1 · Ek−1
−1
· . . . · E1−1
cioé ogni matrice non singolare é invertibile e si puó esprimere come prodotto di
opportune matrici elementari.
Teorema 2.43. Una matrice quadrata A di ordine n é invertibile se e solo se
det(A) 6= 0.

Dim. Grazie al Lemma 2.40, non ci resta che dimostrare che se A é invertibile al-
lora il suo determinante é non nullo.
Al contrario, supponiamo che det(A) = 0. Quindi il rango di A é r < n. Siano
E1 , . . . , Ek le matrici elementari tali che E1 · · · Ek A = A0 sia la forma ridotta a gradini
di A. Per quanto detto in precedenza, la matrice A0 presenta tutte le ultime n − r righe
nulle.
Inoltre, poiché A é invertibile, esiste una matrice B di ordine n tale che AB = I.
Allora E1 · · · Ek AB = E1 · · · Ek I, cioé A0 B = E1 · · · Ek I. Ma questo non potrá verificar-
si, poiché la matrice A0 B avrá almeno le ultime n − r righe nulle, quindi avrá rango
≤ r, mentre la matrice E1 · · · Ek I ha certamente rango n. t
u
Terminiamo il paragrafo con il seguente risultato (noto come Teorema di Binet):
Teorema 2.44. Siano A, B matrici quadrate di ordine n. Allora det(A · B) = det(A) ·
det(B).

Dim. Dividiamo la dimostrazione in due casi:


1. Sia det(A) = 0; Supponiamo sia det(AB) 6= 0, quindi AB é una matrice quadrata
invertibile, cioé (AB)(AB)−1 = I, da cui A(B(AB)−1 ) = I. Questo significherebbe
che A é invertibile, contro l’ipotesi che det(A) = 0. Tale contraddizione implica
che det(AB) = 0. Da qui segue che 0 = det(A)det(B) = det(AB), come richiesto.
2. Sia ora det(A) 6= 0; essendo in questo caso A invertibile, dall’Osservazione 2.42,
esistono opportune matrici elementari E1 , . . . , Ek tali che A = E1 · E2 · · · Ek ; inol-
tre sappiamo anche che det(A) = det(E1 )det(E2 ) · · · det(Ek ) (vedi Osservazione
2.41). Quindi

det(AB) = det(E1 E2 · · · Ek B) = det(E1 )det(E2 ) · · · det(Ek )det(B) = det(A)det(B).

t
u
2.12 Matrici ortogonali. 31

Osservazione 2.45. Facile conseguenza del Teorema di Binet é la seguente:


Sia A invertibile tale che A · B = I, allora

1 = det(I) = det(A · B) = det(A) · det(B)


1
da cui det(B) = det(A) .

Di seguito descriviamo alcune proprietá sui prodotti di matrici, utili in seguito.


Teorema 2.46. Siano A ∈ Mmn (R) e B ∈ Mnp (R).
Allora rank(AB) ≤ min{rank(A), rank(B)}.

Dim. Siano C = AB, rank(A) = rA e rank(B) = rB .


Supponiamo di ridurre per righe A e sia A0 la sua forma ridotta a gradini. Esistono
allora opportune matrici elementari E1 , . . . , Ek ∈ Mm (R) tali che

A0 = E1 · · · Ek · A

da cui
E1 · · · Ek ·C = E1 · · · Ek · A · B = A0 · B.
Poiché rank(A0 ) = rank(A) = rA , la matrice A0 avrá le ultime m − rA righe nulle, per
cui, a seguito del prodotto righe per colonne, la matrice A0 B avrá almeno le ultime
m − rA righe nulle, ovvero avrá rango al piú pari a rA . Quindi E1 · · · Ek ·C ha rango ≤
rA . Poiché E1 · · · Ek rappresenta una sequenza finita di operazioni elementari, segue
che
rank(C) = rank(E1 · · · Ek ·C) ≤ rA .
In modo analogo si dimostra che rank(C) ≤ rB . Infatti, poiché CT = BT AT ed
utilizzando il risultato dimostrato nella prima parte del presente Teorema,

rank(C) = rank(CT ) ≤ rank(BT ) = rank(B).

t
u
Teorema 2.47. Siano A ∈ Mn (R) e B ∈ Mnp (R). Se A é invertibile allora rank(AB) =
rank(B).

Dim. Sia C = AB. Per il Teorema precedente, rank(C) ≤ rank(B). Dall’inver-


tibilitá di A, avremo anche B = A−1C. Ancora dal precedente risultato, avremo
rank(B) ≤ rank(C). Da ció segue rank(C) = rank(B). t
u

2.12 Matrici ortogonali.

Diamo adesso qualche cenno ad alcune classi di matrici che scopriremo in seguito
essere di fondamentale importanza:
32 2 Le Matrici

Sia A una matrice con n righe e colonne, A ∈ Mn (R). A é detta ortogonale se AT =


A−1 . Ció vuol dire che

1 = det(I) = det(A · A−1 ) = det(A · AT ) = det(A) · det(AT ) = det(A)2 .

Quindi condizione necessaria (ma non sufficiente) affinché una matrice sia ortogo-
nale é che il suo determinanate sia pari a +1 oppure −1.
 
cos(ϕ) sen(ϕ)
Esempio 30. La matrice A = é ortogonale.
−sen(ϕ) cos(ϕ)
Osservazione 2.48. Da semplici calcoli si ottiene una proprietá delle matrici orto-
gonali: se A e B sono entrambe ortogonali e dello stesso ordine, allora il prodotto
AB é ancora una matrice ortogonale.

2.13 Matrici simili.

Diciamo che due matrici quadrate A, B, entrambe di ordine n, sono simili se esiste
una matrice M quadrata di ordine n, non singolare, tale che A = M −1 · B · M. Si noti
che valgono le seguenti:
1. Ogni matrice quadrata A é simile a se stessa, infatti

A = I −1 · A · I = I · A · I.

2. Se la matrice A é simile alla B allora B é simile ad A, infatti

A = M −1 · B · M ⇒ B = (M −1 )−1 · A · (M −1 ).

3. Se A é simile a B e B é simile a C, allora A é simile a C, infatti

A = M −1 · B · M e B = N −1 ·C · N

implica
A = (N · M)−1 ·C · (N · M).
Quindi la relazione di similitudine tra matrici soddisfa le proprietá riflessiva (1),
simmetrica (2) e transitiva (3), ed é una relazione di equivalenza.
L’insieme della matrici quadrate di ordine n é allora suddiviso in classi di equivalen-
za, nel senso che tutte le matrici tra loro simili costituiscono un’unica classe, avente
come rappresentante una qualsiasi delle matrici che ne fanno parte. Inoltre due di-
stinte classi di equivalenza non possono aver alcuna matrice in comune.
Le proprietá piú rilevanti che, per il momento, possiamo elencare sono le seguenti:
1. Matrici tra loro simili hanno lo stesso determinante (dal Teorema 2.44).
2. Matrici tra loro simili hanno lo stesso rango (Applicando il Teorema 2.47).
Altre importanti proprietá delle matrici simili saranno evidenziate in seguito.
2.14 Fattorizzazione A = LU. 33

2.14 Fattorizzazione A = LU.

Sia A una qualsiasi matrice quadrata di ordine n. Abbiamo giá visto che operando
sulle righe di A si puó ridurre la matrice A in una forma a gradini, ovvero ottenere
una matrice triangolare superiore U che sia ridotta, il cui rango t ≤ n coincide con
quello della matrice di partenza ed é pari al numero di righe non nulle:
 0 0 0
a11 a12 a13 . . . . . . . . . . . . a01n

 0 a022 a023 . . . . . . . . . . . . a02n 
 0 0 a033 . . . . . . . . . . . . a03n 
 
 
 ... ... ... ... ... ... ... ... 
U =  0 0 0 .

 0 0 0 . . . att . . . at,n−1 at,n 
 0 0 0 ... ... ... ... 0 
 
 ... ... ... ... ... ... ... ... 
0 0 0 ... ... ... ...0

Ricordiamo inoltre che abbiamo formalizzato il procedimento per ottenere la ma-


trice U tramite i prodotti per opportune matrici elementari E1 , E2 , . . . , Ek , ovvero
(E1 · E2 · · · Ek )A = U.

Osservazione 2.49. Supporremo in questa prima parte che la forma ridotta finale
’a gradini’ U sia stata ottenuta senza alcuno scambio di righe. Inoltre utilizzeremo
esclusivamente la seguente operazione elementare sulle righe:
• sostituzione di una riga con se stessa, sommata ad una riga che si trovi al di sopra
di essa, la quale sia stata precedentemente moltiplicata per uno scalare non nullo.
É quindi essenziale NON ricorrere ad operazioni elementari che prevedano la
sostituzione di una riga con la somma di se stessa e di un multiplo di una riga
sottostante.
Richiamiamo adesso alcune proprietá, la cui verifica si ottiene attraverso semplici
calcoli (per cui ci limiteremo al loro semplice enunciato):
1. Ogni matrice elementare riferita alle operazioni elementari descritte nell’Osser-
vazione 2.49 é triangolare inferiore.
2. Ogni matrice elementare é invertibile.
3. Il prodotto di matrici triangolari inferiori é ancora una matrice triangolare infe-
riore.
4. Il prodotto di matrici elementari descritte nell’Osservazione 2.49 é quindi una
matrice triangolare inferiore invertibile.
5. L’inversa di una qualsiasi matrice triangolare inferiore é ancora una matrice
triangolare inferiore.
Per quanto appena detto, é evidente che nell’uguaglianza (E1 · E2 · . . . · Ek )A = U,
la matrice E1 · E2 · . . . · Ek = T é invertibile ed é triangolare inferiore. Quindi
34 2 Le Matrici
 
−1
A = (E1 · E2 · . . . · Ek )−1U = Ek−1 · Ek−1 · · · · · · E1−1 U.

Ancora una volta, per le proprietá precedentemente descritte, ciascuna delle matrici
Ek−1 , Ek−1
−1
, . . . E1−1 é triangolare inferiore, per cui il prodotto Ek−1 · Ek−1
−1
· · · · · · E1−1 =
L é una matrice triangolare inferiore.
Abbiamo quindi che A = LU, dove L é una matrice triangolare inferiore ed U una
matrice triangolare superiore.
É importante adesso comprendere quando una matrice ammetta una tale decompo-
sizione, ovvero quando possiamo essere certi a priori che nella riduzione per righe
non venga effettuato alcuno scambio di righe.
Iniziamo con la seguente:
Definizione 2.50. Sia A una matrice quadrata di ordine n. Diremo che A é infe-
riormente riducibile se puó essere trasformata in forma a gradini per righe, ovve-
ro ridotta per righe in forma a gradini, senza utilizzare scambi di righe né alcuna
sostituzione di una riga con la somma di se stessa e di un multiplo di una riga
sottostante.
Se A é una matrice quadrata inferiormente riducibile, allora possiamo decompor-
re A = LU, dove L é una matrice triangolare inferiore invertibile ed U é una matri-
ce triangolare superiore, ovvero la matrice ridotta in forma a gradini per righe ed
ottenuta dalla trasformazione di A tramite le operazioni elementari sopra descritte.
Definizione 2.51. Sia A una matrice quadrata di ordine n. Una sottomatrice quadrata
di A, ottenuta considerando le prime k righe e colonne di A prende il nome di sot-
tomatrice principale alta di ordine k. Quindi A possiede esattamente n sottomatrici
principali, tenendo in considerazione anche A nella sua interezza.
Il seguente risultato fornisce una condizione sufficiente affinché una matrice
quadrata sia fattorizzabile nella forma LU:
Teorema 2.52. Sia A una matrice quadrata reale di ordine n e siano Ak le sue
sottomatrici principali alte di ordine k = 1, . . . , n. Se Ak é non singolare, per ogni
k = 1, . . . , n − 1 allora esiste ed é unica la fattorizzazione A = LU.
Dobbiamo quindi sottolineare che, poiché il Teorema precedente fornisce una
condizione sufficiente alla fattorizzazione LU, esistono matrici che, pur avendo una
qualche sottomatrice principale alta con determinante nullo, ammettono comunque
una fattorizzazione del tipo LU. In questo caso peró, la fattorizzazione non é in
genere unica.
Diversamente, per le matrici invertibili (non singolari), vale il seguente:
Teorema 2.53. Sia A una matrice quadrata reale invertibile di ordine n e siano Ak
le sue sottomatrici principali alte di ordine k = 1, . . . , n. La matrice A ammette una
fattorizzazione LU se e solo se ogni Ak é non singolare, per k = 1, . . . , n − 1.
Quindi, nel caso di matrici invertibili, la condizione é necessaria oltre che
sufficiente.
2.14 Fattorizzazione A = LU. 35

Esempio 31. Consideriamo la matrice


 
1 −4 1
A =  2 −6 5  .
1 −2 5

Osserviamo inizialmente che la matrice ha determinante diverso da zero, quindi é


invertibile. Inoltre le sue sottomatrici principali alte (fino all’ordine n − 1 = 2) sono
 
1 −4
A1 = [1], A2 =
2 −6

entrambe non singolari. Per cui A ammette una fattorizzazione LU. Procediamo
allora con la decomposizione.
Dapprima costruiamo U, ovvero la matrice ottenuta da A dopo la riduzione per righe.
Le prime due operazioni saranno R2 −→ R2 − 2R1 (alla quale associamo la matrice
elementare E1 ), e R3 −→ R3 − R1 (alla quale associamo la matrice elementare E2 ).
La matrice A si trasforma nella
 
1 −4 1
A0 =  0 2 3  .
0 2 4

La successiva operazione é R3 −→ R3 − R2 (alla quale associamo la matrice elemen-


tare E3 ), in seguito alla quale A0 si trasforma nella
 
1 −4 1
U =  0 2 3 .
0 0 1

Descriviamo adesso le matrici elementari utilizzate per trasformare la matrice A. In


base alle operazioni effettuate, abbiamo
     
1 00 1 00 1 0 0
E1 =  −2 1 0  , E2 =  0 1 0  , E3 =  0 1 0 
0 01 −1 0 1 0 −1 1

tali che
E3 · E2 · E1 · A = U
ovvero      
1 0 0 1 −4 1 1 −4 1
 −2 1 0  ·  2 −6 5 = 0 2 3
1 −1 1 1 −2 5 0 0 1
dove  
1 0 0
E3 · E2 · E1 =  −2 1 0  .
1 −1 1
36 2 Le Matrici

La matrice L nella decomposizione A = LU é allora l’inversa della matrice E3 · E2 ·


E1 , la quale, a conti fatti sará:
 
100
L =  2 1 0 .
111

Osservazione 2.54. Sia


 
l11 0 0 ... ... 0
 l12
 l22 0 ... ... 0  
 l13 l23 l33 0 ... 0 
L=
 ...

 ... ... ... ... ...  
 ... ... ... ... ... 0 
ln1 ln2 . . . . . . . . . lnn

la matrice L della fattorizzazione A = LU.


Si puó facilmente osservare che L si puó comporre nel modo seguente:
1. ogni elemento della diagonale principale lii é pari a 1.
2. tutti gli elementi posti al di sopra della diagonale principale li j (i < j) sono nulli.
3. ciascun elemento li j ( con i > j, ovvero posto al di sotto della diagonale prin-
cipale) é pari −αi j , dove αi j é il coefficiente dell’operazione Ri −→ Ri + αi j R j
utilizzata per ridurre a gradini la matrice A.
Nell’esempio precedentemente esposto, abbiamo utilizzato le seguenti operazio-
ni elementari sulle righe:

R2 −→ R2 − 2R1 =⇒ α21 = −2 =⇒ l21 = −α21 = 2

R3 −→ R3 − R1 =⇒ α31 = −1 =⇒ l31 = −α31 = 1


R3 −→ R3 − R2 =⇒ α32 = −1 =⇒ l32 = −α32 = 1
da cui appunto
 
100
L =  2 1 0 .
111
Esempio 32. Consideriamo la matrice
 
121
A =  1 2 2 .
110

Osserviamo inizialmente che la matrice ha determinante diverso da zero, quindi é


invertibile. Inoltre le sue sottomatrici principali alte (fino all’ordine n − 1 = 2) sono
2.14 Fattorizzazione A = LU. 37
 
12
A1 = [1], A2 = .
12

In tal caso det(A2 ) = 0 e viene cosı́ a cadere la condizione necessaria affinché A sia
fattorizzabile in forma LU.
Infatti se supponessimo che A sia fattorizzabile in forma LU, dovrebbero esistere
   
100 xyz
L =  a 1 0 , U =  0 t u 
bc1 00v

tali che      
121 100 xyz
 1 2 2  =  a 1 0 · 0 t u .
110 bc1 00v
Ma eseguendo i calcoli ed eguagliando elemento per elemento la matrice A con il
prodotto LU, avremmo:
1 = x, , 2 = y, 1 = z
1 = ax =⇒ a = 1
2 = ay + t =⇒ t = 0
2 = az + u =⇒ u = 1
1 = bx =⇒ b = 1
1 = by + ct
e da quest’ultima otterremmo l’incongruenza 2 = 0.
Portiamo adesso un esempio di una matrice non invertibile, le cui sottomatrici
principali alte non sono tutte non singolari, e che ammette comunque una decom-
posizione LU (come giá sottolineato la condizione sulle sottomatrici principali é
sufficiente ma non necessaria per matrici non invertibili). Come precedentemente
detto, in tali casi la decomposizione non é necessariamente unica:
Esempio 33. Consideriamo la matrice
 
121
A =  1 2 2 .
120

Osserviamo inizialmente che la matrice ha determinante nullo, quindi non é inverti-


bile. Inoltre le sue sottomatrici principali alte (fino all’ordine n − 1 = 2) sono
 
12
A1 = [1], A2 = .
12

Nonostante det(A2 ) = 0, esiste la decomposizione A = LU. Procediamo con la ri-


duzione di A.
38 2 Le Matrici

Dapprima costruiamo U, ovvero la matrice ottenuta da A dopo la riduzione per ri-


ghe.
Le due operazioni saranno R2 −→ R2 − R1 e R3 −→ R3 − R1 . La matrice A si
trasforma nella  
12 1
U =  0 0 1 .
0 0 −1
In base alle operazioni operazioni elementari effettuate sulle righe, abbiamo:

R2 −→ R2 − R1 =⇒ α21 = −1 =⇒ l21 = −α21 = 1

R3 −→ R3 − R1 =⇒ α31 = −1 =⇒ l31 = −α31 = 1


da cui
 
100
L = 1 1 0
101
ed infatti
     
121 100 12 1
 1 2 2  =  1 1 0 · 0 0 1 .
120 101 0 0 −1
La matrice ammette quindi una fattorizzazione LU, sebbene una delle sottomatrici
principali alte abbia determinante nullo. Si verifica peró che tale fattorizzazione non
é unica, infatti:
     
121 1 0 0 12 1
1 2 2 = 1 1 0  ·  0 0 1  . ∀α ∈ R
120 1 −1 − α 1 00α

2.15 Fattorizzazione PA = LU.

Sia A una qualsiasi matrice quadrata di ordine n. Sappiamo che una qualsiasi matrice
puó essere trasformata, operando sulle righe, in una matrice triangolare superiore U.
Se la trasformazione avviene senza ricorrere a scambi di righe, allora la matrice A
ammette la fattorizzazione del tipo LU, rammentando che NON si deve ricorrere ad
operazioni elementari che prevedano la sostituzione di una riga con la somma di se
stessa e di un multiplo di una riga sottostante.
L’algoritmo di decomposizione LU non puó peró essere applicato a quelle matrici
per le quali, al fine di ottenere una forma ridotta triangolare superiore, sono necessari
gli scambi di righe.
Vediamo come procedere nei casi in cui gli scambi di righe siano indispensabili per
2.15 Fattorizzazione PA = LU. 39

la riduzione della matrice A.


1. Supponiamo che siano stati effettuati k scambi di righe ed indichiamo con
E1 , . . . , Ek , nell’ordine, le matrici elementari corrispondenti a ciascuno scambio
di righe effettuato durante la riduzione.
2. Determiniamo la matrice P = Ek · Ek−1 · · · · · E1 . Tale matrice é invertibile, e la sua
inversa é P−1 = E1 · E2 · · · · · Ek (in quanto ogni matrice Ei é inversa di se stessa).
3. Calcoliamo il prodotto PA: questa é la matrice che otterremmo dalla matrice A
eseguendo, nell’ordine dato, solo gli scambi di righe presenti nella riduzione.
4. La matrice PA si riduce senza scambi di righe, per cui otterremo la fattorizzazione
PA = LU, dalla quale A = P−1 LU.
Esempio 34. Consideriamo la matrice
 
0 1 2
A =  2 −2 1  .
1 −1 6

Dapprima costruiamo U, ovvero la matrice ottenuta da A dopo la riduzione per righe.


La prima operazione é certamente lo scambio della prima riga con la terza: in caso
non effettuassimo tale operazione, non potremmo annullare gli elementi della prima
colonna della matrice, posti sotto la diagonale principale. Quindi R1 ←→ R3 e la
matrice A si trasforma nella  
1 −1 6
A1 =  2 −2 1 
0 1 2
ottenuta dal prodotto P1 A = A1 , dove P1 é la matrice elementare di permutazione
ottenuta dalla matrice identitá attraverso lo scambio della prima riga con la terza,
ovvero  
001
P1 =  0 1 0  .
100
Operando sulle righe di A1 , abbiamo:

R2 −→ R2 − 2R1 =⇒ α21 = −2 =⇒ l21 = −α21 = 2

per cui otteniamo la matrice


 
1 −1 6
A2 =  0 0 −11  .
0 1 2

Infine é sufficiente scambiare fra loro la seconda e la terza riga di A2 , per ottenere
la forma ridotta finale:
40 2 Le Matrici
 
1 −1 6
U = 0 1 2 
0 0 −11
ottenuta dal prodotto P2 A1 = U, dove P2 é la matrice elementare di permutazione
ottenuta dalla matrice identitá attraverso lo scambio della seconda riga con la terza,
ovvero  
100
P2 =  0 0 1  .
010
Calcoliamo adesso P = P2 · P1 :
     
100 001 001
P = 0 0 1·0 1 0 = 1 0 0
010 100 010

e quindi PA = P2 · P1 · A:
     
001 0 1 2 1 −1 6
PA =  1 0 0  ·  2 −2 1 = 0 1 2 .
010 1 −1 6 2 −2 1

A questo punto abbiamo ottenuto una matrice (la PA) che sará riducibile senza al-
cuno scambio di righe.
Operando sulle righe di PA, abbiamo:

R3 −→ R3 − 2R1 =⇒ α31 = −2 =⇒ l31 = −α31 = 2

per cui otteniamo la matrice


 
1 −1 6
U =  0 1 2 .
0 0 −11

Per quanto fatto, avremo quindi che PA = LU, dove


 
100
L = 0 1 0
201

inoltre A = P−1 LU, dove P−1 = P1 · P2 , ovvero:


     
001 100 010
P−1 =  0 1 0  ·  0 0 1  =  0 0 1 
100 010 100

ed infatti
2.15 Fattorizzazione PA = LU. 41
       
0 1 2 010 100 1 −1 6
 2 −2 1  =  0 0 1  ·  0 1 0  ·  0 1 2  .
1 −1 6 100 201 0 0 −11
Esempio 35. Consideriamo la matrice
 
1 1 3 1
2 2 5 3
A=
1
.
1 3 2
3 1 3 1

Dapprima costruiamo U, ovvero la matrice ottenuta da A dopo la riduzione per righe.


Operando sulle righe di A, abbiamo:

R2 −→ R2 − 2R1

R3 −→ R3 − R1
R4 −→ R4 − 3R1
per cui otteniamo la matrice
 
1 1 3 1
 0 0 −1 1 
 0 0 0 1 .
A1 =  

0 −2 −6 −2

A questo punto, operiamo su A1 lo scambio della seconda riga con la quarta: R2 ←→


R4 , ottenendo la matrice  
1 1 3 1
 0 −2 −6 −2 
A2 = 0 0 0 1 

0 0 −1 1
ed ancora operiamo su A2 lo scambio della terza riga con la quarta: R3 ←→ R4 ,
ottenendo la matrice ridotta a gradini
 
1 1 3 1
 0 −2 −6 −2 
A3 = 
 0 0 −1 1  .

0 0 0 1
Le matrici di permutazione utilizzate sono, nell’ordine,
   
1000 100 0
0 0 0 1 0 1 0 0
P1 = 
 0 0 1 0 ,
 P2 = 
0 0 0
.
1
0100 001 0

Calcoliamo adesso P = P2 · P1 :
42 2 Le Matrici
     
1 0 0 0 1 0 0 0 1 0 0 0
0 1 0 0·0 0 0 1 = 0 0 0 1
 
P=
 
0 0 0 1 0 0 1 0 0 1 0 0
0 0 1 0 0 1 0 0 0 0 1 0

ed anche P−1 = P1 · P2 :
     
1 0 0 0 1 0 0 0 1 0 0 0
0 0 0 1
−1 ·0 1 0 0 = 0 0 1 0
 
P = .
0 0 1 0 0 0 0 1 0 0 0 1
0 1 0 0 0 0 1 0 0 1 0 0

Quindi PA = P2 · P1 · A, cioé:
     
10 0 0 1 1 3 1 1 1 3 1
0 0 0 1 2 2 5 3 3 1 3 1
PA =  · = .
0 1 0 0 1 1 3 2 2 2 5 3
00 1 0 3 1 3 1 1 1 3 2

A questo punto abbiamo ottenuto una matrice (la PA) che sará riducibile senza al-
cuno scambio di righe.
Operando sulle righe di PA, abbiamo:

R2 −→ R2 − 3R1 =⇒ α21 = −3 =⇒ l21 = −α21 = 3

R3 −→ R3 − 2R1 =⇒ α31 = −2 =⇒ l31 = −α31 = 2


R4 −→ R4 − R1 =⇒ α41 = −1 =⇒ l41 = −α41 = 1.
Otteniamo quindi la matrice ridotta
 
1 1 3 1
0 −2 −6 −2 
U = .
0 0 −1 1 
0 0 0 1

e simultaneamente la matrice triangolare inferiore


 
1000
3 1 0 0
L= 2 0 1 0

1001

tali che PA = LU ovvero A = P−1 LU cioé:


       
1131 1000 1 0 0 0 1 1 3 1
 ·  0 −2 −6 −2  .
2 2 5 3 0 0 1 0 3 1 0 0  
A= 1 1 3 2 = 0 0 0 1·2
   
0 1 0   0 0 −1 1 
3131 0100 1 0 0 1 0 0 0 1
2.16 Un algoritmo di inversione. 43

2.16 Un algoritmo di inversione.

Sia A una matrice quadrata non singolare, quindi invertibile. Ci proponiamo di de-
scrivere un metodo per ottenere la matrice inversa di A, utilizzando le operazioni
elementari sulle righe.
Poiché det(A) 6= 0, sappiamo che esistono opportune operazioni elementari che tra-
sformano A nella matrice identitá I. Siano, nell’ordine, E1 , . . . , Ek le matrici ele-
mentari che rappresentano le operazioni elementari utilizzate e sia E = Ek · · · · · E1 .
Quindi EA = I, da cui segue che EI = A−1 .
Concludiamo allora che, le stesse operazioni elementari che trasformano A → I, tra-
sformano I → A−1 .
Per ottenere la matrice inversa di A sará allora sufficiente ricopiare la matrice I ac-
canto alla A, ottenendo la matrice rettangolare [A|I]. Quindi si riduce la matrice A
al fine di trasformarla in I. Con tale procedimento si agisce anche sulla matrice I,
trasformandola nella A−1 :
[A|I] −→ [I|A−1 ].
Esempio 36. Consideriamo la seguente matrice non singolare
 
121
A =  2 4 1 .
224

Costruiamo la matrice rettangolare [A|I]:


 
121|100
B = 2 4 1 | 0 1 0
224|001

e riduciamola per righe, con il fine ultimo di trasformare A → I.


Operando sulle righe di B, abbiamo:

R2 −→ R2 − 2R1

R3 −→ R3 − 2R1
per cui otteniamo la matrice
 
1 2 1 | 1 00
B1 =  0 0 −1 | −2 1 0  .
0 −2 2 | −2 0 1

Quindi scambiamo fra loro la seconda e la terza riga di B1 :


 
1 2 1 | 1 00
B2 =  0 −2 2 | −2 0 1  .
0 0 −1 | −2 1 0
44 2 Le Matrici

Torniamo ad operare sulle righe:

R1 −→ R1 + R3

R2 −→ R2 − 2R3
da cui:  
1 2 0 | −1 1 0
B3 =  0 −2 0 | −6 2 1  .
0 0 −1 | −2 1 0
La successiva operazione su B3 sará R1 → R1 + R2 , in seguito alla quale abbiamo:
 
1 0 0 | −7 3 1
B4 =  0 −2 0 | −6 2 1  .
0 0 −1 | −2 1 0

Infine, in B4 , moltiplichiamo la seconda riga per − 12 e la terza riga per −1, ovvero

R2
R2 → − R3 → −R3
2
ottenendo quindi la forma finale B5 = [I|A−1 ]:
 
1 0 0 | −7 3 1
[I|A−1 ] =  0 1 0 | 3 −1 − 21 
0 0 1 | 2 −1 0

per cui  
−7 3 1
A−1 =  3 −1 − 12  .
2 −1 0

2.17 Esercizi svolti.

Esercizio 1. Si determini il rango della seguente matrice:


 
1 1 2
 3 1 1 .
1 −1 −3

Svolgimento: La matrice é quadrata. Possiamo controllare inizialmente se il suo


rango é massimo, semplicemente calcolando il determinante. Osserviamo quindi
che la matrice ha determinante nullo ed inoltre essa possiede minori di ordine 2 non
nulli. Concludiamo che il suo rango é 2.
Esercizio 2. Si determini il rango della seguente matrice:
2.17 Esercizi svolti. 45
 
111
 2 0 1 .
120

Svolgimento: La matrice é quadrata ed ha determinante non nullo, per cui il suo


rango é pari al suo ordine, ovvero 3.
Esercizio 3. Si determini il rango della seguente matrice, usando il metodo della
riduzione per righe:  
3 1 2 1
1 2 1 2 
 
A=  1 1 1 1 .

4 4 3 4 
1 −2 0 −2

Svolgimento: Inizialmente scambiamo la prima e la quinta riga, ottenendo la


matrice  
1 −2 0 −2
1 2 1 2 
0
 
A = 1 1 1 1 .

4 4 3 4 
3 1 2 1
Eseguiamo su A0 le seguenti operazioni elementari:

R2 → R2 − R1 , R3 → R3 − R1 , R4 → R4 − 4R1 , R5 → R5 − 3R1 .

La matrice risultante é  
1 −2 0 −2
0 4 1 4 
A00 = 
 
0 3 1 3 .
 0 12 3 12 
0 7 2 7
Eseguiamo adesso su A00 le seguenti operazioni elementari:
3 7
R3 → R3 − R2 , R4 → R4 − 3R2 , R5 → R5 − R2
4 4
ottenendo quindi la matrice
 
1 −2 0 −2
0 4 1 4 
000 1
 
A =  0 0 4 0 .

0 0 0 0 
0 0 14 0

Scambiando la quarta e la quinta riga avremo


46 2 Le Matrici
 
1 −2 0 −2
0 4 1 4 
iv 1
 
A = 0 0 14 0  .

0 0
4 0

0 0 0 0

Ed infine, sottraendo la terza riga dalla quarta riga:


 
1 −2 0 −2
0 4 1 4 
v
 1

A =  0 0 4 0 .

0 0 0 0 
0 0 0 0

La matrice é adesso in forma ridotta a gradini. Concludiamo quindi che il suo rango
é 3, poiché 3 sono le righe non nulle.
Esercizio 4. Si determini il rango della seguente matrice, usando il metodo della
riduzione per righe:  
11101
2 1 3 1 2
 2 2 2 1 1 .
 

32422

Svolgimento: Iniziamo eseguendo le seguenti operazioni elementari:

R2 → R2 − 2R1 , R3 → R3 − 2R1 , R4 → R4 − 3R1

da cui  
1 1 10 1
 0 −1 1 1 0 
A0 = 
 0 0 0 1 −1  .

0 −1 1 2 −1
Su A0 operiamo con R4 → R4 − R2 , ottenendo
 
1 1 10 1
 0 −1 1 1 0 
A00 =  0 0 0 1 −1  .

0 0 0 1 −1

Ed infine, su A00 , eseguiamo R4 → R4 − R3 :


 
1 1 10 1
 0 −1 1 1 0 
A000 = 
 0 0 0 1 −1  .

0 0 00 0
2.17 Esercizi svolti. 47

Quest’ultima matrice é in forma ridotta e presenta 3 righe non nulle, quindi il rango
é pari a 3.
Esercizio 5. Si determini il rango della seguente matrice, usando il Teorema degli
orlati  
1121
 2 1 3 1 .
1010

Svolgimento: Individuiamo un minore di ordine 2 non nullo:



1 1
2 1 6= 0.

Tutti i suoi possibili orlati, ovviamente di ordine 3, sono



1 1 2 1 1 1

2 1 3 = 0, 2 1 1 = 0

1 0 1 1 0 0

per cui il rango della matrice é 2.


Esercizio 6. Si determini il rango della seguente matrice, usando il teorema degli
orlati  
1 0 12
1 3 0 0
 1 −3 2 4  .
 

2 3 12

Svolgimento: Vi é un minore di ordine 2 non nullo:



1 0
1 3 6= 0.

Tutti i suoi possibili orlati sono



1 0 1 1 0 2

1 3 0 = 0, 1 3 0 = 0

1 −3 2 1 −3 4

1 0 1 1 0 2

1 3 0 = 0, 1 3 0 = 0

2 3 1 2 3 2
per cui il rango della matrice é pari a 2.
Esercizio 7. Si determini il rango della seguente matrice al variare del parametro
reale k:
48 2 Le Matrici
 
1211
 k 1 k 1 .
k1kk

Svolgimento: Vi é un minore di ordine 2 non nullo, che scaturisce dall’incrocio


delle righe R1 , R2 e delle colonne C2 ,C4 :

2 1
1 1 6= 0.

Consideriamo adesso tutti i possibili minori di ordine 3, cancellando, una alla volta,
le colonne della matrice:

2 1 1
1 k 1 = 2k2 − 3k + 1, il quale si annulla per k ∈ {1, 1 }


1 k k
2

1 1 1

k k 1 = 0 ∀k ∈ R

k k k

1 2 1
k 1 1 = −2k2 + 3k − 1, il quale si annulla per k ∈ {1, 1 }


k 1 k
2

1 2 1

k 1 k = 0, ∀k ∈ R.

k 1 k

Concludiamo allora che il rango della matrice é pari 3 per ogni k 6= 1, 21 , ed é


pari a 2 per k ∈ {1, 21 }.
Esercizio 8. Si determini il rango della seguente matrice al variare dei valori reali
di k (usando eventualmente il teorema degli orlati)
 
210k
 3 1 2 2 .
10k1

Svolgimento: Vi é un minore di ordine 2 non nullo, che scaturisce dall’incrocio


delle righe R1 , R2 e delle colonne C1 ,C2 :

2 1
3 1 6= 0.

Consideriamo adesso tutti i possibili minori orlati del precedente, di ordine 3:



2 1 0

3 1 2 = 2−k

1 0 k
2.17 Esercizi svolti. 49

e
2 1 k

3 1 2 = 1 − k.

1 0 1
Quindi, se si annulla uno dei due minori orlati, l’altro é non nullo, per cui esiste
sempre un minore di ordine 3 non nullo. Concludiamo che il rango della matrice é
pari a 3, per ogni k ∈ R.
Esercizio 9. Si determini il rango della seguente matrice al variare dei valori reali
di k (usando eventualmente il teorema degli orlati)
 
k−1 2 1 1
 1 3 2 k .
1 110

Svolgimento: Vi é un minore di ordine 2 non nullo, che scaturisce dall’incrocio


delle righe R1 , R2 e delle colonne C2 ,C3 :

2 1
3 2 6= 0.

Consideriamo adesso tutti i possibili minori orlati del precedente, di ordine 3:



2 1 1

3 2 k = 1−k

1 1 0

e
2 1 k−1

3 2 1 = k − 1.

1 1 1
Quindi, se k = 1 ogni orlato si annulla, per cui concludiamo che il rango della
matrice é pari a 3, per ogni k 6= 1, ed il rango é pari a 2 per k = 1.
Esercizio 10. Si determini il rango della seguente matrice al variare dei valori reali
di k  
k110
 0 0 1 k .
2101

Svolgimento: Inizialmente scambiamo la seconda e la terza riga, ottenendo la


matice  
k110
A0 =  2 1 0 1  .
001k
Per k 6= 0, operiamo su A0 come segue: R2 → R2 − 2k R1 . La matrice risultante é
50 2 Le Matrici
 
k 1 1 0
A00 =  0 k−2 2 
k −k 1 .
0 0 1 k

Se k 6= 2, la matrice A00 é ridotta e presenta 3 righe non nulle. Se k = 2 allora


 
21 1 0
A00 =  0 0 −1 1  .
00 1 2

Sommando terza e seconda riga otteniamo


 
21 1 0
A000 =  0 0 −1 1
00 0 3

che é ancora in forma ridotta e presenta 3 righe non nulle.


Possiamo allora inizialmente affermare che per ogni k 6= 0 il rango della matrice
iniziale é pari a 3.
Resta da controllare il caso k = 0. Ripartiamo quindi dalla matrice A0 , la quale per
k = 0 assume la seguente forma:
 
0110
A0 =  2 1 0 1  .
0010

Scambiando la prima e la seconda riga avremo


 
2101
A0 =  0 1 1 0 
0010

che é in forma ridotta e presenta 3 righe non nulle.


Concludiamo quindi che il rango della matrice é pari a 3, per ogni valore di k ∈ R.
Esercizio 11. Si determini il rango della seguente matrice al variare dei valori reali
di k  
k110
A =  2 2 2 0 .
0101

Svolgimento: Per k 6= 0, eseguiamo la seguente operazione elementare R2 → R2 −


2
k , ottenendo la matrice  
k 1 1 0
A0 =  0 2k−2
k
2k−2 
k 0 .
0 1 0 1
2.17 Esercizi svolti. 51

Per k 6= 1, la matrice A0 é in forma ridotta, per cui il rango della matrice é pari a 3.
Nel caso di k = 1, la matrice A0 assume la seguente forma
 
1110
A0 =  0 0 0 0 
0101

e, presentando una riga nulla, il suo rango é pari a 2.


Resta da discutere il caso k = 0, ovvero quando la matrice iniziale é
 
0110
A =  2 2 2 0 .
0101

Riordinando opportunamente le righe, avremo


 
2220
0 1 1 0
0101

ed infine, sottraendo la seconda dalla terza riga,


 
22 2 0
 0 1 1 0 .
0 0 −1 1

Quest’ultima é in forma ridotta e presenta 3 righe non nulle.


In conclusione: il rango di A é pari a 3 per ogni k 6= 1, ed é pari a 2 per k = 1.
Esercizio 12. Si determini il rango della seguente matrice, al variare del parametro
reale k  
k+1 k+2 k+3
A= 1 2 3 .
1 − 2k 2 − 2k 3 − 2k

Svolgimento: Dapprima scambiamo la prima e la seconda riga:


 
1 2 3
A0 =  k + 1 k + 2 k + 3  .
1 − 2k 2 − 2k 3 − 2k

Per k 6= −1, 21 , eseguiamo le seguenti operazioni elementari su A0

R2 → R2 − (k + 1)R1 , R3 → R3 − (1 − 2k)R1

da cui
52 2 Le Matrici
 
1 2 3
A00 =  0 −k −2k  .
0 2k 4k
Se k = 0, segue banalmente che il rango é pari a 1.
Nel caso k 6= 0, eseguendo su A00 l’operazione R3 → R3 + 2R2 la matrice si trasforma
in  
1 2 3
A000 =  0 −k −2k 
0 0 0
che é in forma ridotta con 2 righe non nulle, per cui il suo rango é 2.
Consideriamo adesso i casi inizialmente esclusi.
Per k = −1 la matrice A si riscrive
 
012
1 2 3
345

ed il suo determinante é nullo. É facile individuare un minore non nullo di ordine 2,


quindi il rango é pari a 2.
Per k = 21 , la matrice A diventa
3 5 7
2 2 2
 1 2 3 .
0 1 2
Anche in quest’ultimo caso, il determinante é nullo ed il rango é 2.
In conclusione: per k = 0 il rango di A é 1, per ogni k 6= 0 il suo rango é 2.
Esercizio 13. Si trasformi la seguente matrice non singolare nella matrice identitá,
usando il metodo della riduzione per righe:
 
112
 0 1 1 .
311

Svolgimento: Iniziamo con l’operazione elementare R3 → R3 − 3R1 , da cui


 
1 1 2
A0 =  0 1 1  .
0 −2 −5

Procediamo adesso con R3 → R3 + 2R2 :


 
11 2
A00 =  0 1 1  .
0 0 −3

Occupiamoci ora della parte alta della matrice:


2.17 Esercizi svolti. 53

1 2
R2 → R2 + R3 , R1 → R1 + R3 .
3 3
Otteniamo la matrice:  
11 0
A000 =  0 1 0  .
0 0 −3
Quindi eseguiamo R1 → R1 − R2 , da cui
 
10 0
Aiv =  0 1 0 
0 0 −3

ed infine R3 → − 31 R3 , ovvero dividiamo la terza riga per −3, ottenendo


 
100
Av =  0 1 0  .
001

Esercizio 14. Si trasformi la seguente matrice non singolare nella matrice identitá,
usando il metodo della riduzione per righe:
 
211
 0 3 2 .
404

Svolgimento: Iniziamo con la trasformazione R3 → R3 − 2R1 , in seguito alla quale


avremo  
2 1 1
A0 =  0 3 2  .
0 −2 2
Quindi eseguiamo R3 → R3 + 23 R2 , da cui
 
21 1
A00 =  0 3 2  .
0 0 10
3

Occupiamoci della parte alta della matrice: le operazioni


6 3
R2 → R2 − R3 , R1 → R1 − R3
10 10
forniscono la matrice  
21 0
A000 =  0 3 0  .
0 0 10
3
54 2 Le Matrici

Quindi l’operazione R1 → R1 − 13 R2 , determina la trasformazione nella matrice


 
20 0
Aiv =  0 3 0  .
0 0 10
3

Infine, eseguendo le operazioni


1 1 3
R1 → R1 , R2 → R2 , R3 → R3
2 3 10
otteniamo la matrice  
100
Av =  0 1 0  .
001

Esercizio 15. Si determini (se possibile) la matrice inversa della seguente


 
1 2 0
A =  −1 2 2  .
1 −1 −1

Svolgimento: La matrice ha determinante pari a 2, quindi é invertibile.


La sua trasposta é  
1 −1 1
AT =  2 2 −1  .
0 2 −1
Da questa calcoliamo la matrice aggiunta di A, ovvero la matrice i cui ingressi (i, j)
sono i complementi algebrici degli elementi (i, j) della trasposta AT :
 
0 2 4
ad j(A) =  1 −1 −2  .
−1 3 4

Infine, dividendo quest’ultima per il determinante di A, otteniamo l’inversa:


 
0 1 2
A−1 =  12 − 21 −1  .
− − 12 32 2

Esercizio 16. Si determinino i valori reali di k per cui la seguente matrice sia
invertibile  
0 k10
 0 1 2 1
 k +1 0 0 0 .
 

0 213
2.17 Esercizi svolti. 55

Svolgimento: Il determinante della matrice é (k + 1)(5k − 1), per cui la matrice é


invertibile per ogni valore reale k 6= −1, 51 .
Capitolo 3
Sistemi Lineari

3.1 Sistemi lineari equivalenti.

Una equazione del tipo a1 x1 + a2 x2 + a3 x3 + . . . + an xn = b, con a1 , . . . , an , b ∈ R,


si dice lineare; x1 , . . . , xn sono le incognite, b é il termine noto, a1 , . . . , an sono i
coefficienti delle incognite. Risolvere una equazione lineare vuol dire determinare
una n-upla di valori in R, (c1 , . . . , cn ), da attribuire alle incognite x1 , . . . , xn , tali che
a1 c1 + a2 c2 + . . . + an cn = b. Un sistema lineare di m equazioni in n incognite sul
campo R é un insieme di m equazioni lineari nelle stesse incognite x1 , . . . , xn , cioé

 a11 x1 + a12 x2 + . . . + a1n xn = b1

a21 x1 + a22 x2 + . . . + a2n xn = b2

.

 .........
am1 x1 + am2 x2 + . . . + amn xn = bm

Una soluzione di un tale sistema é una n-upla di valori reali (c1 , . . . , cn ) da attribuire
alle incognite x1 , . . . , xn , tali che essi verifichino ciascuna delle m equazioni, cioé:

 a11 c1 + a12 c2 + . . . + a1n cn = b1

a21 c1 + a22 c2 + . . . + a2n cn = b2

.

 .........
am1 c1 + am2 c2 + . . . + amn cn = bm

Se consideriamo le seguenti matrici:


  
 x1
 b1
a11 a12 . . . a1n  x2   b2 
 a21 a22 . . . a2n     
 ... ... ... ... ,X =  ... ,B =  ... 
A=     
 ...   ... 
am1 am2 . . . amn
xn bm
allora il sistema lineare si puó riscrivere in forma compatta nel modo seguente:

57
58 3 Sistemi Lineari

A · X = B.
 
a11 a12 . . . a1n b1
 a21 a22 . . . a2n b2 
Chiameremo A matrice incompleta e C =  ...
 matrice completa
... ... ... ... 
am1 am2 . . . amn bm
del sistema lineare.

Siano A e C le matrici associate ad un sistema lineare. Supponiamo che la matrice


C non sia ridotta per righe. Dopo aver effettuato la riduzione della matrice C, indi-
chiamo con A0 e C0 le matrici trasformate e ridotte. Ad esse viene associato ancora
un sistema lineare A0 · X = B0 . Vale il seguente:
Teorema 3.1. Le soluzioni di un sistema lineare A · X = B sono le stesse di cia-
scun sistema lineare A0 · X = B0 ottenuto da esso attraverso operazioni elementari
effettuate sulle righe delle matrici caratteristiche del sistema. Tali sistemi sono detti
equivalenti.
Esempio 37. Si discuta il sistema lineare

 x1 + x2 + x3 = 3
x1 + x2 − x3 = 2 .
x1 − x2 + x3 = 2

Svolg. Le matrici associate sono


   
1 1 1 1 1 1 3
A =  1 1 −1  ,C =  1 1 −1 2 .
1 −1 1 1 −1 1 2

Riduciamo la matrice C:
 
11 1 3
R3 → R3 + R2 , C0 =  1 1 −1 2 
20 0 4
 
11 1 3
R2 → R2 − R1 , C0 =  0 0 −2 −1  .
20 0 4
Il sistema associato a questa matrice é

 x1 + x2 + x3 = 3
−2x3 = −1
2x1 = 4

la cui soluzione é (x1 = 2, x2 = 21 , x3 = 21 ). Poiché il sistema trasformato é equiva-


lente a quello di partenza, quest’ultima é anche soluzione del primo sistema. t
u
3.1 Sistemi lineari equivalenti. 59

Ma non tutti i sistemi lineari hanno necessariamente una soluzione, per esempio
basti pensare al seguente: 
x1 + x2 = 2
.
2x1 + 2x2 = 1
Diremo compatibili i sistemi lineari che ammettono soluzione ed incompatibili quel-
li che non ne ammettono alcuna. Parallelamente, non é detto che se un sistema
lineare ha una soluzione, esso abbia solo quella, eccone un esempio:

 x1 + x2 + x3 = 2
2x1 + 2x2 + 2x3 = 4
x1 − x2 = 0

che ammette come soluzioni le infinite terne (α, α, 2 − 2α), per ogni α ∈ R.
Diremo indeterminati i sistemi lineari che ammettono infinite soluzioni.
Teorema 3.2. (di Rouché-Capelli) Sia A·X = B un sistema lineare di m equazioni in
n incognite, con matrici associate A ∈ Mmn (R), C ∈ Mmn+1 (R). Esso é compatibile
se e solo se il rango della matrice incompleta A é uguale al rango della matrice
completa C. Inoltre se il sistema é compatibile, detto r il rango delle matrici, le
soluzioni sono in numero di ∞n−r , nel caso n > r. Infine, se n = r il sistema ammette
un’unica soluzione.

Dim. Ipotizziamo dapprima che il sistema sia compatibile. Sia A0 ·X = B0 il sistema


lineare ottenuto da A · X = B dopo aver ridotto per righe la matrice completa [A|B],
in modo tale che rango(A) = rango(A0 ) e rango([A|B]) = rango([A0 |B0 ]) ed i due
sistemi siano equivalenti.
Supponiamo per assurdo che r = rango(A0 ) < rango([A0 |B0 ]). Ordiniamo le righe del
sistema in modo che le prime r righe di A0 siano proprio quelle non nulle. Allora tutte
le righe di A0 dalla r +1-esima in poi sono nulle. Poiché rango(A0 ) < rango([A0 |B0 ]),
esiste almeno una tra le righe di ([A0 |B0 ]) comprese tra la r +1-esima e la n-esima che
non é nulla. Possiamo per esempio supporre che sia la riga di posto r +1. In tal modo
la (r + 1)-esima equazione del sistema condurrebbe alla seguente incongruenza:

0 = 0 · x1 + 0 · x2 + . . . . . . + 0 · xn = br+1 6= 0.

Viceversa supponiamo ora che rango(A) = rango([A|B]) = r. Ció vuol dire che la
colonna dei termini noti B non incide nel calcolo dei ranghi, in altre parole la co-
lonna B é combinazione lineare delle colonne della matrice A. Allora ogni elemento
bi che si trovi su una qualsiasi riga i-esima di B é combinazione lineare di tutti gli
elementi che si trovano sulla riga i-esima di A. Quindi esistono α1 , . . . , αn ∈ R tali
che:
a11 α1 + a12 α2 + . . . + a1n αn = b1
a21 α1 + a22 α2 + . . . + a2n αn = b2
a31 α1 + a32 α2 + . . . + a3n αn = b3
60 3 Sistemi Lineari

.......................................
am1 α1 + am2 α2 + . . . + amn αn = bm
cioé il sistema ammette almeno la soluzione (x1 , . . . , xn )=(α1 , . . . , αn ).
Omettiamo la dimostrazione dell’ultima parte del Teorema: ne faremo una verifica
diretta tramite gli Esempi. t
u
Esempio 38. Si discuta il sistema

 x1 + x2 + x3 = 1
2x1 − 2x2 + 2x3 = 0 .
x1 − x3 = 1

 
1 1 1
Svolg. La matrice incompleta é A =  2 −2 2  che ha rango 3. Ovviamente
1 0 −1
anche la matrice completa avrá rango 3. Il sistema é compatibile ed ammette una
sola soluzione. t
u
Esempio 39. Si discuta il sistema

 x1 + x2 − x3 + x4 = 0
3x1 + x2 − x3 = 1 .
2x1 − x4 = 1

 
1 1 −1 1
Svolg. La matrice incompleta é A =  3 1 −1 0  che ha rango 2. La matrice
 2 0 0 −1

1 1 −1 1 0
completa é C =  3 1 −1 0 1  che ha ancora rango 2. Il sistema é compatibile ed
2 0 0 −1 1
ammette ∞4−2 = ∞2 soluzioni. Il rango 2 ci é dato dalle prime due righe, le terza é
combinazione lineare di esse. Allora un sistema equivalente
 a quello dato é quello
1 1 −1 1 0
associato alla seguente matrice: C0 =  3 1 −1 0 1  e si scrive
00 0 00

x1 + x2 = x3 − x4
.
3x1 + x2 = 1 + x3

Dalla prima otteniamo x1 = −x2 + x3 − x4 e sostituendo nella seconda:

−3x2 + 3x3 − 3x4 + x2 = 1 + x3 cioé − 2x2 = 1 − 2x3 + 3x4


1 − 2x3 + 3x4
x2 = −
2
ed ancora sostituendo il valore di x2 nella espressione di x1 :
3.2 Sistemi omogenei. 61

1 − 2x3 + 3x4
x1 = + x3 − x4 .
2
Allora la generica soluzione del sistema lineare é
1 1 1 3
x1 = + β x2 = − + α − β x3 = α x4 = β
2 2 2 2
per ogni valore reale dei parametri α e β . t
u
Esempio 40. Si discuta il sistema

 x1 + x2 − x3 = 2
x1 + 2x2 − x3 = 0 .
2x1 + 3x2 − 2x3 = 1

 
1 1 −1
Svolg. La matrice incompleta é A =  1 2 −1  che ha rango 2. La matrice
  2 3 −2
1 1 −1 2
completa é C =  1 2 −1 0  che ha rango 3. Il sistema é incompatibile. t
u
2 3 −2 1

3.2 Sistemi omogenei.

Sia A ∈ Mmn (R), matrice con m righe  e n colonne. Il sistema lineare A · X = B é detto
0
 0 
 
omogeneo se B = 0 cioé se B =   . . .  é la m-upla nulla. Tali sistemi hanno sempre

 ... 
0
la soluzione banale, (x1 = 0, . . . , xn = 0). Poiché in tale caso la matrice completa e
quella incompleta hanno sempre lo stesso rango, la soluzione banale é anche l’unica
soluzione se e solo se il rango della matrice A é pari al numero n di incognite. Al
contrario, se il rango di A é r < n allora le soluzioni sono in numero di ∞n−r .
Esempio 41. Si discuta il sistema

 x1 + x2 + 2x3 = 0
2x1 + 2x2 + 3x3 = 0 .
3x1 + x2 + 2x3 = 0


112
Svolg. La matrice associata é A =  2 2 3  che ha rango 3. Allora vi é la sola
312
soluzione banale. t
u
62 3 Sistemi Lineari

Esempio 42. Si discuta il sistema



 x1 + x2 + 2x3 + x4 = 0
2x1 + 2x2 + 3x3 + 2x4 = 0 .
x1 + x2 + x3 + x4 = 0

 
1121
Svolg. La matrice associata é A =  2 2 3 2  che ha rango 2. Allora vi sono ∞2
1111
soluzioni. Il rango é 2 poiché l’ultima riga é combinazione lineare delle precedenti
due. Quindi un sistema equivalente
 al precedente é quello che ha come matrice
1121
associata la seguente: A0 =  2 2 3 2 . Tale sistema si scrive:
0000

x1 + 2x3 = −x2 − x4
.
2x1 + 3x3 = −2x2 − 2x4

Dalla prima otteniamo x1 = −x2 − 2x3 − x4 . Sostituendo nella seconda otteniamo

−2x2 − 4x3 − 2x4 + 3x3 = −2x2 − 2x4 cioé x3 = 0.

Un generica soluzione del sistema é allora

x1 = −α − β x2 = α x3 = 0 x4 = β

al variare dei parametri α e β in R. t


u
Caso particolare é quello in cui r = n − 1. In tal caso le soluzioni non banali del
sistema lineare omogeneo si ottengono nel modo seguente:
• si riordinano le righe della matrice associata al sistema iniziale, in modo che le
prime n − 1 siano linearmente indipendenti;
• si cancellano le ultime m − n + 1 righe, ovvero quelle linearmente dipendenti dalle
prime n − 1;
• indichiamo la matrice ottenuta nel modo seguente:
 
a11 a12 a13 . . . . . . . . . . . . a1n
 a21 a22 a23 . . . . . . . . . . . . a2n 
 
 a31 a32 a33 . . . . . . . . . . . . a3n 
 a41 a42 a43 . . . . . . . . . . . . a4n  ;
 
 
 ...... ...... ...... ... ... ...... ...... 
an−11 an−12 an−13 . . . . . . . . . . . . an−1n

• A partire dalla precedente, consideriamo ora la seguente matrice:


3.2 Sistemi omogenei. 63
 
a11 a12 a13 . . . . . . . . . . . . a1n
 a21
 a22 a23 . . . . . . . . . . . . a2n  
 a31
 a32 a33 . . . . . . . . . . . . a3n  
A=
 a41 a42 a43 . . . . . . . . . . . . a4n  
 ...... ...... ...... ... ... ...... ...... 
 
 an−11 an−12 an−13 . . . . . . . . . . . . an−1n 
an1 an2 an3 . . . . . . . . . . . . ann
nella quale abbiamo aggiunto un’ultima riga linearmente dipendente dalle prece-
denti, scelta arbitrariamente tra le m − n + 1 righe precedentemente scartate. Per
semplicitá abbiamo scelto di aggiungere la n-esima riga. Osserviamo subito che
det(A) = 0, qualunque sia la riga che scegliamo di aggiungere.
Siano Ai j i complementi algebrici degli elementi della matrice A sopra descritta.
Se applichiamo il secondo teorema di Laplace alla matrice A, in ogni sua riga ad
esclusione dell’ultima, otteniamo

a j1 An1 + a j2 An2 + . . . + a jn Ann = 0

per ogni indice j = 1, . . . , n − 1. Inoltre, poiché det(A) = 0, abbiamo anche

an1 An1 + an2 An2 + . . . + ann Ann = 0

ed a fortiori
α(a j1 An1 + a j2 An2 + . . . + a jn Ann ) = 0
per ogni α ∈ R e per j = 1, . . . , n. Questo vuol dire che la n-upla α(An1 , An2 , . . . , Ann )
é soluzione del sistema lineare omogeneo originario, al variare di α ∈ R.
Esempio 43. Si discuta il sistema


 2x1 + 3x2 − x3 + x4 = 0
x1 − x2 + x3 = 0

.

 3x 1 − 2x3 + x4 = 0
2x2 + 2x3 = 0

 
2 3 −1 1
 1 −1 1 0 
Svolg. La matrice associata é A =   3 0 −2 1  che ha rango 3. In particolare il

0 2 2 0
rango é dato dalle prime tre righe della matrice, quindi le ∞1 soluzioni del sistema
sono proporzionali ai complementi algebrici degli elementi a41 = 0, a42 = 2, a43 =
2, a44 = 0, cioé
 
3 −1 1 2 −1 1 2 3 1 2 3 −1

α  −1 1 0 , − 1 1 0 , 1 −1 0 , − 1 −1 1 
0 −2 1 3 −2 1 3 0 1 3 0 −2

al variare di α parametro reale, cioé α(4, 2, −2, −16), ovvero β (2, 1, −1, −8) al
64 3 Sistemi Lineari

variare di β ∈ R. t
u

3.3 Metodo di Cramer per la risoluzione di un sistema lineare.

Supponiamo di aver un sistema al quale venga associata un matrice incompleta A


quadrata di ordine n e non singolare:
   
  x1 b1
a11 a12 . . . a1n  x2   b2 
 a21 a22 . . . a2n     
A=  ... ... ... ... ,X =  ... ,B =  ... .
    
 ...   ... 
an1 an2 . . . ann
xn bn

Il sistema sia A · X = B.
Sappiamo dal teorema di Rouché-Capelli che in tal caso il sistema ammette un’unica
soluzione. Essa puó essere determinata nel modo seguente:

A · X = B → A−1 · A · X = A−1 · B → X = A−1 · B

da cui
1 ∆1
x1 = (A11 b1 + A21 b2 + . . . + An1 bn ) · =
det(A) det(A)
1 ∆2
x2 = (A12 b1 + A22 b2 + . . . + An2 bn ) · =
det(A) det(A)
1 ∆3
x3 = (A13 b1 + A23 b2 + . . . + An3 bn ) · =
det(A) det(A)
..................
1 ∆n
xn = (A1n b1 + A2n b2 + . . . + Ann bn ) · =
det(A) det(A)
dove Ai j é il complemento algebrico dell’elemento ai j ∈ A.
In sostanza, il termine al numeratore ∆1 = (A11 b1 + A21 b2 + . . . + An1 bn ) é il
determinante della matrice
 
b1 a12 . . . a1n
 b2 a22 . . . a2n 
 
 ... ... ... ... 
bn an2 . . . ann
ottenuta scambiando la prima colonna di A con la colonna dei termini noti.
In generale, il termine ∆i = (A1i b1 + A2i b2 + . . . + Ani bn ) é il determinante della
matrice
3.3 Metodo di Cramer per la risoluzione di un sistema lineare. 65
 
a11 a12 . . . b1 . . . a1n
 a12
 a22 . . . b2 . . . a2n 

 ... ... ... bj ... ... 
a1n an2 . . . bn . . . ann
ottenuta scambiando la colonna i di A con la colonna dei termini noti.

Supponiamo ora che la matrice A incompleta non sia quadrata e che il sistema sia
compatibile, diciamo r il rango del sistema. Consideriamo A0 la matrice di ordine r
che fornisce il rango al sistema, essa é ovviamente quadrata e non singolare.
Il sistema relativo alla matrice A0

 a11 x1 + a12 x2 + . . . + a1r xr = b1 − a1(r+1) xr+1 − . . . − a1n xn

a21 x1 + a22 x2 + . . . + a2r xr = b2 − a2(r+1) xr+1 − . . . − a2n xn


 ......
ar1 x1 + ar2 x2 + . . . + arr xr = bm − ar(r+1) xr+1 − . . . − arn xn

é equivalente a quello di partenza, inoltre é risolvibile col metodo di Cramer. Le


soluzioni (x1 , . . . , xr ) saranno date in funzione dei parametri reali xr+1 , . . . , xn , i quali
variano in R in modo arbitrario (in tal modo giustifichiamo anche la seconda parte
del Teorema di Rouché-Capelli).
Esempio 44. Ripetiamo un esempio precedentemente esposto, ma applichiamo ora
il metodo di Cramer. Si discuta il sistema

 x1 + x2 + 2x3 + x4 = 0
2x1 + 2x2 + 3x3 + 2x4 = 0 .
x1 + x2 + x3 + x4 = 0

 
1121
Svolg. La matrice associata é A =  2 2 3 2  che ha rango 2. Allora vi sono ∞2
1111
soluzioni. Il rango é 2 poiché l’ultima riga é combinazione lineare delle precedenti
due. Quindi un sistema equivalente
 al precedente é quello che ha come matrice
1121
associata la seguente: A0 =  2 2 3 2 . Tale sistema si scrive:
0000

x1 + 2x3 = −x2 − x4
.
2x1 + 3x3 = −2x2 − 2x4

Applichiamo
 ora il metodo di Cramer. La matrice incompleta del nuovo sistema
00 12
é A = , con det(A00 ) = −1. Le variabili x2 , x4 diventano parametri reali, ai
23
quali possiamo attribuire un qualsiasi valore in R. Calcoliamo

−x2 − x4 2
∆1 = = x2 + x4
−2x2 − 2x4 3
66 3 Sistemi Lineari

1 −x2 − x4
∆3 =
=0
2 −2x2 − 2x4
e quindi
∆1 ∆3
x1 = = −x2 − x4 x3 = =0
−1 −1
con x2 e x4 parametri reali liberi. t
u

3.4 Metodo di eliminazione di Gauss.

Concludiamo ora facendo vedere come un qualsiasi sistema compatibile si possa


risolvere utilizzando le operazioni elementari sulle righe della matrice completa,
associata ad esso, per riportarla in forma ridotta.
Siano A ∈ Mmn (R), B ∈ Mm1 (R), rispettivamente la matrice incompleta e la colonna
dei termini noti del sistema. Al solito indichiamo con C = [A|B] la matrice completa
del sistema.
Se operiamo sulle righe della matrice C, riportandola nella forma ridotta C0 , auto-
maticamente avremo ridotto anche la matrice A nella forma A0 . Il sistema associato
alle matrici A0 e C0 é equivalente a quello iniziale, ma viene espresso in una forma
ridotta, quindi piú facilmente risolvibile, con il metodo della sostituzione.
Esempio 45. Si discuta il sistema


 x1 + 2x2 − x3 + x4 = 8
x1 + 3x2 − 5x3 − x4 = 9

.

 2x1 + 4x2 − 4x3 + x4 = 16
4x1 + 10x2 − 9x3 + x4 = 33

Svolg. La matrice completa associata é


 
1 2 −1 1 8
 1 3 −5 −1 9
C=  2 4 −4 1
.
16 
4 10 −9 1 33

Cominciamo con le operazioni elementari sulle righe per rendere speciale l’elemen-
to di posto (1, 1):
 
1 2 −1 1 8
1  0 1 −3 − 3 1 
R2 → R2 − R3 C0 =   2 4 −4 1 16 
2 
2
4 10 −9 1 33
3.4 Metodo di eliminazione di Gauss. 67
 
1 2 −1 1 8
1  0 1 −3 − 3 1 
R3 → R3 − R4 C0 =  2
 0 −1 1 1 − 1 

2 2 2 2
4 10 −9 1 33
 
1 2 −1 1 8
 0 1 −3 − 3 1 
R4 → R4 − 4R1 C0 =  2
 0 −1 1 1 − 1  .

2 2 2
0 2 −5 −3 1
Passiamo ora a determinare un elemento speciale sulla seconda riga, e come in
precedenza, scegliamo l’elemento sulla diagonale principale (quello di posto (2, 2)):
 
1 2 −1 1 8
 0 1 −3 − 3 1 
R3 → R3 + R2 , R4 → R4 − 2R2 , C0 =  2
 0 0 − 5 −1 1  .

2 2
0 0 1 0 −1
Infine passiamo alla terza riga:
 
1 2 −1 1 8
2 0
 0 1 −3 − 32 1 
R4 → R4 + R3 C =
0 0 −5
.
5 2 −1 12 
00 0 − 52 − 45
La matrice é ora ridotta. Si noti che tanto il rango della incompleta che della com-
pleta é 4, quindi il sistema é compatibile ed ammette una sola soluzione. Inoltre il
sistema é ora riscrivibile come segue:

 x1 +2x2 −x3 +x
 4 =8
x2 −3x3 − 32 x4 = 1

.

 − 52 x3 −x4 = 12
− 52 x4 = − 45

Dall’ultima equazione si ha x4 = 2. Sostituendo nella terza otteniamo x3 = −1, e


continuando a sostituire nelle prime due, x2 = 1, x1 = 3. t
u
Esempio 46. Si discuta il sistema

 2x1 − x2 + 7x3 = 1
3x1 − 3x2 + 10x3 = 0 .
4x1 − 5x2 + 13x3 = 7

 
2 −1 7 1
Svolg. La matrice completa associata é C =  3 −3 10 0 . Cominciamo con la
4 −5 13 7
prima riga:
68 3 Sistemi Lineari
 
2 −1 7 1
3
R2 → R2 − R1 C0 =  0 − 32 − 12 − 23 
2
4 −5 13 7
 
2 −1 7 1
R3 → R3 − 2R1 C0 =  0 − 32 − 12 − 32 
0 −3 −1 5
 
2 −1 7 1
R3 → R3 − 2R2 C0 =  0 − 32 − 12 − 32  .
0 0 0 2
La matrice é ora ridotta. Si noti che il rango della incompleta é 2, mentre quello
della matrice completa é 3. Pertanto il sistema é incompatibile. t
u
Esempio 47. Si discuta il sistema

 x1 − 2x2 + 3x3 − 4x4 = 4
x1 − x2 + 2x3 − 3x4 = 1 .
−x1 − x2 + x3 + 3x4 = −1

 
1 −2 3 −4 4
Svolg. La matrice completa associata é C =  1 −1 2 −3 1 .
−1 −1 1 3 −1
 
1 −2 3 −4 4
R2 → R2 − R1 C0 =  0 1 −1 1 −3 
−1 −1 1 3 −1
 
1 −2 3 −4 4
R3 → R3 + R1 C0 =  0 1 −1 1 −3 
0 −3 4 −1 3
 
1 −2 3 −4 4
R3 → R3 + 3R2 C0 =  0 1 −1 1 −3  .
0 0 1 2 −6
La matrice é ora ridotta. Si noti che tanto il rango della incompleta che della com-
pleta é 3, quindi il sistema é compatibile ed ammette ∞1 soluzioni. Inoltre il sistema
é ora riscrivibile come segue:

 x1 −2x2 +3x3 −4x4 = 4
x2 −x3 +x4 = −3 .
x3 +2x4 = −6

Dall’ultima equazione si ha
x3 = −2x4 − 6.
Sostituendo nella seconda otteniamo
3.5 Esercizi svolti. 69

x2 = x3 − x4 − 3 = −2x4 − 6 − x4 − 3 = −3x4 − 9

ed infine dalla prima equazione:

x1 = 4 + 2x2 − 3x3 + 4x4 = 4 + 4x4 .

Quindi le soluzioni sono date dalle quaterne

(4 + 4α, −9 − 3α, −6 − 2α, α)

al variare del parametro α in R. t


u

3.5 Esercizi svolti.

Esercizio 17. Si determinino, se possibile, le soluzioni del seguente sistema lineare:



 x1 + x2 − x3 = 1
2x1 + 2x2 + x3 = 0
x1 + x2 + 2x3 = −1

Svolgimento: La matrice completa associata al sistema é


 
1 1 −1 1
C =  2 2 1 0 .
1 1 2 −1

Iniziamo a ridurla in forma a gradini, eseguendo le seguenti operazioni elementari:

R2 → R2 − 2R1 , R3 → R3 − R1

da cui  
1 1 −1 1
0
C =  0 0 3 −2  .
0 0 3 −2
Infine sottraiamo la seconda dalla terza riga:
 
1 1 −1 1
C00 =  0 0 3 −2  .
00 0 0

Abbiamo ottenuto in tal modo la matrice associata ad un sistema equivalente a quel-


lo iniziale. Il rango della matrice incompleta coincide con quello della completa, ed
é pari a 2. Vi sono quindi ∞1 soluzioni, il sistema é compatibile indeterminato. Tale
sistema, associato a C00 , é:
70 3 Sistemi Lineari

x1 + x2 − x3 = 1
3x3 = −2
per cui:
2 1
x3 = − , x1 = 1 − x2 + x3 = − x2 .
3 3
Le soluzioni del sistema sono
1 2
− α, α, − , ∀α ∈ R.
3 3

Esercizio 18. Si determinino, se possibile, le soluzioni del seguente sistema lineare:



 x1 + x2 + x3 = 3
x1 + x2 − x3 = 2
x1 − x2 + x3 = 2

Svolgimento: La matrice completa associata al sistema é


 
1 1 1 3
C =  1 1 −1 2  .
1 −1 1 2

Iniziamo a ridurla in forma a gradini, eseguendo le seguenti operazioni elementari:

R2 → R2 − R1 , R3 → R3 − R1

da cui  
1 1 1 3
C0 =  0 0 −2 −1  .
0 −2 0 −1
Infine scambiamo la seconda e la terza riga:
 
1 1 1 3
C00 =  0 −2 0 −1  .
0 0 −2 −1

Abbiamo ottenuto in tal modo la matrice associata ad un sistema equivalente a quel-


lo iniziale. Il rango della matrice incompleta coincide con quello della completa,
ed é pari a 3. Vi é quindi una sola soluzione, il sistema é compatibile determiato.
Dobbiamo risolvere il seguente:

 x1 + x2 + x3 = 3
2x2 = 1
2x3 = 1

per cui:
3.5 Esercizi svolti. 71

1 1
x3 = , x2 = , x1 = 3 − x2 − x3 = 2.
2 2
La soluzione del sistema é
1 1
2, , .
2 2

Esercizio 19. Si determinino le soluzioni del seguente sistema lineare:




 2x1 + 3x2 − x3 + x4 = 0
x1 − x2 + x3 = 0


 3x 1 − 2x3 + x4 = 0
x2 + x3 = 0

Svolgimento: La matrice associata al presente sistema omogeneo é


 
2 3 −1 1
 1 −1 1 0 
A=  3 0 −2 1  .

0 1 1 0

Inizialmente scambiamo la prima e la seconda riga:


 
1 −1 1 0
 2 3 −1 1 
A0 =  3 0 −2 1  .

0 1 1 0

Quindi eseguiamo le seguenti operazioni sulle righe:

R2 → R2 − 2R1 , R3 → R3 − 3R1

dalle quali segue che  


1 −1 1 0
 0 5 −3 1 
A00 = 
 0 3 −5 1  .

0 1 1 0
Per semplificare i calcoli, scambiamo la seconda e la quarta riga (in tal modo il
prossimo pivot da utilizzare per i calcoli sará 1):
 
1 −1 1 0
0 1 1 0
A000 = 
 0 3 −5 1  .

0 5 −3 1

Procediamo adesso con le seguenti:


72 3 Sistemi Lineari

R3 → R3 − 3R2 , R4 → R4 − 5R2 .

La matrice risultante é  
1 −1 1 0
iv
0 1 1 0
A = 0 0 −8
.
1
0 0 −8 1
Infine, per R4 → R4 − R3 , avremo
 
1 −1 1 0
0 1 1 0
Av = 

.
 0 0 −8 1
0 0 0 0

Il rango della matrice é quindi 3, per cui il sistema lineare omogeneo ammette ∞1
soluzioni. Riscrivendo il sistema equivalente avremo:

 x1 − x2 + x3 = 0
x2 + x3 = 0
−8x3 + x4 = 0

da cui x4 = 8x3 , x2 = −x3 e x1 = x2 − x3 = −2x3 . Quindi le soluzioni del sistema


sono le 4-uple (−2α, −α, α, 8α), al variare di α ∈ R.
Esercizio 20. Si determinino le soluzioni del seguente sistema lineare:

 x1 + x2 + 2x3 = 0

2x1 + 2x2 + 3x3 = 0


 3x1 + x2 + 2x3 = 0
x2 + x3 = 0

Svolgimento: La matrice associata al presente sistema omogeneo é


 
112
2 2 3
A=  3 1 2 .

011

Inizialmente eseguiamo le seguenti operazioni sulle righe:

R2 → R2 − 2R1 , R3 → R3 − 3R1

dalle quali segue che  


1 1 2
 0 0 −1 
A0 = 
 0 −2 −4  .

0 1 1
3.5 Esercizi svolti. 73

Scambiamo la seconda e la quarta riga:


 
1 1 2
0 1 1 
A00 = 
 0 −2 −4  .

0 0 −1

Procediamo quindi con la seguente R3 → R3 + 2R2 , dalla quale segue:


 
11 2
0 1 1 
A000 = 
 0 0 −2  .

0 0 −1

Il rango di A000 é banalmente pari a 3, per cui, per il sistema omogeneo associato
avremo la sola soluzione banale (0, 0, 0).
Esercizio 21. Si determinino le soluzioni del seguente sistema lineare:


 x1 + 2x2 − x3 + x4 = 0
2x1 + 4x2 − 2x3 − x4 = 0


 x1 − x2 + 2x4 = 0
2x1 + x2 − x3 = 0

Svolgimento: La matrice associata al presente sistema omogeneo é


 
1 2 −1 1
 2 4 −2 −1 
A=  1 −1 0 2  .

2 1 −1 0

Inizialmente eseguiamo le seguenti operazioni sulle righe:

R2 → R2 − 2R1 , R3 → R3 − R1 , R4 → R4 − 2R1

dalle quali segue che  


1 2 −1 1
 0 0 0 −3 
A0 = 
 0 −3 1 1  .

0 −3 1 −2
Scambiamo la seconda e la quarta riga:
 
1 2 −1 1
 0 −3 1 −2 
A00 =  0 −3 1 1 

0 0 0 −3
74 3 Sistemi Lineari

e sottraiamo la seconda dalla terza riga:


 
1 2 −1 1
 0 −3 1 −2 
A000 = 
 0 0 0 3 .

0 0 0 −3

Il rango della matrice é 3, per cui avremo ∞1 soluzioni. Osserviamo che un minore
non nullo di ordine 3 é quello individuato dall’incrocio delle righe R1 , R2 , R3 e delle
colonne C2 ,C3 ,C4 , ovvero
2 −1 1

−3 1 −2 6= 0.

0 0 3
Quindi possiamo scegliere come parametro libero, in funzione del quale otterremo
le soluzioni del sistema, la variabile x1 . Riscriviamo quindi il sistema associato alla
matrice A000 come segue: 
 2x2 − x3 + x4 = −x1
−3x2 + x3 − 2x4 = 0
3x4 = 0

da cui x4 = 0, x3 = 3x2 e x2 = x1 , per cui le soluzioni del sistema sono:



α, α, 3α, 0 , ∀α ∈ R.

Esercizio 22. Si determinino, se possibile, le soluzioni del seguente sistema lineare:



 −2x1 + x2 + x3 = 1
x1 − 2x2 + x3 = −2 .
x1 + x2 − 2x3 = 4

Svolgimento: La matrice completa associata al presente sistema é


 
−2 1 1 1
C =  1 −2 1 −2  .
1 1 −2 4

Inizialmente scambiamo la prima e la terza riga:


 
1 1 −2 4
C0 =  1 −2 1 −2  .
−2 1 1 1

Quindi eseguiamo le seguenti operazioni sulle righe:

R2 → R2 − R1 , R3 → R3 + 2R1
3.5 Esercizi svolti. 75

da cui otteniamo  
1 1 −2 4
C00 =  0 −3 3 −6  .
0 3 −3 9
Infine operiamo sulla seconda e terza riga nel modo seguente: R3 → R3 + R2 , per cui
 
1 1 −2 4
C000 =  0 −3 3 −6  .
0 0 0 3

Osserviamo che la matrice completa ha rango 3, mentre la matrice incompleta, for-


mata dalle prime tre colonne, ha rango 2. Il sistema é quindi incompatibile (privo di
soluzioni).
Esercizio 23. Si determinino, se possibile, le soluzioni del seguente sistema lineare:


 x1 + x2 − x3 + 2x4 = 1
x2 + 2x3 − x5 + x6 = 0

.

 x1 3x3 + 2x4 + x5 − x6 = 1

x1 + 2x2 + x3 + 2x4 − x5 + x6 = 1

Svolgimento: La matrice completa associata al presente sistema é


 
1 1 −1 2 0 0 1
 0 1 2 0 −1 1 0 
C=  1 0 −3 2 1 −1 1  .

1 2 1 2−1 1 1

Eseguiamo le seguenti operazioni sulle righe:

R3 → R3 − R1 , R4 → R4 − R1

dalle quali segue  


1 1 −1 2 0 0 1
0 1 2 0 −1 1 0
C0 = 

.
 0 −1 −2 0 1 −1 0
0 1 2 0−1 1 0
Su C0 applichaimo adesso le seguenti trasformazioni:

R3 → R3 + R2 , R4 → R4 − R2

ottenendo la seguente:
 
1 1 −1 2 0 01
01 2 0 −1 1 0
C00 = 

.
0 0 0 0 0 0 0
00 0 00 0 0
76 3 Sistemi Lineari

Tale matrice ha rango 2, pari anche al rango della matrice incompleta. Quindi il
sistema é compatibile indeterminato e presenta ∞4 soluzioni. Il sistema equivalente
a quello iniziale, associato alla matrice C00 é

x1 + x2 − x3 + 2x4 = 1
.
x2 + 2x3 − x5 + x6 = 0

Osservando che il rango della matrice C00 é garantito, ad esempio, dal minore di or-
dine 2 ottenuto dalle prime 2 righe e prime 2 colonne, possiamo riscrivere il sistema
come segue: 
x1 + x2 = x3 − 2x4 + 1
.
x2 = −2x3 + x5 − x6
Da ció otteniamo

x1 = −x2 + x3 − 2x4 + 1 = 3x3 − 2x4 − x5 + x6 + 1.

Le soluzioni del sistema sono quindi:



3α − 2β − γ + δ + 1, −2α + γ − δ , α, β , γ, δ ∀α, β , γ, δ ∈ R.

Esercizio 24. Si determinino, quando possibile, le soluzioni del seguente sistema


lineare, al variare del parametro reale k:

 2x1 + kx2 = 2 − k
kx1 + 2x2 = k .
kx2 + kx3 = k − 1

Svolgimento: La matrice completa associata al presente sistema é


 
2 k 0 2−k
C =  k 2 0 k .
0 k k k−1

Consideriamo inizialmente la matrice incompleta associata al sistema:


 
2k0
A =  k 2 0 .
0kk

Osserviamo che essa é quadrata. In tal caso é conveniente, al fine di discutere il


sistema lineare, calcolare il determinante di A:

|A| = k(4 − k2 ).

Quindi, per ogni k 6= 0, 2, −2, le matrici completa ed incompleta hanno entrambe


rango pari a 3 ed il sistema lineare risulta compatibile determinato. Applichiamo il
3.5 Esercizi svolti. 77

metodo di Cramer per determinarne le soluzioni:



2−k k 0

k 2 0

k − 1 k k 4 − 2k − k2
x1 = =
k(4 − k2 ) 4 − k2

2 2−k 0

k k 0

0 k−1 k k2
x2 = 2
=
k(4 − k ) 4 − k2

2 k 2−k

k 2 k

0 k k − 1 −2k3 + k2 + 4k − 4
x3 = = .
k(4 − k2 ) k(4 − k2 )
Passiamo adesso a discutere i casi k = 0, 2, −2.
Per k = 0 la matrice associata al sistema é
 
200 2
C = 0 2 0 0 
0 0 0 −1

ed ha rango pari a 3, mentre la matrice incompleta ha evidentemente rango 2: il


sistema é incompatibile.
Per k = 2 la matrice associata al sistema é
 
2200
C = 2 2 0 2
0221

ed ha ancora rango pari a 3. Anche in questo caso la matrice incompleta ha rango 2:


il sistema é incompatibile.
Infine, per k = −2 la matrice associata al sistema é
 
2 −2 0 4
C =  −2 2 0 −2 
0 −2 −2 −3

ed ancora una volta ha rango pari a 3. Poiché la matrice incompleta ha rango 2, il


sistema risulta anche in questo caso incompatibile.
Esercizio 25. Si determinino, quando possibile, le soluzioni del seguente sistema
lineare, al variare del parametro reale k:
78 3 Sistemi Lineari

 (3 − k)x1 − 2x2 + (k − 2)x3 = 4
2x1 − 6x2 − 3x3 = 0 .
kx1 + 4x2 + 2x3 = 7

Svolgimento: La matrice completa associata al presente sistema é


 
3 − k −2 k − 2 4
C =  2 −6 −3 0  .
k 4 2 7

Consideriamo la matrice incompleta associata al sistema:


 
3 − k −2 k − 2
A =  2 −6 −3  .
k 4 2

Poiché il suo determinante é |A| = (6k + 8)(k − 1), avremo che il rango di A é pari
a 3, per ogni k 6= 1, − 34 . In questo caso, poiché anche il rango di C é 3, il sistema é
compatibile determinato, ovvero otterremo un’unica soluzione in corrispondenza di
ogni fissato valore di k 6= 1, − 34 . Applicando il metodo di Cramer, abbiamo:

4 −2 k − 2

0 −6 −3

7 4 2 42(k − 1) 21
x1 = = =
(6k + 8)(k − 1) (6k + 8)(k − 1) 3k + 4

3−k 4 k−2

2 0 −3

k 7 2 19(1 − k) 19
x2 = = =−
(6k + 8)(k − 1) (6k + 8)(k − 1) 6k + 8

3 − k −2 4

2 −6 0

k 4 7 66(k − 1) 33
x3 = = = .
(6k + 8)(k − 1) (6k + 8)(k − 1) 3k + 4
Analizziamo ora o casi restanti, k = 1 e k = − 43 .
Per k = 1, la matrice associata al sistema é
 
2 −2 −1 4
C =  2 −6 −3 0  .
1 4 2 7

Riduciamola per righe ed iniziamo scambiando la prima e la terza riga:


3.5 Esercizi svolti. 79
 
1 4 2 7
 2 −6 −3 0  .
2 −2 −1 4

Quindi eseguiamo le seguenti operazioni elementari:

R2 → R2 − 2R1 , R3 → R3 − 2R1

in seguito alle quali la matrice si trasforma in


 
1 4 2 7
 0 −14 −7 −14  .
0 −10 −5 −10

Su quest’ultima eseguiamo le seguenti ulteriori operazioni:


1 1
R2 → − R2 , R3 → − R3
7 5
dalle quali segue che la matrice associata al sistema equivalente é
 
1427
 0 2 1 2 .
0222

Osserviamo allora che il rango della matrice completa é pari a 2, come quello della
matrice incompleta. Quindi il sistema é compatibile indeterminato e presenta ∞1
soluzioni. Un minore non nullo di ordine 2 é dato dall’incrocio delle prime due
righe e prime due colonne, ovvero
 
14
.
02

Riscriviamo allora il sistema come segue:



x1 + 4x2 = 7 − 2x3
.
2x2 = 2 − x3

Da ció segue che


2 − x3 2 − x3
x2 = , x1 = 7 − 2x3 − 4 = 7 − 2x3 − 4 + 2x3 = 3.
2 2
Le soluzioni quindi sono date da:
α 
3, 1 − ,α ∀α ∈ R.
2
Passiamo adesso al caso k = − 43 e riscriviamo quindi la matrice associata al sistema
80 3 Sistemi Lineari

come segue:
13
−2 − 10
 
3 3 4
C =  2 −6 −3 0  .
− 43 4 2 7
Iniziamo moltiplicando entrambe la prima e la terza riga per 3:
 
13 −6 −10 12
 2 −6 −3 0  .
−4 12 6 21

Quindi eseguiamo la seguente operazione: R3 → R3 + 2R2 , dalla quale segue che la


matrice associata al sistema equivalente é
 
13 −6 −10 12
 2 −6 −3 0  .
0 0 0 21

Poiché evidentemente il rango della parte incompleta é 2, mentre quello della


matrice completa é 3, il sistema risulta incompatibile.
Esercizio 26. Si determinino, quando possibile, le soluzioni del seguente sistema
lineare, al variare del parametro reale k:

 x1 + 2x2 − x3 + kx4 = 1
2x1 + x2 + x4 = 1 .
3x1 + 3x2 − x3 + (3 − k)x4 = −1

Svolgimento: La matrice completa associata al presente sistema é


 
1 2 −1 k 1
C =  2 1 0 1 1 .
3 3 −1 3 − k 1

Eseguiamo le seguenti operazioni elementari su C:

R2 → R2 − 2R1 , R3 → R3 − 3R1

dalle quali segue che  


1 2 −1 k 1
C0 =  0 −3 2 1 − 2k −1  .
0 −3 2 3 − 4k −4
Ancora, su C0 eseguiamo R3 → R3 − R2 ed otteniamo
 
1 2 −1 k 1
C00 =  0 −3 2 1 − 2k −1  .
0 0 0 2 − 2k −3
3.5 Esercizi svolti. 81

Osserviamo che, per k = 1 la matrice C00 si presenta nella forma


 
1 2 −1 1 1
 0 −3 2 −1 −1 
0 0 0 0 −3

ed ha rango 3. Al contrario la matrice incompleta


 
1 2 −1 1
 0 −3 2 −1 
0 0 0 0

ha rango 2, per cui il sistema é incompatibile.


Nel caso di k 6= 1, i ranghi della incompleta
 
1 2 −1 k
A00 =  0 −3 2 1 − 2k 
0 0 0 2 − 2k

e della completa C00 coincidono e sono pari a 3. Quindi il sistema é compatibile


indeterminato e presenta ∞1 soluzioni. Individuiamo un minore non nullo di ordine
3 della matrice incompleta A00 , in particolare scegliamo l’incrocio delle tre righe e
delle colonne 1, 2, 4: 
1 2 k

0 −3 1 − 2k 

0 0 2 − 2k

e riscriviamo il sistema associato a C00 , scegliendo quindi x3 come parametro libero:



 x1 + 2x2 + kx4 = 1 + x3
−3x2 + (1 − 2k)x4 = −1 − 2x3
(2 − 2k)x4 = −3

da cui
3
x4 =
2k − 2
1 3 (4k − 4)x3 + 1 − 4k
x2 = − −1 − 2x3 + (2k − 1) =
3 2k − 2 6(k − 1)
(4k − 4)x3 + 1 − 4k 3 (5k − 8) + 2(1 − k)x3
x1 = 1 + x3 − 2 −k = .
6(k − 1) 2k − 2 6(k − 1)
Quindi, per ogni k 6= 1, le soluzioni sono:
 
(5k − 8) + 2(1 − k)α 4(k − 1)α + 1 − 4k 3
, , α, ∀α ∈ R.
6(k − 1) 6(k − 1) 2k − 2
82 3 Sistemi Lineari

Esercizio 27. Risolvere il seguente sistema lineare al variare dei parametri reali h, k

 x1 + x2 + hx3 = 1
x1 + 2x2 + kx3 = 3 .
x1 + x3 = 1

Svolgimento: La matrice completa associata al presente sistema é


 
11h1
C =  1 2 k 3 .
1011

La matrice incompleta ha determinante



1 1 h

1 2 k = k − 2h + 1

1 0 1

il quale si annulla per k = 2h − 1. Quindi, per ogni h, k ∈ R tali che k = 2h − 1, il


rango della matrice incompleta é 2. In tale caso, riscriviamo la matrice completa
come segue:  
11 h 1
 1 2 2h − 1 3  .
10 1 1
Eseguendo le seguenti operazioni

R2 → R2 − R1 , R3 → R3 − R1

avremo la matrice  
1 1 h 1
C0 =  0 1 h − 1 2  .
0 −1 1 − h 0
Inoltre, per R3 → R3 + R2 in C0 , otteniamo la matrice
 
11 h 1
C00 =  0 1 h − 1 2  .
00 0 2

Quindi, per k = 2h − 1, il rango di C00 é pari a 3, per qualsiasi valore di h ∈ R. Il


sistema é in questo caso incompatibile.
Per ogni h, k ∈ R tali che k 6= 2h − 1, il determinante della incompleta é diverso da
zero, quindi il suo rango é 3, come quello della completa. In tal caso il sistema é
compatibile determinato e le soluzioni si trovano applicando il metodo di Cramer:
3.5 Esercizi svolti. 83

1 1 h

3 2 k

1 0 1 k − 2h − 1
x1 = =
k − 2h + 1 k − 2h + 1

1 1 h

1 3 k

1 1 1 2 − 2h
x2 = =
k − 2h + 1 k − 2h + 1

1 1 1

1 2 3

1 0 1 2
x3 = = .
k − 2h + 1 k − 2h + 1
Quindi, per ogni scelta di h, k ∈ R tali che k 6= 2h − 1, la soluzione é
 
k − 2h − 1 2 − 2h 2
, , .
k − 2h + 1 k − 2h + 1 k − 2h + 1

Esercizio 28. Si determinino, quando possibile, le soluzioni del seguente sistema


lineare, al variare del parametro reale k:

 x1 + x2 + 2x4 = 2
−x − x2 + x3 + x4 = k2 .
 1
3x1 + 2x3 + 3x4 = k − 2

Svolgimento: La matrice completa associata al sistema é


 
1 1 02 2
C =  −1 −1 1 1 k2  .
3 0 2 3 k−2

Eseguiamo su C le seguenti operazioni elementari:

R2 → R2 + R1 , R3 → R3 − 3R1 .

La matrice si trasforma nella seguente:


 
1 1 0 2 2
C 0 =  0 0 1 3 2 + k2  .
0 −3 2 −3 k − 8

É sufficiente adesso scambiare la seconda e la terza riga di C0 , per ottenere una


matrice in forma ridotta a gradini:
84 3 Sistemi Lineari
 
1 1 0 2 2
C00 =  0 −3 2 −3 k − 8  .
0 0 1 3 2 + k2

Quindi osserviamo che il rango della matrice incompleta


 
1 1 0 2
 0 −3 2 −3 
0 0 1 3

é pari a 3, per cui anche il rango di C00 é 3. Il sistema é compatibile indeterminato e


possiede ∞1 soluzioni, qualsiasi sia il valore di k.
Inoltre é facile individuare un minore non nullo di ordine 3, ad esempio quello
individuato dall’incrocio delle tre righe e delle prime tre colonne:

1 1 0

0 −3 2 6= 0.

0 0 1

Scegliamo quindi x4 come parametro libero e determiniamo le soluzioni in funzione


di esso. Riscriviamo allora il sistema equivalente a quello iniziale, associato alla
matrice C00 : 
 x1 + x2 = 2 − 2x4
−3x2 + 2x3 = (k − 8) + 3x4 .
x3 = (2 + k2 ) − 3x4

Partendo dall’ultima equazione e sostituendo nelle precedenti otteniamo:

2k2 − k + 12 − 9x4
 
1
x2 = − k − 8 + 3x4 − 2(2 + k2 ) + 6x4 =
3 3

2k2 − k + 12 − 9x4 2k2 − k


x1 = 2 − 2x4 − = −2 − + x4 .
3 3
Concludiamo quindi che, per qualsiasi valore di k, le soluzioni sono

2k2 − k 2k2 − k + 12 − 9α
 
2
−2 − + α, , (2 + k ) − 3α, α ∀α ∈ R.
3 3

Esercizio 29. Si determinino, quando possibile, le soluzioni del seguente sistema


lineare, al variare del parametro reale k:

 x1 + 2x2 − kx3 = k
x1 + x2 − kx3 = 1
3x1 + (2 + k)x2 − (2 + k)x3 = 0

Svolgimento: La matrice incompleta associata al sistema é quadrata:


3.5 Esercizi svolti. 85
 
1 2 −k
A = 1 1 −k  .
3 2 + k −2 − k

In tal caso puó essere conveniente calcolarne subito il determinante:

|A| = 2 − 2k.

In questo modo possiamo giá osservare che per ogni k 6= 1 il rango di A é 3, e tale
dovrá essere anche il rango della matrice completa. Quindi il sistema é compatibile
determinato. Per ottenere l’unica soluzione applichiamo il metodo di Cramer:

k 2
−k
1 1
−k
0 2 + k −2 − k k3 − 2k + 4
x1 = =
2 − 2k 2 − 2k

1 k −k

1 1 −k

3 0 −2 − k k2 − 2k + 1
x2 = =
2 − 2k k−1

1 2 k

1 1 1

3 2 + k 0 k2 − 2k + 4
x3 = = .
2 − 2k 2 − 2k
Quindi, per qualsiasi scelta di k 6= 1, la soluzione é
 3
k − 2k + 4 k2 − 2k + 1 k2 − 2k + 4

, , .
2 − 2k 2 − 2k 2 − 2k

Occupiamoci infine del caso k = 1. Per quanto visto, il rango di A é pari a 2. La


matrice completa é:  
1 2 −1 1
C =  1 1 −1 1  .
3 3 −3 0
Eseguiamo su di essa le seguenti operazioni elementari:

R2 → R2 − R1 , R3 → R3 − 3R1

dalle quali avremo  


1 2 −1 1
C0 =  0 −1 0 0  .
0 −3 0 −3
Infine, per R3 → R3 − 3R2 , otteniamo la matrice
86 3 Sistemi Lineari
 
1 2 −1 1
C00 =  0 −1 0 0 
0 0 0 −3

che é ridotta in forma a gradini e presenta 3 righe non nulle. Il suo rango é quindi 3
ed il sistema risulta incompatibile.
Esercizio 30. Si determinino, quando possibile, le soluzioni del seguente sistema
lineare, al variare del parametro reale k:

 3x1 − x2 − kx3 = k

x1 + x2 − kx3 = 1


 −x 1 + 2x2 − x3 = 0
x1 − x2 = 1

Svolgimento: In questo caso, la matrice completa associata al sistema é quadrata:


 
3 −1 −k k
 1 1 −k 1 
C=  −1 2 −1 0  .

1 −1 0 1

Puó quindi essere coveniente dapprima calcolarne il determinante:

|C| = (3 − k)(k − 2).

In tal modo possiamo giá osservare che, per qualsiasi valore k 6= 2, 3 il rango di
C sará 4. Poiché il rango della matrice incompleta dovrá in ogni caso essere ≤ 3,
concludiamo che il sistema é incompatibile.
Restano quindi da analizzare i singoli casi di k = 2 e k = 3.
Per k = 2 la matrice completa C si esprime nel modo seguente:
 
3 −1 −2 2
 1 1 −2 1 
C=  −1 2 −1 0  .

1 −1 0 1

Scambiamo la prima e la quarta riga:


 
1 −1 0 1
 1 1 −2 1 
 
 −1 2 −1 0 
3 −1 −2 2

ed eseguiamo sulle righe le seguenti operazioni elementari:

R2 → R2 − R1 , R3 → R3 + R1 , R4 → R4 − 3R1 .
3.5 Esercizi svolti. 87

La matrice risultante é  
1 −1 0 1
 0 2 −2 0 
 0 1 −1 1  .
 

0 2 −2 −1
Scambiano, in quest’ultima, la seconda e la terza riga:
 
1 −1 0 1
 0 1 −1 1 
 
 0 2 −2 0 
0 2 −2 −1

e adesso eseguiamo le seguenti operazioni:

R3 → R3 − 2R2 , R4 → R4 − 2R2

tramite le quali otteniamo la matrice


 
1 −1 0 1
 0 1 −1 1 
 0 0 0 −2  .
 

0 0 0 −3

Appare evidente che tale matrice ha rango 3, mentre la matrice incompleta


 
1 −1 0
 0 1 −1 
 
0 0 0 
0 0 0

ha rango 2. Quindi, per k = 2 il sistema é incompatibile.


Terminiamo con il caso k = 3, per il quale la matrice C assume la seguente forma:
 
3 −1 −3 3
 1 1 −3 1 
C=  −1 2 −1 0  .

1 −1 0 1

Iniziamo scambiando la prima a la quarta riga:


 
1 −1 0 1
 1 1 −3 1 
 
 −1 2 −1 0 
3 −1 −3 3

e quindi eseguiamo le seguenti operazioni elementari sulle righe:


88 3 Sistemi Lineari

R2 → R2 − R1 , R3 → R3 + R1 , R4 → R4 − 3R1

in seguito alle quali otteniamo la matrice


 
1 −1 0 1
 0 2 −3 0
 .
 0 1 −1 1
0 2 −3 0

Scambiamo adesso la seconda e la terza riga


 
1 −1 0 1
 0 1 −1 1
 
 0 2 −3 0
0 2 −3 0

ed operiamo R4 → R4 − R3 , in seguito alla quale operazione otteniamo


 
1 −1 0 1
 0 1 −1 1 
 0 2 −3 0  .
 

0 0 0 0

Infine eseguiamo l’operazione R3 → R3 − 2R1 , in seguito alla quale abbiamo la


matrice  
1 −1 0 1
 0 1 −1 1 
 0 0 −1 −2  .
 

0 0 0 0
Possiamo allora concludere che sia la matrice completa che quella incompleta han-
no rango pari a 3, per cui il sistema é compatibile determinato.
Abbiamo quindi dimostrato che il sistema ha soluzione solo per k = 3. Inoltre ri-
scriviamo il sistema equivalente a quello iniziale ed associato alla matrice in forma
ridotta appena ottenuta: 
 x1 − x2 = 1
x2 − x3 = 1
x3 = 2

la cui soluzione é  
4, 3, 2 .
Capitolo 4
Gli Spazi Vettoriali.

4.1 Introduzione

Siano K un campo e V un insieme non vuoto in cui sono definite le seguenti


operazioni:
+ : V ×V → V
· : K ×V → V.
Diremo che (V, +, ·) é uno spazio vettoriale sul campo K se valgono le seguenti:
1. (V, +) é un gruppo commutativo (vedi Definizione 1.1);
2. (α + β )v = αv + β v, per ogni α, β ∈ K, v ∈ V ;
3. α(v1 + v2 ) = αv1 + αv2 , per ogni α ∈ K, v1 , v2 ∈ V ;
4. α(β v) = (αβ )v = (β α)v = β (αv), per ogni α, β ∈ K, v ∈ V .
Chiameremo vettori gli elementi di uno spazio vettoriale V e scalari gli elementi del
campo K su cui V é costruito.
Esempio 48. L’insieme delle matrici Mmn (R) é uno spazio vettoriale su R, rispetto
alle operazioni di somma tra matrici e prodotto per uno scalare.
Esempio 49. R é uno spazio vettoriale su se stesso.
Esempio 50. L’insieme dei polinomi di grado minore o uguale ad un fissato n,

R[X] = {a0 + a1 x + a2 x2 + a3 x3 + . . . . + an xn a0 , . . . ., an ∈ R}

a coefficienti reali, é uno spazio vettoriale su R, rispetto alle operazioni di somma


tra polinomi e di prodotto di un polinomio per uno scalare.
Esempio 51. Sia
Rn = {(x1 , . . . , xn ) |x1 , . . . , xn ∈ R}.
Definiamo le seguenti operazioni

(x1 , . . . , xn ) + (y1 , . . . , yn ) = (x1 + y1 , . . . , xn + yn )

89
90 4 Gli Spazi Vettoriali.

α(x1 , . . . , xn ) = (αx1 , . . . , αxn ).


Allora Rn é uno spazio vettoriale sul campo dei reali. Ogni vettore é una n-upla del
tipo (x1 , . . . , xn ).
Osservazione 4.1. Nel seguito ci occuperemo esclusivamente di spazi vettoriali
definiti sul campo K = R dei numeri reali.
Definizione 4.2. Sia W ⊆ V , sottoinsieme dello spazio vettoriale V . Diremo che W
é un sottospazio vettoriale di V , se é uno spazio vettoriale rispetto alle operazioni
definite in V , sul medesimo campo dei reali.
Osservazione 4.3. Da tale definizione deriva che, condizione necessaria e sufficien-
te affinché W sia sottospazio di V é che valgano le due seguenti:

w1 + w2 ∈ W

αw ∈ W
per ogni α ∈ R, w, w1 , w2 ∈ W , e queste si possono compattare nell’unica condizione

αw1 + β w2 ∈ W

per ogni α, β ∈ R, w1 , w2 ∈ W .
 
xy
Esempio 52. W = { x, y ∈ R} é un sottospazio dello spazio vettoriale delle
00
matrici quadrate di ordine 2 su R, M2 (R).
Esempio 53. W = {(x, 0, z) x, z ∈ R} é un sottospazio vettoriale di R3 .
Esempio 54. R é un sottospazio banale di se stesso.
Esempio 55. W = {a0 + a1 x1 + a2 x2 + . . . . + am xm , a0 , . . . , am ∈ R}, insieme dei
polinomi di grado minore o uguale a m, con m ≤ n, é sottospazio vettoriale di V =
{a0 + a1 x1 + a2 x2 + . . . . + an xn , a0 , . . . , an ∈ R}.

4.2 Intersezione, unione e somma di sottospazi.

Siano U,W sottospazi dello spazio vettoriale V . Consideriamo l’intersezione di U e


W:
U ∩W = {v ∈ V : v ∈ U e v ∈ W }.
Esso é ancora un sottospazio di V . Infatti, se v1 , v2 ∈ U ∩W , allora avremo v1 ∈ U e
v2 ∈ U, da cui v1 +v2 ∈ U (poiché U é un sottospazio vettoriale di V ). Analogamente
avremo v1 ∈ W e v2 ∈ W , quindi v1 + v2 ∈ W . Per cui v1 + v2 ∈ U ∩ W . In modo
del tutto simile, se v ∈ U ∩ W ed α ∈ K, allora αv ∈ U ed anche αv ∈ W , ovvero
αv ∈ U ∩W .
Vengono quindi soddisfatte in U ∩W le proprietá esposte nell’Osservazione 4.3, la
qual cosa ci fa concludere che U ∩W é un sottospazio vettoriale di V .
4.2 Intersezione, unione e somma di sottospazi. 91

Esempio 56. Siano U = {(x, y, 0), x, y ∈ R} e W = {(x, 0, z), x, z ∈ R} sotto-


spazi di R3 . Allora un generico vettore che appartenga ad entrambi é espresso dalla
terna di componenti (x, 0, 0), al variare di x ∈ R. Scriveremo quindi

U ∩W = {v ∈ V : v = (x, 0, 0), x ∈ R}.

Esempio 57. Siano U = {(x, y, 0), x, y ∈ R} e W = {(x, x, x), x ∈ R} sottospazi


di R3 . Allora l’unico vettore che appartenga ad entrambi é dato dalle componenti
(0, 0, 0), quindi scriveremo

U ∩W = {(0, 0, 0)}.

Definiamo adesso l’unione di dei due sottospazi U e W :

U ∪W = {v ∈ V : v∈U oppure v ∈ W }.

Al contrario dell’intersezione, tale insieme non é un sottospazio di V (eccetto che in


casi particolari che vedremo in seguito). Per dimostrare tale affermazione, portiamo
un controesempio. Consideriamo

U = {(x, 0), x ∈ R}

W = {(0, y), y ∈ R}
sottospazi di R2 . Scegliamo il vettore (1, 0) ∈ U ed il vettore (0, 1) ∈ W : ovviamente
entrambi appartengono a U ∪W ma

(1, 0) + (0, 1) = (1, 1) ∈


/ U ∪W.

Piú in generale, U ∪ W é sottospazio vettoriale di V solo nei due casi banali in cui
u ⊆ W oppure W ⊆ U.
Definiamo ora il seguente sottoinsieme dello spazio vettoriale V :

U +W = {v ∈ V : v = u + w, u∈U e w ∈ W }.

Esso é un sottospazio di V , detto somma di U e W , piú precisamente é il piú piccolo


sottospazio di V contenente U ∪W .
Il fatto che la somma sia un sottospazio vettoriale discende direttamente dalla sua
definizione, infatti:
– se v1 , v2 ∈ U +W allora esistono u1 , u2 ∈ U e w1 , w2 ∈ W tali che v1 = u1 + w1 e
v2 = u2 + w2 . Quindi

v1 + v2 = u1 + w1 + u2 + w2 = (u1 + u2 ) + (w1 + w2 ) ∈ U +W.

– Dato v ∈ U + W esistono u ∈ U e w ∈ W tali che v = u + w. Quindi, per ogni


α ∈ R,
αv = αu + αw ∈ U +W.
92 4 Gli Spazi Vettoriali.

Ancora una volta abbiamo verificato la validitá delle proprietá esposte nella Osser-
vazione 4.3, per cui U +W é un sottospazio di V .
Diremo che U + W é somma diretta (e la indicheremo U ⊕ W ) se U ∩ W = {0},
ovvero se U e W hanno in comune il solo vettore nullo di V .
Esempio 58. Siano U = {(x, y, 0), x, y ∈ R} e W = {(x, 0, z), x, z ∈ R} sotto-
spazi di R3 . Allora

U +W = {(x, y, z), x, y, z ∈ R} = R3

inoltre U ∩W = {(x, 0, 0)}, quindi la somma non é diretta.


Esempio 59. Siano U = {(x, y, 0), x, y ∈ R} e W = {(z, z, z), z ∈ R} sottospazi
di R3 . Allora
U +W = {(x + z, y + z, z), x, y, z ∈ R} = R3
inoltre U ∩W = {(0, 0, 0)}, quindi la somma é diretta.
Esempio 60. Siano U = {(x, 0, z), x, z ∈ R} e W = {(y, 0, y), y ∈ R} sottospazi
di R3 . Allora
U +W = {(x + y, 0, y + z), x, y, z ∈ R}
inoltre U ∩W = {(x, 0, x)}, quindi la somma non é diretta.
Consideriamo ancora un esempio di somma non diretta:
Esempio 61. Siano U = {(x + y, y, 0), x, y ∈ R} e W = {(x + y, 0, y), x, y ∈ R}
sottospazi di R3 . Allora

U ∩W = {(α, 0, 0), α ∈ R}

quindi la somma non é diretta. Scegliamo un vettore che appartenga alla somma: in
particolare sia w = (6, 1, 1) ∈ U + W . Si noti che il vettore w si puó esprimere in
diversi modi come somma di due vettori scelti rispettivamente in U e W :

(6, 1, 1) = (3, 1, 0) + (3, 0, 1) con (3, 1, 0) ∈ U, (3, 0, 1) ∈ W

ed anche

(6, 1, 1) = (2, 1, 0) + (4, 0, 1) con (2, 1, 0) ∈ U, (4, 0, 1) ∈ W.

Questo accade per ogni vettore appartenente allo spazio somma, nel caso in cui
questa non sia diretta.
Alla luce di ció, vale la seguente:
Proposizione 4.4. Siano U e W sottospazi vettoriali dello spazio V . La loro somma
é diretta se e solo se ogni vettore di essa si puó esprimere in modo unico come
somma di un vettore di U e di uno di W .

Dim. Rimandiamo la dimostrazione al paragrafo successivo. t


u
4.2 Intersezione, unione e somma di sottospazi. 93

Definizione 4.5. Una combinazione lineare di vettori {v1 , . . . , vn } di V tramite gli


scalari {α1 , . . . , αn } di R é un vettore v ∈ V ottenuto dalla somma dei prodotti αi vi ,
ovvero
v = α1 v1 + α2 v2 + . . . + αn vn ∈ V.
Gli scalari {α1 , . . . , αn } di R sono dette componenti o coordinate di v rispetto
all’insieme ordinato di vettori {v1 , . . . , vn }.
Definizione 4.6. Sia S ⊆ V , un sottoinsieme dello spazio V . Definiamo Span(S) =<
S >, e lo chiamiamo sottospazio generato da S, il sottospazio di V composto da tutte
le possibili combinazioni lineari di vettori di S e scalari in R.
Esempio 62. Sia V = R3 , S = {(1, 0, 0), (0, 1, 0)}. Allora

Span(S) =< S >= {x(1, 0, 0) + y(0, 1, 0), x, y ∈ R} = {(x, y, 0), x, y ∈ R}.

Definizione 4.7. Diremo che i vettori v1 , . . . , vn ∈ V sono linearmente dipendenti se


esistono α1 , . . . , αn ∈ R, non tutti nulli tali che

α1 v1 + α2 v2 + . . . + αn vn = 0.

Sono detti linearmente indipendenti se

α1 v1 + α2 v2 + . . . + αn vn = 0 ⇐⇒ α1 = α2 = . . . , αn = 0.

Definizione 4.8. Un insieme S = {v1 , . . . , vn } di vettori di V , é detto insieme li-


nearmente indipendente (rispettivamente dipendente) se v1 , . . . , vn ∈ V sono vettori
linearmente indipendenti (risp. dipendenti).
Esempio 63. v1 = (−1, 2, 3), v2 = (0, −1, 0), v3 = (1, 0, 1) vettori di R3 sono
linearmente indipendenti.
Esempio 64. v1 = (1, 2, 1, 0), v2 = (1, −1, 0, 1), v3 = (−1, 2, −1, 0), v4 = (−1, 1, 0, −1),
v5 = (1, 1, 0, 1) vettori di R4 sono linearmente dipendenti.
Esempio 65. v1 = (1, 2, 0), v2 = (0, 1, a), v3 = (1, a, −1) vettori di R3 , con a
parametro reale, sono indipendenti per a 6= 1 e sono dipendenti per a = 1.
Osservazione 4.9. Se in un insieme S di vettori di V é contenuto il vettore nul-
lo, allora tali vettori sono tra loro linearmente dipendenti. Infatti, ponendo S =
{0, v1 , . . . , vk }, ne deriva che

α0 · 0 + 0 · v1 + 0 · v2 + · · · + 0 · vn = 0, ∀0 6= α0 ∈ R.

Osservazione 4.10. Sia S un insieme linearmente indipendente di vettori di V .


Allora ogni sottoinsieme di S é linearmente indipendente.
Osservazione 4.11. Sia S un insieme linearmente dipendente di vettori di V . Allora,
dato un qualsiasi sottoinsieme di S, la sua eventuale dipendenza od indipendenza
lineare deve essere verificata, caso per caso.
94 4 Gli Spazi Vettoriali.

Osservazione 4.12. Sia S un insieme linearmente indipendente di vettori di V e


sia T un insieme di vettori di V , tale che S ⊂ T . Allora l’eventuale dipendenza od
indipendenza lineare di T deve essere verificata.
Osservazione 4.13. Sia S un insieme linearmente dipendente di vettori di V e sia T
un insieme di vettori di V , tale che S ⊂ T . Allora T é linearmente dipendente.

4.3 Basi e dimensione di uno spazio vettoriale.

Sia V uno spazio vettoriale su R. Un insieme B di vettori é detta base di V se:


1. i vettori di B sono linearmente indipendenti;
2. Span(B) = V .
Sia quindi B = {e1 , . . . , en } una base per V . Ogni vettore di V si puó esprimere come
combinazione lineare dei vettori della base, tramite l’utilizzo di opportuni scalari:

∀v ∈ V ∃α1 , . . . , αn ∈ R : v = α1 e1 + . . . + αn en .

Gli scalari (α1 , . . . , αn ) sono dette componenti o coordinate di v rispetto alla base B
(od in termini della base B, od in riferimento alla base B).
Osservazione 4.14. Sia B = {e1 , . . . , en } una base per V . Le componenti di un
qualsiasi vettore v ∈ V in termini di B, sono uniche. Infatti, se supponessimo
v = ∑ni=1 αi ei e contestualmente v = ∑ni=1 βi ei , con αi , βi ∈ R per ogni i = 1, . . . , n, al-
lora avremmo ∑ni=1 (αi − βi )ei = 0. Dalla indipendenza lineare tra i vettori e1 , . . . , en ,
segue che αi = βi , per ogni i = 1, . . . , n.
Esempio 66. Se V = R, allora per ogni a ∈ R, B = {a} é una base di V .
 
x x
Esempio 67. Sia V = { 1 2 , x1 , x2 , x3 , x4 ∈ R}.
x3 x4
Allora una base per V é data da
       
10 01 00 00
B={ , , , }
00 00 10 01

Esempio 68. Sia V = R3 , allora una base é data da

B = {(1, 0, 0), (0, 1, 0), (0, 0, 1)}

Esempio 69. Sia

V = R[X] = {a0 + a1 x + a2 x2 + a3 x3 + . . . . + an xn a0 , . . . ., an ∈ R}

allora una base é data da


B = {1, x, x2 , x3 , . . . , xn }
4.3 Basi e dimensione di uno spazio vettoriale. 95

Esempio 70. Sia V = R2 e consideriamo due distinte basi di V :

B1 = {(1, 0), (0, 1)}

B2 = {(1, −2), (4, 1)}.


Sia v ∈ V un vettore che abbia componenti (0, −1) rispetto alla base B1 , cioé v =
(0)(1, 0) + (−1)(0, 1) = (0, −1). Calcoliamo le sue componenti (a1 , a2 ) rispetto alla
base B2 :
v = a1 (1, −2) + a2 (4, 1)
cioé
(0, −1) = (a1 + 4a2 , −2a1 + a2 )
4
da cui a1 = 9 e a2 = − 91 .
Teorema 4.15. Sia V uno spazio vettoriale e B = {e1 , e2 , . . . , en } una sua base.
Siano v1 , v2 , . . . , vr vettori linearmente indipendenti di V . Allora r ≤ n ed esistono
n − r vettori e j1 , e j2 , . . . , e jn−r di B tali che l’insieme

B0 = {v1 , v2 , . . . , vr , e j1 , e j2 , . . . , e jn−r }

costituisca una base per V .

Dim. Dimostriamo dapprima che r ≤ n. Supponiamo al contrario che r > n. Poiché


B é una base di V , ciascuno dei vettori vi ammette n componenti rispetto ad essa.
Diciamo
v1 = (v11 , . . . , v1r ), . . . , vr = (vr1 , . . . , vrn ).
Costruiamo la matrice con r righe ed n colonne nella quale siano inserite le
componenti degli r vettori vi :
 
v11 v12 . . . . . . v1n
 v21 v22 . . . . . . v2n 
 
 v31 . . . . . . . . . v3n 
 
 ... ... ... ... ... 
vr1 vr2 . . . . . . vrn

Tale matrice dovrebbe avere r righe indipendenti, quindi rango r, ma ció é impossi-
bile poiché n < r.
Passiamo ora alla seconda parte del teorema. Poiché B é una base di V , esistono
α1 , . . . , αn non tutti nulli tali che

v1 = α1 e1 + α2 e2 + . . . + αn en

e supponiamo per esempio α1 6= 0. Segue che


1
e1 = (v1 − α2 e2 − . . . − αn en ) .
α1
96 4 Gli Spazi Vettoriali.

Quindi {v1 , e2 , e3 , . . . , en } é un insieme di generatori di V . Supponiamo che esista


una combinazione lineare

β1 v1 + β2 e2 + . . . + βn en = 0.

Se fosse β1 = 0 avremmo anche β2 = β3 = . . . = βn = 0, poiché e2 , . . . , en sono


indipendenti tra loro. Nel caso in cui β1 6= 0 allora
1
v1 = −(β2 e2 + . . . + βn en )
β1

cioé l’insieme {e2 , . . . , en } sarebbe un insieme di generatori di V , il che porta alla


falsa conclusione che il vettore e1 debba dipendere linearmente da {e2 , . . . , en }.
Ne segue che {v1 , e2 , e3 , . . . , en } devono essere tra loro linearmente indipendenti,
quindi sono una base per V .
Completiamo la dimostrazione per induzione sul numero r di vettori v1 , v2 , . . . , vr .
Supponiamo allora che il Teorema sia vero per r − 1 di essi, cioé

{v1 , v2 , . . . , vr−1 , er , . . . , en }

sia una base di V .


Quindi devono esistere α10 , . . . , αn0 non tutti nulli, tali che

vr = α10 v1 + α20 v2 + . . . + αr−1


0
vr−1 + αr0 er + . . . + αn0 en .

In particolare, poiché v1 , . . . , vr−1 sono indipendenti, almeno uno tra αr0 , . . . , αn0 deve
essere non nullo. Supponiamo per esempio αr0 6= 0. Segue che
r−1 n
1
er = (vr − ∑ αi0 vi − ∑ α 0j e j ) .
i=1 j=r+1 αr0

Quindi {v1 , . . . , vr , er+1 . . . , en } é un insieme di generatori di V . Come prima, sup-


poniamo ora che esista una combinazione lineare

β10 v1 + . . . + βr0 vr + βr+1


0
er+1 + . . . + βn0 en = 0.

Se fosse βr0 = 0 avremmo anche β10 = β20 = . . . = βr−1 = βr+1 = . . . = βn0 = 0, poiché
{v1 , v2 , . . . , vr−1 , er , . . . , en } é una base di V . Nel caso in cui βr0 6= 0 allora
r−1 n
1
vr = (− ∑ βi0 vi − ∑ β j0 e j )
i=1 j=r+1 βr0

cioé l’insieme {v1 , . . . , vr−1 , er+1 . . . , en } sarebbe un insieme di generatori di V , che


porterebbe alla falsa conclusione che il vettore er debba dipendere linearmente da
{v1 , . . . , vr−1 , er+1 . . . , en }. t
u
Dal Teorema 4.15 segue immediatamente che:
4.3 Basi e dimensione di uno spazio vettoriale. 97

Teorema 4.16. Due distinte basi di uno spazio vettoriale contengono lo stesso
numero di elementi.
Teorema 4.17. Sia V uno spazio vettoriale di dimensione n e siano v1 , v2 , . . . , vr
vettori linearmente indipendenti di V , con r ≤ n. Allora esistono n − r vettori
vr+1 , vr+2 , . . . , vn di V tali che l’insieme

B = {v1 , v2 , . . . , vr , vr+1 , vr+2 , . . . , vn }

costituisca una base per V .


Definiamo dimensione di uno spazio vettoriale V , e la indichiamo con dim(V ), il
numero di elementi di una qualsiasi base di V .
Corollario 4.18. Sia V uno spazio vettoriale di dimensione n. Ogni insieme di
vettori linearmente indipendenti di V é contenuto in una base di V .
Corollario 4.19. Sia V uno spazio vettoriale di dimensione n e sia m > n.
Ogni insieme di m vettori di V é linearmente dipendente.
Corollario 4.20. Sia V uno spazio vettoriale di dimensione n.
Ogni insieme di n vettori generatori di V é una base di V .
Corollario 4.21. Sia V uno spazio vettoriale di dimensione n.
Ogni insieme di n vettori linearmente indipendenti di V é una base di V .
Osservazione 4.22. Sia V uno spazio vettoriale di dimensione n.
Per determinare una base di V é allora sufficiente partire da un qualsiasi vettore non
nullo di V ed aggiungere uno alla volta altri vettori, in modo tale che ognuno sia
linearmente indipendente con quelli precedentemente aggiunti, fino ad ottenere un
insieme di n vettori linearmente indipendenti.
Una base {e1 , . . . , en } di V é quindi un insieme massimale di vettori linearmente
indipendenti, nel senso che, dato un qualsiasi vettore v ∈ V , l’insieme {e1 , . . . , en , v}
é linearmente dipendente.
Esempio 71. Sia

W = {(x1 , x2 , x3 , x4 ) ∈ R4 x1 − x4 = 0, x2 + x3 = 0}.

Il generico vettore di W é (x1 , x2 , −x2 , x1 ), quindi

dim(W ) = 2 e W =< (1, 0, 0, 1), (0, 1, −1, 0) > .

Esempio 72. Sia

W = {(x1 , x2 , x3 , x4 ) ∈ R4 x4 − x2 + x3 = 0}.

Il generico vettore di W é (x1 , x2 , x3 , x2 − x3 ), quindi

dim(W ) = 3 e W =< (1, 0, 0, 0), (0, 1, 0, 1), (0, 0, 1, −1) > .


98 4 Gli Spazi Vettoriali.

Terminiamo questo paragrafo con la dimostrazione della Proposizione 1, enunciata


nel paragrafo precedente:
Dimostrazione della Proposizione 4.4
Supponiamo dapprima che la somma U +V sia diretta. Indichiamo com {u1 , . . . , uh }
e {w1 , . . . , wk } rispettivamente una base per U ed una per W . Sia v ∈ U +W e sup-
poniamo che esso si possa esprimere in almeno due modi come somma di vettori di
U e W:
v = u+w
con u = ∑hi=1 αi ui ∈ U e u0 = ∑hi=1 βi ui ∈ U ed anche

v = u0 + w0

con w = ∑kj=1 γ j w j ∈ W e w0 = ∑kj=1 δ j w j ∈ W , per opportuni scalari αi , βi , γ j , δ j .


Quindi
h k h k
v = ∑ αi ui + ∑ γ j w j = ∑ βi ui + ∑ δ j w j
i=1 j=1 i=1 j=1

da cui
h k
∑ (αi − βi )ui = ∑ (γ j − δ j )w j ∈ U ∩W
i=1 j=1

poiché il primo membro appartiene a U ed il secondo a W . Dal fatto che la somma


é diretta segue che
h
∑ (αi − βi )ui = 0
i=1
k
∑ (γ j − δ j )w j = 0
j=1

cioé αi = βi per ogni indice i, γ j = δ j per ogni indice j. Quindi u = u0 e w = w0 .


Supponiamo ora che ogni vettore della somma sia esprimibile in modo unico e
scegliamo un qualsiasi vettore v ∈ U ∩W . Esistono opportuni scalari αi , β j tali che

h
v = ∑ αi ui ∈ U
i=1

ed anche
k
v = ∑ β j w j ∈ W.
i=1

Sottraendo le due espressioni otteniamo


h k
0 = ∑ αi ui − ∑ β j w j ∈ U +W.
i=1 j=1
4.4 Una nota sull’intersezione di sottospazi vettoriali. 99

Poiché il vettore 0 ∈ U + W si deve esprimere in modo unico, esso deve essere


necessariamente la somma del vettore nullo in U e del vettore nullo in W . Ne segue
che
h
∑ αi ui = 0 ∈ U
i=1

e quindi αi = 0 per ogni indice i, ed anche


k
∑ β jw j = 0 ∈ W
j=1

e quindi β j = 0 per ogni indice j. Da tutto ció otteniamo la conclusione v = 0.

4.4 Una nota sull’intersezione di sottospazi vettoriali.

Alla luce delle definizioni di base e dimensione (e per fugare ogni dubbio sul metodo
da utilizzare), diamo ora due esempi di come si possa determinare l’intersezione di
spazi vettoriali nei casi piú rilevanti:
1. il primo caso é quello in cui gli spazi siano descritti tramite le relazioni lineari
che intercorrono tra le componenti dei vettori appartenenti ad essi (usualmente
dette equazioni cartesiane di un sottospazio vettoriale);
Ecco un esempio: siano A e B due sottospazi di R4 definiti da

A = {(x1 , x2 , x3 , x4 ) : x1 − x4 = 0}

B = {(x1 , x2 , x3 , x4 ) : 2x1 + x3 = 0, x2 + x4 = 0}.


Per determinare A ∩ B, serve dapprima determinare la generica espressione di
un vettore (x1 , x2 , x3 , x4 ) ∈ R4 che appartenga ad entrambi. Le componenti di un
tale vettore devono soddisfare contemporaneamente a tutte le condizioni lineari
dettate da entrambe le definizioni di A e B. Quindi la quaterna (x1 , x2 , x3 , x4 ) deve
essere una soluzione del sistema lineare omogeneo in 4 incognite:

 x1 − x4 = 0
2x1 + x3 = 0 .
x2 + x4 = 0

La matrice ad esso associata ha rango 3, per cui ci aspettiamo ∞1 soluzioni, cioé


la dimensione dell’intersezione é 1. Il generico vettore risolutivo del sistema é
dato da (α, −α, −2α, α), al variare di α ∈ R, e concludiamo che una base per
l’intersezione é {(1, −1, −2, 1)}.
2. L’altro caso é quello in cui non venga data (almeno apparentemente) alcuna rela-
zione che leghi tra loro le componenti dei vettori degli spazi. Eccone un esempio:
Siano ancora A e B due sottospazi di R4 definiti da:
100 4 Gli Spazi Vettoriali.

A = {(a, a + b, c, a) : a, b, c ∈ R}

B = {(d, e, e, e) : d, e ∈ R}.
Ancora una volta dobbiamo dapprima determinare la generica espressione del
vettore che appartenga ad entrambi gli spazi: per un tale vettore deve accadere
che
(a, a + b, c, a) = (d, e, e, e) (4.1)
quindi
a = d, a + b = e, c = e, a=e
che si riduce ancora ad un sistema lineare omogeneo nelle incognite a, b, c, d, e:


 a−d = 0
a+b−e = 0

.

 c−e = 0
a−e = 0

La matrice associata al sistema ha rango 4, quindi vi sono ∞1 soluzioni, per cui


lo spazio intersezione ha dimensione 1. Riscrivendo il sistema in funzione del
parametro e, otteniamo: 

 a−d = 0
a+b = e


 c=e
a=e

da cui a = d = c = e, b = 0. Sostituendo nella (4.1) i valori qui ottenuti, si ot-


tiene il vettore appartenente all’intersezione: esso é (α, α, α, α), al variare del
parametro α ∈ R. Per cui la base per lo spazio intersezione é {(1, 1, 1, 1)}.

4.5 Formula di Grassmann.

Siano A, B sottospazi vettoriali dello spazio V . Vogliamo considerare ora la relazione


che intercorre tra le dimensioni di A, B, A + B e A ∩ B.
Premettiamo il seguente:
Lemma 4.23. Siano A e B sottospazi vettoriali dello spazio V , e siano CA e CB ri-
spettivamente una base di A ed una di B. Allora l’unione dei vettori delle due basi,
cioé CA ∪CB , costituisce un insieme di generatori per il sottospazio A + B. Inoltre,
se t é il numero massimo di vettori di CA ∪ CB che possano essere linearmente in-
dipendenti, allora ogni sottoinsieme di CA ∪ CB costituito da t vettori linearmente
indipendenti, é una base per A + B.
Teorema 4.24. (Formula di Grassmann) dim(A + B) = dim(A) + dim(B) − dim(A ∩
B).
4.5 Formula di Grassmann. 101

Dim. Dividiamo la dimostrazione in due casi:


1. Sia A ⊕ B somma diretta. Se EA = {c1 , . . . , cr }, EB = {d1 , . . . , ds } sono rispet-
tivamente basi per A e B, allora EA ∪ EB é un insieme di generatori per A ⊕
B. É sufficiente quindi dimostrare che {c1 , . . . , cr , d1 , . . . , ds } sono linearmente
indipendenti. Consideriamo la combinazione lineare

α1 c1 + . . . . + αr cr + β1 d1 + . . . . + βs ds = 0.

Poiché la somma é diretta, il vettore 0 si puó esprimere in modo unico come


somma di un vettore di A e di uno in B. Certamente 0 si puó esprimere come

0 = 0 · c1 + . . . + 0 · cr + 0 · d1 + . . . . + 0 · ds

Per cui, confrontando quest’ultima con la precedente combinazione lineare, si


ottiene
α1 = . . . . = αr = β1 = . . . . = βs = 0.
Quindi EA ∪ EB é una base per A ⊕ B, per cui dim(A ⊕ B) = dim(A) + dim(B).
2. Supponiamo ora che A ∩ B =< c1 , . . . , ck >, dim(A ∩ B) = k. Per il teorema sul
completamento di una base, esistono vettori indipendenti tra loro d1 , . . . , dq , tali
che {c1 , . . . , ck , d1 , . . . , dq } sia una base per B. Consideriamo un vettore v ∈<
c1 , . . . , ck > ∩ < d1 , . . . , dq >:

k q
v = ∑ αi ci = ∑ β jd j
i=1 j=1

e quindi
k q
∑ αi ci − ∑ β j d j = 0.
i=1 j=1

Poiché {c1 , . . . , ck , d1 , . . . , dq } sono indipendenti, segue che αi = β j = 0, per ogni


i, j. Allora v = 0, cioé < c1 , . . . , ck > ∩ < d1 , . . . , dq >= 0 e

B =< c1 , . . . , ck > ⊕ < d1 , . . . , dq >= (A ∩ B)⊕ < d1 , . . . , dq >

quindi per il punto precedente dim(B) = k + q. Inoltre, se w ∈ A∩ < d1 , . . . , dq >,


allora w ∈ A, w ∈ B e w ∈< d1 , . . . , dq >, quindi w ∈ (A ∩ B)∩ < d1 , . . . , dq >, per
cui w = 0. Alla luce di tutto ció:

A + B = A + ((A ∩ B)⊕ < d1 , . . . , dq >) = A⊕ < d1 , . . . , dq >

e passando alle dimensioni:

dim(A + B) = dim(A) + dim(< d1 , . . . , dq >) = dim(A) + (dim(B) − k) =

dim(A) + dim(B) − dim(A ∩ B).


102 4 Gli Spazi Vettoriali.

t
u
Esempio 73. Siano V = R4 ,

A = {(x, y, z,t) ∈ R4 , y = 0, 2z − t = 0}

B = {(x, y, z,t) ∈ R4 , x − t = 0, y + z = 0}
e calcoliamo dim(A + B).

Svolg. Il primo passo é quello di calcolare basi e dimensioni di A e B. Il generico


vettore di A si esprime (x, 0, z, 2z), al variare di x, z ∈ R. Allora dim(A) = 2 ed una
sua base é la seguente
(1, 0, 0, 0), (0, 0, 1, 2).
Il generico vettore di B si esprime (x, y, −y, x), al variare di x, y ∈ R. Allora dim(B) =
2 ed una sua base é
(1, 0, 0, 1), (0, 1, −1, 0).
Quindi se v ∈ A ∩ B, esso deve essere esprimibile contemporaneamente in due modi,
cioé
v = (a, 0, b, 2b) = (c, d, −d, c) con a, b, c, d ∈ R.
Uguagliando le due quaterne si ottiene

a=b=c=d=0

che significa A ∩ B = {0} e dim(A ∩ B) = 0, da cui

dim(A + B) = dim(A) + dim(B) − dim(A ∩ B) = 2 + 2 − 0 = 4.

Concludiamo allora che A + B = R4 , come somma diretta. t


u
Esempio 74. Siano V = R3 ,

A = {(a + b, b, a), a, b ∈ R}

B = {(x, y, z), x − y = 0}.

Svolg. Si ha che dim(A) = 2 ed una sua base é data da

(1, 0, 1), (1, 1, 0).

Inoltre il generico vettore di B si esprime (x, x, z), quindi dim(B) = 2 ed una sua
base é
(1, 1, 0), (0, 0, 1).
Quindi se v ∈ A ∩ B, esso deve essere esprimibile contemporaneamente in due modi,
cioé
v = (a + b, b, a) = (c, c, d) con a, b, c, d ∈ R.
4.5 Formula di Grassmann. 103

Uguagliando le due terne si ottiene

a=d=0 e b=c da cui v = (b, b, 0).

Ció vuol dire che dim(A ∩ B) = 1 ed una sua base é data dal vettore (1, 1, 0).
Applicando la formula di Grassmann otteniamo:

dim(A + B) = 2 + 2 − 1 = 3

quindi A + B = R3 ma non come somma diretta. t


u
Esempio 75. Siano U =< (0, 1, 1), (2, 0, 1) > e W =< (1, 1, 2) > sottospazi di R3 .
Determiniamo dim(U +W ).

Svolg. Il generico vettore v ∈ U ∩W si deve esprimere nei due seguenti modi

v = a(0, 1, 1) + b(2, 0, 1) = (2b, a, a + b) ∈ U

v = c(1, 1, 2) = (c, c, 2c) ∈ W.


Uguagliando le due terne otteniamo a = b = c = 0, cioé U ∩ W = {0}, quindi in
base alla formula di Grassmann dim(U +W ) = 2 + 1 − 0 = 3, e U +W = R3 come
somma diretta. t
u
Esempio 76. Siano

A =< (2, 0, 0, 1), (0, 0, −2, 0), (0, 0, 1, −1) >

B =< (0, 1, 0, 0), (1, 1, 0, 0) >


sottospazi di R4 .

Svolg. Un generico vettore v ∈ A ∩ B si esprime nei due seguenti modi

v = a(2, 0, 0, 1) + b(0, 0, −2, 0) + c(0, 0, 1, −1) = (2a, 0, −2b + c, a − c) ∈ A

v = d(0, 1, 0, 0) + e(1, 1, 0, 0) = (e, d + e, 0, 0) ∈ B.


Uguagliando le due quaterne si ottiene

d e
a = 2b = c = − =
2 2
quindi v = (e, 0, 0, 0), al variare di e ∈ R. Per cui dim(A ∩ B) = 1 e dim(A + B) =
3 + 2 − 1 = 4, cioé A + B = R4 , ma non come somma diretta. t
u
Esempio 77. Siano

A =< (2, −1, 0, 1), (1, 3, 1, −1), (0, 1, −1, −1) >

B =< (2, 0, 1, 0), (1, 2, 2, 0) >


104 4 Gli Spazi Vettoriali.

sottospazi di R4 . Determiniamo dim(A + B).

Svolg. Un generico vettore v ∈ A ∩ B si esprime nei due seguenti modi

v = a(2, −1, 0, 1) + b(1, 3, 1, −1) + c(0, 1, −1, −1)


= (2a + b, −a + 3b + c, b − c, a − b − c) ∈ A
e
v = d(2, 0, 1, 0) + e(1, 2, 2, 0)
= (2d + e, 2e, d + 2e, 0) ∈ B.
Uguagliando le due 4-uple si ottiene

a=d=0 b = −c = e

quindi v = (e, 2e, 2e, 0), al variare di e ∈ R. Per cui dim(A ∩ B) = 1 e dim(A + B) =
3 + 2 − 1 = 4, cioé A + B = R4 , ma non come somma diretta. t
u

4.6 Cambiamento di base in uno spazio vettoriale.

Sia V uno spazio vettoriale di dimensione n sul campo R e siano

B = {e1 , e2 , . . . , en }, e B0 = {e01 , e02 , . . . , e0n }

due distinte basi di V . Per ogni vettore v ∈ V avremo:

v = x1 e1 + x2 e2 + . . . + xn en x1 , . . . , xn ∈ R

v = x10 e01 + x20 e02 + . . . + xn0 e0n x10 , . . . , xn0 ∈ R.


Indichiamo allora X = [x1 , . . . , xn ]T il vettore contenente le componenti di v rispetto
alla base B e X 0 = [x10 , . . . , xn0 ]T quello contenente le componenti di v rispetto alla
base B0 .
Il passaggio dalle componenti di un vettore in base B a quelle del medesimo vettore
in base B0 é regolato dalla seguente relazione vettoriale

X0 = A · X

nella quale a matrice A é detta matrice del cambiamento di base ed é costruita come
segue:
– nella prima colonna vi sono le componenti del vettore e1 rispetto alla base B0 ;
– nella seconda colonna vi sono le componenti del vettore e2 rispetto alla base B0 ;
– in generale, nella colonna j vi sono le componenti del vettore e j della base B,
calcolate rispetto alla base B0 .
4.6 Cambiamento di base in uno spazio vettoriale. 105

Poiché gli n vettori di una base sono sempre linearmente indipendenti, il sistema
sopra citato ha rango massimo, cioé n, quindi la matrice A é invertibile, da cui
otteniamo le formule inverse per il cambiamento di base:

X = A−1 · X 0

che costituiscono le formule di passaggio dalle componenti di un vettore in base B0


a quelle del medesimo vettore in base B.
La matrice A−1 é costruita come segue:
– nella prima colonna vi sono le componenti del vettore e01 rispetto alla base B;
– nella seconnda colonna vi sono le componenti del vettore e02 rispetto alla base B;
– in generale, nella colonna j vi sono le componenti del vettore e0j della base B0 ,
calcolate rispetto alla base B.
Esempio 78. Siano V = R2 e B = {e1 = (1, 1), e2 = (0, 1)}, B0 = {e01 = (1, 0), e02 =
(2, 1)} due basi di V . Determiniamo le formule di cambiamento di base in entrambi
i versi. Calcoliamo le componenti dei vettori della prima base rispetto alla seconda.

Svolg. Le componenti di e1 = (1, 1) rispetto a B0 sono (e1 )B0 = (−1, 1) infatti

(1, 1) = (−1)(1, 0) + (1)(2, 1)

analogamente le componenti
 di e2 = (0, 1) rispetto a B0 sono (e2 )B0 = (−2, 1).
−1 −2
Per cui, la matrice A = é quella che determina il passaggio dalla base B
1 1
a quella B0 , cioé  0    
x1 −1 −2 x
= · 1
x20 1 1 x2
e le formule di passaggio sono

x10 = −x1 − 2x2



.
x20 = x1 + x2

Le formule inverse sono date da


     0
x1 1 2 x
= · 10
x2 −1 −1 x2

x1 = x10 + 2x20


x2 = −x10 − x20
 
1 2
in cui la matrice di passaggio é A−1 = . Per esempio consideriamo il
−1 −1
vettore v ∈ V che abbia componenti X = (x1 , x2 ) = (2, −3) rispetto alla base B.
Determiniamo le sue componenti X 0 = (x10 , x20 ) rispetto alla base B0 :
106 4 Gli Spazi Vettoriali.

x10 = −2 + 6 = 4

.
x20 = 2 − 3 = −1
t
u
Esempio 79. Siano V = R3 ,

B = {(1, 1, 0), (1, 0, 1), (2, 0, 1)}, B0 = {(0, 1, 1), (2, −1, 0), (1, 0, 2)}

due basi di V . Determiniamo le formule di cambiamento di base.

Svolg. Calcoliamo le componenti dei vettori della prima base B rispetto alla
seconda B0 :
(1, 1, 0) → (2, 1, −1)B0
1 1 1
(1, 0, 1) → ( , , )B0
3 3 3
(2, 0, 1) → (1, 1, 0)B0
per cui
x10 2 13 1
     
x1
 x20  =  1 1 1  ·  x2 
3
x30 −1 13 0 x3
 0
 x1 = 2x1 + 13 x2 + x3
x0 = x + 1 x + x .
 2 0 1 3 21 3
x3 = −x1 + 3 x2
Per esempio il vettore v di componenti (1, 0, 3) rispetto alla base B avrá componenti
(5, 4, −1) rispetto alla base B0 . t
u
Esempio 80. Siano V = R3 e B = {e1 , e2 , e3 }, B0 = {e01 , e02 , e03 } due basi di V tali
che  0
 e1 = e1 + 3e2 + 2e3
e02 = e1 + e3 .
e03 = e2

Svolg. In tale caso le formule di passaggio dalla base B0 alla base B sono
     0
x1 110 x1
 x2  =  3 0 1  ·  x20 
x3 210 x30
e calcolando l’inversa della matrice che compare nel sistema precedente:
 0    
x1 −1 0 1 x1
 x20  =  2 0 −1  ·  x2 
x30 3 1 −3 x3

Esempio 81. Siano V = R3 ,


4.7 Esercizi svolti. 107

B = {(1, 1, 0), (1, 0, 1), (2, 0, 1)}, B0 = {(1, 1, 1), (0, 0, 1), (1, 0, 2)}

due basi di V . Sia v ∈ V un vettore di componenti (1, 1, 2) rispetto alla base B.


Indichiamo (a, b, c) le componenti di v rispetto alla base B0 . Per determinare (a, b, c)
applichiamo ora esattamente la definizione di componenti:

(1)(1, 1, 0) + (1)(1, 0, 1) + (2)(2, 0, 1) = a(1, 1, 1) + b(0, 0, 1) + c(1, 0, 2)

cioé
(6, 1, 3) = (a + c, a, a + b + 2c)
da cui a = 1, b = −8, c = 5. t
u

4.7 Esercizi svolti.

Esercizio 31. Dati i seguenti vettori di R3 [X], lo spazio vettoriale dei polinomi di
grado 3 nella variabile x

2x3 + x2 + x + 1, x3 + x2 + 2, x3 + 1

si verifichi se essi sono linearmente dipendenti od indipendenti.

Svolgimento: Scegliamo una base B per R3 [X], in modo da poter esprimere i


polinomi tramite le loro componenti rispetto a tale base. Sia

B = {1, x, x2 , x3 }

per cui

2x3 + x2 + x + 1 = (1, 1, 1, 2)B , x3 + x2 + 2 = (2, 0, 1, 1)B , x3 + 1 = (1, 0, 0, 1)B .

É quindi sufficiente verificare l’eventuale dipendenza od indipendenza lineare tra


le 4-uple rappresentative dei vettori. Per farlo, poniamo le componenti di ciascun
polinomio in una riga di una matrice, ottenendo quindi
 
1112
A = 2 0 1 1
1001

il cui rango é 3 (facile verifica). Concludiamo allora che i 3 vettori (polinomi) sono
tra loro linearmente indipendenti in R3 [X].
Esercizio 32. Dati i seguenti vettori di R3 [X], lo spazio vettoriale dei polinomi di
grado 3 nella variabile x

2x3 + x2 + x + 1, x3 + x2 + 2, 3x3 + x2 + 2x
108 4 Gli Spazi Vettoriali.

si verifichi se essi sono linearmente dipendenti od indipendenti.

Svolgimento: Come nel precedente esercizio, esprimiamo i polinomi tramite le


lorocomponenti rispetto alla base B = {1, x, x2 , x3 } di R3 [X]:

2x3 +x2 +x+1 = (1, 1, 1, 2)B , x3 +x2 +2 = (2, 0, 1, 1)B , 3x3 +x2 +2x = (0, 2, 1, 3)B .

Quindi introduciamo una matrice, le cui righe siano le 4-uple delle componenti dei
polinomi:  
1112
A =  2 0 1 1 .
0213
Determiniamo il rango di A, procedendo con opportune operazioni elementari sulle
righe:  
1 1 1 2
R2 → R2 − 2R1 =⇒ A0 =  0 −2 −1 −3 
0 2 1 3
ed infine  
1 1 1 2
R3 → R3 + R2 =⇒ A00 =  0 −2 −1 −3  .
0 0 0 0
Il rango di A00 é ovviamente 2, quindi i 3 vettori sono tra loro linearmente dipendenti
in R3 [X]. In particolare, osserviamo che la terza riga della matrice si é annullata. Ció
vuol dire che il vettore 3x3 + x2 + 2x = (0, 2, 1, 3)B , le cui componenti riempivano
la terza riga della matrice iniziale, é combinazione lineare dei restanti 2. Per deter-
minare le componenti scalari tramite le quali esprimere tale combinazione lineare,
procediamo come segue. Poniamo α, β ∈ R tali che

(0, 2, 1, 3) = α(1, 1, 1, 2) + β (2, 0, 1, 1) = (α + 2β , α, α + β , 2α + β ).

Eguagliando le componenti omologhe del primo ed ultimo vettore nella precedente


relazione, avremo 

 α + 2β = 0
α =2


 α + β =1
2α + β = 3

dalle quali concludiamo che α = 2 e β = −1. Infatti é facile verificare che

3x3 + x2 + 2x = 2(2x3 + x2 + x + 1) − (x3 + x2 + 2).

Esercizio 33. Nello spazio vettoriale R3 si considerino le seguenti basi:

B = {(1, 1, 1), (1, 0, 1), (1, 0, 0)}, B0 = {(2, 1, 0), (0, 0, 1), (1, 1, 0)}.
4.7 Esercizi svolti. 109

Si determinino entrambe le matrici di transizione da B a B0 e viceversa.

Svolgimento: Determiniamo dapprima la matrice di transizione C da B a B0 . Tale


matrice é costruita una colonna alla volta. In ciascuna colonna vi sono le componenti
di un vettore della base B rispetto alla base B0 . L’ordine con cui costruire le colonne
della matrice é strettamente connesso all’ordine con cui i vettori compaiono nella
base B. Quindi la prima colonna della matrice di transizione C sará composta dalle
componenti del vettore (1, 1, 1) rispetto a B0 , la seconda colonna tramite le com-
ponenti di (1, 0, 1) rispetto a B0 , la terza colonna tramite le componenti di (1, 0, 0)
rispetto a B0 . Procedendo in tal senso dovremo porre

(1, 1, 1) = x1 (2, 1, 0) + x2 (0, 0, 1) + x3 (1, 1, 0) = (2x1 + x3 , x1 + x3 , x2 )

da cui 
 2x1 + x3 = 1
x1 + x3 = 1
x2 = 1

dalle quali concludiamo che x1 = 0, x2 = 1, x3 = 1.


Analogamente

(1, 0, 1) = x1 (2, 1, 0) + x2 (0, 0, 1) + x3 (1, 1, 0) = (2x1 + x3 , x1 + x3 , x2 )

da cui 
 2x1 + x3 = 1
x1 + x3 = 0
x2 = 1

dalle quali concludiamo che x1 = 1, x2 = 1, x3 = −1.


Ed infine

(1, 0, 0) = x1 (2, 1, 0) + x2 (0, 0, 1) + x3 (1, 1, 0) = (2x1 + x3 , x1 + x3 , x2 )

da cui 
 2x1 + x3 = 1
x1 + x3 = 0
x2 = 0

dalle quali concludiamo che x1 = 1, x2 = 0, x3 = −1. Quindi la matrice di transizione


C é data da  
0 1 1
C =  1 1 0 .
1 −1 −1
Se indichiamo con X e X 0 rispettivamente le coordinate di un vettore v ∈ R3 , rispet-
tivamente in termini della base B e B0 , allora la matrice C permette di effettuare la
transizione X 0 = CX. L’inversa di C é invece la matrice di transizione da B0 a B,
ovvero quella che consente di effettuare la transizione X = C−1 X 0 . Essa é
110 4 Gli Spazi Vettoriali.
 
1 0 1
C−1 =  −1 1 −1  .
2 −1 1

Esercizio 34. Si considerino le seguenti due basi in R2 [X]:

B = {x2 + x + 1, x2 + x, x2 + 2}, E = {x, x + 1, x2 + x}

ed il polinomio p(x) di componenti (2, 1, 1)B rispetto alla base B. Si determinino le


componenti di p(x) rispetto alla base E e si costruisca la matrice di transizione da B
a E.

Svolgimento: Siano (α, β , γ)E le componenti di p(x) rispetto alla base C. Espri-
mendo p(x) tramite le prorpie componenti rispetto ad entrambe le basi, avremo
che
2(x2 + x + 1) + (x2 + x) + (x2 + 2) = α(x) + β (x + 1) + γ(x2 + x)
e svolgendo i relativi calcoli, segue

4x2 + 3x + 4 = γx2 + (α + β + γ)x + β .

Eguagliando adesso i coefficienti dei monomi simili, avremo γ = 4, α + β + γ = 3


e β = 4, da cui α = −5.
Quindi le componenti di p(x) rispetto alla base E sono (−5, 4, 4)E .
Per ottenere la matrice di transizione da B a E dovremo calcolare le componenti
di ciascuno dei vettori della base B rispetto alla base E. Potremmo operare come
precedentemente fatto, ripetendo per ciascuno dei vettori di B il medesimo procedi-
mento (lasciamo al lettore tale compito).
In alternativa, utilizziamo un metodo del tutto equivalente, che equipara il computo
della matrice di transizione in R2 [X] a quello della matrice di transizione in R3 . Per
farlo, dobbiamo dapprima esprimere ciascun vettore delle due basi tramite le rispet-
tive componenti rispetto ad una ulteriore base, scelta arbitrariamente. Ovviamente,
per facilitare al massimo i calcoli, la nostra scelta ricade sulla base F = {1, x, x2 }.
In tal modo, possiamo riscrivere B ed E, pensandole come basi in R3 :

B = {(1, 1, 1)F , (0, 1, 1)F , (2, 0, 1)F }, E = {(0, 1, 0)F , (1, 1, 0)F , (0, 1, 1)F }.

A questo punto iniziamo a determinare le componenti delle 3-uple di B in termini


delle 3-uple di E:

(1, 1, 1) = x1 (0, 1, 0) + x2 (1, 1, 0) + x3 (0, 1, 1) = (x2 , x1 + x2 + x3 , x3 )

da cui
x1 = −1, x2 = 1, x3 = 1.
Quindi
4.7 Esercizi svolti. 111
   
x1 −1
 x2  =  1 
x3 1
é la prima colonna della matrice di transizione cercata. Per il secondo vettore di B i
calcoli sono banali, poiché ponendo

(0, 1, 1) = x1 (0, 1, 0) + x2 (1, 1, 0) + x3 (0, 1, 1)

segue ovviamente che


x1 = 0, x2 = 0, x3 = 1.
Quindi    
x1 0
 x2  =  0 
x3 1
é la seconda colonna della matrice di transizione. Infine

(2, 0, 1) = x1 (0, 1, 0) + x2 (1, 1, 0) + x3 (0, 1, 1) = (x2 , x1 + x2 + x3 , x3 )

implica che
x1 = −3, x2 = 2, x3 = 1.
Quindi    
x1 −3
 x2  =  2 
x3 1
é la terza colonna della matrice di transizione.
Concludiamo che, se indichiamo con X e X 0 rispettivamente le coordinate di un
vettore p(x) ∈ R2 [X], rispettivamente in termini della base B ed E, allora la matrice
C, che permette di effettuare la transizione X 0 = CX, é la seguente:
 
−1 0 −3
C =  1 0 2 .
1 1 1

Ad esempio, ritornando al primo punto dell’esercizio, possiamo verificare che


     
−1 0 −3 2 −5
 1 0 2 ·1 =  4  .
1 1 1 1 B 4 E

Esercizio 35. Nello spazio vettoriale R4 si consideri il seguente sottospazio:

V = {(x, y, z,t) ∈ R4 : x − y + z = 0; x + t = 0}.

Si determini una base per un sottospazio W di R4 , tale che V ⊕ W = R4 (somma


112 4 Gli Spazi Vettoriali.

diretta).

Svolgimento: Inizialmente serve determinare una base per il sottospazio V . Per


farlo é necessario esprimere le relazioni che intercorrono tra le componenti del ge-
nerico vettore di V , individuando in particolare quante e quali siano le componenti
libere. In altre parole, prima di tutto determiniamo la dimensione di V . Dobbiamo
quindi risolvere il sistema formato dalle relazioni lineari che definiscono V :

x−y+z = 0
.
x+t = 0

Il sistema ha banalmente 4 incognite e rango 2, per cui avremo ∞2 soluzioni. Da


qui segue che dim(V ) = 2. Inoltre tramite le relazioni t = −x e y = x + z, possiamo
descrivere il generico vettore di V come segue

v ∈ V ⇐⇒ α, β ∈ R : v = (α, α + β , β , −α).

Da ció abbiamo che


v = α(1, 1, 0, −1) + β (0, 1, 1, 0)
per cui V =< (1, 1, 0, −1), (0, 1, 1, 0) >. Per costruire un opportuno sottospazio W
di R4 , tale che V ⊕W = R4 é sufficiente completare la base di V ad una base di R4 .
Aggiungiamo allora all’insieme {(1, 1, 0, −1), (0, 1, 1, 0)} un terzo vettore (a, b, c, d) ∈
R4 , in modo tale che l’insieme {(1, 1, 0, −1), (0, 1, 1, 0), (a, b, c, d)} sia linearmente
indipendente. La scelta di (a, b, c, d) é arbitraria, a patto che sia garantita l’indipen-
denza lineare coi precedenti due vettori. Il vettore (a, b, c, d) deve quindi essere tale
da garantire che la matrice  
1 1 0 −1
0 1 1 0 
abc d
abbia rango 3. La nostra scelta ricade sul vettore (0, 0, 1, 0).
Abbiamo in sostanza costruito un sottospazio vettoriale V 0 di R4 di dimensione 3,
avente come base l’insieme {(1, 1, 0, −1), (0, 1, 1, 0), (0, 0, 1, 0)}, ovvero

V 0 = V ⊕ < (0, 0, 1, 0) >

= < (1, 1, 0, −1), (0, 1, 1, 0) > ⊕ < (0, 0, 1, 0) >

= < (1, 1, 0, −1), (0, 1, 1, 0), (0, 0, 1, 0) > .

Per concludere, partendo adesso dal sottospazio V 0 , ripetiamo il precedente pro-


cedimento, al fine di determinare un opportuno vettore (a0 , b0 , c0 , d 0 ) ∈ R4 tale
che
4.7 Esercizi svolti. 113
 
1 1 0 −1
0 1 1 0 
 
0 0 1 0 
a0 b0 c0 d0
abbia rango 4. Anche in tale caso la scelta é arbitraria, a patto che venga garantita
l’indipendenza lineare. Come prima, la nostra scelta ricade su di un vettore estratto
dalla base canonica, ovvero (a0 , b0 , c0 , d 0 ) = (1, 0, 0, 0).
Abbiamo quindi ottenuto

R4 = V 0 ⊕ < (1, 0, 0, 0) >= V ⊕ < (0, 0, 1, 0) > ⊕ < (1, 0, 0, 0) >

ovvero abbiamo costruito un sottospazio W =< (0, 0, 1, 0), (1, 0, 0, 0) > di R4 ,


avente dimensione 2, tale che

R4 = V ⊕W

= < (1, 1, 0, −1), (0, 1, 1, 0) > ⊕ < (0, 0, 1, 0), (1, 0, 0, 0) >

= < (1, 1, 0, −1), (0, 1, 1, 0), (0, 0, 1, 0), (1, 0, 0, 0) > .

Esercizio 36. Dati i seguenti sottospazi V =< (2, 1, 0), (0, 2, 0) > e W = {(x, y, z) ∈
R3 : x + 2y + 3z = 0} di R3 , si determinino le basi dei sottospazi V ∩W e V +W .

Svolgimento: Inizialmente determiniamo una base per il sottospazio W . La sua


dimensione é 2, poiché esiste una unica relazione lineare intercorrente tra le com-
ponenti di un generico vettore di W . Ponendo x = −2y − 3z, possiamo descrivere un
vettore w ∈ W come segue

w = (−2α − 3β , α, β ) α, β ∈ R

per cui W =< (−2, 1, 0), (−3, 0, 1) >. Unendo i vettori delle basi dei due sottospazi
vettoriali otteniamo un insieme di generatori per la somma V + W . Tale insieme é
quindi
B = {(2, 1, 0), (0, 2, 0), (−2, 1, 0), (−3, 0, 1)}.
Costruiamo una matrice, ponendo in ciascuna riga le componenti dei vettori di B:
 
2 10
 0 2 0
A=  −2 1 0  .

−3 0 1

Tale matrice ha rango 3 (facile verifica), in particolare possiamo individuare il


seguente minore non nullo di ordine 3
114 4 Gli Spazi Vettoriali.

0 2 0

−2 1 0 6= 0.

−3 0 1

Quindi il vettore che inizialmente occupava la prima riga di A é linearmente di-


pendente dai restanti 3 vettori, i quali sono tra loro linearmente indipendenti. Con-
cludiamo allora che dim(V +W ) = 3 e V +W =< (0, 2, 0), (−2, 1, 0), (−3, 0, 1) >.
Osserviamo a margine che la somma dei sottospazi ricopre interamente R3 .
Grazie alla formula di Grassmann, dim(V +W ) = dim(V ) + dim(W ) − dim(V ∩W ),
sappiamo adesso che dim(V ∩W ) = 1. Determiniamo infine una base per V ∩W .
Supponiamo u ∈ V ∩W . Allora esso é esprimibile in 2 modi differenti. Poiché u ∈ V
avremo che
u = x1 (2, 1, 0) + x2 (0, 2, 0) = (2x1 , x1 + 2x2 , 0).
Parallelamente, poiché u ∈ W avremo anche

u = x3 (−2, 1, 0) + x4 (−3, 0, 1) = (−2x3 − 3x4 , x3 , x4 ).

Quindi i parametri x1 , x2 , x3 , x4 devono essere tali che

(2x1 , x1 + 2x2 , 0) = (−2x3 − 3x4 , x3 , x4 )

ovvero 
 2x1 = −2x3 − 3x4
x1 + 2x2 = x3
0 = x4

e riordinando le variabili nelle precedenti relazioni lineari, otteniamo il sistema


omogeneo 
 2x1 + 2x3 + 3x4 = 0
x1 + 2x2 − x3 = 0 .
x4 = 0

Risolvendolo, otteniamo

x4 = 0, x1 = −x3 , x2 = x3 .

Riportiamo adesso i valori ottenuti nelle espressioni di u ∈ V ed u ∈ W :

u ∈ V =⇒ u = −x3 (2, 1, 0) + x3 (0, 2, 0) = (−2x3 , x3 , 0)

u ∈ W =⇒ u = x3 (−2, 1, 0) = (−2x3 , x3 , 0).


Abbiamo quindi verificato che (−2x3 , x3 , 0), al variare del parametro libero x3 ∈ R,
é l’espressione di un qualsiasi vettore u ∈ V ∩ W . Per cui V ∩ W =< (−2, 0, 1) >.
Se ne deduce che la somma V +W = R3 non é diretta.
Esercizio 37. Nello spazio vettoriale R4 si considerino i seguenti sottospazi:

V = {(x, y, z,t) ∈ R4 : x − y + z = 0; x + t = 0, y − t = 0}
4.7 Esercizi svolti. 115

W = {(x, y, z,t) ∈ R4 : x − y = 0; z − t = 0}
Si determini una base per ciascuno degli spazi A + B e A ∩ B.

Svolgimento: Determiniamo dapprima una base per ciascuno dei due sottospazi
vettoriali.
Se v = (x, y, z,t) ∈ V , allora 
x−y+z = 0
x+t = 0
y−t = 0

da cui x = −t, y = t e z = 2t. La dimensione di V é pari ad 1 ed il generico vettore


appartenente a V si esprime (−α, α, 2α, α), al variare di α ∈ R. Quindi V =<
(−1, 1, 2, 1) >.
Se w ∈ W , allora 
x−y = 0
z−t = 0
da cui x = y e z = t. In questo caso, la dimensione di W é pari a 2 ed il generico
vettore appartenente a W si esprime (β , β , γ, γ), al variare di β , γ ∈ R. Una base per
W é quindi {(1, 1, 0, 0), (0, 0, 1, 1)}.
Iniziamo a discutere la somma tra i due sottospazi. Partiamo da un insieme di
generatori di V +W : esso é certamente dato dall’unione delle due nasi:

{(−1, 1, 2, 1), (1, 1, 0, 0), (0, 0, 1, 1)}.

É di facile verifica il fatto che tali vettori sono tra loro linearmente indipendenti (si
costruisca ad esempio un matrice avente per righe le 4-uple delle componenti dei
vettori e si verifichi che il rango di tale matrice e 3).
Applichiamo adesso la formula di Grassmann, per cui

3 = 1 + 2 − dim(V ∩W ) =⇒ dim(V ∩W ) = 0.

Concludiamo allora che V ∩ W = {0} ed é quindi privo di base, e da ció possiamo


affermare che V e W sono in somma diretta:

V ⊕W =< (−1, 1, 2, 1), (1, 1, 0, 0), (0, 0, 1, 1) > .

Esercizio 38. Nello spazio vettoriale R4 si considerino i seguenti sottospazi:

V = {(x, y, z,t) ∈ R4 : x − y + z = 0}

W = {(x, y, z,t) ∈ R4 : x + t = 0; y + z = 0}
Si determini una base per ciascuno degli spazi V +W e V ∩W .

Svolgimento: Dalla relazione lineare che intercorre tra le componenti (x, y, z,t) di
116 4 Gli Spazi Vettoriali.

un generico vettore in V , abbiamo x = y − z. Quindi un qualsiasi vettore di V si


esprime
(α − β , α, β , γ) α, β , γ ∈ R.
La dimensione di V é pari a 3 ed una sua base é

BV = {(1, 1, 0, 0), (−1, 0, 1, 0), (0, 0, 0, 1)}.

Passiamo al secondo sottospazio: un vettore in W é tale che le sue componenti


(x, y, z,t) debbano soddisfare entrambe le relazioni x = −t, y = −z. Per cui un tale
vettore ha la seguente forma:

(−δ , −η, η, δ ) δ , η ∈ R.

La dimensione di W é pari a 2 ed una sua base é

BW = {(−1, 0, 0, 1), (0, −1, 1, 0)}.

Consideriamo BV ∪BW = {(1, 1, 0, 0), (−1, 0, 1, 0), (0, 0, 0, 1), (−1, 0, 0, 1), (0, −1, 1, 0)}.
Tale insieme genera V + W , ma chiaramente non potrá esserne una base, visto
che V + W é sottospazio di R4 . Studiamo il rango della matrice formata dalle
componenti, poste in riga, dei vettori generatori di V +W :
 
1 1 00
 −1 0 1 0 
 
 0 0 0 1 .
 
 −1 0 0 1 
0 −1 1 0

Tale rango é 4, ad esempio un minore non nullo di ordine 4 é dato da



1 1 0 0

−1 0 1 0
0 0 0 1 6= 0.


−1 0 0 1

Ció implica che dim(V +W ) = 4 e

V +W =< (1, 1, 0, 0), (−1, 0, 1, 0), (0, 0, 0, 1), (−1, 0, 0, 1) > .

Dall’applicazione della formula di Grassmann deduciamo che dim(V ∩W ) = 1. Per


ottenere l’espressione del generico vettore appartenente a V ∩ W , scegliamo u ∈
V ∩W ed esprimiamolo in due diversi modi. Poiché u ∈ V

u = (α − β , α, β , γ) α, β , γ ∈ R

ma anche, dal fatto che supponiamo u ∈ W ,


4.7 Esercizi svolti. 117

u = (−δ , −η, η, δ ) δ , η ∈ R.

Uguagliando le due scritture abbiamo



 α − β = −δ

α = −η


 β =η
γ =δ

ovvero 

 α −β +δ = 0
α +η = 0


 β −η = 0
γ − δ = 0.

Sappiamo giá che 4 dei precedenti 5 parametri α, β , γ, δ , η dipenderanno dal quin-


to. Infatti, poiché dim(V ∩W ) = 1, nell’espressione del generico vettore u ∈ V ∩W
comparirá soltanto un parametro libero .
Sebbene la soluzione del precedente sistema sia banale, preferiamo discuterlo tra-
mite la matrice associata, affinché possa essere esemplificativo di come si debba
lavorare in generale. Nello specifico, analizzare la matrice é utile per comprendere
quali siano i parametri liberi e quelli dipendenti.
Costruiamo la matrice associata al sistema lineare omogeneo nelle incognite α, β , γ, δ , η:
 
1 −1 0 1 0
1 0 0 0 1 
 0 1 0 0 −1  .
 

0 0 1 −1 0

Sappiamo giá di doverci aspettare un rango pari a 4 (Rouché-Capelli), per cui


osserviamo semplicemente che il minore formato dalle prime 4 colonne é

1 −1 0 1

1 0 0 0
0 1 0 0 6= 0.


0 0 1 −1

Quindi sceglieremo η come parametro libero, in quanto corrispondente ai termini


posti in quinta colonna. Adesso possiamo esprimere u ∈ V ∩W , in funzione di η ∈
R:
α = −η, β = η, γ = 2η, δ = 2η.
Da ció segue che

u ∈ V ⇒ u = (−2η, −η, η, 2η) η ∈ R

ed analogamente
118 4 Gli Spazi Vettoriali.

u ∈ W ⇒ u = (−2η, −η, η, 2η) η ∈ R.

Le due forme ovviamente coincidono, ma la verifica andava eseguita, per esser certi
del risultato.
Tale verifica ci consente di asserire che V ∩W =< (−2, −1, 1, 2) >.
Concludiamo infine che V +W non é somma diretta.
Esercizio 39. Nello spazio vettoriale R4 si considerino i seguenti sottospazi:

V = {(x, y, z,t) ∈ R4 : x − y + z = 0}

W = {(x, y, z,t) ∈ R4 : 2x + t = 0}
Si determini una base per ciascuno degli spazi V +W e V ∩W .

Svolgimento: Lo spazio vettoriale V é identico a quello introdotto nell’esercizio


precedente, per cui, un qualsiasi vettore di V si esprime

(α − β , α, β , γ) α, β , γ ∈ R.

La dimensione di V é pari a 3 ed una sua base é

BV = {(1, 1, 0, 0), (−1, 0, 1, 0), (0, 0, 0, 1)}.

Passiamo al secondo sottospazio: un vettore in W é tale che le sue componenti


(x, y, z,t) debbano soddisfare alla relazione t = −2x. Per cui un tale vettore ha la
seguente forma:
(δ , η, θ , −2δ ) δ , η, θ ∈ R.
La dimensione di W é pari a 3 ed una sua base é

BW = {(1, 0, 0, −2), (0, 1, 0, 0), (0, 0, 1, 0)}.

Questa volta scegliamo di iniziare con la discussione del sottospazio V ∩ W . Le


componenti di un vettore (x, y, z,t) ∈ V ∩W devono soddisfare contemporaneamente
a tutte le condizioni lineari poste nelle definizioni di V e W , per cui

x−y+z = 0
.
2x + t = 0

Tale sistema lineare omogeneo ha una matrice associata


 
1 −1 1 0
2 0 01

di rango certamente 2, quindi prevede ∞2 soluzioni, ovvero uno spazio delle solu-
zioni di dimensione 2. Tale spazio delle soluzioni é appunto V ∩ W . La scelta dei
2 parametri liberi puó essere diversificata. La nostra ricade su x, y, poiché facciamo
riferimento al minore non nullo
4.7 Esercizi svolti. 119

1 0
0 1 =
6 0

proveniente dalle ultime due colonne della matrice del sistema. Abbiamo allora che
t = −2x, z = y − x. Quindi il generico vettore di V ∩W si puó scrivere

(α, β , β − α, −2α) α, β ∈ R

da cui
V ∩W =< (1, 0, −1, −2), (0, 1, 1, 0) > .
La somma V +W non é diretta.
Passiamo adesso alla somma dei due sottospazi vettoriali. Consideriamo BV ∪ BW =
{(1, 1, 0, 0), (−1, 0, 1, 0), (0, 0, 0, 1), (1, 0, 0, −2), (0, 1, 0, 0), (0, 0, 1, 0)}. Tale insie-
me genera V + W , ma chiaramente non potrá esserne una base, visto che V + W é
sottospazio di R4 . Studiamo il rango della matrice formata dalle componenti, poste
in riga, dei vettori generatori di V +W :
 
1 10 0
 −1 0 1 0 
 
 0 00 1 
 1 0 0 −2  .
 
 
 0 10 0 
0 01 0

La formula di Grassmann e la conoscenza della dim(V ∩W ) = 2 suggeriscono che


il rango della matrice precedente (ovvero la dimensione della somma) debba essere
4. Un minore non nullo di ordine 4 é dato da

1 1 0 0

−1 0 1 0
0 0 0 1 6= 0.


0 0 1 0

Ció implica che

V +W =< (1, 1, 0, 0), (−1, 0, 1, 0), (0, 0, 0, 1), (0, 0, 1, 0) > .

Esercizio 40. Nello spazio vettoriale R5 si considerino i seguenti sottospazi:

V =< (1, 1, 0, 1, 0), (1, 1, 2, 1, 2), (2, 1, 0, 0, 0) >

W = {(x1 , x2 , x3 , x4 , x5 ) ∈ R5 |x3 − x5 = 0, x1 + x2 − 2x4 = 0}.


Si determinino una base per lo spazio intersezione V ∩ W ed una per lo spazio
somma V +W .
120 4 Gli Spazi Vettoriali.

Svolgimento: Un qualsiasi vettore di V si esprime

α(1, 1, 0, 1, 0) + β (1, 1, 2, 1, 2) + γ(2, 1, 0, 0, 0) = (α + β + 2γ, α + β + γ, 2β , α + β , 2β )


α, β , γ ∈ R.

Un qualsiasi vettore di W , le cui componenti soddisfano le relazioni x3 = x5 , x1 =


−x2 + 2x4 , si esprime

(2δ − η, η, θ , δ , θ ), δ , η, θ ∈ R

per cui
W =< (2, 0, 0, 1, 0), (−1, 1, 0, 0, 0), (0, 0, 1, 0, 1) > .
Iniziamo analizzando V ∩W . Scegliendo un vettore u ∈ V ∩W , esso si potrá espri-
mere in entrambi i modi precedentemente descritti. Tali espressioni devono quindi
coincidere, ovvero

(α + β + 2γ, α + β + γ, 2β , α + β , 2β ) = (2δ − η, η, θ , δ , θ ). (4.2)

Uguagliano le componenti omologhe abbiamo




 α + β + 2γ = 2δ − η
α +β +γ = η


 2β = θ
α +β = δ

che si traduce in un sistema lineare omogeneo nelle 6 incognite (α, β , γ, δ , η, θ ):




 α + β + 2γ − 2δ + η = 0
α +β +γ −η = 0


 2β − θ = 0
α +β −δ = 0

la cui matrice associata é  


1 1 2 −2 1 0
 1 1 1 0 −1 0 
 0 2 0 0 0 −1  .
 

1 1 0 −1 0 0
Riduciamola per righe: eseguiamo dapprima le operazioni R2 → R2 − R1 e R4 →
R4 − R1 , in seguito alle quali otteniamo
 
1 1 2 −2 1 0
 0 0 −1 2 −2 0 
 0 2 0 0 0 −1  .
 

0 0 −2 1 −1 0

Scambiamo adesso la seconda e terza riga:


4.7 Esercizi svolti. 121
 
1 1 2 −2 1 0
 0 2 0 0 0 −1 
 
 0 0 −1 2 −2 0 
0 0 −2 1 −1 0

e su quest’ultima matrice eseguiamo l’operazione R4 → R4 − 2R3 , ottenendo


 
1 1 2 −2 1 0
 0 2 0 0 0 −1 
 0 0 −1 2 −2 0  .
 

0 0 0 −3 3 0

La matrice ha quindi rango 4. Un minore non nullo del quarto ordine é fornito dalle
prime 4 colonne
1 1 2 −2

0 2 0 0
0 0 −1 2 6= 0


0 0 0 −3
per cui riscriviamo un sistema equivalente, nel quale η, θ sono parametri liberi:


 α + β + 2γ − 2δ = −η
2β = θ


 −γ + 2δ = 2η
−3δ = −3η

da cui segue
1 1
δ = η, β = θ, α = η − θ.
γ = 0,
2 2
Sostituendo tali valori in uno qualsiasi dei due membri della relazione (4.2) ottenia-
mo l’espressione del generico vettore di V ∩W :

(η, η, θ , η, θ )

per cui dim(V ∩W ) = 2 e V ∩W =< (1, 1, 0, 1, 0), (0, 0, 1, 0, 1) >.


Dall’applicazione della formula di Grassmann, segue che dim(V + W ) = 4. Al so-
lito, per ottenerne una base, uniamo le due distinte basi di V e W e, nell’insieme
ricavato, individuiamo 4 vettori linearmente indipendenti. L’unione delle due basi
genera il seguente insieme di vettori

{(1, 1, 0, 1, 0), (1, 1, 2, 1, 2), (2, 1, 0, 0, 0), (2, 0, 0, 1, 0), (−1, 1, 0, 0, 0), (0, 0, 1, 0, 1)}.

I primi 3 vettori sono tra loro linearmente indipendenti poiché costituiscono una ba-
se per V . Per concludere é allora sufficiente individuare uno dei 3 successivi vettori
(la base di W ), in modo tale che sia linearmente indipendente con i precedenti.
É di facile verifica il fatto che tale vettore puó essere (2, 0, 0, 1, 0), infatti la matrice
122 4 Gli Spazi Vettoriali.
 
1 1 0 1 0
1 1 2 1 2
 
2 1 0 0 0
2 0 0 1 0

ha rango 4, poiché possiede il seguente minore non nullo di ordine 4:



1 1 0 0

1 1 2 1
2 1 0 0 6= 0.


2 0 0 1

Quindi l’insieme

{(1, 1, 0, 1, 0), (1, 1, 2, 1, 2), (2, 1, 0, 0, 0), (2, 0, 0, 1, 0)}

é linearmente indipendente e costituisce una base per V +W .


Esercizio 41. Determinare somma ed intersezione dei seguenti sottospazi di M2 (R):
  
a b
V= |a, b, c ∈ R
2a c
   
11 21
W =< , >
10 01

Svolgimento: Lo spazio vettoriale delle matrici M2 (R) é isomorfo allo spazio R4 :


ogni matrice quadrata di ordine 2 é infatti una 4-upla, opportunamente ordinata, di
numeri reali:  
a11 a12 
Longrightarrow a11 , a12 , a21 , a22 .
a21 a22
Per semplificare, riscriviamo quindi entrambi gli spazi vettoriali nella seguente
forma:
V = {(a, b, 2a, c)|a, b, c ∈ R}
da cui
V =< (1, 0, 2, 0), (0, 1, 0, 0), (0, 0, 0, 1) >
e
W =< (1, 1, 1, 0), (2, 1, 0, 1) > .
Un qualsiasi vettore di W si esprime quindi nella forma

d(1, 1, 1, 0) + e(2, 1, 0, 1) = (d + 2e, d + e, d, e) d, e ∈ R.

Iniziamo analizzando V ∩W . Scegliendo un vettore u ∈ V ∩W , esso si potrá espri-


mere in entrambi i modi precedentemente descritti. Tali espressioni devono quindi
coincidere, ovvero
4.7 Esercizi svolti. 123

(a, b, 2a, c) = (d + 2e, d + e, d, e). (4.3)


Uguagliano le componenti omologhe abbiamo


 a = d + 2e
b = d +e


 2a = d
c=e

che si traduce in un sistema lineare omogeneo nelle 5 incognite (a, b, c, d, e):




 a − d − 2e = 0
b−d −e = 0


 2a − d = 0
c−e = 0

la cui matrice associata é  


1 0 0 −1 −2
0
 1 0 −1 −1 
.
2 0 0 −1 0 
0 0 1 0 −1
Operando sulla matrice tramite R3 → R3 − 2R1 otteniamo la seguente:
 
1 0 0 −1 −2
 0 1 0 −1 −1 
 0 0 0 1 4 .
 

0 0 1 0 −1

É sufficiente scambiare la terza e la quarta riga per avere una forma ridotta a gradini:
 
1 0 0 −1 −2
 0 1 0 −1 −1 
 
 0 0 1 0 −1 
000 1 4

il cui rango é 4 ed é fornito, ad esempio, dal minore non nullo di ordine 4 indivi-
duato dalle prime sue 4 colonne. Scegliamo allora e ∈ R come parametro libero e
riscriviamo il sistema: 

 a − d = 2e
b−d = e


 c=e
d = −4e

da cui
d = −4e, c = e, b = −3e, a = −2e.
Sostituendo tali valori in uno qualsiasi dei due membri della relazione (4.3) ottenia-
mo l’espressione del generico vettore di V ∩W :
124 4 Gli Spazi Vettoriali.

(−2e, −3e, e, −4e)

per cui dim(V ∩W ) = 1 e V ∩W =< (−2, −3, −4, 1) >. Volendo riscrivere il tutto
sotto forma matriciale avremo
    
−2e −3e −2 −3
V ∩W = |e ∈ R = .
−4e e −4 1

Applicando la formula di Grassmann, abbiamo inoltre che dim(V +W ) = 4, per cui


V +W = M2 (R) (osserviamo anche che la somma non é diretta).
Esercizio 42. Determinare somma ed intersezione dei seguenti sottospazi di R5 [X]:

V =< 3x4 + x2 + x + 1, 2x4 + x + 1 >, W =< 2x4 + 2, 4x4 + x2 + 2x + 1 >

Svolgimento: Lo spazio vettoriale dei polinomi R5 [X] di grado ≤ 5 é isomorfo


allo spazio R6 : ogni polinomio della forma a0 + a1 x + a2 x2 + a3 x3 + a4 x4 + a5 x5
é infatti esprimibile tramite una 6-upla di numeri reali (a0 , a1 , a2 , a3 , a4 , a5 ): es-
si sono i coefficienti dei monomi ordinati dal grado minore al maggiore, ovvero
le componenti del vettore a0 + a1 x + a2 x2 + a3 x3 + a4 x4 + a5 x5 rispetto alla base
{1, x, x2 , x3 , x4 , x5 } di R5 [X]. Per semplificare, riscriviamo quindi entrambi gli spazi
vettoriali nella seguente forma:

V =< (1, 1, 1, 0, 3, 0), (1, 1, 0, 0, 2, 0) >

e
W =< (2, 0, 0, 0, 2, 0), (1, 2, 1, 0, 4, 0) > .
Un qualsiasi vettore di V si esprime quindi nella forma

a(1, 1, 1, 0, 3, 0) + b(1, 1, 0, 0, 2, 0) = (a + b, a + b, a, 0, 3a + 2b, 0) a, b ∈ R

ed un qualsiasi vettore di W si esprime nella forma

c(2, 0, 0, 0, 2, 0) + d(1, 2, 1, 0, 4, 0) = (2c + d, 2d, d, 0, 2c + 4d, 0) c, d ∈ R.

Iniziamo analizzando V ∩W . Scegliendo un vettore u ∈ V ∩W , esso si potrá espri-


mere in entrambi i modi precedentemente descritti. Tali espressioni devono quindi
coincidere, ovvero

(a + b, a + b, a, 0, 3a + 2b, 0) = (2c + d, 2d, d, 0, 2c + 4d, 0). (4.4)

Uguagliano le componenti omologhe abbiamo




 a + b = 2c + d
a + b = 2d


 a=d
3a + 2b = 2c + 4d

4.7 Esercizi svolti. 125

che si traduce in un sistema lineare omogeneo nelle 4 incognite (a, b, c, d):




 a + b − 2c − d = 0
a + b − 2d = 0


 a−d = 0
3a + 2b − 2c − 4d = 0

la cui matrice associata é  


1 1 −2 −1
1
 1 0 −2 .
1 0 0 −1 
3 2 −2 −4
Operando sulla matrice tramite

R2 → R2 − R1 , R3 → R3 − R1 , R4 → R4 − 3R1

otteniamo la seguente:  
1 1 −2 −1
0 0
 2 −1 
.
 0 −1 2 0 
0 −1 4 −1
Scambiamo ora seconda e quarta riga
 
1 1 −2 −1
 0 −1 4 −1 
 
 0 −1 2 0 
0 0 2 −1

ed operiamo tramite R3 → R3 − R2 :
 
1 1 −2 −1
 0 −1 4 −1 
 0 0 −2 1  .
 

0 0 2 −1

Infine é sufficiente operare tramite R4 → R4 + R3 per ottenere la forma ridotta


 
1 1 −2 −1
 0 −1 4 −1 
 
 0 0 −2 1 
0 0 0 0

il cui rango é 3 ed é fornito, ad esempio, dal minore non nullo di ordine 3 individuato
dall’incrocio delle sue prime 3 righe e delle colonne C1 ,C2 ,C4 . Scegliamo allora
c ∈ R come parametro libero e riscriviamo il sistema:
126 4 Gli Spazi Vettoriali.

 a + b − d = 2c
−b − d = −4c
d = 2c

da cui
d = 2c, b = 2c, a = 2c.
Sostituendo tali valori in uno qualsiasi dei due membri della relazione (4.4) ottenia-
mo l’espressione del generico vettore di V ∩W :

(4c, 4c, 2c, 0, 10c, 0)

per cui dim(V ∩ W ) = 1 e V ∩ W =< (2, 2, 1, 0, 5, 0) >. Volendo riscrivere il tutto


sotto forma polinomiale avremo

V ∩W = 2 + 2x + x2 + 5x4 .

Applicando la formula di Grassmann, abbiamo inoltre che dim(V +W ) = 3. Per de-


terminarne una base di V +W partiamo dalla base {(1, 1, 1, 0, 3, 0), (1, 1, 0, 0, 2, 0)}
di V . Osserviamo semplicemente che il vettore (2, 0, 0, 0, 2, 0), scelto nella base di
W , é linearmente indipendente dai due precedenti, per cui {(1, 1, 1, 0, 3, 0), (1, 1, 0, 0, 2, 0), (2, 0, 0, 0, 2, 0)}
é una base di V +W . In termini polinomiali avremo:

V +W = 1 + x + x2 + 3x4 , 1 + x + 2x4 , 2 + 2x4 .




Capitolo 5
Prodotti scalari, basi ortonormali e proiezioni
ortogonali.

5.1 Prodotto interno.

In tutto ció che segue, sia V uno spazio vettoriale reale di dimensione n, senza
perdita di generalitá poniamo V = Rn . Introduciamo una base B = {e1 , . . . , en } di V
e denotiamo con    
x1 y1
 x2   y2 
   
X = ... , Y =  ... 
  
 ...   ... 
xn yn
due qualsiasi vettori in V , espressi in componenti rispetto alla base B.
Definizione 5.1. Si chiama prodotto interno o scalare, una qualsiasi funzione f :
V ×V → R con le proprietá:
1. f (X, X) ≥ 0, per ogni X ∈ Rn e f (X, X) = 0 se e solo se X = 0;
2. f (X,Y ) = f (Y, X), per ogni X,Y ∈ Rn ;
3. f (αX + βY, Z) = α f (X, Z) + β f (Y, Z), per ogni X,Y, Z ∈ Rn e α, β ∈ R.
Esempio 82. Un particolare prodotto interno, chiamato prodotto scalare standard é
il seguente:
n
f (X,Y ) = X T Y = x1 y1 + x2 y2 + . . . + xn yn = ∑ xi yi
i=1

nella quale scrittura indichiamo con X T = [x1 , . . . , xn ] il vettore trasposto di X.


A margine dell’esempio appena esposto, osserviamo che, se A ∈ Mn (R) é una
matrice quadrata reale di ordine n, allora

f (AX,Y ) = (AX)T Y = X T AT Y = f (X, AT Y ).

In particolare notiamo che, se A é simmetrica, vale la seguente:

127
128 5 Prodotti scalari, basi ortonormali e proiezioni ortogonali.

f (AX,Y ) = f (X, AY ).

Definizione 5.2. Una norma di uno spazio vettoriale reale é una funzione q : V → R
che abbia le seguenti proprietá:
1. q(X) ≥ 0, per ogni X ∈ V e q(X) = 0 se e solo se X = 0;
2. q(αX) = |α|q(X), per ogni X ∈ V e α ∈ R;
3. q(X +Y ) ≤ q(X) + q(Y ), per ogni X,Y ∈ V .
Se f : V × V → R é il prodotto scalare standard, allora possiamo introdurre una
particolare norma, detta norma euclidea, nel modo seguente:
s
p n
q(X) = ||X|| = f (X, X) = ∑ xi2 .
i=1

Si noti che ||X|| ≥ 0, per ogni vettore X di V , e ||X|| = 0 solo se X = 0.


Inoltre, per ogni α ∈ R, ||αX|| = |α| · ||X||.

Definizione 5.3. Uno spazio vettoriale reale V che sia dotato di un prodotto scalare,
é chiamato spazio euclideo.
Definizione 5.4. Sia f : V × V → R un prodotto scalare. Diremo che i due vettori
X,Y ∈ V sono ortogonali rispetto a f in V , se f (X,Y ) = 0.
Dalla definizione di prodotto scalare, segue che se f (X,Y ) = 0 allora f (αX, βY ) =
0, per ogni α, β ∈ R.
Osservazione 5.5. Siano X,Y ∈ V due vettori ortogonali ed entrambi non nulli.
Segue facilmente che essi sono anche linearmente indipendenti.
Infatti, supponendo che esistano α, β ∈ R (non nulli) tali che αX + βY = 0, allora
otterremmo che

0 = f (αX + βY, X) = α f (X, X) + β f (Y, X) = α f (X, X)

da cui, essendo X 6= 0, seguirebbe la contraddizione α = 0.


Definizione 5.6. Un vettore si dirá normalizzato se la sua norma é pari a 1.
Osservazione 5.7. In generale, dato un vettore qualsiasi X ∈ Rn qualsiasi, é sem-
pre possibile normalizzarlo, cioé costruire un suo multiplo con norma pari a 1. É
sufficiente infatti dividere ogni componente di X per ||X||.
Definizione 5.8. Un insieme di vettori tutti non nulli S = {X1 , . . . , Xm } é detto in-
sieme ortogonale di vettori se f (Xi , X j ) = 0, per ogni Xi , X j ∈ S con i 6= j.
S é detto insieme ortonormale di vettori se é un insieme ortogonale ed inoltre
f (Xi , Xi ) = 1, per ogni i = 1, . . . , m. In altre parole tutti i vettori di S devono ave-
re norma unitaria.
Una base B = {e1 , . . . , en } di V é detta base ortonormale di V se essa é un insieme
ortonormale di vettori.
5.2 Costruzione di basi ortonormali. 129

Osservazione 5.9. Se {X1 , . . . , Xm } é un insieme ortogonale di vettori, allora


 
X1 Xm
,...,
||X1 || ||Xm ||

é un insieme ortonormale di vettori.

5.2 Costruzione di basi ortonormali.

Il seguente risultato ci consente di ottenere una base di uno spazio vettoriale formata
da vettori ortogonali (Algoritmo di Gram-Schmidt).

Teorema 5.10. Siano v1 , . . . , vm vettori dello spazio euclideo V , f : V ×V → R sia


il prodotto scalare definito in V . Allora esiste un insieme di vettori w1 , . . . , wm di V
tali che
1. Span{v1 , . . . , vm } = Span{w1 , . . . , wm }.
2. {w1 , . . . , wm } é un insieme di vettori ortogonali tra loro.
Vediamo come costruire i vettori {w1 , . . . , wm } che soddisfino alle condizioni 1 e 2
del Teorema 5.10.
Dapprima notiamo che se v, w ∈ V , allora il vettore w− ff(v,w)
(v,v) v é ortogonale al vettore
v infatti:
f (v, w)  f (v, w)
f v, w − v = f (v, w) − f (v, v) = f (v, w) − f (v, w) = 0.
f (v, v) f (v, v)

Partiamo quindi dai vettori della base iniziale {v1 , . . . , vm } e definiamo:

w1 = v1

f (w1 , v2 )
w2 = v2 − w1
f (w1 , w1 )
f (w1 , v3 ) f (w2 , v3 )
w3 = v3 − w1 − w2
f (w1 , w1 ) f (w2 , w2 )
f (w1 , v4 ) f (w2 , v4 ) f (w3 , v4 )
w4 = v4 − w1 − w2 − w3
f (w1 , w1 ) f (w2 , w2 ) f (w3 , w3 )
ed in generale
i
f (w j , vi+1 )
wi+1 = vi+1 − ∑ wj
j=1 f (w j , w j )

I vettori ottenuti {w1 , . . . , wn } costituiscono una base ortogonale per V .


130 5 Prodotti scalari, basi ortonormali e proiezioni ortogonali.

Osservazione 5.11. Come immediata conseguenza otteniamo che, se {v1 , . . . , vn } é


una base di V allora esistono w1 , . . . , wn tali che
 
w1 wn
,...,
||w1 || ||wn ||

sia una base ortonormale di V .


Osservazione 5.12. Sia {X1 , . . . , Xn } una base ortonormale di Rn . Per quanto
precedentemente detto
XiT Xi = 1 ∀i = 1, . . . , n
e
XiT X j = 0 ∀i 6= j.
Ció implica che la matrice C ∈ Mn (R), costruita ponendo ordinatamente in riga
(oppure in colonna) le componenti dei vettori X1 , . . . , Xn , é ortogonale.
Viceversa, sia C ∈ Mn (R) una matrice ortogonale. Dal fatto che CT C = CCT = I
deriva che l’insieme dei vettori riga (ed analogamente l’insieme dei vettori colonna)
costituisce una base ortonormale di Rn .

5.3 Complemento ortogonale e proiezione ortogonale.

Iniziamo con la seguente:


Definizione 5.13. Siano V = Rn con prodotto scalare f (X,Y ) = X T Y e sia 0 6= W
un sottospazio di V . Definiamo complemento ortogonale di W in V , il seguente
sottospazio di V

W ⊥ = {v ∈ V : f (v, w) = 0, ∀w ∈ W }.

Valgono le seguenti:
1. W ⊥ é un sottospazio di V . Infatti, per ogni X1 , X2 ∈ W ⊥ e per ogni α, β ∈ R
avremo che, per qualsiasi Y ∈ V :

Y T (αX1 + β X2 ) = αY T X1 + βY T X2 = 0

cioé αX1 + β X2 ∈ W ⊥ .
2. W ∩ W ⊥ = {0}, cioé W e W ⊥ sono in somma diretta. Infatti, se X ∈ W ∩ W ⊥ ,
allora segue che X T X = 0, da cui X = 0.
3. W ⊕ W ⊥ = V . Per dimostrarlo, scegliamo una base B = {e1 , . . . , ek } di W , con
k < n (il caso k = n sarebbe banale, poiché significherebbe W = V e W ⊥ = {0}).
Esprimiamo ogni vettore della base B tramite le sue componenti in V :

e1 = (a11 , a12 , . . . , a1n ), . . . , . . . , . . . , ek = (ak1 , . . . , akn ).


5.3 Complemento ortogonale e proiezione ortogonale. 131

Sia ora X = (x1 , . . . , xn ) un qualsiasi vettore di W ⊥ , quindi ortogonale con


ciascuno dei vettori e1 , . . . , ek . Per determinare le componenti x1 , . . . , xn di X,
esplicitiamo la ortogonalitá tramite l’annullarsi dei prodotti scalari f (e1 , X) =
0,. . . , f (ek , X) = 0. Otteniamo il seguente sistema lineare omogeneo nelle inco-
gnite x1 , . . . , xn : 

 a11 x1 + a12 x2 + . . . + a1n xn = 0
a21 x1 + a22 x2 + . . . + a2n xn = 0

.

 ... ... ...
ak1 x1 + am2 x2 + . . . + akn xn = 0

Il sistema ha ovviamente rango k, poiché le k righe della matrice associata sono


linearmente indipendenti (in quanto componenti dei vettori e1 , . . . , ek ). Quindi la
dimensione dello spazio delle soluzioni é n − k, ovvero dim(W ⊥ ) = n − k, da cui
dim(W ⊕W ⊥ ) = n.
Definizione 5.14. Alla luce di quanto precedentemente detto, dato un sottospazio
W di V , ogni elemento v ∈ V = Rn si puó esprimere in modo unico come v = w + w0 ,
dove w ∈ W e w0 ∈ W ⊥ , poiché V = W ⊕W ⊥ si esprime come somma diretta.
In tale decomposizione, il vettore w ∈ W é detto proiezione ortogonale di v sullo
spazio W .
Vogliamo considerare adesso il caso in cui {e1 , . . . , ek } sia una base ortogonale di W
e {c1 , . . . , cn−k } una qualsiasi base di W ⊥ . Come detto precedentemente, possiamo
scrivere in modo unico:

v = (α1 e1 + · · · + αk ek ) + (β1 c1 + · · · + βn−k cn−k )

per opportuni scalari α1 , . . . , αk , β1 , . . . , βn−k ∈ R, ovvero le componenti del vettore


v rispetto alla nuova base {e1 , . . . , ek , c1 , . . . , cn−k } di V . Aabbiamo dato forma alla
decomposizione v = w + w0 , determinando w = α1 e1 + · · · + αk ek ∈ W e w0 = β1 c1 +
· · · + βn−k cn−k ∈ W ⊥ .
L’eventualitá di considerare una base ortogonale di W , ci consente di esprimere una
formula analitica per il calcolo della proiezione ortogonale di un vettore su W :
Teorema 5.15. Siano V uno spazio vettoriale euclideo di dimensione n, f : V ×V →
R il prodotto scalare introdotto, W un sottospazio di V di dimensione k e {e1 , . . . , ek }
una base ortogonale di W .
Dato il vettore v ∈ V , la proiezione ortogonale w di v su W si ottiene nel modo
seguente:
f (v, e1 ) f (v, ek )
w= e1 + . . . . . . + ek .
f (e1 , e1 ) f (ek , ek )
Nota 5.16. Riassumendo, dati uno spazio vettoriale V , un suo sottospazio W ed un
vettore v ∈ V , possiamo determinare la proiezione di v su W in due distinti modi:
1. Determinare una base ortogonale di W ed utilizzare la formula espressa nel
Teorema 5.15.
2. Determinare W ⊥ ed esprimere v = w + w0 , tali che w ∈ W e w0 ∈ W ⊥ .
132 5 Prodotti scalari, basi ortonormali e proiezioni ortogonali.

5.4 Esercizi svolti.

Esercizio 43. Siano V = R4 , con prodotto scalare standard. Siano v1 = (0, 1, 0, 1),
v2 = (2, 1, 0, 1), v3 = (−1, 0, 0, 1), v4 = (0, 0, 1, 0).
B = {v1 , v2 , v3 , v4 } é una base di V ma non é ortogonale. A partire da essa costruiamo
una base ortogonale.

Svolgimento:
w1 = v1 = (0, 1, 0, 1)
f (w1 , v2 )
w2 = v2 − w1 = v2 − v1 = (2, 0, 0, 0)
f (w1 , w1 )
f (w1 , v3 ) f (w2 , v3 ) 1 1 1 1
w3 = v3 − w1 − w2 = v3 − (0, 1, 0, 1)+ (2, 0, 0, 0) = (0, − , 0, )
f (w1 , w1 ) f (w2 , w2 ) 2 2 2 2
f (w1 , v4 ) f (w2 , v4 ) f (w3 , v4 )
w4 = v4 − w1 − w2 − w3 = v4 = (0, 0, 1, 0).
f (w1 , w1 ) f (w2 , w2 ) f (w3 , w3 )
I vettori w1 , w2 , w3 , w4 costituiscono una base ortogonale per V e quindi i vettori
w1 1 1
= (0, √ , 0, √ )
||w1 || 2 2
w2
= (1, 0, 0, 0)
||w2 ||
w3 1 1
= (0, − √ , 0, √ )
||w3 || 2 2
w4
= (0, 0, 1, 0)
||w4 ||
costituiscono una base ortonormale per V .
Esercizio 44. Siano V = R3 , v1 = (1, 1, 1), v2 = (0, 0, 1), v3 = (1, 0, 1). B =
{v1 , v2 , v3 } una base di V , che non é ortogonale rispetto al prodotto scalare standard.
A partire da essa costruiamo una base ortogonale.

Svolgimento:
w1 = v1 = (1, 1, 1)
f (w1 , v2 ) 1 1 2
w2 = v2 − w1 = (− , − , )
f (w1 , w1 ) 3 3 3
f (w1 , v3 ) f (w2 , v3 ) 1 1
w3 = v3 − w1 − w2 = ( , − , 0)
f (w1 , w1 ) f (w2 , w2 ) 2 2
allora B0 = {w1 , w2 , w3 } é una base ortogonale. Inoltre
p √
||w1 || = f (w1 , w1 ) = 3
5.4 Esercizi svolti. 133

p 6
||w2 || = f (w2 , w2 ) =
3
p 1
||w3 || = f (w3 , w3 ) = √
2
quindi i vettori
w1 1 1 1
= (√ , √ , √ )
||w1 || 3 3 3
w2 1 1 2
= (− √ , − √ , √ )
||w2 || 6 6 6
w3 1 1
= ( √ , − √ , 0)
||w3 || 2 2
costituiscono una base ortonormale per V .
Osservazione 5.17. Il Teorema 5.10 fornisce quindi una dimostrazione dell’esi-
stenza di una base ortogonale per qualsiasi spazio vettoriale euclideo e qualsiasi suo
sottospazio.
Vogliamo infine suggerire un metodo per la determinazione di una base ortogonale,
che prescinda dall’algoritmo descritto precedentemente. A tal fine, proponiamo un
paio di ulteriori esempi.
Esercizio 45. Siano V = R4 e W = {(x1 , x2 , x3 , x4 ) ∈ V : x1 + x2 − x3 = 0}. Si
determini una base ortonormale di W .

Svolgimento: Un generico vettore di W puó essere scritto (α, β , α + β , γ), al


variare di α, β , γ ∈ R. Una base di W é quindi

B = {(1, 0, 1, 0), (0, 1, 1, 0), (0, 0, 0, 1)}

ed é ovviamente costituita da vettori non ortogonali tra loro. Per costruire una base
B0 = {w1 , w2 , w3 } di W , formata da vettori ortogonali, dapprima fissiamo arbitraria-
mente il primo, scegliendolo nella precedente base B: sia w1 = (1, 0, 1, 0).
Quindi determiniamo il secondo vettore w2 = (α, β , α +β , γ) ∈ W , tale che f (w1 , w2 ) =
0, ovvero
2α + β = 0 =⇒ w2 = (α, −2α, −α, γ), ∀α, γ ∈ R.
Possiamo quindi scegliere w2 = (1, −2, −1, 0) ∈ W , ortogonale a w1 (cioé per α = 1
e γ = 0).
Il terzo vettore dovrá quindi essere w3 = (α 0 , β 0 , α 0 +β 0 , γ 0 ) ∈ W , tale che f (w1 , w3 ) =
0 e f (w2 , w3 ) = 0, ovvero

2α 0 + β 0 = 0, −3β 0 = 0 =⇒ w3 = (0, 0, 0, γ 0 ), ∀γ 0 ∈ R.

Per cui possiamo scrivere w3 = (0, 0, 0, 1) ∈ W , ortogonale a w1 e w2 (per γ 0 = 1).


I vettori w1 , w2 , w3 costituiscono una base ortogonale per W e quindi i vettori
134 5 Prodotti scalari, basi ortonormali e proiezioni ortogonali.

w1 1 1
= ( √ , 0, √ , 0)
||w1 || 2 2
w2 1 2 1
= (( √ , − √ , − √ , 0)
||w2 || 6 6 6
w3
= (0, 0, 0, 1)
||w3 ||
costituiscono una base ortonormale per W .
Esercizio 46. Siano V = R5 e

W =< (1, 1, 0, 1, 0), (2, 0, 1, 1, 0), (0, 0, 1, 0, 0), (1, 1, 1, 0, 1) > .

un suo sottospazio. Si determini una base ortonormale di W .

Svolgimento: Lo svolgiamo utilizzando entrambi i metodi precedentemente de-


scritti.
Metodo 1.
Indichiamo v1 = (1, 1, 0, 1, 0), v2 = (2, 0, 1, 1, 0), v3 = (0, 0, 1, 0, 0), v4 = (1, 1, 1, 0, 1)
e scegliamo w1 = v1 = (1, 1, 0, 1, 0).
Otteniamo
f (w1 , v2 )
w2 = v2 − w1 = v2 = (1, −1, 1, 0, 0)
f (w1 , w1 )
f (w1 , v3 ) f (w2 , v3 ) 1 1 2
w3 = v3 − w1 − w2 = (− , , , 0, 0)
f (w1 , w1 ) f (w2 , w2 ) 3 3 3
f (w1 , v4 ) f (w2 , v4 ) f (w3 , v4 ) 1 1 2
w4 = v4 − w1 − w2 − w3 = ( , , 0, − , 1).
f (w1 , w1 ) f (w2 , w2 ) f (w3 , w3 ) 3 3 3
Quindi B0 = {w1 , w2 , w3 } é una base ortogonale. Inoltre
p √
||w1 || = f (w1 , w1 ) = 3
p √
||w2 || = f (w2 , w2 ) = 3

p 2
||w3 || = f (w3 , w3 ) = √
3

p 5
||w4 || = f (w4 , w4 ) = √
3
quindi i vettori
w1 1 1 1
= ( √ , √ , 0, √ , 0)
||w1 || 3 3 3
w2 1 1 1
= ( √ , − √ , √ , 0, 0)
||w2 || 3 3 3
5.4 Esercizi svolti. 135

w3 1 1 2
= (− √ , √ , √ , 0, 0)
||w3 || 6 6 6
w4 1 1 2 3
= ( √ , √ , 0, − √ , √ )
||w4 || 15 15 15 15
costituiscono una base ortonormale per V .
Metodo 2.
Un generico vettore di W puó essere scritto

(α + 2β + δ , α + δ , β + γ + δ , α + β , δ ) ∈ W

al variare di α, β , γ, δ ∈ R. Per costruire una base B0 = {w1 , w2 , w3 , w4 } di W , for-


mata da vettori ortogonali, dapprima fissiamo arbitrariamente il primo, scegliendolo
nella precedente base B: sia w1 = (1, 1, 0, 1, 0).
Quindi determiniamo il secondo vettore w2 ∈ W , tale che f (w1 , w2 ) = 0, ovvero

2
3α + 3β + 2δ = 0 =⇒ α = −β − δ
3
da cui
1 1 2
w2 = (β + δ , −β + δ , β + γ + δ , − δ , δ ), ∀β , γ, δ ∈ R.
3 3 3
Possiamo quindi scegliere β = 1, γ = 0, δ = 0 ed ottenere w2 = (1, −1, 1, 0, 0) ∈ W ,
ortogonale a w1 .
Il terzo vettore dovrá quindi essere w3 ∈ W , tale che f (w1 , w3 ) = 0 e f (w2 , w3 ) = 0,
ovvero
2
3α + 3β + 2δ = 0, 3β + γ + δ = 0 =⇒ α = β + γ, δ = −3β − γ
3
da cui
1 1 2
w3 = (− γ, −2β − γ, −2β , 2β + γ, −3β − γ), ∀β , γ ∈ R.
3 3 3
Scegliendo β = 0 e γ = −3 otteniamo w3 = (1, 1, 0, −2, 3) ∈ W , ortogonale a w1 e
w2 .
Infine l’ultimo vettore w4 ∈ W , dovrá essere tale che f (w1 , w4 ) = 0, f (w2 , w4 ) = 0
e f (w3 , w4 ) = 0, ovvero

3α + 3β + 2δ = 0, 3β + γ + δ = 0, δ = 0 =⇒ α = −β , γ = −3β

da cui
w4 = (β , −β , −2β , 0, 0), ∀β ∈ R.
Scegliendo β = −1 otteniamo w4 = (−1, 1, 2, 0, 0). I vettori w1 , w2 , w3 , w4 costitui-
scono una base ortogonale per W e quindi i vettori
136 5 Prodotti scalari, basi ortonormali e proiezioni ortogonali.

w1 1 1 1
= ( √ , √ , 0, √ , 0)
||w1 || 3 3 3
w2 1 1 1
= ( √ , − √ , √ , 0, 0)
||w2 || 3 3 3
w3 1 1 2 3
= ( √ , √ , 0, − √ , √ )
||w3 || 15 15 15 15
w4 1 1 2
= (− √ , √ , √ , 0, 0)
||w4 || 6 6 6
costituiscono una base ortonormale per W .
Notiamo che, a meno dell’ordine con cui compaiono i vettori, le due basi, ottenute
con i due differenti metodi, coincidono.
Esercizio 47. Siano V = R4 ,

W = {(x1 , x2 , x3 , x4 ) ∈ R4 : x1 + 2x3 = 0, x2 + x4 = 0}.

Si determini la proiezione ortogonale del vettore v = (2, 1, 1, 1) ∈ V sullo spazio W .

Svolgimento: Per prima cosa determiniamo una base di W :

W =< (−2, 0, 1, 0), (0, 1, 0, −1) > .

Osserviamo che tale base é formata da vettori ortogonali, e quindi proseguiamo.


Costruiamo adesso W ⊥ . Sia X = (x1 , x2 , x3 , x4 ) ∈ W ⊥ ed imponiamo l’ortogonalitá
con i vettori della base di W :

−2x1 + x3 = 0, x2 − x4 = 0 =⇒ X = (α, β , 2α, β ) ∀α, β ∈ R.

Quindi W ⊥ =< (1, 0, 2, 0), (0, 1, 0, 1) >, da cui ricaviamo una base per V = R4 come
unione delle basi di W e W ⊥ :
V =< (−2, 0, 1, 0), (0, 1, 0, −1), (1, 0, 2, 0), (0, 1, 0, 1) >=
< (−2, 0, 1, 0), (0, 1, 0, −1) > ⊕ < (1, 0, 2, 0), (0, 1, 0, 1) > .

Determiniamo infine le componenti del vettore v = (2, 1, 1, 1) rispetto alla nuova


base B = {(−2, 0, 1, 0), (0, 1, 0, −1), (1, 0, 2, 0), (0, 1, 0, 1)} di V = R4 . A conti fatti
otterremo
 
3
(2, 1, 1, 1) = (− 5 )(−2, 0, 1, 0) + (0)(0, 1, 0, −1) +
  ∈W
( 54 )(1, 0, 2, 0) + (1)(0, 1, 0, 1) .
∈W ⊥

Le componenti di v = (2, 1, 1, 1) rispetto alla base B sono quindi (− 35 , 0, 45 , 1)B . In


particolare, nella decomposizione avremo
5.4 Esercizi svolti. 137

6 3  4 8 
(2, 1, 1, 1) = , 0, − , 0 ∈W + , 1, , 1 ∈W ⊥
5 5 5 5

per cui la proiezione di v su W é data dal vettore v0 = ( 65 , 0, − 35 , 0) ∈ W .


Esercizio 48. Siano V = R4 e W =< (1, −1, 0, 1), (1, 0, 1, 0) > un suo sottospazio.
Si determini la proiezione ortogonale del vettore v = (2, 1, −1, 1) ∈ V su W .

Svolgimento: Per prima cosa determiniamo una base ortogonale di W . Fissia-


mo il primo vettore e1 = (1, 0, 1, 0). Sia e2 un qualsiasi vettore di W , per cui la
sua espressione sará una combinazione lineare dei vettori della base nota: e2 =
(a + b, −a, b, a), al variare di a, b ∈ R. Imponiamo adesso l’ortogonalitá con e1 ,
per cui a + 2b = 0. Possiamo quindi riscrivere e2 = (−b, 2b, b, −2b) e scegliere
e2 = (1, −2, −1, 2). Partiamo allora dalla base ortogonale

W =< (1, 0, 1, 0), (1, −2, −1, 2) > .

Costruiamo adesso W ⊥ . Sia X = (x1 , x2 , x3 , x4 ) ∈ W ⊥ ed imponiamo l’ortogonalitá


con i vettori della base di W :

x1 + x3 = 0, x1 − 2x2 − x3 + 2x4 = 0 =⇒ X = (x1 , x1 + x4 , −x1 , x4 ) ∀x1 , x4 ∈ R.

Quindi W ⊥ =< (1, 1, −1, 0), (0, 1, 0, 1) >, da cui ricaviamo una base per V = R4
come unione delle basi di W e W ⊥ :
V =< (1, 0, 1, 0), (1, −2, −1, 2), (1, 1, −1, 0), (0, 1, 0, 1) >=
< (1, 0, 1, 0), (1, −2, −1, 2) > ⊕ < (1, 1, −1, 0), (0, 1, 0, 1) > .

Determiniamo infine le componenti (y1 , y2 , y3 , y4 ) del vettore v = (2, 1, −1, 1) ri-


spetto alla nuova base B = {(1, 0, 1, 0), (1, −2, −1, 2), (1, 1, −1, 0), (0, 1, 0, 1)} di
V = R4 , ovvero

(2, 1, −1, 1) = y1 (1, 0, 1, 0) + y2 (1, −2, −1, 2) + y3 (1, 1, −1, 0) + y4 (0, 1, 0, 1)


   
= y1 (1, 0, 1, 0) + y2 (1, −2, −1, 2) + y3 (1, 1, −1, 0) + y4 (0, 1, 0, 1)

da cui y1 = 21 , y2 = 3
= 56 , y4 = 25 . In particolare, nella decomposizione avremo
10 , y3
 
1 3
(2, 1, −1, 1) = 2 (1, 0, 1, 0) + 10 (1, −2, −1, 2) +
  W
6 2
5 (1, 1, −1, 0) + 5 (0, 1, 0, 1)
W⊥

per cui la proiezione ortogonale di v su W é data dal vettore v0 = ( 45 , − 53 , 15 , 35 ) ∈ W .


Esercizio 49. Ripetiamo l’Esercizio precedente, utilizzando quanto detto nel Teo-
rema 5.15.
138 5 Prodotti scalari, basi ortonormali e proiezioni ortogonali.

Siano V = R4 e W =< (1, −1, 0, 1), (1, 0, 1, 0) > un suo sottospazio. Si determini la
proiezione ortogonale del vettore v = (2, 1, −1, 1) ∈ V su W .

Svolgimento: Conosciamo giá una base ortogonale di W

W =< (1, 0, 1, 0), (1, −2, −1, 2) > .

Poniamo e1 = (1, 0, 1, 0) e e2 = (1, −2, −1, 2). Alla luce della formula precedente-
mente dimostrata, avremo che la proiezione di v su W é la seguente:

f (v, e1 ) f (v, e2 )
w= e1 + e2 .
f (e1 , e1 ) f (e2 , e2 )

Osserviamo quindi che

f (e1 , e1 ) = 2, f (e2 , e2 ) = 10, f (v, e1 ) = 1, f (v, e2 ) = 3

da cui
1 3 1 1 3 3 3 3 4 3 1 3
w = e1 + e2 = ( , 0, , 0) + ( , − , − , ) = ( , − , , ).
2 10 2 2 10 5 10 5 5 5 5 5
Capitolo 6
Equazioni cartesiane di un sottospazio vettoriale.

Siano V = Rn e W =< e1 , . . . , ek > un suo sottospazio vettoriale di dimensione k. Le


equazioni cartesiane di W sono le n − k equazioni lineari nelle incognite x1 , . . . , xn
che permettono di individuare le relazioni intercorrenti tra le componenti di un qual-
siasi vettore X = [x1 , . . . , xn ]T ∈ W .
Per ottenere tali relazioni faremo riferimento ad alcune proprietá degli spazi vetto-
riali, giá viste in precedenza. Esponiamo adesso due diversi metodi per determinare
le equazioni cartesiane di un sottospazio vettoriale.

6.1 Equazioni cartesiane ottenute tramite il complemento


ortogonale del sottospazio.

Si costruisca il complemento ortogonale W ⊥ di W , tale che W ⊕ W ⊥ = V , e sia


{c1 , . . . , cn−k } una base di W ⊥ .
Tutti e soli i vettori di W ⊆ V sono quelli ortogonali ad ognuno dei vettori c1 , . . . , cn−k .
Per cui, detto X = [x1 , . . . , xn ]T un qualsiasi vettore di V , esso appartiene a W se e
solo se:
X T · c1 = 0
X T · c2 = 0
......
......
T
X · cn−k = 0.
Esplicitando gli n − k prodotti scalari precedenti, otterremo n − k equazioni lineari
nelle incognite x1 , . . . , xn , ovvero le n − k equazioni cartesiane di W .

139
140 6 Equazioni cartesiane di un sottospazio vettoriale.

6.1.1 Esercizi svolti.

Esercizio 50. Siano V = R4 e W =< (1, 1, 0, 0), (1, 0, 0, 1), (0, 1, 1, 0) > un suo
sottospazio. Si determinino le equazioni cartesiane di W .

Svolgimento: Per prima cosa determiniamo il complemento ortogonale W ⊥ . Sia


Y = (a, b, c, d) ∈ W ⊥ ed imponiamo l’ortogonalitá con i vettori della base di W :

a + b = 0, a + d = 0, b + c = 0 =⇒ Y = (−d, d, −d, d) ∀d ∈ R.

Quindi W ⊥ =< (−1, 1, −1, 1) >. Allora X = [x1 , x2 , x3 , x4 ]T ∈ R4 é un vettore di W


se e solo se é ortogonale al vettore base di W ⊥ , ovvero

−x1 + x2 − x3 + x4 = 0.

Quest’ultima é l’espressione cartesiana di W .


Esercizio 51. Siano V = R4 e W =< (1, 1, 0, 0), (1, 0, 0, 1) > un suo sottospazio. Si
determinino le equazioni cartesiane di W .

Svolgimento: Per prima cosa determiniamo il complemento ortogonale W ⊥ . Sia


Y = (a, b, c, d) ∈ W ⊥ ed imponiamo l’ortogonalitá con i vettori della base di W :

a + b = 0, a + d = 0 =⇒ Y = (a, −a, c, −a) ∀a, c ∈ R.

Quindi W ⊥ =< (1, −1, 0, −1), (0, 0, 1, 0) >. Allora X = [x1 , x2 , x3 , x4 ]T ∈ R4 é un


vettore di W se e solo se é ortogonale a ciascuno dei vettori base di W ⊥ , ovvero

x1 − x2 − x4 = 0
.
x3 = 0

Queste sono le equazioni cartesiane di W .


Esercizio 52. Siano V = R4 e W =< (1, 2, −2, −2), (1, 1, −1, −1) > un suo sotto-
spazio. Si determinino le equazioni cartesiane di W .

Svolgimento: Per prima cosa determiniamo il complemento ortogonale W ⊥ . Sia


Y = (a, b, c, d) ∈ W ⊥ ed imponiamo l’ortogonalitá con i vettori della base di W :

a + 2b − 2c − 2d = 0, a + b − c − d = 0 =⇒ Y = (0, c + d, c, d) ∀a, c ∈ R.

Quindi W ⊥ =< (0, 1, 1, 0), (0, 1, 0, 1) >. Allora X = [x1 , x2 , x3 , x4 ]T ∈ R4 é un vettore


di W se e solo se é ortogonale a ciascuno dei vettori base di W ⊥ , ovvero

x2 + x3 = 0
.
x2 + x4 = 0

Queste sono le equazioni cartesiane di W .


6.2 Equazioni cartesiane ottenute tramite lo studio del rango di una matrice. 141

6.2 Equazioni cartesiane ottenute tramite lo studio del rango di


una matrice.

Siano
e1 = (a11 , a21 , . . . , an1 )
e2 = (a12 , a22 , . . . , an2 )
... ... ...
ek = (a1k , a2k , . . . , ank )
le componenti dei vettori della base di W , e sia X = [x1 , . . . , xn ]T un qualsiasi vettore
di W . Tale vettore é ovviamente linearmente dipendente da e1 , . . . , ek , per cui la
n-upla (x1 , . . . , xn ) é linearmente dipendente dalle n-uple

(a11 , a21 , . . . , an1 ), (a12 , a22 , . . . , an2 ), . . . , (a1k , a2k , . . . , ank ).

Quindi la matrice  
a11 a12 . . . . . . . . . a1k x1
 a21
 a22 . . . . . . . . . a2k x2 
 a31
 a32 . . . . . . . . . a3k x3 
 a41
 a42 . . . . . . . . . a4k x4 
 ... ... ... ... ... ... ... 
 
 ... ... ... ... ... ... ... 
 
 ... ... ... ... ... ... ... 
an1 an2 . . . . . . . . . ank xn
ottenuta disponendo i vettori generatori di W nelle prime k colonne, ed aggiungendo
una ulteriore colonna formata dalle componenti (x1 , . . . , xn ) del generico vettore X
di W , deve avere rango k. In particolare tale rango deve provenire dalle prime k
colonne.
Quindi, riducendo tale matrice, noteremo che le ultime n − k righe si annulleranno
completamente, eccetto che nella colonna k + 1:
 0 0
a11 a12 . . . . . . . . . a01k ∑ α1,i xi

 0 a0 a0 . . . . . . a0 ∑ α2,i xi 
22 23 2k
 0 0 a0 a0 . . . a0 ∑ α2,i xi 
 
 33 34 3k 
 0 0 ... ... ... ... ... 
 0 0 . . . . . . . . . a0 ∑ αk,i xi 
 
 kk 
 0 0 . . . . . . 0 0 ∑ αk+1,i xi 
 
 0 0 . . . . . . 0 0 ∑ αk+2,i xi 
 
 ... ... ... ... ... ... ... 
 
 ... ... ... ... ... ... ... 
0 0 . . . . . . 0 0 ∑ αn,i xi

In quest’ultima colonna compariranno infatti i polinomi lineari nelle indeterminate


x1 , . . . , xn , provenienti dalle operazioni elementari sulle righe che avremo effettuato
142 6 Equazioni cartesiane di un sottospazio vettoriale.

per ridurre la matrice.


Ma poiché il rango della matrice é k, tutti i polinomi presenti nelle ultime n − k
righe devono essere identicamente nulli. Le relazioni lineari che otteniamo sono
esattamente le equazioni cartesiane di W .

6.2.1 Esercizi svolti.

Ripetiamo i precedenti Esercizi, sviluppandoli col metodo appena esposto:


Esercizio 53. Siano V = R4 e W =< (1, 1, 0, 0), (1, 0, 0, 1), (0, 1, 1, 0) > un suo
sottospazio. Si determinino le equazioni cartesiane di W .

Svolgimento: Sia X = [x1 , x2 , x3 , x4 ]T un generico vettore di W , per cui linearmnte


dipendente dai vettori base di W .
Quindi la matrice  
1 1 0 x1
 1 0 1 x2 
 
 0 0 1 x3 
0 1 0 x4
deve avere rango 3. Riducendola per righe otteniamo la matrice
 
1 1 0 x1
 0 −1 1 x2 − x1 
 
 0 0 1 x4 + x2 − x1 
0 0 0 x3 − x4 − x2 + x1
Poiché l’ultima riga deve essere nulla, otteniamo l’equazione x1 − x2 + x3 − x4 = 0.
Esercizio 54. Siano V = R4 e W =< (1, 1, 0, 0), (1, 0, 0, 1) > un suo sottospazio. Si
determinino le equazioni cartesiane di W .

Svolgimento: Sia X = [x1 , x2 , x3 , x4 ]T un generico vettore di W , per cui linearmnte


dipendente dai vettori base di W .
Quindi la matrice  
1 1 x1
 1 0 x2 
 
 0 0 x3 
0 1 x4
deve avere rango 2. Riducendola per righe otteniamo la matrice
 
1 1 x1
 0 −1 x2 − x1 
 
0 0 x3 
0 0 x4 + x2 − x1
6.2 Equazioni cartesiane ottenute tramite lo studio del rango di una matrice. 143

Poiché le ultime 2 righe devono essere nulle, otteniamo le equazioni



−x1 + x2 + x4 = 0
.
x3 = 0
Esercizio 55. Siano V = R4 e W =< (1, 2, −2, −2), (1, 1, −1, −1) > un suo sotto-
spazio. Si determinino le equazioni cartesiane di W .

Svolgimento: Sia X = [x1 , x2 , x3 , x4 ]T un generico vettore di W , per cui linearmnte


dipendente dai vettori base di W .
Quindi la matrice  
1 1 x1
 2 1 x2 
 
 −2 −1 x3 
−2 −1 x4
deve avere rango 2. Riducendola per righe otteniamo la matrice
 
1 1 x1
 0 −1 x2 − 2x1 
 0 0 x3 + x2  .
 

0 0 x4 + x2
Poiché le ultime 2 righe devono essere nulle, otteniamo le equazioni

x2 + x3 = 0
.
x2 + x4 = 0
Capitolo 7
Le applicazioni lineari.

7.1 Introduzione.

Siano U e V due spazi vettoriali sul campo K, rispettivamente di dimensioni n e


m e siano B = {e1 , . . . , en } una base di U, C = {c1 , . . . , cm } una base di V su K.
La corrispondenza f : U → V , tra i vettori di U espressi in componenti rispetto alla
base B e quelli di V , espressi in componenti rispetto alla base C, é detta applicazione
lineare o anche omomorfismo se, per ogni α, β ∈ K e u1 , u2 ∈ U si ha:

f (αu1 + β u2 ) = α f (u1 ) + β f (u2 ).

Come usualmente avviene in tutte le corrispondenze, lo spazio vettoriale U é detto


dominio di f , V é detto codominio di f .
Esempio 83. f : U → U, tale che f (u) = u, per ogni u ∈ U, é una applicazione
lineare, detta identitá di U.
Esempio 84. f : U → U, tale che f (u) = 0, vettore nullo di U, per ogni u ∈ U, é
una applicazione lineare, detta applicazione nulla in U.
Esempio 85. Sia α ∈ R, f : U → U, tale che f (u) = αu, per ogni u ∈ U, é una
applicazione lineare, detta omotetia in U di rapporto α.
Esempio 86. f : R2 → R3 , tale che, per ogni X = (x1 , x2 ) ∈ R, f (X) = f (x1 , x2 ) =
(x1 , x1 + x2 , x1 − x2 ) ∈ R3 , é una applicazione lineare.
Nota 7.1. Per rimanere in linea con la precedente trattazione degli Spazi Vettoriali,
ci occuperemo esclusivamente di applicazioni lineari tra spazi vettoriali definiti sul
campo R dei numeri reali.
Definizione 7.2. Sia f : U → V una applicazione lineare. Diciamo Immagine di f
il seguente sottoinsieme di V :

Im( f ) = {v ∈ V : ∃u ∈ U, f (u) = v}.

145
146 7 Le applicazioni lineari.

Osservazione 7.3. É facile notare che Im( f ) é un sottospazio vettoriale di V . Infatti,


data l’applicazione lineare f : U → V e scelti due vettori v1 , v2 ∈ Im( f ), avremo che
esistono u1 , u2 ∈ U tali che f (u1 ) = v1 e f (u2 ) = v2 . Per cui v1 + v2 = f (u1 ) +
f (u2 ) = f (u1 + u2 ). Da ció segue che v1 + v2 é immagine del vettore u1 + u2 ∈ U.
Inoltre, per ogni α ∈ R e per ogni v ∈ Im( f ), avremo che esiste u ∈ U tale che
v = f (u) e quindi αv = α f (u) = f (αu), da cui segue che αv ∈ Im( f ).
Esempio 87. Sia f : R3 → R4 definita da f (x1 , x2 , x3 ) = (x1 , 0, x3 , 0), scegliendo
di esprimere i vettori di dominio e codominio in componenti rispetto alle rispettive
basi canoniche. Allora si ha

Im( f ) = {(α, 0, β , 0) : α, β ∈ R} =< (1, 0, 0, 0), (0, 0, 1, 0) >

sottospazio di dimensione 2 di R4 .
Esempio 88. Sia f : R4 → R3 definita da f (x1 , x2 , x3 , x4 ) = (x1 +x2 , x2 −x3 , x1 +x3 ),
scegliendo di esprimere i vettori di dominio e codominio in componenti rispetto alle
rispettive basi canoniche. Sia w ∈ Im( f ), allora w = (a1 , a2 , a3 ) tali che

 x1 + x2 = a1
x2 − x3 = a2
x1 + x3 = a3

da cui ricaviamo che a1 = a2 + a3 , per cui w = (a2 + a3 , a2 , a3 ), per ogni a1 , a2 , a3 ∈


R.

Im( f ) = {(a2 + a3 , a2 , a3 ) : a1 , a2 , a3 ∈ R} =< (1, 1, 0), (1, 0, 1) >

sottospazio di dimensione 2 di R3 .
Esempio 89. Sia f : R3 → R4 definita da f (x1 , x2 , x3 ) = (x1 , x1 + x2 − x3 , 2x1 + x2 −
x3 , x2 − x3 ), scegliendo di esprimere i vettori di dominio e codominio in componenti
rispetto alle rispettive basi canoniche. Sia w ∈ Im( f ), allora w = (a1 , a2 , a3 , a4 ) tali
che 

 x1 = a1
x1 + x2 − x3 = a2


 2x1 + x2 − x3 = a3
x2 − x3 = a4

da cui ricaviamo che a2 = a1 + a4 e a3 = 2a1 + a4 per cui w = (a1 , a1 + a4 , 2a1 +


a4 , a4 ), per ogni a1 , a4 ∈ R.

Im( f ) = {(a1 , a1 + a4 , 2a1 + a4 , a4 ) : a1 , a4 ∈ R} =< (1, 1, 2, 0), (0, 1, 1, 1) >

sottospazio di dimensione 2 di R4 .
Esempio 90. Sia f : R4 → R3 definita da f (x1 , x2 , x3 , x4 ) = (x1 − x2 , 2x1 − x2 +
x4 , −x1 + x2 + x4 ), scegliendo di esprimere i vettori di dominio e codominio in com-
ponenti rispetto alle rispettive basi canoniche. Sia w ∈ Im( f ), allora w = (a1 , a2 , a3 )
tali che
7.1 Introduzione. 147

 x1 − x2 = a1
2x1 − x2 + x4 = a2
−x1 + x2 + x4 = a3

da cui ricaviamo che a2 = a1 + x1 + x4 e a3 = −a1 + x4 per cui, detti x1 = b1 e


x4 = b2 , w = (a1 , a1 + b1 + b2 , −a1 + b2 ), per ogni a1 , b1 , b2 ∈ R.

Im( f ) = {(a1 , a1 + b1 + b2 , −a1 + b2 ) : a1 , b1 , b2 ∈ R}


= < (1, 1, 1), (0, 1, 0), (0, 1, 1) >
cioé Im( f ) = R3 .
Definizione 7.4. Una applicazione f : U → V é detta suriettiva quando Im( f ) = V .
Come ben noto, una applicazione (quindi anche una lineare) f : U → V é detta
iniettiva quando ad ogni scelta di u1 6= u2 in U corrispondono f (u1 ) 6= f (u2 ) in V .
Definizione 7.5. Diciamo Nucleo di una applicazione lineare f : U → V , il seguente
sottoinsieme del dominio:

N( f ) = {u ∈ U : f (u) = 0V }

cioé l’insieme dei vettori di U che hanno come immagine in V il vettore nullo.
Osservazione 7.6. Anche in questo caso é facile notare che N( f ) é un sottospazio
vettoriale di U. Infatti, data l’applicazione lineare f : U → V e scelti due vettori
u1 , u2 ∈ N( f ), avremo che 0 = f (u1 ) + f (u2 ) = f (u1 + u2 ), per cui u1 + u2 ∈ N( f ).
Inoltre, per ogni α ∈ R e per ogni u ∈ N( f ), avremo 0 = α f (u) = f (αu), da cui
segue che αu ∈ N( f ).
Esempio 91. Sia f : R3 → R4 definita da f (x1 , x2 , x3 ) = (x1 , 0, x3 , 0), scegliendo
di esprimere i vettori di dominio e codominio in componenti rispetto alle rispettive
basi canoniche. Sia u ∈ N( f ), allora f (u) = (0, 0, 0, 0) quindi

x1 = 0
x3 = 0

da cui u = (0, x2 , 0)

N( f ) = {(0, a, 0) : a ∈ R} =< (0, 1, 0) >

sottospazio di dimensione 1 di R3 .
Esempio 92. f : R4 → R3 sia definita da f (x1 , x2 , x3 , x4 ) = (x1 +x2 , x2 −x3 , x1 +x3 ).
Sia u ∈ N( f ), allora f (u) = (0, 0, 0) quindi

 x1 + x2 = 0
x2 − x3 = 0
x1 + x3 = 0

da cui x1 = −x2 = −x3 e u = (x1 , −x1 , −x1 , x4 )


148 7 Le applicazioni lineari.

N( f ) = {(a, −a, −a, b) : a, b ∈ R} =< (1, −1, −1, 0), (0, 0, 0, 1) >

sottospazio di dimensione 2 di R4 .
Esempio 93. Sia f : R3 → R4 definita da f (x1 , x2 , x3 ) = (x1 , x1 + x2 − x3 , 2x1 + x2 −
x3 , x2 − x3 ), scegliendo di esprimere i vettori di dominio e codominio in componenti
rispetto alle rispettive basi canoniche. Sia u ∈ N( f ), allora f (u) = (0, 0, 0, 0) quindi


 x1 = 0
x1 + x2 − x3 = 0


 2x1 + x2 − x3 = 0
x2 − x3 = 0

da cui u = (0, x2 , x2 )

N( f ) = {(0, a, a) : a ∈ R} =< (0, 1, 1) >

sottospazio di dimensione 1 di R3 .
Esempio 94. Sia f : R4 → R3 definita da f (x1 , x2 , x3 , x4 ) = (x1 − x2 , 2x1 − x2 +
x4 , −x1 + x2 + x4 ), scegliendo di esprimere i vettori di dominio e codominio in com-
ponenti rispetto alle rispettive basi canoniche. Sia u ∈ N( f ), allora f (u) = (0, 0, 0, 0)
quindi 
 x1 − x2 = 0
2x1 − x2 + x4 = 0
−x1 + x2 + x4 = 0

da cui x1 = x2 = x4 = 0 e u = (0, 0, x3 , 0)

N( f ) = {(0, 0, a, 0) : a ∈ R} =< (0, 0, 1, 0) >

sottospazio di dimensione 1 di R4 .
Teorema 7.7. Una applicazione lineare f : U → V é iniettiva se e solo se N( f ) =
{0U }, cioé se il nucleo di f é il sottospazio banale di U.

Dim. Supponendo f iniettiva, e osservando che f (0U ) = 0V , segue immediata-


mente che non potrá esistere in U un vettore diverso da 0U che abbia immagine
nulla in V .
Viceversa, sia N( f ) = {0U } e scegliamo u1 6= u2 in U. Se fosse f (u1 ) = f (u2 ), allo-
ra f (u1 − u2 ) = 0V , ovvero u1 − u2 ∈ N( f ) = {0U }. Per cui avremmo u1 − u2 = 0U
e da ció seguirebbe la contraddizione u1 = u2 . t
u

7.2 Applicazioni lineari e matrici.

Siano U e V spazi vettoriali su R, tali da avere dim(U) = n e dim(V ) = m e


7.2 Applicazioni lineari e matrici. 149

B = {e1 , . . . , en } una base per U

B0 = {e01 , . . . , e0m } una base per V.


Sia f : U → V una applicazione lineare. Ogni vettore X ∈ U ha delle componenti
(x1 , . . . , xn ) rispetto alla base B, cioé X = x1 e1 + . . . . + xn en , ed ogni vettore Y ∈ V
ha delle componenti (y1 , . . . , ym ) rispetto alla base B0 , cioé Y = y1 e01 + . . . . + ym e0m .
In particolare se Y = f (X) ∈ V allora Y ∈ Im( f ) e

Y = f (x1 e1 + . . . . + xn en ) = x1 f (e1 ) + x2 f (e2 ) + . . . + xn f (en ).

Notiamo quindi che Im( f )é generato dall’insieme { f (e1 ), . . . , f (en )}.
Diremo matrice associata all’applicazione lineare f rispetto alle basi B nel dominio
e B0 nel codominio, la matrice che si ottiene nel seguente modo: la sua colonna j
é formata dalle componenti rispetto alla base B0 dell’immagine f (e j ) del vettore e j
della base B. Concludiamo allora che ad ogni applicazione lineare f é associata una
matrice che dipende dalle basi scelte per il dominio e codominio di f .
Sia quindi  
a11 a12 . . . a1n
 a21 a22 . . . a2n 
A=  ... ... ... ... 

am1 am2 . . . amn


la matrice associata a f rispetto alle basi B e B0 rispettivamente in dominio e
codominio, e siano
   
x1 y1
 x2   y2 
   
X = . . .  ∈ U, Y =  . . .  ∈ V
  
 ...   ... 
xn B ym B0

rispettivamente il generico vettore del dominio (espresso per componenti rispetto


a B) e la sua immagine nel codominio (espressa per componenti rispetto a B0 ). La
corrispondenza f (X) = Y si puó scrivere nel modo seguente:

A · X = Y.

Viceversa sia data una qualsiasi matrice A ∈ Mmn (R) e siano al solito U e V spazi
vettoriali rispettivamente di dimensioni n e m. Siano B = {e1 , . . . , en } una base per
U, e B0 = {e01 , . . . , e0m } una per V . Costruiamo la seguente corrispondenza tra i due
spazi vettoriali:
f (X) = A · X ∈ V, per ogni X ∈ U
nella quale il vettore X ∈ U sia espresso per componenti rispetto alla base B e Y ∈ V
per componenti rispetto alla base B0 . La corrispondenza f é ovviamente lineare.
150 7 Le applicazioni lineari.

Quindi ad ogni matrice é associata una applicazione lineare, rispetto ad opportune


basi scelte nel domino e nel codominio.

Quanto detto fin’ora si puó riassumere nel modo seguente: dati due spazi vettoriali
U e V di dimensioni rispettivamente n e m su di un campo K, e fissate due basi
per ciascuno di essi, ogni matrice m × n corrisponde ad una applicazione lineare e
viceversa.
Inoltre, la matrice associata ad una applicazione lineare f : U → V , una volta fissate
le basi per il dominio ed il codominio, é unica. Infatti, se supponessimo di avere
due matrici A, A0 associate a f , rispetto alle basi B = {e1 , . . . , en } del dominio U, e
B0 = {e01 , . . . , e0m } del codominio V , seguirebbe che (A − A0 )u = 0, per ogni u ∈ U.
Ma ogni vettore ei ∈ B é espresso per componenti rispetto a B tramite il vettore

Xi = [0, . . . , 0, 1, 0, . . . , 0], ∀i = 1, . . . , n.
| {z } | {z }
i−1 n−i

Per cui, in particolare avremo che (A − A0 )Xi = 0, per ogni i = 1, . . . , n. Sviluppando


tutti tali prodotti, ne consegue che ogni riga della matrice A − A0 é nulla, cioé l’intera
matrice A − A0 é nulla, ovvero A = A0 .
Osservazione 7.8. Poiché le colonne della matrice associata ad un omomorfismo
sono vettori generatori di Im( f ) (espressi per componenti rispetto alla base scelta nel
codominio), allora una base dell’immagine si puó ricavare facilmente, considerando
i vettori colonna della matrice associata, che siano tra loro linearmente indipendenti.
Teorema 7.9. Sia data una applicazione lineare f : U → V .
Ad un qualsiasi sottoinsieme S = {v1 , . . . , vr } ⊆ U formato da vettori linearmente
dipendenti corrisponde un sottoinsieme f (S) = { f (v1 ), . . . , f (vr )} ⊆ V anche esso
formato da vettori linearmente dipendenti.

Dim. Siano {v1 , . . . , vr } vettori linearmente dipendenti, quindi esistano α1 , . . . , αr


scalari non tutti nulli tali che ∑ri=1 αi vi = 0. Applichiamo la f alla precedente rela-
zione: 0 = f (∑ri=1 αi vi ) = ∑ri=1 αi f (vi ) che implica la dipendenza lineare dei vettori
{ f (v1 ), . . . , f (vr )}. t
u
Teorema 7.10. Sia data una applicazione lineare f : U → V e sia S = {v1 , . . . , vr } ⊆
U un sottoinsieme del dominio, formato da vettori linearmente indipendenti. Se f é
iniettiva allora il sottoinsieme f (S) = { f (v1 ), . . . , f (vr )} ⊆ V é anche esso formato
da vettori linearmente indipendenti.

Dim. Se { f (v1 ), . . . , f (vr )} fossero linearmente dipendenti allora esisterebbero


opportuni scalari α1 , . . . , αr non tutti nulli tali che ∑ri=1 αi f (vi ) = 0, cioé f (∑i αi (vi )) =
0. Questo implica che il vettore ∑ri=1 αi (vi ) sia nel nucleo di f . Per ipotesi, f é iniet-
tiva con nucleo banale, quindi avremo che ∑ri=1 αi (vi ) = 0, cioé i vettori {v1 , . . . , vr }
costituiscono un sottoinsieme di formato da vettori linearmente dipendenti, che
contraddice l’ipotesi. t
u
7.2 Applicazioni lineari e matrici. 151

Teorema 7.11. Sia data una applicazione lineare f : U → V . Allora dim(U) =


dim(N( f )) + dim(Im( f )).

Dim. Indichiamo dim(U) = n. Se N( f ) = 0 allora f é iniettiva e sappiamo che


dato un insieme di vettori indipendenti B = {e1 , . . . , en } nel dominio, i vettori
{ f (e1 ), . . . , f (en )} sono indipendenti nel codominio. Inoltre se B é una base di U
allora é noto anche che { f (e1 ), . . . , f (en )} sono generatori di Im( f ).
Unendo le due cose otteniamo che { f (e1 ), . . . , f (en )} é esattamente una base per
Im( f ), da cui l’asserto.
Poniamoci allora nel caso in cui N( f ) 6= 0. Essendo N( f ) un sottospazio di U, in-
dichiamo con {b1 , . . . , br } una sua base. Siano ora {cr+1 , . . . , cn } vettori indipen-
denti in U tali che {b1 , . . . , br , cr+1 , . . . , cn } sia una base per U. Ancora richiamia-
mo il risultato per cui l’insieme { f (b1 ), . . . , f (br ), f (cr+1 ), . . . , f (cn )} é un insie-
me di generatori per Im( f ). Poiché b1 , . . . , br appartengono al nucleo di f , allora
f (b1 ) = f (b2 ) = . . . = f (br ) = 0, per cui l’insieme { f (cr+1 ), . . . , f (cn )} é un in-
sieme di generatori per Im( f ). Dimostriamo ora che essi sono anche linearmente
indipendenti. Consideriamo la combinazione lineare

αr+1 f (cr+1 ) + . . . . . . + αn f (cn ) = 0;

dalla linearitá di f abbiamo che

f (αr+1 cr+1 + . . . . . . + αn cn ) = 0

cioé il vettore αr+1 cr+1 + . . . . . . + αn cn appartiene al nucleo N( f ). Se tale vettore


fosse non nullo, esisterebbero β1 , . . . , βr ∈ R tali che

αr+1 cr+1 + · · · + αn cn = β1 b1 + · · · + βr br

che contraddice il fatto che i vettori {b1 , . . . , br , cr+1 , . . . , cn } debbano essere li-
nearmente indipendenti. Questo vuol dire che αr+1 cr+1 + . . . . . . + αn cn = 0, e dal-
la indipendenza lineare dei vettori cr+1 , . . . , cn segue che αi = 0, per ogni i =
r + 1, . . . , n.
Infine, essendo { f (cr+1 ), . . . , f (cn )} una base per Im( f ), allora dim(Im( f )) = n −
r = dim(U) − dim(N( f )), da cui la formula. t
u
Esempio 95. Sia f : R3 → R4 definita da f (x1 , x2 , x3 ) = (x1 , x1 + x2 − x3 , 2x1 + x2 −
x3 , x2 − x3 ), scegliendo di esprimere i vettori di dominio e codominio in componenti
rispetto alle rispettive basi canoniche. Abbiamo visto in un esempio precedente che

Im( f ) = {(a1 , a1 + a4 , 2a1 + a4 , a4 ) : a1 , a4 ∈ R} =< (1, 1, 2, 0), (0, 1, 1, 1) >

sottospazio di dimensione 2 di R4 . Inoltre

N( f ) = {(0, a, a) : a ∈ R} =< (0, 1, 1) >

é sottospazio di dimensione 1 di R3 , ovvero


152 7 Le applicazioni lineari.

dim(Im( f )) + dim(N( f )) = 3 = dimR3 .

Esempio 96. Sia f : R2 → R3 definita da

f (x1 , x2 ) = (x1 + 2x2 , 3x2 , x1 )

rispetto alle basi canoniche sia nel dominio che nel codominio. Determiniamo la
matrice associata alla f .

Svolg.
f (1, 0) = (1, 0, 1)
f (0, 1) = (2, 3, 0)
 
12
A =  0 3 .
10
Il rango della matrice é 2, quindi dim(Im( f )) = 2 e Im( f ) =< (1, 0, 1), (2, 3, 0) >.
Perció dim(N( f )) = 0 e N( f ) = {0}. L’applicazione é iniettiva ma non suriettiva.
t
u
Esempio 97. Ripetiamo l’esempio precedente ma ora esprimiamo la matrice asso-
ciata a due basi differenti da quelle canoniche:

B2 = {(1, 1), (2, 1)} base nel dominio

B3 = {(1, 1, 0), (1, 0, 0), (1, 2, 1)} base nel codominio.

Svolg. Per costruire tale matrice dobbiamo calcolare le immagini dei vettori della
base B2 . Tali immagini si possono calcolare sfruttando la matrice A associata alla f
rispetto alle basi canoniche:
   
12   3
1
f (1, 1) =  0 3  · = 3
1
10 1
   
12   4
2
f (2, 1) =  0 3  · =  3 .
1
10 2
Ora conosciamo le immagini,ma esse sono espresse rispetto alla base canonica di
R3 , dobbiamo quindi convertirle rispetto alla base B3 :

(3, 3, 1) = (1, 1, 1)B3

(4, 3, 2) = (−1, 3, 2)B3


quindi la matrice associata alla f rispetto alle basi B2 e B3 é
7.2 Applicazioni lineari e matrici. 153
 
1 −1
A0 =  1 3 
1 2
da cui    
1 −1   x1 − x2
x
f (X) =  1 3  · 1 =  x1 + 3x2 
x2
1 2 x1 + 2x2
é l’espressione dell’applicazione lineare rispetto a tali basi. Si noti che le dimen-
sioni di Nucleo e Immagine sono invarianti rispetto ad un cambiamento di basi nel
dominio e nel codominio. t
u
Esempio 98. Siano B = {e1 , e2 , e3 } la base canonica di R3 e f : R3 → R4 una
applicazione lineare tale che

f (e1 ) = (1, 1, 1, 0)

f (e2 ) = (1, 0, 1, 0)
f (e3 ) = (0, 0, 0, 1).
La matrice associata a f rispetto alle basi canoniche in R3 ed in R4 é allora
 
110
1 0 0
A= 1 1 0

001

che ha rango 3, quindi dim(Im( f )) = 3 ( f non é suriettiva) e dim(N( f )) = 0 ( f é


iniettiva), inoltre l’espressione di f rispetto alle basi canoniche sia nel dominio che
nel codominio é    
110   x1 + x2
 1 0 0  x1
    x2 
 
 1 1 0  · x2 =  x1 + x2  .
f (X) = 
x3
001 x3
Esempio 99. Sia f : R5 → R3 definito da

f (x1 , x2 , x3 , x4 , x5 ) = (x1 − x4 , x2 − x4 , x3 ).

Determiniamo la matrice associata rispetto alle basi canoniche in R5 e R3 .

Svolg.  
1 0 0 −1 0
A =  0 1 0 −1 0 
001 0 0
che ha rango 3. Quindi dim(Im( f )) = 3 ( f é suriettiva) e dim(N( f )) = 2 ( f non é
iniettiva).
154 7 Le applicazioni lineari.

Una base dell’immagine é data da

(1, 0, 0), (0, 1, 0), (0, 0, 1).

Il generico vettore del Nucleo, X = (x1 , x2 , x3 , x4 , x5 ) é dato da x3 = 0, x1 = x2 = x4 ,


per cui

N( f ) = {(x1 , x1 , 0, x1 , x5 ), x1 , x5 ∈ R} =< (1, 1, 0, 1, 0), (0, 0, 0, 0, 1) > .

t
u
Esempio 100. Sia f : R3 → R4 definito da

f (x1 , x2 , x3 ) = (5x1 + 4x2 − 9x3 , 4x1 + 5x2 − 9x3 , −9x1 − 9x2 + 9x3 , x1 + x2 + x3 ).

Determiniamo la matrice associata rispetto alle basi canoniche in R3 e R4 .

Svolg.  
5 4 −9
 4 5 −9 
 −9 −9 9  .
A= 

1 1 1
Determiniamo la matrice associata a f rispetto alla base

B = {(1, 1, 0), (1, 0, −1), (0, 1, −1)} di R3

ed alla base canonica in R4 .


Si devono calcolare le immagini dei vettori di B:
   
5 4 −9   9
 4 5 −9  1
    9 
 
 −9 −9 9  · 1 =  −18 
f (1, 1, 0) = 
0
1 1 1 2

ed in modo analogo

f (1, 0, −1) = (14, 13, −18, 0) f (0, 1, −1) = (13, 14, −18, 0).

Allora la matrice associata a f rispetto a tali basi é


 
9 14 13
 9 13 14 
A0 =  −18 −18 −18  .

2 0 0

t
u
Esempio 101. Sia f : R4 → R2 con matrice associata
7.2 Applicazioni lineari e matrici. 155
 
1 00 1
A=
−1 1 2 −1

rispetto alle basi

B4 = {(1, 1, 0, 0), (1, 0, 0, 0), (2, 0, 0, 1), (0, 0, 1, 0)} in R4

B2 = {(1, 1), (1, 0)} in R2 .


Determiniamo la matrice associata a f rispetto alle basi canoniche sia nel dominio
che nel codominio.

Svolg. Per prima cosa esprimiamo i vettori della base canonica di R4 per compo-
nenti rispetto alla base B4 :

(1, 0, 0, 0) = (0, 1, 0, 0)B4

(0, 1, 0, 0) = (1, −1, 0, 0)B4


(0, 0, 1, 0) = (0, 0, 0, 1)B4
(0, 0, 0, 1) = (0, −2, 1, 0)B4 .
Quindi calcoliamo le immagini di tali vettori utilizzando la matrice A:
 
  0  
1 00 1 1 0
f (0, 1, 0, 0) = ·   =
−1 1 2 −1  0  1
0

ed analogamente
f (1, −1, 0, 0) = (1, −2)
f (0, 0, 0, 1) = (1, −1)
f (0, −2, 1, 0) = (0, 0).
Tali immagini sono espresse per componenti rispetto alla base B2 e quindi dobbiamo
ora convertirle per componenti rispetto alla base canonica C2 di R2 :

(0, 1)B2 = (1, 0)C2

(1, −2)B2 = (−1, 1)C2


(1, −1)B2 = (0, 1)C2
(0, 0)B2 = (0, 0)C2 .
La matrice associata rispetto alle basi canoniche é allora
 
1 −1 0 0
A0 =
0 1 10
156 7 Le applicazioni lineari.

e l’espressione dell’applicazione lineare nelle basi canoniche di dominio e codomi-


no é  
  x1  
1 −1 0 0  x2  x1 − x2
f (x1 , x2 , x3 , x4 ) = ·   = .
0 1 1 0  x3  x2 + x3
x4
t
u
Esempio 102. Siano f : R4 → R4 e

B = {(1, 0, 0, −1), (1, 0, 0, 0), (0, 1, 1, 0), (0, 0, 1, 0)}

un base di R4 , tali che


f (1, 0, 0, −1) = (2, 0, 0, −1)
f (1, 0, 0, 0) = (−1, 0, 1, 0)
f (0, 1, 1, 0) = (1, 0, 1, −1)
f (0, 0, 1, 0) = (1, 1, 2, 0).
La matrice associata a f rispetto alla base B nel dominio e alla base canonica nel
codominio é  
2 −1 1 1
 0 0 0 1
A=  0 1 1 2 .

−1 0 −1 0
Determiniamo la matrice associata a f rispetto alla base B anche nel codominio.

Svolg. Si devono esprimere le immagini dei vettori di B per componenti rispetto


alla base B stessa, per cui, detta C la base canonica di R4 , si ha:

(2, 0, 0, −1)C = (1, 1, 0, 0)B

(−1, 0, 1, 0)C = (0, −1, 0, 1)B


(1, 0, 1, −1)C = (1, 0, 0, 1)B
(1, 1, 2, 0)C = (0, 1, 1, 1)B
e la matrice associata é  
1 0 10
1 −1 0 1
A0 = 

.
0 0 0 1
0 1 11
Infine determiniamo la matrice relativa ad f rispetto alla base canonica sia nel
dominio che nel codominio:
Per prima cosa esprimiamo i vettori della base canonica C per componenti rispetto
alla base B:
7.2 Applicazioni lineari e matrici. 157

(1, 0, 0, 0) = (0, 1, 0, 0)B


(0, 1, 0, 0) = (0, 0, 1, −1)B
(0, 0, 1, 0) = (0, 0, 0, 1)B
(0, 0, 0, 1) = (−1, 1, 0, 0)B
quindi calcoliamo le immagini di tali vettori tramite la matrice A:
     
2 −1 1 1 0 −1
 0 0 0 1 1  0 
 0 1 1 2·0 =  1 
f (0, 1, 0, 0) =      

−1 0 −1 0 0 0

ed analogamente
f (0, 0, 1, −1) = (0, −1, −1, −1)
f (0, 0, 0, 1) = (1, 1, 2, 0)
f (−1, 1, 0, 0) = (−3, 0, 1, 1)
ed é chiaro che, avendo usato la matrice A, i risultati sono giá espressi per compo-
nenti rispetto alla base canonica. Quindi la matrice associata a f rispetto alla base
canonica sia nel dominio che nel codominio é
 
−1 0 1 −3
 0 −1 1 0 
A00 = 
 1 −1 2 1 

0 −1 0 1

ed esprimiamo f come segue:


     
−1 0 1 −3 x1 −x1 + x3 − 3x4
 0 −1 1 0  −x2 + x3
 ·  x2  = 
   
f (x1 , x2 , x3 , x4 ) = 
 .
1 −1 2 1   x3   x1 − x2 + 2x3 + x4 
0 −1 0 1 x4 −x2 + x4

t
u
Esempio 103. Sia f : R3 → R3 associato alla matrice
 
112
A = 2 1 3
314

rispetto alle basi

B = {(1, 1, 0), (1, 0, 1), (0, 0, 1)} nel dominio

C = {(1, 0, −1), (1, 0, 1), (0, 1, 0)} nel codominio.


158 7 Le applicazioni lineari.

Determiniamo una base per l’immagine in base canonica.

Svolg. Poiché il rango della matrice A é 2, la dimensione dell’immagine é appunto


2.
Le colonne (1, 2, 3) e (1, 1, 1) sono indipendenti, quindi esse rappresentano i 2
vettori che generano l’immagine.
Per definizione, essi compaiono nella matrice espressi per componenti rispetto alla
base C, quindi per ottenere i vettori che generano l’immagine dobbiamo esprimerli
rispetto alla base canonica:

(1, 2, 3)C = 1(1, 0, −1) + 2(1, 0, 1) + 3(0, 1, 0) = (3, 3, 1)

(1, 1, 1)C = 1(1, 0, −1) + 1(1, 0, 1) + 1(0, 1, 0) = (2, 1, 0).


Allora Im( f ) =< (3, 3, 1), (2, 1, 0) >. t
u
Esempio 104. Siano f : R3 → R2 ,

B3 = {(1, 0, 1), (0, 2, 0), (1, 0, −1)} una base di R3

B2 = {(1, 1), (2, 3)} una base di R2


 
10 4 −2
A=
−3 −2 1
la matrice associata a f rispetto alla base B3 nel dominio ed alla base B2 nel co-
dominio. Determiniamo la matrice associata a f rispetto alle basi canoniche sia nel
dominio che nel codominio.

Svolg. Dapprima si devono esprimere i vettori della base canonica C3 di R3 per


componenti rispetto alla base B3 :
1 1
(1, 0, 0) = ( , 0, )B3
2 2
1
(0, 1, 0) = (0, , 0)B3
2
1 1
(0, 0, 1) = ( , 0, − )B3 .
2 2
Di tali vettori calcoliamo adesso le immagini:
1 1
f ( , 0, ) = (4, −1)B2
2 2
1
f (0, , 0) = (2, −1)B2
2
1 1
f ( , 0, − ) = (6, −2)B2 .
2 2
7.2 Applicazioni lineari e matrici. 159

Riportiamo tali immagini per componenti rispetto alla base canonica C2 di R2 :

(4, −1)B2 = (2, 1)C2

(2, −1)B2 = (0, −1)C2


(6, −2)B2 = (2, 0)C2
per cui la matrice associata a f rispetto alle basi canoniche C3 e C2 é:
 
0 2 0 2
A = .
1 −1 0

t
u
Esempio 105. Siano V = R2 [x], W = R3 [x] rispettivamente gli spazi vettoriali
dei polinomi di secondo e terzo grado nella variabile x. Definiamo la seguente
applicazione lineare f : V → W , rispetto alle basi

BV = {1, x, x2 } nel dominio

BW = {1, x, x2 , x3 } nel codominio


f (p(x)) = x · p0|x+1
2

dove p0|x+1 indica la derivata prima del polinomio p(x) calcolata in x + 1.


1. Calcoliamo la matrice associata a f nelle basi scelte;
2. Determiniamo nucleo ed immagine di f ;
3. Determiniamo la matrice associata a f rispetto alle basi

CV = {x + 1, x2 + 1, x2 + x} nel dominio

CW = {x, x3 + 1, x + 1, x2 } nel codominio.

Svolg. Iniziamo calcolando le immagini dei vettori in BV :

f (1) = x2 · 0 = 0W = (0, 0, 0, 0)BW

f (x) = x2 · 1 = x2 = (0, 0, 1, 0)BW


f (x2 ) = x2 (2x + 2) = 2x3 + 2x2 = (0, 0, 2, 2)BW .
La matrica associata é quindi  
0 0 0
0 0 0
A=
0
.
1 2
0 0 2
Il nucleo di f si ricava risolvendo il sistema lineare omogeneo:
160 7 Le applicazioni lineari.
 
0 0 0  
0 x1
 0 0 ·  x2  = 0
0 1 2
x3
0 0 2

da cui x1 = α, x2 = 0, x3 = 0. Quindi i polinomi di R2 [x] che appartengono al nucleo


sono generati dal vettore di componenti (1, 0, 0) rispetto alla base BV , cioé sono tutti
e soli gli scalari in R: Ker( f ) =< α >, α ∈ R.
Segue inoltre che l’immagine ha dimensione 2 ed osservando le colonne della
matrice A, una base dell’immagine é individuata dai vettori di componenti:

(0, 0, 1, 0), (0, 0, 2, 2)

quindi Im( f ) =< x2 , 2x2 + 2x3 >.


Per determinare la matrice rispetto alle nuove basi, dapprima calcoliamo le compo-
nenti dei vettori di CV rispetto alla base BV :

x + 1 = (1, 1, 0)BV

x2 + 1 = (1, 0, 1)BV
x2 + x = (0, 1, 1)BV .
L’immagine di tali vettori tramite f é il prodotto di ciascuno di essi per la matrice A
associata:  
000  
0 0 0 1 2
 0 1 2  · 1 = (0, 0, 1, 0)BW = x
f (1, 1, 0) =    
0
002
 
000  
0 0 0 1 2 3
f (1, 0, 1) =  0 1 2  · 0 = (0, 0, 2, 2)BW = 2x + 2x
  
1
002
 
000  
0 0 0 0 2 3
f (0, 1, 1) =  0 1 2  · 1 = (0, 0, 3, 2)BW = 3x + 2x
  
1
002
Esprimiamo infine tali vettori per componenti rispetto alla base CW .

x2 = αx + β (x3 + 1) + γ(x + 1) + δ x2

cioé
x2 = (β + γ) + (α + γ)x + δ x2 + β x3
ed uguagliando i monomi simili, segue β = 0, γ = −β = 0, α = −γ = 0, δ = 1:
7.3 Applicazioni lineari e cambiamento di base. 161

x2 = (0, 0, 0, 1)CW

Analogamente:
2x2 + 2x3 = (2, 2, −2, 2)CW
3x2 + 2x3 = (2, 2, −2, 3)CW
e la matrice associata é:  
0 2 2
0 2 2 
A0 = 


0 −2 −2 
1 2 3
t
u

7.3 Applicazioni lineari e cambiamento di base.

Siano U,V spazi vettoriali di dimensione rispettivamente n e m sul campo K. Sia


f : U → V una applicazione lineare con matrice associata A ∈ Mmn (K), rispetto alle
basi B nel dominio U, E nel codominio V . Ad ogni vettore u ∈ U, corrisponde la
propria immagine f (u) = v ∈ V . Tale corrispondenza é ottenuta tramite l’utilizzo
della matrice A. Siano X = [x1 , . . . , xn ]T il vettore delle componenti di u rispetto alla
base B del dominio, ed Y = [y1 , . . . , ym ]T il vettore delle componenti dell’immagine
v rispetto alla base E del codominio. La corrispondenza é determinata da

AX = Y. (7.1)

Al variare della scelta delle basi, varia anche la matrice che rappresenta f . Sup-
poniamo quindi di scegliere le basi B0 nel dominio U, E 0 nel codominio V e sia
A0 ∈ Mmn (K) la matrice associata a f rispetto a tali basi. Il vettore u ∈ U sará rap-
presentato dalla n-upla di componenti X 0 = [x10 , . . . , xn0 ]T rispetto alla base B0 , mentre
l’immagine v ∈ V sará rappresentato dalla m-upla di componenti Y 0 = [y01 , . . . , y0m ]T
rispetto alla base E 0 . In tali condizioni, la corrispondenza f (u) = v é ottenuta tramite
l’utilizzo della matrice A0 , ovvero

A0 X 0 = Y 0 . (7.2)

Ci proponiamo di osservare in quale modo le matrici A ed A0 siano tra loro correlate.


Supponiamo quindi di costruire le matrici che determinano il cambiamento di basi
nel dominio e nel codominio. Sia C ∈ Mn (K) la matrice di cambiamento di base
nel dominio, tale che X = CX 0 . Sia inoltre D ∈ Mm (K) la matrice di cambiamneto
di base nel codominio, tale che Y 0 = DY . Da quest’ultima in particolare otteniamo
Y = D−1Y 0 . Sostituendo X ed Y nell relazione (7.1) otteniamo

f (u) = A(CX 0 ) = D−1Y 0 .


162 7 Le applicazioni lineari.

ovvero
(DAC)X 0 = Y 0 . (7.3)
Infine, confrontando (7.3) e (7.2), e ricordando che la matrice associata a f , rispetto
alle basi B0 nel dominio ed E 0 nel codominio, é unica, otteniamo

A0 = DAC.

Poniamoci adesso nel caso particolare in cui U = V . In letteratura una tale appli-
cazione lineare f : V → V é detta endomorfismo od anche operatore lineare. In tali
condizioni, possiamo scegliere una stessa base sia per il dominio che per il codo-
minio. Facendo riferimento alle basi introdotte nel caso generale, poniamo B = E e
B0 = E 0 . Per cui, la matrice A rappresenta l’endomorfismo f in relazione alla scelta
della base B (in dominio e codominio), mentre la matrice A0 rappresenta l’endomor-
fismo in relazione alla scelta della base B0 in dominio e codominio.
Appare chiaro che le matrici di cambiamento di basi, C nel domino e D nel codomi-
nio, sono l’una l’inversa dell’altra. Concludiamo allora che, nel caso di endomorfi-
smi, la relazione che intercorre tra le matrici A e A0 , associate a due distinte basi di
V é la seguente:
A0 = C−1 · A ·C.
Ció vuol dire che le matrici associate ad uno stesso endomorfismo, relativamente a
basi differenti, sono tra loro tutte simili.
Esempio 106. Siano V = R3 ,

B = {(1, 1, 0), (0, 0, 1), (1, 0, 1)}

B0 = {(0, 1, 0), (0, 1, 1), (1, 0, 0)}


due basi di R3 e f : R3 → R3 un endomorfismo con matrice associata rispetto alla
base B  
101
A =  1 0 0 .
011
Rispetto a tale base l’endomorfismo si esprime:
     
101 x1 x1 + x3
f (X) =  1 0 0  ·  x2  =  x1  .
011 x3 x2 + x3

Determiniamo la matrice associata a f rispetto alla base B0 , utilizzando sia il metodo


esposto nei paragrafi precedenti, che quello appena illustrato.

Svolg. Metodo 1. Per prima cosa esprimiamo i vettori di B0 per componenti


rispetto alla base B:
(0, 1, 0) = (1, 1, −1)B
7.3 Applicazioni lineari e cambiamento di base. 163

(0, 1, 1) = (1, 2, −1)B


(1, 0, 0) = (0, −1, 1)B
quindi calcoliamo le immagini di tali vettori tramite la matrice A:

w1 = f (1, 1, −1) = (0, 1, 0)

w2 = f (1, 2, −1) = (0, 1, 1)


w3 = f (0, −1, 1) = (1, 0, 0).
Ricordiamo che per definizione della matrice A, tali immagini sono espresse per
componenti rispetto alla base B, quindi

w1 = (0, 1, 0)B = (0, 0, 1)

w2 = (0, 1, 1)B = (1, 0, 2)


w3 = (1, 0, 0)B = (1, 1, 0)
e per ottenere la matrice A0 non resta altro che esprimere tali vettori per componenti
rispetto alla base B0 :
w1 = (0, 0, 1) = (−1, 1, 0)B0
w2 = (1, 0, 2) = (−2, 2, 1)B0
w3 = (1, 1, 0) = (1, 0, 1)B0 .
La matrice A0 associata a f in base B0 é allora
 
−1 −2 1
A0 =  1 2 0  .
0 1 1

Metodo 2. Calcoliamo la matrice C di cambiamento di base da B a B0 . Essa ha per


colonne le componenti dei vettori di B rispetto alla base B0 :
   
1 −1 −1 1 1 0
C =  0 1 1  , C−1 =  1 2 −1 
1 0 1 −1 −1 1

da cui
     
1 −1 −1 101 1 1 0
A0 = C · A ·C−1 =  0 1 1  ·  1 0 0  ·  1 2 −1  =
1 0 1 011 −1 −1 1
 
−1 −2 1
 1 2 0 .
0 1 1
164 7 Le applicazioni lineari.

t
u

Ci chiediamo allora se e quando esiste una base di V tale che un endomorfismo


f : V → V sia rappresentabile nel modo piú semplice possibile, cioé tramite una
matrice nella quale compaia il numero piú elevato possibile di elementi nulli. La
condizione ideale sarebbe quella in cui la matrice associata a f si presenti in forma
diagonale cioé
 
a11 0 0 . . . . . . . . . 0
 0 a22 0 . . . . . . . . . 0 
 
 0 0 a33 . . . . . . . . . 0 
 
A=  ... ... ... ... ... ... ... 

 ... ... ... ... ... ... ... 
 
 ... ... ... ... ... ... ... 
0 0 0 0 . . . . . . ann
in modo tale che
     
a11 0 0 ... ... ... 0 x1 a11 x1
 0
 a22 0 ... ... ... 0    x2   a22 x2 
   
 0
 0 a33 ... ... ... 0   x3   a33 x3 
    
 ...
f (X) =  ... ... ... ... ... ...  · ...  =  ... .
   
 ... ... ... ... ... ... ...   ...   ... 
     
 ... ... ... ... ... ... ...   ...   ... 
0 0 0 0 . . . . . . ann xn ann xn

Per cui il problema che ci proponiamo di risolvere é il seguente : in quali casi esiste
una base di V rispetto alla quale la matrice associata ad un endomorfismo di V
sia diagonale? Osserviamo che nel caso ció sia possibile, la forma diagonale della
matrice é rappresentativa di ogni altra matrice associata a f in ogni altra base, e tali
matrici sono tra di esse tutte simili.
Esempio 107. Siano V = R3 ,

B = {(0, 1, 0), (1, −1, −1), (0, 1, 1)}

B0 = {(1, 1, 0), (1, 0, 0), (0, 1, 1)}


due basi di R3 e f : R3 → R3 un endomorfismo con matrice associata rispetto alla
base B  
1 00
A =  −1 2 0  .
−1 1 1
Rispetto a tale base l’endomorfismo si esprime:
     
1 00 x1 x1
f (X) =  −1 2 0  ·  x2  =  −x1 + 2x2  .
−1 1 1 x3 −x1 + x2 + x3
7.3 Applicazioni lineari e cambiamento di base. 165

Determiniamo la matrice associata a f rispetto alla base B0 .

Svolg. Calcoliamo la matrice C di cambiamento di base da B a B0 . Essa ha per


colonne le componenti dei vettori di B rispetto alla base B0 :
   
1 0 0 100
C =  −1 1 0  , C−1 =  1 1 0 
0 −1 1 111

da cui      
1 0 0 1 00 100
A0 = C · A ·C−1 =  −1 1 0  ·  −1 2 0  ·  1 1 0  =
0 −1 1 −1 1 1 111
 
100
0 2 0
001
e l’endomorfismo rispetto alla base B0 si esprime:
     
100 x1 x1
f (X) =  0 2 0  ·  x2  =  2x2  .
001 x3 x3

t
u

7.3.1 Restrizione di un endomorfismo.

Definizione 7.12. Sia f : V → V un endomorfismo sullo spazio vettoriale reale V


di dimensione n e sia W ⊂ V un sottospazio vettoriale di V . Diremo che W é un
sottospazio f -invariante se f (W ) ⊆ W .
In presenza di un sottospazio proprio W ⊂ V che sia f -invariante, é allora possibile
determinare l’endomorfismo indotto da f su W , che denotiamo con il simbolo f|W e
chiamiamo restrizione di f in W . Se dim(V ) = n e poiché W é un sottospazio proprio
di V , allora dim(W ) = m < n. Quindi la restrizione f|W , che é un endomorfismo in
W , é rappresentata da una matrice quadrata di ordine m, che si ottiene esattamente
come giá descritto per gli endomorfismi in generale, in dipendenza della base scelta
per W .
Esempio 108. Siano f : R4 → R4 un endomorfismo rappresentato dalla matrice
 
1200
2 1 0 0
A= 
1 1 1 2
1121
166 7 Le applicazioni lineari.

rispetto alla base canonica di R4 , e W =< (0, 0, 1, 1), (1, −1, 0, 0) > un sottospazio
di R4 .
Osserviamo inizialmente che un qualsiasi vettore X ∈ W si esprime con la 4-upla
(β , −β , α, α) al variare di α, β ∈ R. L’immagine di un tale vettore é quindi
         
1200 β −β 0 1
 2 1 0 0   −β   β  0  −1 
f (X) =  1 1 1 2  ·  α  =  3α  = 3α  1  − β  0  ∈ W
        

1121 α 3α 1 0

per cui f (W ) ⊆ W , ovvero W é f -invariante.


Costruiamo allora la matrice AW di ordine 2 (poiché tale é la dimensione di W )
associata a f|W rispetto alla base precedentemente scelta per W :

B = {(0, 0, 1, 1), (1, −1, 0, 0)}.

In accordo con la definizione di matrice associata ad un endomorfismo, dovremo cal-


colare le immagini dei vettori della base B. Le componenti di tali immagini, rispetto
ancora a B, saranno le colonne della matrice AW . Avremo:
       
  1200 0 0 0
2 1 0 0 0 0 0
f (0, 0, 1, 1) =   1 1 1 2 · 1  =  3  = 3 1 
      

1121 1 3 1

per cui  
f (0, 0, 1, 1) = (3, 0)B .

Analogamente
       
  1 2 0 0 1 −1 1
 ·  −1  =  1  = −  −1 
2 1 0 0      
f (1, −1, 0, 0) = 
1 1 1 2   0   0   0 
1 1 2 1 0 0 0

per cui  
f (−1, 1, 0, 0) = (0, −1)B .

Allora la matrice associata a f|W relativamente alla base B é



3 0
AW = .
0 −1

Quindi ogni vettore X ∈ W , di componenti (x1 , x2 )B rispetto alla base B, ha imma-


gine tramite f|W pari a
7.4 Esercizi svolti. 167
     
      0 1 −x2
3 0 3x1  − x2  −1  =  x2 
x 0
· 1 =
   
f (X) = = 3x1 
0 −1 x2 B −x2 B  1   0   3x1 
1 0 3x1

la quale ovviamente coincide con l’immagine di X, visto come vettore di R4 , tramite


la f inizialmente definita.
In conclusione del presente Capitolo introduciamo gli strumenti successivamente
necessari per determinare in quali casi sia possibile che un endomorfismo possa
essere espresso tramite una matrice diagonale, rispetto ad una opportuna base dello
spazio vettoriale

7.4 Esercizi svolti.

Esercizio 56. Sia f : R2 → R4 una applicazione lineare definita da



f (x1 , x2 ) = (x1 + x2 , x1 − x2 , x2 , x1 )

rispetto alle basi


B = {(1, 1), (2, 1)} nel dominio
e
D = {(1, 2, 0, 0), (0, 1, 0, 0), (0, 1, 1, 0), (0, 0, 0, 1)} nel codominio.
Si determinino una base per il nucleo ed una per l’immagine di f .

Svolgimento: La matrice associata all’omomorfismo f é:


 
1 1
 1 −1 
A=  0 1 .

1 0

L’immagine é quindi generata dai vettori le cui componenti rispetto alla base D sono
espresse nelle colonne di A. Quindi Im( f ) é generata da

{(1, 1, 0, 1)D , (1, −1, 1, 0)D }.

Tali vettori sono anche linearmente indipendenti, per cui costituiscono una ba-
se per Im( f ). Determiniamo quindi quali siano i vettori in questione, calcolan-
do le combinazioni lineari dei vettori di D tramite le componenti (1, 1, 0, 1)D e
(1, −1, 1, 0)D :

(1, 1, 0, 1)D = (1, 2, 0, 0) + (0, 1, 0, 0) + (0, 0, 0, 1) = (1, 3, 0, 1)


168 7 Le applicazioni lineari.

e
(1, −1, 1, 0)D = (1, 2, 0, 0) − (0, 1, 0, 0) + (0, 1, 1, 0) = (1, 2, 1, 0).
Abbiamo quindi che Im( f ) =< (1, 3, 0, 1), 1, 2, 1, 0) >.
Passiamo al nucleo N( f ). Sarebbe sufficiente risolvere il sistema lineare omogeneo
AX = 0, in cui X = [x1 x2 ]TB é il vettore del dominio espresso per componenti
dim(R2 ) = dim N( f ) + dim Im( f ) ,

rispetto alla base B. Ciononostante, poiché

in questo caso é chiaro che dim N( f ) = 0, cioé N( f ) = {0}.
Esercizio 57. Sia f : R5 → R4 una applicazione lineare definita da

f (x1 , x2 , x3 , x4 , x5 ) = (x1 − x2 , x2 + x3 , x2 + x3 , x1 − x5 )

rispetto alle basi

B = {(1, 1, 0, 0, 0), (2, 1, 0, 0, 0), (0, 0, 1, 1, 0), (0, 0, 0, 1, 0), (0, 0, 0, 0, 1)}

nel dominio e

D = {(1, 2, 0, 0), (0, 1, 0, 0), (0, 1, 1, 0), (0, 0, 0, 1)} nel codominio.

Si determinino una base per il nucleo ed una per l’immagine di f .

Svolgimento: La matrice associata all’omomorfismo f é:


 
1 −1 0 0 0
0 1 1 0 0 
A=  0 1 1 0 0 .

1 0 0 0 −1

Iniziamo dal nucleo N( f ). É sufficiente risolvere il sistema lineare omogeneo


AX = 0, in cui X = [x1 x2 x3 x4 x5 ]TB é il vettore del dominio espresso per
componenti rispetto alla base B. Quindi poniamo
 
  x1  
1 −1 0 0 0  x2  0
0 1 1 0 0    0
 0 1 1 0 0  ·  x3  =  0 
     
 x4 
1 0 0 0 −1 0
x5 B

da cui risulta facilmente

x1 = x2 = x5 , x3 = −x2 .

Un generico vettore del nucleo h́a componenti rispetto alla base B

(α, α, −α, β , α)B α, β ∈ R


7.4 Esercizi svolti. 169

quindi possiamo esprimere i vettori di una base di N( f ) per componenti rispetto a


B:
N( f ) =< (1, 1, −1, 0, 1)B , (0, 0, 0, 1, 0)B > .
Infine determiniamo quali siano tali vettori, esprimendoli come combinazione linea-
re delle loro componenti rispetto a B:

(1, 1, −1, 0, 1)B = (1, 1, 0, 0, 0) + (2, 1, 0, 0, 0) − (0, 0, 1, 1, 0) + (0, 0, 0, 0, 1)

= (3, 2, −1, −1, 1)

mentre il secondo vettore (0, 0, 0, 1, 0)B , avendo solo la quarta componente non
nulla, é pari a (0, 0, 0, 1, 0). In definitiva

N( f ) =< ((3, 2, −1, −1, 1), (0, 0, 0, 1, 0) > .

Passiamo all’immagine e dapprima determiniamone la dimensione:

dim(R5 ) = dim N( f ) + dim Im( f ) =⇒ dim Im( f ) = 3.


  

Constatiamo che le prime tre colonne di A sono linearmente indipendenti, per cui
una base di Im( f ) é data da

{(1, 0, 0, 1)D , (−1, 1, 1, 0)D , (0, 1, 1, 0)D }

dove i vettori sono espressi per componenti rispetto alla base D del codominio.
Esprimendo i vettori come combinazione lineare delle loro componenti rispetto ai
vettori di D, otteniamo

(1, 0, 0, 1)D = (1, 2, 0, 0) + (0, 0, 0, 1) = (1, 2, 0, 1)

(−1, 1, 1, 0)D = −(1, 2, 0, 0) + (0, 1, 0, 0) + (0, 1, 1, 0) = (−1, 0, 1, 0)


e
(0, 1, 1, 0)D = (0, 1, 0, 0) + (0, 1, 1, 0) = (0, 2, 1, 0).
Quindi
Im( f ) =< (1, 2, 0, 1), (−1, 0, 1, 0), (0, 2, 1, 0) > .

Esercizio 58. Sia f : R2 → R2 una applicazione lineare definita da



f (x1 , x2 ) = (x1 , x1 + x2 )

rispetto alle basi B = {(1, 1), (1, 0)} nel dominio e D = {(2, 1), (0, 1)} nel codomi-
nio. Si determini la matrice A0 associata alla f rispetto alle basi B0 = {(1, 2), (0, 1)}
nel dominio e D0 = {(1, 1), (−1, 1)} nel codominio.
170 7 Le applicazioni lineari.

Svolgimento: La matrice associata all’endomorfismo f rispetto alle basi B nel


dominio e D nel codominio é:  
10
A= .
11
Per determinare la nuova matrice A0 , é necessario inizialmente esprimere i vetto-
ri della nuova base B0 del dominio per componenti rispetto a quella inizialmente
introdotta B:
(1, 2) = (2, −1)B , (0, 1) = (1, −1)B .
A questo punto possiamo effettuare il calcolo delle immagini dei vettori della base
B0 tramite l’utilizzo della matrice A. Tali immagini saranno espresse per componenti
rispetto alla base D del codominio:
     
 10 2 2
f (1, 2) = · = = 2(2, 1) + (0, 1) = (4, 3)
11 −1 B 1 D
     
 10 1 1
f (0, 1) = · = = (2, 1).
11 −1 B 0 D
Non ci resta che esprimere le immagini ottenute tramite le rispettive componenti
rispetto alla nuova base D0 del codominio:
7 1
(4, 3) = α(1, 1) + β (−1, 1) =⇒ (α, β )D0 = ( , − )D0
2 2
3 1
(2, 1) = γ(1, 1) + δ (−1, 1) =⇒ (γ, δ )D0 = ( , − )D0 .
2 2
Quindi la matrice associata a f rispetto alle basi B nel dominio e D0 nel codominio
0

é  7 3 
A = 21 21 .
−2 −2

Esercizio 59. Sia f : R2 → R3 una applicazione lineare definita da



f (x1 , x2 ) = (x1 + x2 , x1 + 2x2 , x2 )

rispetto alle basi B = {(1, 1), (1, 2)} nel dominio e D = {(1, 1, 1), (1, 0, 0), (1, 0, 2)}
nel codominio. Si determini la matrice A0 associata alla f rispetto alle basi B0 =
{(0, 1), (2, 1)} nel dominio e D0 = {(1, , 0, 1), (1, 0, 0), (1, 1, 2)} nel codominio.

Svolgimento: La matrice associata all’omomorfismo f rispetto alle basi B nel


dominio e D nel codominio é:  
11
A =  1 2 .
01
Inizialmente esprimiamo i vettori della base B0 del dominio per componenti rispetto
7.4 Esercizi svolti. 171

a B:
(0, 1) = (−1, 1)B , (2, 1) = (3, −1)B .
Calcolo delle immagini dei vettori della base B0 tramite l’utilizzo della matrice A:
   
11   0
 −1
f (0, 1) =  1 2  · =  1  = (1, 0, 0) + (1, 0, 2) = (2, 0, 2)
1 B
01 1 D
 
1 1  
 3
f (2, 1) =  1 2 ·

−1 B
0 1
 
2
=  1  = 2(1, 1, 1) + (1, 0, 0) − (1, 0, 2) = (2, 2, 0).
−1 D
Per concludere esprimiamo le immagini ottenute tramite le rispettive componenti
rispetto alla nuova base D0 del codominio:

(2, 0, 2) = α(1, , 0, 1) + β (1, 0, 0) + γ(1, 1, 2) =⇒ (α, β , γ)D0 = (2, 0, 0)D0

(2, 2, 0) = δ (1, , 0, 1) + η(1, 0, 0) + θ (1, 1, 2) =⇒ (δ , η, θ )D0 = (−4, 4, 2)D0 .


Quindi la matrice associata a f rispetto alle basi B0 nel dominio e D0 nel codominio
é  
2 −4
A =  0 4 .
0 2

Esercizio 60. Sia f : R3 [X] → R2 [X] una applicazione lineare alla quale sia asso-
ciata la seguente matrice  
−1 0 −1 2
A =  1 0 1 0
0 3 0 0
rispetto alle basi B = {x, x3 + 1, x + 1, x2 } nel dominio e D = {x, 1 + x, x2 } nel
codominio.
1. Si determini la matrice A0 associata alla f rispetto alle basi B0 = {x − 1, x2 +
1, x, x3 } nel dominio e D0 = {x − 1, x2 + 1, 1} nel codominio.
2. Si determinino una base per il nucleo ed una per l’immagine di f .

Svolgimento: Dapprima esprimiamo i vettori di ciascuna delle basi indicate, tra-


mite le loro componenti rispetto ad una base qualsiasi di R3 [X] ed una di R2 [X].
Ovviamente le nostre scelte ricadono sulle basi canoniche

{1, x, x2 , x3 } per R3 [X]


172 7 Le applicazioni lineari.

{1, x, x2 } per R2 [X].


Quindi avremo

B = {(0, 1, 0, 0), (1, 0, 0, 1), (1, 1, 0, 0), (0, 0, 1, 0)}

D = {(0, 1, 0), (1, 1, 0), (0, 0, 1)}


B0 = {(−1, 1, 0, 0), (1, 0, 1, 0), (0, 1, 0, 0), (0, 0, 0, 1)}
D0 = {(−1, 1, 0), (1, 0, 1), (1, 0, 0)}.
Esprimiamo i vettori di B0 per componenti rispetto alla base B:

(−1, 1, 0, 0) = (2, 0, −1, 0)B

(1, 0, 1, 0) = (−1, 0, 1, 1)B


(0, 1, 0, 0) = (1, 0, 0, 0)B
(0, 0, 0, 1) = (1, 1, −1, 0)B .
Adesso calcoliamo le immagini di tali vettori:
 
  2
 −1 0 −1 2  0 
f (−1, 1, 0, 0) =  1 0 1 0  ·  
 −1 
0 3 0 0
0 B
 
−1
=  1  = −(0, 1, 0) + (1, 1, 0) = (1, 0, 0)
0 D



 −1  
−1 0 −1 2  2
 0 
f (1, 0, 1, 0) =  1 0 1 0  · 
 1 
 =  0  = 2(0, 1, 0) = (0, 2, 0)
0 3 0 0 0 D
1 B
 
 1 
−1 0 −1 2  
 0
f (0, 1, 0, 0) =  1 0 1 0  · 
0
0 3 0 0
0 B
 
−1
=  1  = −(0, 1, 0) + (1, 1, 0) = (1, 0, 0)
0 D
ed infine
7.4 Esercizi svolti. 173
 
  −1  
 −1 0 −1 2  0  2
f (1, 0, 1, 0) =  1 0 1 0  ·   =  0  = 2(0, 1, 0) = (0, 2, 0)
 1 
0 3 0 0 0 D
1 B

 
 1   
−1 0 −1 2  0
 1 
f (0, 0, 0, 1) =  1 0 1 0  ·   =  0  = 3(0, 0, 1) = (0, 0, 3).
 −1 
0 3 0 0 3 D
0 B

concludiamo determinando le componenti delle immagini ottenute, rispetto alla


nuova base D0 scelta per il codominio:

(1, 0, 0) = (0, 0, 1)D0 questo calcolo si ripete anche per il terzo vettore immagine

(0, 2, 0) = (2, 0, 2)D0


(0, 0, 3) = (0, 3, −3)D0 .
Quindi la matrice A0 associata all’omomorfismo rispetto alle basi B0 e D0 rispettiva-
mente in dominio e codominio, é la seguente:
 
020 0
A0 =  0 0 0 3  .
1 2 1 −3

Per determinare il nucleo di f , ricordiamo che potremmo fare riferimento indiffe-


rentemente alla matrice A oppure alla A0 . Preferiamo scegliere A0 .
Risolviamo il sistema lineare omogeneo A0 X = 0, in cui X = [x1 x2 x3 x4 ]TB é il
vettore del dominio espresso per componenti rispetto alla base B0 . Quindi poniamo
 
  x1  
020 0  x2  0
0 0 0 3 ·  = 0
 x3 
1 2 1 −3 0
x4 B0

da cui x2 = x4 = 0 e x1 = −x3 , ovvero X ∈ N( f ) se e solo se X = (α, 0, −α, 0)B0 .


Quindi N( f ) =< (1, 0, −1, 0)B0 >. Esprimendo tale vettore come combinazione del-
le sue componenti rispetto alla base B0 , otteniamo (1, 0, −1, 0)B0 = (−1, 0, 0, 0), ov-
vero N( f ) =< (1, 0, 0, 0) >. In termini polinomiali il nucleo di f é generato da un
qualsiasi polinomio di grado 0, ovvero da un qualsiasi scalare. La sua dimensione é
pari a 1 e possiamo porre N( f ) = R.
La dimensione dell’immagine é quindi 3. Osservando la matrice A0 , notiamo che
le sue ultime 3 colonne formano un minore non nullo e quindi sono linearmente
indipendenti. Allora

Im( f ) =< (2, 0, 2)D0 , (0, 0, 1)D0 , (0, 3, −3)D0 >


174 7 Le applicazioni lineari.

ed esprimendo ciascuno dei precedenti vettori come combinazione lineare delle


proprie componenti rispetto alla base D0 avremo

(2, 0, 2)D0 = (0, 2, 0)

(0, 0, 1)D0 = (1, 0, 0)


(0, 3, −3)D0 = (0, 0, 3)
e
Im( f ) =< (0, 2, 0), (1, 0, 0), (0, 0, 3) > .
In termini polinomiali
Im( f ) =< 2x, 1, 3x2 > .

Esercizio 61. Sia f : R3 → R2 una applicazione lineare alla quale sia associata la
seguente matrice  
121
A=
112
rispetto alle basi

B = {(1, 1, 0), (0, 1, 1), (2, 0, 1)} nel dominio

e
D = {(1, 1), (2, 1)} nel codominio.
Si determini la matrice A0 associata alla f rispetto alle basi

B0 = {(1, 0, 1), (1, 0, 0), (2, 1, 0)} nel dominio

e
D0 = {(1, 2), (1, 0)} nel codominio.

Svolgimento: In tale caso vogliamo sfruttare la relazione che intercorre tra le ma-
trici associate ad un omomorfismo, rispetto a scelte differenti delle basi di dominio
e/o codominio. Precisamente, dette C ∈ M3 (R) la matrice di transizione da B0 a B, e
P ∈ M2 (R) la matrice di transizione da D a D0 , avremo

A0 = PAC.

Per determinare C dobbiamo calcolare le componenti di ciascuno dei vettori della


base B0 rispetto alla base B:
1 1 2
(1, 0, 1) = (− , , )B
3 3 3
1 1 1
(1, 0, 0) = ( , − , )B
3 3 3
7.4 Esercizi svolti. 175

4 1 1
(2, 1, 0) = ( , − , )B
3 3 3
da cui
− 13 1 4
 
3 3
1
C= 3 − 13 − 13  .
2 1 1
3 3 3
Analogamente, per determinare P dobbiamo calcolare le componenti di ciascuno
dei vettori della base D rispetto alla base D0 :
1 1
(1, 1) = ( , )D0
2 2
1 3
(2, 1) = ( , )D0
2 2
da cui 1 1

P= 2 2 .
1
2 − 32
Infine abbiamo:
A0 = P · A ·C

 −1 1 4
 
1 1
 
2 2 1 2 1  13 31 31 
= 1 · · 3 −3 −3
2 − 32 112 2 1 1
3 3 3

7 1 4
 
= 6 3 3 .
− 32 −1 −2

Esercizio 62. Sia f : R3 → R3 un endomorfismo al quale sia associata la seguente


matrice  
101
A = 1 0 0
110
rispetto alla base B = {(1, 1, 2), (1, 0, 0), (0, 1, 1)} sia nel dominio che nel codomi-
nio. Si determini la matrice A0 associata alla f rispetto alla base

B0 = {(1, 1, 0), (2, 0, 1), (1, 0, 1)}

sia nel dominio che nel codominio.

Svolgimento: Sostanzialmente dovremmo ripetere il medesimo procedimento del


precedente Esercizio, se non fosse per il fatto che adesso stiamo considerando una
medesima base per dominio e codominio. Nella analogia all’esercizio precedente,
é come se avessimo B = D e B0 = D0 . Ció si traduce nel fatto che le matrici di
176 7 Le applicazioni lineari.

transizione necessarie per poter ottenere A0 , sono l’una l’inversa dell’altra. Ovvero
detta C ∈ M3 (R) la matrice di transizione da B0 a B, allora P = C−1 ∈ M3 (R) é la
matrice di transizione da B a B0 . Dovremo quindi calcolare

A0 = C−1 AC.

Come noto, per determinare C dobbiamo calcolare le componenti di ciascuno dei


vettori della base B0 rispetto alla base B:

(1, 1, 0) = (−1, 2, 2)B

(2, 0, 1) = (1, 1, −1)B


(1, 0, 1) = (1, 0, −1)B
da cui  
−1 1 1
C= 2 1 0 
2 −1 −1
e  
1 0 1
C−1 =  −2 1 −2  .
4 −1 3
Infine abbiamo:
A0 = C−1 · A ·C
     
1 0 1 101 −1 1 1
=  −2 1 −2  ·  1 0 0  ·  2 1 0 
4 −1 3 110 2 −1 −1
 
2 2 1
=  −5 −3 −1  .
8 5 2
Capitolo 8
La forma canonica di un operatore lineare.

Sia f : Rn → Rn un endomorfismo di V = Rn (anche detto operatore lineare reale) e


siano B e B0 due distinte basi di Rn . Indichiamo con A la matrice associata a f rispet-
to alla base B e con A0 quella associata a f rispetto a B0 . Come visto in precedenza,
le matrici A e A0 sono simili e quindi esse hanno gli stessi autovalori. Quindi, piú
in generale, diciamo che tutte le matrici relative ad uno stesso endomorfismo pos-
siedono gli stessi autovalori, indipendentemente dalla base rispetto alla quale sono
costruite.
Sfruttando tali premesse ed alla luce dei risultati discussi nel Capitolo precedente,
ci proponiamo di determinare, qualora sia possibile, una opportuna base per lo spa-
zio vettoriale su cui un operatore lineare é definito, rispetto alla quale la matrice
associata all’operatore sia espressa nel modo piú semplice possibile.

8.1 Autovalori ed autovettori di un endomorfismo.

Siano V uno spazio vettoriale di dimensione n sul campo K, f : V → V un endomor-


fismo di V e A = [ai j ] la matrice quadrata di ordine n associata a f in una certa base
B. Indichiamo con X ∈ V un generico vettore di V , da cui f (X) = A · X.
Ci chiediamo se possano esistere vettori X ∈ V , la cui immagine f (X) sia un vettore
proporzionale a X, cioé f (X) = A · X = λ X, per qualche opportuno λ ∈ K. Chiara-
mente il vettore nullo 0 ∈ V soddisfa tale proprietá (é ovvio infatti che f (0) = λ · 0,
per ogni λ ∈ K), per cui ci proponiamo di determinare gli eventuali vettori X 6= 0
che la soddisfino.
Definizione 8.1. Sia f : V → V un endomorfismo dello spazio vettoriale V sul cam-
po K. Siano λ ∈ K e 0 6= X ∈ V tali che f (X) = λ X. Diremo che λ é un autovalore
di f e che X é un autovettore di f relativo all’autovalore λ .
La determinazione di tali vettori si riduce alla risoluzione del sistema lineare A · X =
λ · X che si puó scrivere come segue:

177
178 8 La forma canonica di un operatore lineare.


 a11 x1 + a12 x2 + . . . + a1n xn = λ x1
a21 x1 + a22 x2 + . . . + a2n xn = λ x2


 ..........
an1 x1 + an2 x2 + . . . + ann xn = λ xn

o meglio ancora, come sistema lineare omogeneo (A − λ I)X = 0:




 (a11 − λ )x1 + a12 x2 + . . . + a1n xn = 0
a21 x1 + (a22 − λ )x2 + . . . + a2n xn = 0

.

 ..........
an1 x1 + an2 x2 + . . . + (ann − λ )xn = 0

Tale sistema ammette soluzioni non banali solo quando p(h) = det(A − λ I) = 0. Il
polinomio p(h) = det(A − λ I) é detto polinomio caratteristico di f (o di A) ed ha
grado n. Analogamente l’equazione p(λ ) = 0 é detta equazione caratteristica di f
(o di A) ed ha n soluzioni (in K o non in K, distinte o coincidenti).
Si osservi che da ora in poi ci riferiremo al polinomio caratteristico di un endomor-
fismo o di una matrice ad esso associata senza alcuna distinzione.
Le radici del polinomio caratteristico che appartengano al campo K, sono quindi
gli autovalori di f . In corrispondenza di ciascun autovalore h0 , il sistema lineare
omogeneo (A − h0 I)X = 0 ammette sempre soluzioni non banali. I vettori X che
ricoprono l’insieme V0 di tali soluzioni non banali sono quindi gli autovettori di f
relativi all’autovalore h0

V0 = {0 6= X ∈ V, (A − h0 I)X = 0}.

Il sottospazio vettoriale V0 ∪ {0} ricoperto da tali autovettori, con l’aggiunta del


vettore nullo di V , é detto autospazio di f relativo all’autovalore h0 .
Osservazione 8.2. Sia A una matrice quadrata di ordine n. Se λ é un autovalore di
A, allora λ n lo é per la matrice An .

Dim. É sufficiente osservare che da AX = λ X, moltiplicando a sinistra ambo i


membri per A, si ottiene A2 X = λ AX = λ 2 X, cioé A2 ammette λ 2 come autovalore.
Allo stesso modo lo si dimostra per ogni potenza della matrice. t
u
Osservazione 8.3. Sia A una matrice quadrata di ordine n. Allora A e la sua trasposta
AT hanno gli stessi autovalori (nulla si puó concludere sugli autovettori relativi).

Dim. Deriva direttamente dal fatto che

det(A − λ I) = det((A − λ I)T ) = det(AT − λ I).

t
u
Esempio 109. Siano V = R2 ,
ef : R2 → R2
con matrice associata rispetto alla base
canonica sia nel dominio che nel codominio:
8.1 Autovalori ed autovettori di un endomorfismo. 179
 
0 −1
A= .
1 0

Il polinomio caratteristico é p(h) = h2 + 1, il quale non ha radici reali. Quindi l’en-


domorfismo reale da noi definito non possiede autovalori. Nel caso avessimo defini-
to f : C2 → C2 , dove C é il campo dei numeri complessi, con la medesima matrice
associata A, avremmo avuto i due autovalori immaginari +i, −i.
Esempio 110. Siano V = R3 e f : R3 → R3 con matrice associata rispetto alla base
canonica di R3 :  
122
A =  1 3 1 .
221
Gli autovalori della matrice A sono le soluzioni dell’equazione caratteristica:

1−h 2 2

1 3−h 1 = 0

2 2 1−h

cioé
(h + 1)(−h2 + 6h − 5) = 0.
Tali soluzioni sono: h1 = −1, h2 = 5, h3 = 1, ciascuna con molteplicitá algebrica 1
come radici del polinomio caratteristico.
Determiniamo gli autospazi relativi a ciascun autovalore:
Per h1 = −1,  
222
A − h1 I =  1 4 1 
222
ed il sistema lineare omogeneo associato é:

 2x1 + 2x2 + 2x3 = 0
x1 + 4x2 + x3 = 0 .
2x1 + 2x2 + 2x3 = 0

Il rango di tale sistema é 2, quindi vi sono ∞1 soluzioni, cioé ∞1 autovettori relativi


a h1 = −1. Essi si ottengono risolvendo il sistema omogeneo: x1 = −x3 , x2 = 0.
Per cui il generico autovettore relativo a h1 é X = (−a, 0, a), al variare di a ∈ R, e
l’autospazio associato a h1 é

V1 = {(−a, 0, a) ∈ R3 , a ∈ R}

con dim(V1 ) = 1.
Per h2 = 5,  
−4 2 2
A − h2 I =  1 −2 1 
2 2 −4
180 8 La forma canonica di un operatore lineare.

ed il sistema lineare omogeneo associato é:



 −4x1 + 2x2 + 2x3 = 0
x1 − 2x2 + x3 = 0 .
2x1 + 2x2 − 4x3 = 0

Il rango di tale sistema é 2, quindi vi sono ∞1 soluzioni, cioé ∞1 autovettori relativi


a h2 = 5. Essi si ottengono risolvendo il sistema omogeneo: x1 = x3 , x2 = x3 . Per cui
il generico autovettore relativo a h2 é X = (a, a, a), al variare di a ∈ R, e l’autospazio
associato a h2 é
V2 = {(a, a, a) ∈ R3 , a ∈ R}
con dim(V2 ) = 1.
Per h3 = 1,  
022
A − h3 I =  1 2 1 
220
ed il sistema lineare omogeneo associato é:

 2x2 + 2x3 = 0
x1 + 2x2 + x3 = 0 .
2x1 + 2x2 = 0

Il rango di tale sistema é 2, quindi vi sono ∞1 soluzioni, cioé ∞1 autovettori relativi


a h3 = 1. Essi si ottengono risolvendo il sistema omogeneo: x1 = x3 , x2 = −x3 .
Per cui il generico autovettore relativo a h3 é X = (a, −a, a), al variare di a ∈ R, e
l’autospazio associato a h3 é

V3 = {(a, −a, a) ∈ R3 , a ∈ R}

con dim(V3 ) = 1.
Definizione 8.4. Chiameremo molteplicitá algebrica di un autovalore, la sua mol-
teplicitá come radice del polinomio caratteristico e molteplicitá geometrica di un
autovalore, la dimensione dell’autospazio associato ad esso.
Osservazione 8.5. Una matrice triangolare (superiore od inferiore) ha come auto-
valori gli elementi presenti sulla sua diagonale principale.
Poniamoci ad esempio nel caso di una triangolare superiore:
 
a11 a12 a13 . . . . . . . . . a1n
 0 a22 a23 . . . . . . . . . a2n 
 
 0 0 a33 . . . . . . . . . a3n 
 
A=  ... ... ... ... ... ... ... 
 ... ... ... ... ... ... ... 
 
 . . . . . . . . . . . . . . . an−1,n−1 an−1,n 
0 0 0 0 ... ... ann
8.1 Autovalori ed autovettori di un endomorfismo. 181

allora
 
a11 − h a12 a13 . . . . . . ... a1n
 0 a22 − h a23 . . . . . . ... a2n 
 
 0
 0 a33 − h . . . . . . ... a3n  
A − hI = 
 ... ... ... ... ... ... ...  
 ... ... ... ... ... ... ... 
 
 ... ... . . . . . . . . . an−1,n−1 − h an−1,n 
0 0 0 0 ... ... ann − h

e det(A − hI) = (a11 − h)(a22 − h) · · · (ann − h), le cui radici sono

h1 = a11 , h2 = a22 , ..., ..., hn = ann .

In particolare, lo stesso vale per matrici diagonali.


Teorema 8.6. Siano A, B ∈ Mn (R) due matrici simili, cioé esista una matrice non
singolare C ∈ Mn (R) tale che B = C−1 · A · C. Siano Yi autovettori di B. Allora A e
B hanno gli stessi autovalori h1 , . . . , hm ed inoltre Xi = C ·Yi sono autovettori di A,
per ogni autovettore Yi di B.

Dim. Siano h1 , . . . , hm gli autovalori di A e X1 , . . . , Xm autovettori di A tali che


AXi = hi Xi , per i = 1, . . . , m. Poiché A = CBC−1 , segue che:

(CBC−1 )Xi = hi Xi

B(C−1 Xi ) = hi (C−1 Xi )
cioé h1 , . . . , hm sono anche autovalori di B, con autovettori corrispondenti Yi =
C−1 Xi . t
u
Teorema 8.7. Sia f : V −→ V un endomorfismo su uno spazio vettoriale V di di-
mensione n sul campo K, con matrice associata A, e sia h0 un autovalore di f con
molteplicitá algebrica r. Indichiamo con t la dimensione dell’autospazio V0 di f
relativo all’autovalore h0 . Allora t ≤ r.

Dim. Denotiamo p(h) il polinomio caratteristico di f (o equivalentemente di A).


Poiché dim(V0 ) = t, esiste un insieme di vettori {b1 , . . . , bt } che cosituiscono una
base per V0 .
Di conseguenza esistono {et+1 , . . . , en } vettori linearmente indipendenti tali che l’in-
sieme B = {b1 , . . . , bt , et+1 , . . . , en } sia una base per V . Costruiamo la matrice A0 as-
sociata a f rispetto alla base B, ricordando che le colonne di tale matrice saranno
determinate dalle componenti rispetto alla base B dei vettori

{ f (b1 ), . . . , f (bt ), f (et+1 ), . . . , f (en )}.


 
0 D A1
Poiché f (bi ) = h0 bi , avremo che A = , con A1 matrice di ordine (t, n − t),
0 A2
182 8 La forma canonica di un operatore lineare.

A2 matrice di ordine (n − t, n − t) e D matrice diagonale di ordine (t,t):


 
h0 0 0 0 0
 0 h0 0 0 0 
 
D=  0 0 h0 0 0  .

 0 0 0 ... 0 
0 0 0 0 h0

Poiché A e A0 sono simili, esse hanno gli stessi autovalori. In particolare il polinomio
caratteristico di A0 é : p(h) = (h − h0 )t · µ(h), dove µ(h) é il polinomio caratteristico
della matrice A2 . Quindi h0 compare almeno t-volte come radice del polinomio
p(h), di conseguenza la sua molteplicitá algebrica é almeno t, anche come radice
del polinomio caratteristico di A. t
u
Teorema 8.8. Siano h1 , . . . , hm autovalori distinti di f e X1 , . . . , Xm autovettori di f
tali che ogni Xi sia autovettore relativo a hi , per ogni i = 1, . . . , m. Allora i vettori
X1 , . . . , Xm sono linearmente indipendenti.

Dim. Possiamo facilmente dimostrarlo nel caso di 2 autovalori distinti h1 6= h2 (in-


duttivamente si puó estendere al caso generico).
Siano X1 6= 0 autovettore di f relativo a h1 e X2 6= 0 autovettore di f relativo a
h2 . Supponiamo al contrario che essi siano linearmente dipendenti, quindi esista
un α 6= 0 tale che X1 = αX2 . Da f (X1 ) = h1 X1 segue che f (αX2 ) = h1 αX2 , cioé
α f (X2 ) = αh1 X2 . Inoltre f (X2 ) = h2 X2 , quindi otteniamo αh2 X2 = αh1 X2 . Essen-
do α 6= 0 e h1 6= h2 , segue l’assurdo che X2 = 0.
Supponiamo il Teorema vero per ogni insieme di m − 1 autovalori distinti di f , ov-
vero supponiamo che per ogni insieme di m − 1 autovalori distinti, i corrispondenti
m − 1 autovettori siano linearmente indipendenti. Ci proponiamo di dimostrare in-
duttivamente che il Teorema é valido per m autovalori distinti di f . Siano X1 , . . . , Xm
autovettori di f , tali che ciascun Xi sia autovettore relativo all’autovalore λi , per
i = 1, . . . , m. Supponiamo che esistano α1 , . . . , αm ∈ R tali che
m
∑ αi Xi = 0. (8.1)
i=1

Applicando f alla (8.1) otteniamo:


m m
∑ αi f (Xi ) = ∑ αi hi Xi = 0. (8.2)
i=1 i=1

Simultaneamente, moltiplichiamo la (8.1) per h1 :


m
∑ h1 αi Xi = 0. (8.3)
i=1

Sottraendo dalla (8.3) la (8.2) otteniamo:


8.2 Endomorfismi diagonalizzabili. 183
m
∑ αi (h1 − hi )Xi = 0.
i=2

Poiché, per ipotesi induttiva, i vettori X2 , . . . , Xm sono linearmente indipendenti,


avremo che α(h1 − hi ) = 0, per ogni i = 2, . . . , m. Inoltre h2 , . . . , hm sono tutti di-
stinti, per cui α2 = α3 = . . . = αm = 0, da cui, sostituendo nella (8.1), avremo anche
α1 = 0. Se ne deduce quindi che X1 , . . . , Xm sono linearmente indipendenti. t
u
Nei successivi paragrafi applicheremo quanto detto su autovalori ed autovettori di
un endomorfismo per determinare in quali casi esistano opportune basi dello spazio
vettoriale rispetto alle quali la matrice associata sia diagonale oppure triangolare
superiore.

8.2 Endomorfismi diagonalizzabili.

Diremo che l’endomorfismo f é diagonalizzabile se esiste una base di Rn , rispetto


alla quale la matrice associata a f sia diagonale.
Analogamente diremo che una matrice A é diagonalizzabile se é simile ad una ma-
trice diagonale, cioé se esiste una matrice non singolare P tale che P−1 · A · P sia
diagonale. La matrice P sará detta matrice diagonalizzante di A.
In base a quanto fin’ora detto, possiamo dimostrare adesso il seguente:
Teorema 8.9. Sia f : Rn −→ Rn un endomorfismo con matrice associata A, tale da
possedere n autovalori reali. Le seguenti affermazioni sono equivalenti:
1. f é diagonalizzabile;
2. ogni autovalore di f possiede una molteplicitá algebrica ed una geometrica
coincidenti;
3. esiste una base dello spazio Rn che é costituita da n autovettori di f .

Dim. Siano h1 , . . . , hm gli autovalori distinti della matrice A associata all’endo-


morfismo e siano (a1 , a2 , . . . , am ) e (g1 , . . . , gm ) rispettivamente le molteplicitá alge-
briche e quelle geometriche di h1 , h2 , . . . , hm . Ricordiamo che a1 + a2 + . . . + am =
n.
(1) ⇒ (2) Nell’ipotesi in cui la matrice A associata all’endomorfismo sia diago-
nalizzabile, esisterá una matrice non sigolare C tale che
 
h1 0 0 0 0 . . . . . . 0
 0 ∗ 0 0 0 ... ... 0 
 
 0 0 ∗ 0 0 ... ... 0 
 
 0 0 0 ∗ 0 ... ... 0 
A0 = C−1 · A ·C = 
 0 0 0 0 ∗ 0 ... 0 .

 
 ... 0 0 0 ... ... ... 0 
 
 0 0 0 0 ... ... ∗ 0 
0 0 0 0 . . . . . . 0 hm
184 8 La forma canonica di un operatore lineare.

La matrice A0 é diagonale ed ogni autovalore compare, sulla diagonale, tante vol-


te quanto é la sua molteplicitá algebrica.
Scegliamo un qualsiasi autovalore λ ∈ {h1 , . . . , hm }, con molteplicitá algebrica
a. Poiché la matrice A0 − λ I ha rango n − a, allora l’autospazio relativo all’auto-
valore λ ha dimensione n − (n − a) = a, cioé la molteplicitá geometrica di λ é
esattamente pari alla molteplicitá algebrica (ed analogamente per ogni altro au-
tovalore di A).

(2) ⇒ (3) Supponiamo ora che per ogni autovalore hi , la sua molteplicitá alge-
brica ai coincida con la sua molteplicitá geometrica gi = dim(Vi ) (la dimensione
dell’autospazio relativo a hi ). Allora si ha che

dim(V1 ) + dim(V2 ) + . . . + dim(Vm ) = n = dim(V )

cioé
V1 ⊕V2 ⊕ . . . . ⊕Vm = V = Rn
ed inoltre l’unione delle basi di tutti gli autospazi costituisce una base di V = Rn .

(3) ⇒ (1) Infine supponiamo che esista una base di Rn formata da autovettori
di A. Esistono allora X1 , . . . , Xn autovettori in Rn che siano tra loro linearmente
indipendenti. Ricordiamo che gli autovalori sono in numero di n, anche se non
tutti necessariamente distinti. Indichiamo adesso λ1 , . . . , λn tali autovalori, m dei
quali siano distinti (m ≤ n). Allora:

AX1 = λ1 X1 , AX2 = λ2 X2 , AX3 = λ3 X3 , . . . . . . , AXn = λn Xn .

Se indichiamo con B = {X1 , X2 , X3 , X4 , . . . , . . . , Xn } la base di V formata dall’u-


nione delle basi di tutti gli autospazi, allora

f (X1 ) = AX1 = λ1 X1 = (λ1 , 0, 0, 0, . . . , 0)B

f (X2 ) = AX2 = λ2 X2 = (0, λ2 , 0, 0, . . . , 0)B


f (X3 ) = AX3 = λ3 X3 = (0, 0, λ3 , 0, . . . , 0)B
............
f (Xn ) = AXn = λn Xn = (0, 0, 0, 0, . . . , λn )B
per cui la matrice associata all’endomorfismo rispetto alla base B in dominio e
codominio é diagonale:
8.2 Endomorfismi diagonalizzabili. 185
 
λ1 0 0 0 0 ... ... 0
 0 λ2
 0 0 0 ... ... 0  
 0 0
 λ3 0 0 . . . . . . 0  
0
 0 0 0 λ4 0 . . . . . . 0 
A = 
 0 0
 0 0 λ5 0 . . . 0  
 ... 0 0 0 ... ... ... 0 
 
 0 0 0 0 . . . . . . λn−1 0 
0 0 0 0 . . . . . . 0 λn
 
dove C = X1 X2 X3 X4 . . . . . . . . . Xn é la matrice nella cui colonna i sono elen-
cate le componenti dell’autovettore Xi . Quindi rispetto alla base di Rn formata da-
gli autovettori linearmente indipendenti, la matrice associata all’endomorfismo é
diagonale. Le colonne della matrice diagonalizzante sono costituite proprio dagli
autovettori di f che costituiscono una base per Rn .
t
u
Esempio 111. Siano V = R3ef : R3 → R3
tale che f (x1 , x2 , x3 ) = (x1 , x2 , x1 +4x3 ).
La matrice associata a f rispetto alla base canonica di R3 é:
 
100
A =  0 1 0 .
104

Gli autovalori della matrice A sono le soluzioni dell’equazione caratteristica:



1−h 0 0

0 1−h 0 = 0

1 0 4−h

cioé
(1 − h)2 (4 − h) = 0.
Tali soluzioni sono: h1 = 4, con molteplicitá algebrica a1 = 1 e h2 = 1, con
molteplicitá algebrica a2 = 2.
Determiniamo gli autospazi relativi a ciascun autovalore:
Per h1 = 4,  
−3 0 0
A − h1 I =  0 −3 0 
1 0 0
ed il sistema lineare omogeneo associato é:

 −3x1 = 0
−3x2 = 0 .
x1 = 0

Il rango di tale sistema é 2, quindi vi sono ∞1 soluzioni, cioé ∞1 autovettori relativi


a h1 = 4. Essi si ottengono risolvendo il sistema omogeneo: x1 = x2 = 0. Per cui il
186 8 La forma canonica di un operatore lineare.

generico autovettore relativo a h1 é X = (0, 0, a), al variare di a ∈ R, e l’autospazio


associato a h1 é
V1 = {(0, 0, a) ∈ R3 , a ∈ R}
con dim(V1 ) = 1 e V1 =< (0, 0, 1) >.
Per h2 = 1,  
000
A − h2 I =  0 0 0 
103
ed il sistema lineare omogeneo associato é:

x1 + 3x3 = 0 .

Il rango di tale sistema é 1, quindi vi sono ∞2 soluzioni, cioé ∞2 autovettori re-


lativi a h2 = 1. Essi si ottengono risolvendo il sistema omogeneo: x1 = −3x3 . Per
cui il generico autovettore relativo a h2 é X = (−3a, b, a), al variare di a, b ∈ R, e
l’autospazio associato a h2 é

V2 = {(−3a, b, a) ∈ R3 , a, b ∈ R}

con dim(V2 ) = 2 e V2 =< (−3, 0, 1), (0, 1, 0) >.


Per ogni autovalore le molteplicitá algebrica e geometrica coincidono, quindi la
matrice é diagonalizzabile, l’unione delle basi degli autospazi é una base per R3

B = {(0, 0, 1), (−3, 0, 1), (0, 1, 0)}

e la matrice associata a f rispetto a tale base é diagonale.


La matrice diagonalizzante é formata dagli autovettori che costituiscono la base per
R3 :
 
0 −3 0
P = 0 0 1
1 1 0
A0 = P−1 · A · P =
1
     
3 0 1 100 0 −3 0
−1 0 0·0 1 0·0 0 1 =
3
0 10 104 1 1 0
 
400
 0 1 0 .
001
Esempio 112. Siano V = R3 e f : R3 → R3 tale che f (x1 , x2 , x3 ) = (x1 + x3 , x2 +
x3 , x3 ).
La matrice associata a f rispetto alla base canonica di R3 é:
8.2 Endomorfismi diagonalizzabili. 187
 
101
A =  0 1 1 .
001

Gli autovalori della matrice A sono le soluzioni dell’equazione caratteristica:



1−h 0 1

0 1−h 1 = 0

0 0 1−h

cioé
(1 − h)3 = 0.
Tali soluzioni sono: h1 = 1, con molteplicitá algebrica a1 = 3.
Determiniamo l’autospazio relativo all’unico autovalore:
 
001
A − h1 I =  0 0 1 
000

ed il sistema lineare omogeneo associato é:



x3 = 0 .

Il rango di tale sistema é 1, quindi vi sono ∞2 soluzioni, cioé ∞2 autovettori relativi


a h1 = 1. Essi si ottengono risolvendo il sistema omogeneo: x3 = 0. Per cui il ge-
nerico autovettore relativo a h1 é X = (a, b, 0), al variare di a, b ∈ R, e l’autospazio
associato a h1 é
V1 = {(a, b, 0) ∈ R3 , a, b ∈ R}
con dim(V1 ) = 2 e V1 =< (1, 0, 0), (0, 1, 0) >. Le molteplicitá algebrica e geometrica
non coincidono, quindi la matrice non é diagonalizzabile, cioé non esiste alcuna
base rispetto alla quale l’endomorfismo sia rappresentabile attraverso una matrice
diagonale.
Esempio 113. Siano V = R3 e f : R3 → R3 tale che f (x1 , x2 , x3 ) = (2x1 + x3 , 2x2 −
x3 , x1 − x2 + x3 ).
La matrice associata a f rispetto alla base canonica di R3 é:
 
2 0 1
A =  0 2 −1  .
1 −1 1

Gli autovalori della matrice A sono le soluzioni dell’equazione caratteristica:



2−h 0 1

0 2 − h −1 = 0

1 −1 1 − h
188 8 La forma canonica di un operatore lineare.

cioé
(2 − h)(h2 − 3h) = 0.
Tali soluzioni sono: h1 = 2, con molteplicitá algebrica a1 = 1, h2 = 0, con moltepli-
citá algebrica a2 = 1, h3 = 3, con molteplicitá algebrica a3 = 1.
Determiniamo gli autospazi relativi a ciascun autovalore:
Per h1 = 2,  
0 0 1
A − h1 I =  0 0 −1 
1 −1 −1
ed il sistema lineare omogeneo associato é:

x3 = 0
.
x1 − x2 − x3 = 0

Il rango di tale sistema é 2, quindi vi sono ∞1 soluzioni, cioé ∞1 autovettori relativi


a h1 = 2. Essi si ottengono risolvendo il sistema omogeneo: x1 = x2 , x3 = 0. Per cui
il generico autovettore relativo a h1 é X = (a, a, 0), al variare di a ∈ R, e l’autospazio
associato a h1 é
V1 = {(a, a, 0) ∈ R3 , a ∈ R}
con dim(V1 ) = 1 e V1 =< (1, 1, 0) >.
Per h2 = 0,  
2 0 1
A − h2 I =  0 2 −1 
1 −1 1
ed il sistema lineare omogeneo associato é:

 2x1 + x3 = 0
2x2 − x3 = 0 .
x1 − x2 + x3 = 0

Il rango di tale sistema é 2, quindi vi sono ∞1 soluzioni, cioé ∞1 autovettori relativi


a h2 = 0. Essi si ottengono risolvendo il sistema omogeneo: x1 = −x2 = − x23 . Per
cui il generico autovettore relativo a h2 é X = (− 2a , a2 , a), al variare di a ∈ R, e
l’autospazio associato a h2 é
a a
V2 = {(− , , a) ∈ R3 , a ∈ R}
2 2
con dim(V2 ) = 1 e V2 =< (− 12 , 12 , 1) >.
Per h3 = 3,  
−1 0 1
A − h3 I =  0 −1 −1 
1 −1 −2
ed il sistema lineare omogeneo associato é:
8.2 Endomorfismi diagonalizzabili. 189

 −x1 + x3 = 0
−x2 − x3 = 0 .
x1 − x2 − 2x3 = 0

Il rango di tale sistema é 2, quindi vi sono ∞1 soluzioni, cioé ∞1 autovettori relativi a


h3 = 3. Essi si ottengono risolvendo il sistema omogeneo: x1 = −x2 = x3 . Per cui il
generico autovettore relativo a h3 é X = (a, −a, a), al variare di a ∈ R, e l’autospazio
associato a h3 é
V3 = {(a, −a, a) ∈ R3 , a ∈ R}
con dim(V3 ) = 1 e V3 =< (1, −1, 1) >.
Per ogni autovalore le molteplicitá algebrica e geometrica coincidono, quindi la
matrice é diagonalizzabile, l’unione delle basi degli autospazi é una base per R3
1 1
B = {(1, 1, 0), (− , , 1), (1, −1, 1)}
2 2
e la matrice associata a f rispetto a tale base é diagonale:
 
200
A0 =  0 0 0  .
003

La matrice diagonalizzante é formata dagli autovettori che costituiscono la base per


R3 :
1 − 12 1
 

P =  1 12 −1  .
0 1 1
Esempio 114. Siano V = R3 e f : R3 → R3 tale che f (x1 , x2 , x3 ) = (3x1 +x2 , 3x2 , 3x1 ).
La matrice associata a f rispetto alla base canonica di R3 é:
 
310
A =  0 3 0 .
300

Gli autovalori della matrice A sono le soluzioni dell’equazione caratteristica:



3−h 1 1

0 3−h 0 = 0

3 0 −h

cioé
−h(3 − h)2 = 0.
Tali soluzioni sono: h1 = 0, con molteplicitá algebrica a1 = 1, h2 = 3, con moltepli-
citá algebrica a2 = 2,
Determiniamo gli autospazi relativi a ciascun autovalore:
Per h1 = 0,
190 8 La forma canonica di un operatore lineare.
 
310
A − h1 I =  0 3 0 
300
ed il sistema lineare omogeneo associato é:

 3x1 + x2 = 0
3x2 = 0 .
3x1 = 0

Il rango di tale sistema é 2, quindi vi sono ∞1 soluzioni, cioé ∞1 autovettori relativi


a h1 = 0. Essi si ottengono risolvendo il sistema omogeneo: x1 = x2 = 0. Per cui il
generico autovettore relativo a h1 é X = (0, 0, a), al variare di a ∈ R, e l’autospazio
associato a h1 é
V1 = {(0, 0, a) ∈ R3 , a ∈ R}
con dim(V1 ) = 1 e V1 =< (0, 0, 1) >.
Per h2 = 3,  
01 0
A − h2 I =  0 0 0 
3 0 −3
ed il sistema lineare omogeneo associato é:

x2 = 0
.
3x1 − 3x3 = 0

Il rango di tale sistema é 2, quindi vi sono ∞1 soluzioni, cioé ∞1 autovettori relativi


a h2 = 3. Essi si ottengono risolvendo il sistema omogeneo: x2 = 0, x1 = x3 . Per cui
il generico autovettore relativo a h2 é X = (a, 0, a), al variare di a ∈ R, e l’autospazio
associato a h2 é
V2 = {(a, 0, a) ∈ R3 , a ∈ R}
con dim(V2 ) = 1 e V2 =< (1, 0, 1) >. L’autovalore h2 = 3 ha le molteplicitá diffe-
renti, quindi la matrice non é diagonalizzabile.
Osservazione 8.10. Se gli autovalori di un endomorfismo f sono tutti distinti
tra loro, cioé ciascuno di essi ha molteplicitá algebrica 1, allora f é certamente
diagonalizzabile.

8.2.1 Matrici simmetriche reali.

Ci occupiamo infine della classe delle matrici simmetriche reali (ovvero della classe
degli endomorfismi rappresentati da matrici simmetriche reali). Se un endomorfismo
f : Rn → Rn é rappresentato, rispetto ad una qualsiasi base di Rn , da una matrice
simmetrica, esso viene detto operatore simmetrico (reale).
8.2 Endomorfismi diagonalizzabili. 191

Premettiamo la seguente:
Definizione 8.11. Diremo che l’endomorfismo f : Rn → Rn é ortogonalmente dia-
gonalizzabile se esiste una base ortonormale di Rn , rispetto alla quale la matrice
associata a f sia diagonale.
Analogamente diremo che una matrice A é ortogonalmente diagonalizzabile se
esiste una matrice ortogonale U tale che U −1 · A ·U sia diagonale.
In quanto segue faremo uso del prodotto scalare tra vettori dello spazio euclideo
V = Rn : Denoteremo con
   
x1 y1
 x2   y2 
   
X =  ... , Y =  ... 
  
 ...   ... 
xn yn

due qualsiasi vettori in V .


Ricordiamo che il prodotto scalare standard tra X e Y é definito come segue:
n
X T Y = x1 y1 + x2 y2 + . . . + xn yn = ∑ xi yi .
i=1

Inoltre é noto che due vettori sono detti ortogonali se il loro prodotto scalare é nullo.
Infine ricordiamo anche che per ogni spazio euclideo (e per ogni suo sottospazio) é
sempre possibile costruire una base formata da vettori ortogonali tra loro e di norma
pari a 1 (vettori ortonormali).
Consideriamo adesso alcune proprietá delle matrici simmetriche.
Lemma 8.12. Siano λ1 6= λ2 autovalori distinti di una matrice A ∈ Mn (R) sim-
metrica e X1 , X2 due autovettori corrispondenti rispettivamnte a λ1 e λ2 . Allora
X2T X1 = 0 (cioé i due autovettori sono ortogonali).

Dim. Consideriamo AX1 = λ1 X1 e AX2 = λ2 X2 . Moltiplichiamo la prima per X2T a


sinistra e la seconda per X1T sempre a sinistra. Otteniamo quindi

X2T AX1 = λ1 X2T X1 (8.4)

e
X1T AX2 = λ2 X1T X2 . (8.5)
Trasponendo ambo i membri dell (8.5) otteniamo:

X2T AX1 = λ2 X2T X1 . (8.6)

Sottraiamo ora la (8.6) dalla (8.4), ottenendo 0 = (λ1 − λ2 )X2T X1 , da cui X2T X1 = 0.
Si noti che la ortogonalitá tra due vettori implica la loro indipendenza lineare. t
u
Lemma 8.13. Tutti gli autovalori di una matrice simmetrica A ∈ Mn (R) sono reali.
192 8 La forma canonica di un operatore lineare.

Dim. Indichiamo di seguito con il simbolo α ∗ il coniugato di un numero comples-


so α. Inoltre, se  
x1
 x2 
 
X = ... 

 ... 
xn
allora indicheremo col simbolo X ∗ il coniugato di un vettore X, ovvero il trasposto
di X, che abbia inoltre come componenti i coniugati delle componenti di X, cioé

X ∗ = [x1∗ , x2∗ , . . . , . . . , xn∗ ].

Sia λ un autovalore di A, ovvero:

AX = λ X, X 6= 0. (8.7)

Coniugando ambo i membri della (8.7), otteniamo X ∗ A = λ ∗ X ∗ . Moltiplichiamo ora


a destra ambo i membri per X:

X ∗ AX = λ ∗ X ∗ X. (8.8)

Moltiplichiamo ora ambo i membri della (8.7) a sinistra per X ∗ :

X ∗ AX = λ X ∗ X (8.9)

e sottraiamo ora la (8.9) dalla (8.8). Segue che 0 = (λ ∗ − λ )X ∗ X. Poiché X 6= 0,


allora anche X ∗ X 6= 0, da cui λ = λ ∗ , cioé λ ∈ R. t
u
Lemma 8.14. Siano A ∈ Mn (R) una matrice simmetrica e C ∈ Mn (R) una matrice
ortogonale, allora C−1 AC é ancora simmetrica.

Dim. Sia B = C−1 AC. Allora

BT = (C−1 AC)T = (CT AC)T = CT AC = B.

t
u
Lemma 8.15. Siano A ∈ Mn (R) una matrice simmetrica, f : Rn → Rn l’endomorfi-
smo associato ad A, λ ∈ R un autovalore di A e W ≤ Rn l’autospazio associato al-
l’autovalore λ . Detto W ⊥ il complemento ortogonale di W , segue che f (W ⊥ ) ⊆ W ⊥
(ovvero f si puó restringere ad un endomorfismo di W ⊥ ).

Dim. Siano X ∈ W e Y ∈ W ⊥ , ovvero X T Y = Y T X = 0. Poiché X é un autovettore


relativo all’autovalore λ , allora f (X) = AX = λ X. Da ció segue che

Y T AX = Y T (λ X) = λY T X = 0. (8.10)
8.2 Endomorfismi diagonalizzabili. 193

Trasponendo il primo e l’ultimo membro della (8.10), otteniamo X T AY = 0, cioé


f (Y ) = AY ∈ W ⊥ , da cui l’asserto. t
u
I precedenti risultati possono essere utilizzati per dimostrare il seguente:
Teorema 8.16. (Teorema spettrale) Sia f : Rn → Rn un endomorfismo. Esiste una
base ortonormale B = {v1 , . . . , vn } di Rn , costituita da autovettori di f , se e solo se
f é un operatore simmetrico.
In altre parole una matrice A ∈ Mn (R) é ortogonalmente diagonalizzabile se e solo
se essa é simmetrica.
Esempio 115. Siano V = R3 e f : R3 → R3 tale che f (x1 , x2 , x3 ) = (x1 + x2 +
x3 , x1 , x1 ).
La matrice associata a f rispetto alla base canonica di R3 é:
 
111
A = 1 0 0
100

che é simmetrica. Gli autovalori della matrice A sono le soluzioni dell’equazione


caratteristica:
1−h 1 1

1 −h 0 = 0

1 0 −h
cioé
h(h − 2)(h + 1) = 0.
Tali soluzioni sono: h1 = 0, con molteplicitá algebrica a1 = 1, h2 = 2, con molte-
plicitá algebrica a2 = 1, h3 = −1 con molteplicitá algebrica a3 = 1. La matrice é
diagonalizzabile ed una sua forma diagonale é
 
2 0 0
A0 =  0 −1 0 
0 0 0
per ottenere la quale si utilizza la matrice diagonalizzante
 
2 1 0
P =  1 −1 1 
1 −1 −1

le cui colonne sono gli autovettori, tra loro ortogonali, che costituiscono una ba-
se di R3 rispetto alla quale A0 é la matrice associata a f . Per ottenere una base
ortonormale, é sufficiente normalizzare i vettori della base.
194 8 La forma canonica di un operatore lineare.

8.3 La forma canonica di Jordan.

Sia f : Rn −→ Rn un endomorfismo. Abbiamo in precedenza analizzato in quali casi


tale endomorfismo sia diagonalizzabile, cioé quando esista una base di Rn compo-
sta da autovettori di f , rispetto alla quale la matrice associata all’endomorfismo si
presenti in forma diagonale. In sostanza abbiamo visto che, quando f é diagonaliz-
zabile, tutte le matrici che si possono ad esso associare, in una qualsiasi base di Rn ,
sono tra loro simili e tutte simili ad una matrice diagonale. Abbiamo concluso che
non tutti gli endomorfismi, e quindi non tutte le matrici, sono diagonalizzabili. Ci
proponiamo ora di rispondere alla domanda di quale possa essere una forma suffi-
cientemente semplice per poter esprimere un endomorfismo, nel caso in cui esso non
sia diagonalizzabile. A tal fine abbiamo la necessitá di richiamare alcuni risultati e
definizioni relativi ad endomorfismi e matrici.

8.3.1 Forma canonica a blocchi.

Definizione 8.17. Diciamo Blocco di Jordan di ordine p e relativo allo scalare α ∈


R, la matrice  
α 1 0 0 0 0 0 0
 0 α 1 0 0 0 0 0 
 
 0 0 α 1 0 0 0 0 
 
 0 0 0 α 1 0 0 0 
J p (α) = 
 .
 ... ... ... ... ... ... ... ... 

 ... ... ... ... ... ... ... ... 
 
 0 0 0 0 0 0 α 1 
0 0 0 0 0 0 0 α
Descrivendo quindi gli elementi ai j della matrice, avremmo:

α se i= j
ai j = 1 se j = i+1
0 negli altri casi

Definizione 8.18. Diremo che una matrice A ∈ Mn (R) é in forma canonica di Jordan
se essa presenta la seguente forma diagonale a blocchi di Jordan
 
Jn1 (α1 ) 0 0 0 0 0 0 0
 0
 Jn2 (α2 ) 0 0 0 0 0 0  
 0
 0 Jn3 (α 3 ) 0 0 0 0 0 

 0 0 0 Jn4 (α 4 ) 0 0 0 0 
A=  
 . . . . . . . . . . . . . . . . . . . . . . . . 

 ... ... ... ... ... ... ... ... 
 
 0 0 0 0 0 0 Jnr−1 (αr−1 ) 0 
0 0 0 0 0 0 0 Jnr (αr )
8.3 La forma canonica di Jordan. 195

dove ogni Jni (αi ) é un blocco di Jordan di ordine ni relativo ad un qualche scalare
αi ed ovviamente ∑i ni = n.
Esempio 116.  
3 1 0 0 0
0 3 1 0 0  
  J3 (3) 0
A1 = 
0 0 3 0 0 =
0
 0 J2 (2)
0 0 2 1
0 0 0 0 2
Esempio 117.
 
2 1 0 0 0 0
0 2 1 0 0 0  

0
 J3 (2) 0 0
0 2 0 0 0 

A2 = 
0 = 0 J2 (4) 0 
0 0 4 1 0

0
 0 0 J1 (3)
0 0 0 4 0
0 0 0 0 0 3

Esempio 118.
   
3 0 0 0 J1 (3) 0 0 0
 =  0 J1 (2) 0
0 2 0 0  0 
A3 = 
 .
0 0 4 0   0 0 J1 (4) 0 
0 0 0 5 0 0 0 J1 (5)

Dato un operatore lineare reale f : V → V , sullo spazio n-dimensionale reale V ,


il nostro intento é quello di determinare una base per V rispetto alla quale f possa
essere associato ad una matrice in forma canonica di Jordan. É bene sottolineare che
le matrici diagonali sono delle particolari matrici di Jordan, nelle quali ogni blocco
ha ordine 1, cioé con un numero di blocchi pari all’ordine della matrice stessa.
Vale il seguente:
Teorema 8.19. Sia A ∈ Mn (R) una matrice il cui polinomio caratteristico abbia
tutte radici reali. Allora esiste una matrice non singolare C ∈ Mn (R) tale che la
matrice C−1 AC sia in forma canonica di Jordan. In altre parole, per ogni endomor-
fismo f : Rn → Rn , avente tutti gli autovalori nel campo reale, esiste una base di Rn ,
rispetto alla quale, la matrice associata a tale endomorfismo sia in forma canonica
di Jordan.
Allora possiamo inizialmente stabilire che:
–Se f possiede esattamente n autovalori reali allora esiste una base di V rispetto
alla quale la matrice associata a f é in forma canonica di Jordan.
Definizione 8.20. L’endomorfismo f é detto Jordanizzabile se esiste una base di V
rispetto alla quale, la matrice associata a f é in forma canonica di Jordan. Ovvero,
una matrice é detta Jordanizzabile se é simile ad una matrice in forma canonica di
Jordan.
196 8 La forma canonica di un operatore lineare.

La base di V rispetto alla quale l’operatore lineare é Jordanizzabile, é detta base di


Jordan.
Sia quindi A ∈ Mn (R) ed indichiamo con

P(X) = (X − λ1 )r1 · (X − λ2 )r2 · · · ·(X − λh )rh

il suo polinomio caratteristico, in cui λ1 , λ2 , λ3 , . . . , λh sono tutti gli autovalori di A,


tra loro distinti e tali che l’autovalore λi abbia molteplicitá algebrica, come radice
del polinomio caratteristico, pari a ri , con ∑i ri = n. Supponiamo che A0 sia la forma
canonica di Jordan di A. Poiché A0 é triangolare superiore, i suoi autovalori sono
esattamente gli elementi che compaiono sulla sua diagonale principale. D’altro can-
to, A e A0 sono simili, per cui posseggono gli stessi autovalori. Quindi gli scalari
rispetto ai quali i blocchi di Jordan della A0 vengono costruiti sono proprio gli au-
tovalori della matrice A, cioé per ogni autovalore di A esiste almeno un blocco di
Jordan in A0 .
Ritornando al Teorema 8.19, ci occuperemo adesso di trasformare la matrice A nella
sua forma canonica di Jordan A0 . Per farlo, é necessario rispondere alle seguenti
domande:
1. Nella forma canonica finale, quale é il numero totale di blocchi di Jordan relativi
a ciascun autovalore?
2. Quale deve essere l’ordine di ogni singolo blocco?
3. Quale é la base di Rn rispetto alla quale l’operatore lineare é associato ad una
matrice in forma canonica di Jordan?
Iniziamo, rispondendo alla prima:
Teorema 8.21. Sia A ∈ Mn (R) una matrice Jordanizzabile. La molteplicitá geome-
trica di un suo autovalore λ ∈ R é pari al numero totale t di blocchi, relativi a λ ,
presenti nella forma canonica.
In sostanza, per ogni autovalore λi , di molteplicitá geometrica ti , avremo esatta-
mente ti blocchi, ciascuno con la propria dimensione. Mettendo insieme tali blocchi
costruiamo una matrice, la cui dimensione é pari alla molteplicitá algebrica di λi :
 
J1 (λi )
 .. .. 
Ci = 
 . . 
 (8.11)
 ... 
Jti (λi )
in cui ognuno dei blocchi di Jordan Jh (h = 1, . . . ,ti ) ha la forma
 
λi 1
 .. .. 
Jh (λi ) = 
 . .  .
 λi 1 
λi
8.3 La forma canonica di Jordan. 197

Ripetiamo tale argomentazione per ogni autovalore della matrice, avremo la seguen-
te forma canonica di Jordan finale:
 
C1 0 0 0 0 0 0 0
 0 C2 0 0 0 0 0 0 
 
 0 0 C3 0 0 0 0
 
 0 0 0 ... 0 0 0 0 
A0 = 
 0 0 0 0 ... 0 0 0 

 
 0 0 0 0 0 ... 0 0 
 
 0 0 0 0 0 0 Cr−1 0 
0 0 0 0 0 0 0 Cr

in cui ogni blocco Ci é una matrice della forma (8.11) ed é relativa all’autovalore
λi ∈ {λ1 , . . . , λr }.

8.3.2 Autospazi generalizzati.

In tutto ció che segue, siano V uno spazio vettoriale reale di dimensione n, f : V −→
V un operatore lineare reale.
Il primo passo é estendere il concetto di autospazio associato ad un autovalore:
Definizione 8.22. Sia λ ∈ R un autovalore di f . Costruiamo il seguente endomorfi-
smo di V : fλ = f − λ I, dove I : V → V indica l’identitá in V . Un vettore v ∈ V tale
che esista k ≥ 1 per cui fλk (v) = 0, é detto autovettore generalizzato di f associato
all’autovalore λ .
Il piú piccolo h ≥ 1 tale che fλh (v) = 0, é detto esponente dell’autovettore generaliz-
zato v.
Sia λ ∈ R un autovalore di f ed introduciamo i seguenti endomorfismi di V :

fλ0 = ( f − λ I)0 = I, fλ1 = f − λ I, fλ2 = fλ ( fλ ), . . . fλt = fλ ( fλt−1 ).

Ciascuno di essi ha un nucleo:

ker( fλi ) = {v ∈ V : fλi (v) = 0}.

La prima osservazione che possiamo fare é certamente la seguente:

(0) = ker( fλ0 ) ⊂ ker( fλ ) ⊂ ker( fλ2 ) ⊂ . . . . . . ⊂ ker( fλs ) ⊂ . . .

é una catena di sottospazi di V . Poiché quest’ultimo ha dimensione finita, tale catena


non puó essere infinita, quindi esiste un certo s ≥ 1 tale che

ker( fλs ) = ker( fλs+1 )

da cui
198 8 La forma canonica di un operatore lineare.

ker( fλs ) = ker( fλt ) ∀t ≥ s.


Definizione 8.23. Siano λ ∈ R un autovalore di f , fλ = f − λ I, s ≥ 1 il minimo
intero tale che ker( fλs ) = ker( fλt ), per ogni t ≥ s. Il sottospazio

ker( fλs ) = {X ∈ V : ∃k ≤ s, fλk (X) = 0}

é detto autospazio generalizzato di f relativo all’autovalore λ . L’intero s é detto


indice dell’autovalore λ .
Osservazione 8.24. Gli elementi di ker( fλs ) precedentemente definito, sono esatta-
mente gli autovettori generalizzati associati all’autovalore λ . L’indice s dell’autova-
lore λ é esattamente il piú grande tra gli esponenti degli autovettori generalizzati ad
esso relativi.
Per semplicitá di notazione, da ora in poi indicheremo con il termine Eλ l’autospazio
generalizzato ker( fλs ) associato all’autovalore λ di f .
Lemma 8.25. Se λ é un autovalore di f , allora dim(Eλ ) é uguale alla molteplicitá
algebrica di λ .
Teorema 8.26. V é somma diretta degli autospazi generalizzati relativi agli auto-
valori distinti di f .
Alla luce dei risultati appena citati, il nostro obiettivo sará quello di determinare una
opportuna base per ciascuno degli autospazi generalizzati di f . L’unione di tali basi
é una base per V e mostreremo che la matrice associata a f , rispetto a quest’ultima
base, é in forma canonica di Jordan.
Teorema 8.27. L’indice di un autovalore é pari all’ordine massimo di un blocco di
Jordan ad esso relativo, nella forma canonica finale.
Osservazione 8.28. Segue da quanto appena detto che la molteplicitá algebrica di
un autovalore é maggiore o uguale del proprio indice.
Sia quindi A ∈ Mn (R) e λ un suo autovalore di molteplicitá algebrica t. Fissiamo
l’intero k ≤ t. Ci chiediamo quanti blocchi di ordine k sono associati all’autovalore
λ.
Useremo la seguente notazione:

ri = rango(A − λ I)i , per ogni i = 1, 2, 3, . . . ,t.

Se k = 1 allora il numero di blocchi di ordine 1 relativi a λ é pari a

n − 2r1 + r2 .

Per k ≥ 2 vale la seguente tabella, in cui sono riportati a sinistra gli ordini di tutti i
blocchi che eventualmente si possono presentare, e a destra il numero di tali blocchi:
8.3 La forma canonica di Jordan. 199

ORDINE DEI BLOCCHI NUMERO DEI BLOCCHI


2 r1 − 2r2 + r3
3 r2 − 2r3 + r4
4 r3 − 2r4 + r5
5 r4 − 2r5 + r6
6 r5 − 2r6 + r7
... ......
... ......
... ......
k rk−1 − 2rk + rk+1
Una volta ottenuto il numero totale di blocchi relativi ad un autovalore λ ed inoltre
la distribuzione delle dimensioni di ciascun blocco, non ci resta che determinare una
base per l’autospazio generalizzato Eλ .
Vale il seguente:
Teorema 8.29. Sia λ ∈ R un autovalore
  lineare f : V → V , e sia vr ∈
dell’operatore
r
V tale che vr ∈ Ker ( f − λ I) − Ker ( f − λ I)r−1 (ovvero sia vr un autovettore
generalizzato corrispondente all’autovalore λ , di esponente r). Poniamo fλ = f −
λ I e costruiamo la seguente catena di vettori in V :

vr−1 = fλ (vr ) vr−2 = f λ (vr−1 ) = fλ2 (vr )

vr−3 = fλ (vr−2 ) = fλ3 (vr ), . . . . . . v1 = fλ (v2 ) = fλr−1 (vr ) 6= 0.


Allora:
1. vr−i ∈ Ker( fλr−i ) − Ker( fλr−i−1 ), per ogni i = 1, . . . , r − 1.
2. I vettori {v1 , v2 , . . . , vr } sono linearmente indipendenti.
Consideriamo quindi la catena di autovettori generalizzati sopra introdotta: {v1 , . . . , vr }.
Completiamo tale insieme di vettori ad una base di V

B = {v1 , . . . , vr , w1 , . . . , wn−r }.

Osserviamo che:
fλ (v1 ) = fλr (vr ) = 0 =⇒ f (v1 ) = λ v1
per cui le componenti di f (v1 ) rispetto a B sono (λ , 0, . . . , 0).

f (v2 ) = v1 + λ v2

per cui le componenti di f (v2 ) rispetto a B sono (1, λ , 0, . . . , 0).

f (v3 ) = v2 + λ v3

per cui le componenti di f (v3 ) rispetto a B sono (0, 1, λ , 0, . . . , 0).


In generale
f (vi ) = vi−1 + λ vi , 2 ≤ i ≤ r
200 8 La forma canonica di un operatore lineare.

per cui le componenti di f (vi ) rispetto a B sono ( 0, . . . , 0 , 1, λ , 0, . . . , 0 ), al va-


| {z } | {z }
(i−2)−volte (n−i)−volte
riare di 2 ≤ i ≤ r.
La matrice associata a f rispetto alla base B avrá quindi r colonne che esprimono le
immagini dei vettori {v1 , . . . , vr }. Per quanto visto, tali immagini compongono un
blocco della di ordine r avente seguente forma
 
λ 1
 .. .. 

 . .  .
 λ 1
λ
Definizione 8.30. L’insieme {v1 , v2 , . . . , vr }, composto dai vettori definiti nel Teo-
rema 8.29 é chiamato catena di autovettori generalizzati relativi all’autovalore λ di
lunghezza r.
Il vettore vr ∈ Ker( fλr ) − Ker( fλr−1 ) da cui ha origine la catena é detto generatore
della catena.
In presenza di piú catene di autovettori generalizzati relative allo stesso autovalore,
la massima lunghezza di una di tali catene é pari all’indice dell’autovalore al quale
esse si riferiscono.
Ció vuol dire che, in corrispondenza di un qualsiasi blocco relativo ad un dato auto-
valore, é possibile costruire una opportuna catena di autovettori generalizzati, asso-
ciati a tale autovalore, da inserire in una base di Jordan. La lunghezza di ogni catena
é pari alla dimensione del blocco cui si riferisce.
Per cui, costruendo una catena per ciascuno dei blocchi di ogni autovalore e unendo
tutte le catene ottenute, si arriva alla composizione di una base di Jordan.
Riassumiamo i passi da compiere per ottenere la forma canonica di una matrice rea-
le di ordine n (o endomorfismo di uno spazio reale n-dimensionale), nell’ipotesi che
essa abbia n autovalori reali.
1. Siano λ1 , . . . , λ p ∈ R gli autovalori distinti di A, E1 , . . . , E p i rispettivi autospazi
generalizzati. Sia mi la molteplicitá algebrica dell’autovalore λi , per i = 1, . . . , r.
2. Puntiamo l’attenzione su un fissato autovalore λ ∈ {λ1 , . . . , λ p }. Sia m la sua
molteplicitá algebrica e sia E il corrispondente autospazio generalizzato.
3. Dalla molteplicitá geometrica dell’autovalore λ otteniamo il numero totale di
blocchi di Jordan ad esso relativi.
4. L’indice dell’autovalore λ é pari al massimo ordine di un blocco di Jordan ad
esso relativo.
5. Detto k tale indice, possiamo calcolare il numero di blocchi di Jordan relativi a
λ e di ordine h, per ogni h ≤ k.
6. A questo punto abbiamo determinato l’insieme dei blocchi di Jordan relativi a λ .
7. Al fine di costruire una base di Jordan, riordiniamo i blocchi dal piú grande fi-
no al piú piccolo e costruiamo per ciascuno di essi la corrispondente catena di
autovettori generalizzati. Ad un blocco di ordine h corrisponderá una catena di
lunghezza h.
8.3 La forma canonica di Jordan. 201

8. L’unione di tutti i vettori presenti nelle catene ottenute é un insieme di m auto-


vettori generalizzati, i quali costituiscono la base per l’autospazio generalizzato
E.
9. Ripetiamo questo processo per ciascun autovalore λ1 , . . . , λ p , determinando quin-
di una base per ciascuno dei corrispondenti E1 , . . . , E p autospazi generalizzati.
10. Riuniamo ora in un unico insieme B i vettori che formano la base di ogni
autospazio generalizzato E1 , . . . , E p . Avremo ottenuto in tal modo

V = E1 ⊕ E2 ⊕ . . . ⊕ E p

ed una base B rispetto alla quale la matrice associata ad f si esprime in forma


canonica di Jordan.
Definizione 8.31. Un endomorfismo f : V −→ V é detto nilpotente se esiste t ≥ 1
tale che f t é l’endomorfismo nullo, cioé se f t (v) = 0, per ogni v ∈ V . Si dice indice
di nilpotenza di f , il minimo t ≥ 1, tale che f t = 0 (analogamente t é l’indice di
nilpotenza della matrice A associata a f , cioé At = 0 e As 6= 0, per ogni s < t).
Lemma 8.32. Sia V uno spazio vettoriale di dimensione n sul campo K e sia f :
V −→ V un endomorfismo. Se f é nilpotente allora il suo polinomio caratteristico
é p(λ ) = (−1)n λ n , cioé f ammette come unico autovalore lo 0 con molteplicitá n.
Osservazione 8.33. Sia λ l’unico autovalore della matrice A ∈ Mn (R), con molte-
plicitá algebrica n. Quindi il polinomio caratteristico é det(A − λ I)n e si consideri
B = A − λ I, dove al solito I sia la matrice identica di ordine n. Supponiamo che
Bm = 0 per qualche intero m (come noto, si dice in tal caso che la matrice B é nilpo-
tente). Sia r = min{t ∈ N : Bt = 0}, l’indice di nilpotenza di B.
Allora esisterá certamente almeno un blocco di Jordan relativo a λ che abbia ordine
r ed ogni altro eventuale blocco di Jordan relativo a λ avrá un ordine minore o al
piú uguale a r.
Sará utile adesso un richiamo al ben noto Teorema di Caley-Hamilton:
Teorema 8.34. Ogni matrice quadrata A di un certo ordine n é una radice del suo
polinomio caratteristico p(λ ) = det(A − λ I). In altre parole se

p(λ ) = a0 + a1 λ + a2 λ 2 + . . . + an λ n

allora
p(A) = a0 I + a1 A + a2 A2 + . . . + an An = 0.

Dim. Sia A associata all’endomorfismo f : V → V , dove V é uno spazio vetto-


riale reale di dimensione n. Riscriviamo il polinomio caratteristico di A nel modo
seguente
p(λ ) = (λ − λ1 )n1 · (λ − λ2 )n2 · · · · · · (λ − λt )nt
dove {λ1 , . . . , λt } siano gli autovalori distinti di A, con molteplicitá algebriche
n1 , . . . , nt . Per ciascuno degli autovalori λi , sia ei il corrispondente indice, ovvero
202 8 La forma canonica di un operatore lineare.
 
sia Ker ( f − λi I)ei l’autospazio generalizzato associato a λi .
Dal Teorema 8.26
   
e1 et
V = Ker ( f − λ1 I) ⊕ · · · · · · ⊕ Ker ( f − λt I) .

Quindi, per ogni v ∈ V esistono


     
v1 ∈ Ker ( f −λ1 I)e1 , v2 ∈ Ker ( f −λ2 I)e2 , ...... vt ∈ Ker ( f −λt I)et

 
tali che v = v1 + · · · · · · + vt . Inoltre, per ogni vi ∈ Ker ( f − λi I)ei abbiamo che

( f − λi I)ni vi = 0 poiché ni ≥ ei

e  
nj ei
( f − λ j I) vi ∈ Ker ( f − λi I) ∀ j 6= i.

Da ció segue che, per ogni v ∈ V ,


 n1  n2  nt
A − λ1 I · A − λ2 I · · · · · · A − λt I ·v = 0

ovvero, la matrice P(A) ∈ Mn (R) é tale che P(A)v = 0, per ogni v ∈ V . Da qui segue
facilmente che P(A) = 0. t
u
Teorema 8.35. Sia V uno spazio vettoriale di dimensione n sul campo K e sia
f : V −→ V un endomorfismo. Esso é nilpotente se e solo se il suo polinomio ca-
ratteristico é p(λ ) = (−1)n λ n , cioé se ammette come unico autovalore lo 0 con
molteplicitá n.

Dim. Sia f nilpotente. La conclusione che il suo polinomio caratteristico sia


p(λ ) = (−1)n λ n deriva dal Lemma 8.32.
Al contrario, supponiamo che A ammetta come unico autovalore λ = 0. In tale ca-
so il polinomio caratteristico di A é p(λ ) = λ n . Infine, 0 = P(A) = An segue dal
Teorema di Caley-Hamilton. t
u

8.3.3 Il polinomio minimo di una matrice (o di un endomorfismo).

Sia A ∈ Mn (R) una matrice quadrata di ordine n ad elementi reali e sia f : Rn −→ Rn


l’endomorfismo ad essa associato. Indichiamo al solito p(λ ) = det(A − λ I) il poli-
nomio caratteristico di A (o equivalentemente di f ). Alla luce del Teorema 8.34 vi
sono chiaramente infiniti polinomi f (x) di grado superiore ad n tali da avere la ma-
8.3 La forma canonica di Jordan. 203

trice A come radice, sono esattamente tutti i polinomi che ammettano il polinomio
p(x) come divisore, cioé quelli del tipo f (x) = p(x) · h(x), per ogni polinomio h(x).
Ci chiediamo allora se esistono polinomi di grado inferiore a n che ammettano la
matrice A come radice. Essi possono esistere, ma non necessariamente.
Definizione 8.36. Il polinomio minimo di A é il polinomio monico (con coefficiente
1 per il monomio di grado massimo) di grado minimo che ammetta A come radice.
Se ne deduce che il polinomio minimo di A deve essere cercato tra tutti i polinomi di-
visori del polinomio caratteristico di A. Se nessuno di tali polinomi dovesse ammet-
tere A come radice, concluderemmo che il polinomio minimo e quello caratteristico
coincidono. Vale inoltre il seguente:
Teorema 8.37. Il polinomio minimo m(λ ) di A deve contenere tutti gli zeri del
polinomio caratteristico p(λ ).
Quindi un metodo per determinare il polinomio minimo m(λ ) é quello di scomporre
in fattori p(λ )
p(λ ) = (λ − λ1 )h1 (λ − λ2 )h2 . . . .(λ − λr )hr
con h1 + h2 + . . . + hr = n, e considerare dapprima il polinomio composto dal
prodotto dei fattori lineari di p:

m1 = (λ − λ1 )(λ − λ2 ) . . . .(λ − λr ).

Se la matrice A é radice di m1 , allora esso é il polinomio minimo. In caso contrario


si aumentano gradualmente ed alternativamente tutti i gradi dei monomi (λ − λi ),
fino a quando non si ottiene un polinomio del tipo

m(λ ) = (λ − λ1 )t1 (λ − λ2 )t2 . . . .(λ − λr )tr

per cui A sia una radice, con 1 ≤ ti ≤ hi , per ogni i = 1, . . . , r. Inoltre ogni esponente
ti é il massimo ordine dei blocchi di Jordan in cui si distribuisce l’autovalore hi nella
forma canonica finale della matrice.
Appare chiaro che, per matrici di ordini relativamente grandi, costruire tutti i possi-
bili divisori di p(λ ) e verificare quale sia il minimo, potrebbe risultare complicato.
É quindi consigliabile risalire al polinomio minimo di una matrice affidandoci ad un
altro metodo. A tal proposito, viene in aiuto il seguente:
Teorema 8.38. Sia f : V → V un operatore lineare, dove V é uno spazio vettoriale
reale di dimensione n. Siano {λ1 , . . . , λt } gli autovalori distinti di A, con moltepli-
citá algebriche n1 , . . . , nt . Per ciascuno degli autovalori λi , sia ei il corrispondente
indice. Allora
m(λ ) = (λ − λ1 )e1 · · · · · · (λ − λt )et
é il polinomio minimo di f .
Esempio 119. Sia f : R4 −→ R4 con matrice A ∈ M4 (R) associata
204 8 La forma canonica di un operatore lineare.
 
0 1 1 0
0 0 1 0
A=
 .
0 0 0 0
0 0 0 0

Il polimomio caratteristico é p(λ ) = λ 4 , ovvero l’unico autovalore é λ = 0 con


molteplicitá algebrica 4. L’autospazio associato allo 0 é:

ker( f ) = {X ∈ R4 : AX = 0} =< (1, 0, 0, 0), (0, 0, 0, 1) >

per cui avremo 2 blocchi di Jordan presenti nella forma canonica finale. Per deter-
minare l’ordine di tali blocchi, iniziamo con il calcolo del numero degli eventuali
blocchi di ordine 1. É necessario calcolare il rango della matrice A2 , poiché giá
conosciamo il rango di A, che é pari a 2. Poiché
 
0010
0 0 0 0
A2 = 0 0 0 0

0000

allora il suo rango é 1. Quindi il numero di blocchi di ordine 1 é esattamente


4 − 4 + 1 = 1. Avremo quindi un blocco di ordine 1 ed un blocco di ordine 3. Il
polinomio minimo é allora m(λ ) = λ 3 e le catene di autovettori sono 2, una di
lunghezza 3 e l’altra di lunghezza 1. Per costruirle si devono calcolare i sottospazi
ker( f 2 ), ker( f 3 ), poiché ker( f ) é giá stato analizzato. Avremo:

ker( f 2 ) = {X ∈ R4 : A2 X = 0} =< (1, 0, 0, 0), (0, 1, 0, 0), (0, 0, 0, 1) >

inoltre A3 = 0 per cui

ker( f 3 ) = R4 =< (1, 0, 0, 0), (0, 1, 0, 0), (0, 0, 1, 0), (0, 0, 0, 1) > .

Iniziamo dalla catena di ordine 3:

v3 = (0, 0, 1, 0) ∈ ker( f 3 ) − ker( f 2 )

v2 = f (v3 ) = (1, 1, 0, 0) ∈ ker( f 2 ) − ker( f )


v1 = f (v2 ) = f 2 (v3 ) = (1, 0, 0, 0).
Passiamo ora alla catena di ordine 1, per la quale é sufficiente scegliere un vettore
v01 ∈ ker( f )− < v1 , v2 , v3 >, prendiamo v01 = (0, 0, 0, 1). Possiamo ora definire la
base rispetto alla quale la matrice A ammette la forma canonica di Jordan:

B = {(1, 0, 0, 0), (1, 1, 0, 0), (0, 0, 1, 0), (0, 0, 0, 1)}.

La matrice
8.3 La forma canonica di Jordan. 205
 
1 1 0 0
0 1 0 0
P=
 
0 0 1 0
0 0 0 1
é tale che  
1 1 0 0
0 1 1 0
P−1 AP = 


0 0 1 0
0 0 0 1
sia la forma canonica di Jordan di A.
Esempio 120. Sia f : R3 −→ R3 con matrice A ∈ M3 (R) associata
 
200
A =  2 2 0 .
132

Lasciamo al lettore la verifica che il polimomio caratteristico e quello minimo coin-


cidono: p(λ ) = m(λ ) = (λ − 2)3 . L’unico autovalore é λ = 2 con una sola catena
di autovettori una di lunghezza 3. Per costruirla si devono calcolare i sottospazi
ker( f − 2I), ker ( f − 2I)2 , ker ( f − 2I)3 .
 

ker( f − 2I) = {X ∈ R3 : (A − 2I)X = 0} =< (0, 0, 1) >


 
000
Inoltre (A − 2I)2 =  0 0 0 , quindi
600

ker ( f − 2I)2 = {X ∈ R3 : (A − 2I)2 X = 0} =< (0, 1, 0), (0, 0, 1) > .




Infine (A − 2I)3 = 0 da cui

ker ( f − 2I)3 = R3 =< (1, 0, 0), (0, 1, 0), (0, 0, 1) > .




Costruiamo la catena di ordine 3:

v3 = (1, 0, 0) ∈ ker ( f − 2I)3 − ker ( f − 2I)2


 

v2 = ( f − 2I)(v3 ) = (0, 2, 1) ∈ ker ( f − 2I)2 − ker f − 2I


 

v1 = ( f − 2I)(v2 ) = ( f − 2I)2 (v3 ) = (0, 0, 6).


La base rispetto alla quale la matrice A ammette la forma canonica di Jordan:

B = {(0, 0, 6), (0, 2, 1), (1, 0, 0)}

e la matrice P tale che P−1 AP = A0 sia la matrice in forma di Jordan é:


206 8 La forma canonica di un operatore lineare.
 
001
P = 0 2 0
610
con  
210
A0 =  0 2 1 
002
Esempio 121. Sia f : R5 −→ R5 con matrice A ∈ M5 (R) associata
 
11111
0 1 0 1 1
 
A=  0 0 1 0 1 .

0 0 0 1 0
00001

Il polimomio caratteristico é p(λ ) = (λ − 1)5 . Poiché

ker( f − I) = {X ∈ R5 : (A − I)X = 0} =< (1, 0, 0, 0, 0), (0, 1, −1, 0, 0) >

avremo 2 blocchi di Jordan. Inoltre:


 
0 0 0 1 2
0 0 0 0 0
(A − I)2 = 
 
0 0 0 0 0
0 0 0 0 0
0 0 0 0 0

e (A − I)3 = 0. Quindi avremo un blocco di ordine 3 ed uno di ordine 2, per cui la


forma canonica finale sará  
11000
0 1 1 0 0
0
 
A =  0 0 1 0 0 .

0 0 0 1 1
00001
Per l’autovalore λ = 1 avremo una catena di ordine 3 ed una di ordine 2. Calcoliamo
la sequenza di tutti i sottospazi generalizzati relativi a λ = 1:

ker( f − I) = {X ∈ R5 : (A − I)X = 0}
= < (1, 0, 0, 0, 0), (0, 1, −1, 0, 0) >

ker ( f − I)2 = {X ∈ R5 : (A − I)2 X = 0}




= < (1, 0, 0, 0, 0), (0, 1, 0, 0, 0), (0, 0, 1, 0, 0), (0, 0, 0, −2, 1) >

ed infine, poiché (A − I)3 = 0,


8.3 La forma canonica di Jordan. 207

ker ( f − I)3 = {X ∈ R5 : (A − I)3 X = 0}




= R5
= < (1, 0, 0, 0, 0), (0, 1, 0, 0, 0), (0, 0, 1, 0, 0), (0, 0, 0, 1, 0), (0, 0, 0, 0, 1) > .

Iniziamo dalla catena di ordine 3:

v3 = (0, 0, 0, 0, 1) ∈ ker ( f − I)3 − ker ( f − I)2


 

v2 = ( f − I)(v3 ) = (1, 1, 1, 0, 0)
v1 = ( f − I)(v2 ) = ( f − I)2 (v3 ) = (2, 0, 0, 0, 0).
Passiamo ora alla catena di ordine 2: il vettore di partenza v02 deve essere scelto in
modo che

v02 ∈ ker ( f − I)2 − ker f − I − < (0, 0, 0, 0, 1), (1, 1, 1, 0, 0), (2, 0, 0, 0, 0) > .
 

La nostra scelta é quindi v02 = (0, 0, 0, −2, 1), da cui

v01 = ( f − I)(v02 ) = (−1, −1, 1, 0, 0)

Possiamo ora definire la base rispetto alla quale la matrice A ammette una forma
canonica di Jordan A0 :

B = {(2, 0, 0, 0, 0), (1, 1, 1, 0, 0), (0, 0, 0, 0, 1), (−1, −1, 1, 0, 0), (0, 0, 0, −2, 1)}.

Esempio 122. Sia f : R5 −→ R5 con matrice A ∈ M5 (R) associata


 
10000
1 2 1 1 1
 
A=  0 0 2 0 1 .

0 0 0 1 0
10011

Il polimomio caratteristico é p(λ ) = (λ − 2)2 (λ − 1)3 . quindi avremo un autovalore


λ1 = 1 ed uno λ2 = 2. Calcoliamo la sequenza di tutti i sottospazi generalizzati
relativi a λ = 1:

ker( f − I) = {X ∈ R5 : (A − I)X = 0} =< (1, 0, 0, −1, 0), (0, 0, 1, 0, −1) > .

Da questo ricaviamo anche che l’autospazio ha dimensione 2, quindi vi sono due


blocchi di Jordan per λ1 = 1, da cui riconosciamo
 la presenza
 di una catena di
00000
2 1 2 2 2
2
 
ordine 2 ed una di ordine 1. Inoltre (A − I) = 
 1 0 1 1 1 , quindi

0 0 0 0 0
00000
208 8 La forma canonica di un operatore lineare.

ker ( f − I)2 = {X ∈ R5 : (A − I)2 X = 0}




= < (1, 0, 0, 0, −1), 0, 0, 1, 0, −1), (0, 0, 0, 1, −1) > .


Iniziamo dalla catena di ordine 2:

v3 = (0, 0, 0, 1, −1) ∈ ker ( f − I)2 − ker( f − I)




v2 = ( f − I)(v3 ) = (0, 0, −1, 0, 1).


Passiamo ora alla catena di ordine 1. Il vettore deve essere scelto in

ker( f − I)− < (0, 0, −1, 0, 1), (0, 0, 0, 1, −1) > .

La nostra scelta ricade quindi su (1, 0, 0, −1, 0).


Consideriamo ora l’autovalore λ2 = 2:

ker( f − 2I) = {X ∈ R5 : (A − 2I)X = 0} =< (0, 1, 0, 0, 0) > .

Poiché vi é un solo blocco relativo a tale autovalore, esso sará di ordine 2. Vi é allora
una catena di ordine 2 di autovettori generalizzati. Dal fatto che
 
1 00 0 0
 0 0 0 −1 0 
2
 
(A − 2I) =   1 0 0 0 −1 

 0 00 1 0 
−2 0 0 0 1

segue che

ker ( f − 2I)2 = {X ∈ R5 : (A − 2I)2 X = 0} =< (0, 1, 0, 0, 0), (0, 0, 1, 0, 0) > .




Da cui, le scelte per la catena sono:

v02 = (0, 0, 1, 0, 0) ∈ ker ( f − 2I)2 − ker( f − 2I)




v01 = ( f − 2I)(0, 0, 1, 0, 0) = (0, 1, 0, 0, 0).


Possiamo ora definire la base R5 rispetto alla quale la matrice A ammette una forma
canonica di Jordan:

B = {(1, 0, 0, −1, 0), (0, 0, −1, 0, 1), (0, 0, 0, 1, −1), (0, 1, 0, 0, 0), (0, 0, 1, 0, 0)}.

In relazione all’ordine scelto per i vettori nella base B, la forma canonica sará
 
10000
0 1 1 0 0
 
 0 0 1 0 0 .
 
0 0 0 2 1
00002
8.3 La forma canonica di Jordan. 209

Esempio 123. Sia f : R3 −→ R3 con matrice A ∈ M3 (R) associata


 
311
A =  0 3 1 .
003

Il polimomio caratteristico é p(λ ) = (λ − 3)3 . L’unico autovalore é λ = 3:

ker( f − 3I) = {X ∈ R3 : (A − 3I)X = 0} =< (1, 0, 0) > .

Vi é un solo blocco di Jordan e quindi una sola catena, chiaramente di ordine 3.


Inoltre  
001
(A − 3I)2 =  0 0 0 
000
quindi

ker ( f − 3I)2 = {X ∈ R3 : (A − 3I)2 X = 0} =< (1, 0, 0), (0, 1, 0) > .




Infine (A − 3I)3 = 0 da cui

ker ( f − 3I)3 = R3 =< (1, 0, 0), (0, 1, 0), (0, 0, 1) > .




Costruiamo la catena di ordine 3:

v3 = (0, 0, 1) ∈ ker ( f − 3I)3 − ker ( f − 3I)2


 

v2 = ( f − 3I)(0, 0, 1) = (1, 1, 0) ∈ ker ( f − 3I)2 − ker( f − 3I)




v1 = ( f − 3I)(1, 1, 0) = ( f − 3I)2 (v3 ) = (1, 0, 0).


Una base rispetto alla quale la matrice A ammette una forma canonica di Jordan:

B = {(1, 0, 0), (1, 1, 0), (0, 0, 1)}.

In relazione a tale base, la forma canonica é


 
310
 0 3 1 .
003

Esempio 124. Sia f : R3 −→ R3 con matrice A ∈ M3 (R) associata


 
1 1 −1
A =  0 1 0 .
92 7

Il polimomio caratteristico: p(λ ) = (λ − 1)(λ − 4)2 . Per l’autovalore λ1 = 1 non


210 8 La forma canonica di un operatore lineare.

abbiamo alcun problema ad individuare l’autovettore (che in tal caso é autovalore


in senso ordinario e generalizzato):

ker( f − I) = {X ∈ R3 : (A − I)X = 0} =< (8, −9, −9) > .

Passiamo all’autovalore λ4 = 4:

ker( f − 4I) = {X ∈ R3 : (A − 4I)X = 0} =< (1, 0, −3) >

per cui vi é un solo blocco di ordine 2 relativo all’autovalore 4, e quindi anche una
sola catena di autovettori di lunghezza 2. Inoltre
 
080
(A − 4I)2 =  0 9 0 
090

quindi

ker ( f − 4I)2 = {X ∈ R3 : (A − 4I)2 X = 0} =< (1, 0, 0), (0, 0, 1) >




Costruiamo la catena di ordine 2:

v2 = (1, 0, 0) ∈ ker ( f − 4I)2 − ker( f − 4I)




v1 = ( f − 4I)(1, 0, 0) = (−3, 0, 9).


Una base rispetto alla quale la matrice A ammette una forma canonica di Jordan é:

B = {(−3, 0, 9), (1, 0, 0), (8, −9, −9)}.

In relazione a tale base, la forma canonica é


 
410
 0 4 0 .
001

Esempio 125. Sia f : R3 −→ R3 con matrice A ∈ M3 (R) associata


 
2 0 0
A =  0 1 2 .
0 −2 −3

Il polimomio caratteristico: p(λ ) = (λ − 2)(λ + 1)2 . Per l’autovalore λ2 = 2 non


abbiamo alcun problema ad individuare l’autovettore (che in tal caso é autovalore
in senso ordinario e generalizzato):

ker( f − 2I) = {X ∈ R3 : (A − 2I)X = 0} =< (1, 0, 0) > .


8.3 La forma canonica di Jordan. 211

Passiamo all’autovalore λ1 = −1:

ker( f + I) = {X ∈ R3 : (A + I)X = 0} =< (0, 1, −1) >

Vi é un solo blocco di ordine 2 relativo all’autovalore −1, e quindi anche una sola
catena di autovettori di lunghezza 2. Inoltre
 
100
(A + I)2 =  0 0 0 
000

quindi

ker ( f + I)2 = {X ∈ R3 : (A + I)2 X = 0} =< (0, 1, 0), (0, 0, 1) >




Costruiamo la catena di ordine 2:

v2 = (0, 1, 0) ∈ ker ( f + I)2 − ker( f + I)




v1 = ( f + I)(0, 1, 0) = (0, 2, −2).


Una base rispetto alla quale la matrice A ammette una forma canonica di Jordan:

B = {(0, 2, −2), (0, 1, 0), (1, 0, 0)}.

In relazione a tale base, la forma canonica é


 
−1 1 0
 0 −1 0  .
0 0 2

Osservazione 8.39. L’ordine con cui i blocchi compaiono nella forma canonica di
Jordan non é fondamentale per l’individuazione della forma canonica stessa. Si dice
infatti che la forma canonica ottenuta é unica a meno di permutazioni dei blocchi di
Jordan di cui é composta. Permutando tali blocchi si ottiene comunque una forma
canonica che sia simile a quella precedente. Tale permutazione corrisponde di fatto
alla scelta di una differente base di Rn rispetto alla quale la matrice che rappresenta
l’endomorfismo é ancora espressa in forma canonica di Jordan. Si tratta in sostanza
di permutare fra loro le catene di autovettori generalizzati che costituiscono la ba-
se, facendo attenzione a mantenere invariato l’ordine dei vettori all’interno di ogni
singola catena.
212 8 La forma canonica di un operatore lineare.

8.4 Esercizi svolti.

Esercizio 63. Determinare gli autovalori, una base per gli autospazi, una forma
canonica ed il polinomio minimo della seguente matrice:
 
2 −3 0
A =  −1 0 0  .
−1 1 1

Svolgimento: Gli autovalori della matrice A sono le soluzioni dell’equazione


caratteristica:
2−λ −3 0

−1 −λ 0 = 0

−1 1 1−λ
cioé
(1 − λ )(−1 − λ )(3 − λ ) = 0.
Tali soluzioni sono: λ1 = 3, λ2 = 1, λ3 = −1, ciascuna con molteplicitá algebrica 1
come radici del polinomio caratteristico.
La matrice é certamente diagonalizzabile, avendo tutti gli autovalori distinti fra loro.
Determiniamo gli autospazi relativi a ciascun autovalore:
Per λ1 = 3,  
−1 −3 0
A − 3I =  −1 −3 0 
−1 1 −2
ed il sistema lineare omogeneo associato é:

x1 + 3x2 = 0
.
x1 − x2 + 2x3 = 0

Il rango di tale sistema é 2, quindi vi sono ∞1 soluzioni, cioé ∞1 autovettori relativi


a λ1 = 3. Essi si ottengono risolvendo il sistema omogeneo: x1 = −3x2 , x3 = 2x2 .
Per cui il generico autovettore relativo a λ1 = 3 é X1 = (−3α, α, 2α), al variare di
α ∈ R, e l’autospazio associato a λ1 = 3 é

V1 =< (−3, 1, 2) > .

Per λ2 = 1,  
1 −3 0
A − I =  −1 −1 0 
−1 1 0
ed il sistema lineare omogeneo associato é:
8.4 Esercizi svolti. 213

 x1 − 3x2 = 0
x1 + x2 = 0
x1 − x2 = 0

da cui x1 = x2 = 0. Il generico autovettore relativo a λ2 = 1 é X2 = (0, 0, α), al


variare di α ∈ R, e l’autospazio associato a λ2 = 1 é

V2 =< (0, 0, 1) > .

Per λ3 = −1,  
3 −3 0
A + I =  −1 1 0 
−1 1 2
ed il sistema lineare omogeneo associato é:

x1 − x2 = 0
x1 − x2 − 2x3 = 0

da cui x1 = x2 e x3 = 0. Il generico autovettore relativo a λ3 = −1 é X3 = (α, α, 0),


al variare di α ∈ R, e l’autospazio associato a λ3 = −1 é

V3 =< (1, 1, 0) > .

Una base di R3 rispetto alla quale la matrice é diagonalizzabile é

B = {(−3, 1, 2), (0, 0, 1), (1, 1, 0)}.

La matrice diagonalizzante é
 
−3 0 1
C =  1 0 1
2 10
ovvero  
30 0
C−1 AC =  0 1 0  .
0 0 −1
Il polinomio minimo coincide con quello caratteristico.
Esercizio 64. Determinare gli autovalori, una base per gli autospazi, una forma
canonica ed il polinomio minimo della seguente matrice:
 
3 20
 −1 0 0  .
0 01

Svolgimento: Gli autovalori della matrice A sono le soluzioni dell’equazione


214 8 La forma canonica di un operatore lineare.

caratteristica:
3−λ 2 0

−1 −λ 0 = 0

0 0 1−λ
cioé
(1 − λ )2 (2 − λ ) = 0.
Abbiamo quindi un autovalore λ1 = 1 con molteplicitá algebrica pari a 2 ed un
autovalore, λ2 = 2, con molteplicitá algebrica 1.
Determiniamo gli autospazi relativi a ciascun autovalore:
Per λ1 = 1,  
2 2 0
A − I =  −1 −1 0  .
0 0 0
Dal sistema lineare omogeneo associato, otteniamo l’unica relazione

x1 + x2 = 0

Per cui il generico autovettore relativo a λ1 = 1 é X1 = (α, −α, β ), al variare di


α, β ∈ R, e l’autospazio associato a λ1 = 1 é

V1 =< (1, −1, 0), (0, 0, 1) > .

Per λ2 = 2,  
1 2 0
A − 2I =  −1 −2 0 
0 0 −1
ed il sistema lineare omogeneo associato é:

x1 + 2x2 = 0
x3 = 0

da cui x1 = −2x2 . Il generico autovettore relativo a λ2 = 2 é X2 = (−2α, α, 0), al


variare di α ∈ R, e l’autospazio associato a λ2 = 2 é

V2 =< (−2, 1, 0) > .

Per entrambi gli autovalori, vi é coincidenza tra le molteplicitá algebrica e geome-


trica, per cui la matrice é diagonalizzabile. Una base di R3 rispetto alla quale la
matrice assume una forma diagonale é

B = {(1, −1, 0), (0, 0, 1), (−2, 1, 0)}.

La matrice diagonalizzante é
8.4 Esercizi svolti. 215
 
1 0 −2
C =  −1 0 1 
0 1 0
ovvero  
100
C−1 AC =  0 1 0  .
002
Il polinomio minimo é m(λ ) = (1 − λ )(2 − λ ).
Esercizio 65. Determinare i valori di k ∈ R per i quali la seguente matrice é
diagonalizzabile:  
3 1 0
A =  0 2k 0  .
k 0 0

Svolgimento: L’equazione caratteristica é:



3−λ 1 0

0 2k − λ 0 = 0

k 0 −λ

cioé
(3 − λ )(2k − λ )(−λ ) = 0.
Quindi gli autovalori della matrice sono {0, 3, 2k}.
Inizialmente possiamo affermare che se k 6= 0, 23 , allora la matrice possiede tutti gli
autovalori distinti, per cui é certamente diagonalizzabile.
Analizziamo allora i casi k = 0 e k = 32 .
Per k = 0, l’autovalore λ = 0 ha molteplicitá algebrica pari a 2. La matrice A − λ I é
ovviamente pari ad A:  
310
A−λI = A =  0 0 0 .
000
Poiché essa ha rango 1, il sistema omogeneo associato avrá ∞2 soluzioni, ovvero la
molteplicitá geometrica di λ é esattamente 2. In tal caso allora la matrice é ancora
diagonalizzabile.
Concludiamo con k = 23 . L’autovalore µ = 3 ha molteplicitá algebrica 2 e la matrice
A − 3I é  
01 0
A − 3I =  0 0 0  .
3
2 0 −3

Essa ha rango 2, per cui il sistema lineare omogeneo associato possiede ∞1 soluzio-
ni. Quindi la molteplicitá geometrica dell’autovalore 3 é pari ad 1 e non coincide
con la sua molteplicitá algebrica. In tal caso la matrice non é diagonalizzabile.
Concludiamo allora che la matrice A é diagonalizzabile per ogni k 6= 32 .
216 8 La forma canonica di un operatore lineare.

Esercizio 66. Sia A ∈ M2 (R) con autovalori λ1 , λ2 ∈ R. Determinare tutte le


possibili forme canoniche simili alla matrice A.

Svolgimento: Se i due autovalori sono distinti, allora sappiamo che la dimensione


geometrica di entrambi é 1, per cui esiste un blocco di Jordan per ognuno dei due
autovalori e ciascun blocco ha ordine 1, cioé la matrice
 é diagonalizzabile,
 la forma
λ1 0
di Jordan coincide con quella diagonale ed essa é .
0 λ2
Se i due autovalori coincidono (λ1 = λ2 = λ ) e la dimensione dell’autospazio asso-
ciato é 2, allora esitono due blocchi di Jordan relativi all’autovalore, ciascuno con
ordine 1. Anche in questo caso la matrice é diagonalizzabile
  e la forma canonica di
λ 0
Jordan si riduce alla semplice forma diagonale .

Se i due autovalori coincidono (λ1 = λ2 = λ ) ma la dimensione dell’autospazio as-
sociato é 1, allora esiste un solo blocco di Jordan relativo all’autovalore, ed esso
deve avere
  necessariamente ordine 2. La forma canonica di Jordan é in tale caso
λ 1
.

Esercizio 67. Sia A ∈ M3 (R) e siano λ1 , λ2 , λ3 ∈ R gli autovalori di A. Determinare
tutte le possibili forme canoniche simili alla matrice A.

Svolgimento: Se i tre autovalori sono tutti distinti allora la matrice é diagonalizza-


bile. Infatti ogni autospazio ha dimensione 1 e quindi esiste un solo blocco relativo
ad ogni autovalore ed ogni blocco ha  ordine 1. La forma canonica di Jordan si riduce
λ1 0 0
a quella diagonale ed é  0 λ2 0 .
0 0 λ3
Supponiamo ora che λ1 = λ2 = λ e λ3 sia distinto dai primi due.
Se la molteplicitá geometrica di λ é 2, allora vi sono due blocchi di Jordan relativi
a λ ed ovviamente uno relativo a λ3 , per cui ciascun  blocco  dovrá avere ordine 1, e
λ 0 0
la matrice é diagonalizzabile, con forma canonica  0 λ 0 .
0 0 λ3
Al contrario, se la molteplicitá geometrica di λ é 1, allora esiste un solo blocco di
Jordan relativo a λ . Poiché il blocco relativo a λ3 deve necessariamente avere ordine

λ 1 0
1, allora il blocco relativo a λ ha ordine due, e la forma canonica é  0 λ 0 .
0 0 λ3
Infine consideriamo il caso in cui i tre autovalori siano tutti coincidenti col valore
λ.
Se la molteplicitá geometrica dell’autovalore
 é anche 3, allora la forma canonica di
λ 0 0
Jordan si riduce a forma diagonale  0 λ 0 .
0 0λ
Se la molteplicitá geometrica dell’autovalore é 2 allora esistono due blocchi di Jor-
8.4 Esercizi svolti. 217

dan relativi ad esso, quindi


 uno
 di questi avrá ordine 2 e l’altro avrá ordine 1. La
λ 1 0
forma canonica sará  0 λ 0 . Se la molteplicitá geometrica dell’autovalore é 1,
0 0λ  
λ 1 0
esiste un solo blocco di Jordan relativo ad esso e la forma canonica é  0 λ 1 .
0 0λ
Esercizio 68. Determinare una base ortogonale rispetto alla quale la seguente
matrice simmetrica  
122
2 1 2
221
assume una forma diagonale.

Svolgimento: Gli autovalori della matrice A sono le soluzioni dell’equazione


caratteristica:
1−λ 2 2

2 1−λ 2 = 0

2 2 1−λ
cioé
(−1 − λ )2 (5 − λ ) = 0.
Abbiamo quindi un autovalore λ1 = −1 con molteplicitá algebrica pari a 2 ed un
autovalore, λ2 = 5, con molteplicitá algebrica 1. Poiché la matrice é simmetrica, non
vi é alcun dubbio che le molteplicitá algebrica e geometrica di ciascun autovalore
coincideranno.
Determiniamo gli autospazi relativi a ciascun autovalore:
Per λ1 = −1,  
222
A+I =  2 2 2 .
222
Dal sistema lineare omogeneo associato, otteniamo l’unica relazione

x1 + x2 + x3 = 0

Per cui il generico autovettore relativo a λ1 = −1 é X1 = (α, β , −α − β ), al variare


di α, β ∈ R, e l’autospazio associato a λ1 = 1 é

V1 =< (1, 0, −1), (0, 1, −1) > .

La base di V1 non é formata da vettori ortogonali. A partire da questa ricaviamo una


base ortogonale di V1 .
Scegliamo un vettore qualsiasi della base: la nostra scelta ricade su X1 = (1, 0, −1).
Costruiamo ora un vettore X2 di V1 che sia ortogonale a X1 :
218 8 La forma canonica di un operatore lineare.

X2 = (α, β , −α − β ) : X1T X2 = 0

ovvero
α + α + β = 0 =⇒ β = −2α.
Quindi, al variare di α ∈ R, il vettore X2 = (α, −2α, α) ∈ V1 é ortogonale a X1 .
Possiamo allora asserire che {(1, 0, −1), (1, −2, 1)} é una base ortogonale di V1 . Da
ció segue che
1 1 1 2 1
{( √ , 0, − √ ), ( √ , − √ , √ )}
2 2 6 6 6
é una base ortonormale di V1 .
Per λ2 = 5,  
−4 2 2
A − 5I =  2 −4 2 
2 2 −4
ed il sistema lineare omogeneo associato é:

 2x1 − x2 − x3 = 0
x1 − 2x2 + x3 = 0
x1 + x2 − 2x3 = 0

da cui x1 = x2 = x3 . Il generico autovettore relativo a λ2 = 5 é (α, α, α), al variare


di α ∈ R, e l’autospazio associato a λ2 = 5 é

V2 =< (1, 1, 1) > .

Tale autovettore é ovviamente ortogonale ai due precedentemente determinati,


poiché relativo ad un autovalore differente. Quindi una base ortogonale di R3
formata da autovettori di A é
1 1 1 2 1
{( √ , 0, − √ ), ( √ , − √ , √ ), (1, 1, 1)}
2 2 6 6 6

ed allora una base ortonormale di R3 formata da autovettori di A é


1 1 1 2 1 1 1 1
{( √ , 0, − √ ), ( √ , − √ , √ ), ( √ , √ , √ )}.
2 2 6 6 6 3 3 3
Rispetto a tale base la matrice assume una forma diagonale ovvero, detta
 1 1 1 
√ √ √
2 6 3
C =  0 − √26 √1
 
3 
− √12 √16 √1
3

la matrice ortogonale diagonalizzante, avremo


8.4 Esercizi svolti. 219
 
−1 0 0
C−1 AC = CT AC  0 −1 0  .
0 0 5

Esercizio 69. Determinare una forma canonica della seguente matrice A ∈ M6 (R):
 
0 1 −1 −1 1 −1
0 0 1 2 0 1 
 
0 0 1 1 0 0 
A=  0 0 0 1 0 1 .

 
 0 0 0 0 2 −1 
00 0 0 1 0

Svolgimento: Il polinomio caratteristico é det(A − λ I) = λ 2 (λ − 1)4 , cioé λ1 = 0


é autovalore con molteplicitá algebrica 2, mentre l’autospazio ad esso associato ha
dimensione 1. Esiste allora un solo blocco di Jordan ad esso relativo.
Infine l’autospazio relativo all’autovalore λ2 = 1 ha dimensione 1, quindi vi é un
solo blocco di Jordan relativo a λ2 , necessariamente di ordine 4.
La forma canonica sará:
 
110000
0 1 1 0 0 0
   
 = J4 (1) 0
0 0 1 1 0 0
A0 =  .
0 0 0 1 0 0
  0 J2 (0)
0 0 0 0 0 1
000000

Esercizio 70. Determinare una forma canonica della seguente matrice A ∈ M3 (R):
 
1 2 5
A =  0 14 39  .
0 −5 −14

Svolgimento: Il polinomio caratteristico é (λ − 1)(1 − λ )(1 + λ ). Quindi un auto-


valore é λ1 = −1 con molteplicitá algebrica e geometrica pari ad 1, per cui ad esso
é associato un solo blocco di Jordan di ordine 1.
L’altro autovalore é λ2 = 1 con molteplicitá algebrica 2. La dimensione dell’auto-
spazio ad esso associato é 1, quindi vi é un solo blocco ad esso corrispondente, ed
ovviamente deve avere ordine 2. La forma canonica della matrice é allora
 
11 0
A0 =  0 1 0  .
0 0 −1

Esercizio 71. Determinare una forma canonica della seguente matrice A ∈ M6 (R):
220 8 La forma canonica di un operatore lineare.
 
0 1 1 00 0
 0
 1 0 0 1 −1 

 −1 1 2 00 0 
A=
 0
.
 0 0 10 0  
 1 0 −1 11 1 
0 0 0 10 1

Svolgimento: Il polinomio caratteristico é det(A − λ I) = (λ − 1)6 , cioé λ = 1 é


autovalore con molteplicitá algebrica 6.
L’autospazio associato é

V = {(a − b, −b, a, 0, b, b), a, b ∈ R}

che ha quindi dimensione 2. Vi sono allora due blocchi relativi all’autovalore λ = 1.


Calcolando l’indice di nilpotenza della matrice B = A − λ I = A − I, si avrá B5 = 0,
cioé uno dei due blocchi ha ordine 5, quindi l’altro deve avere necessariamente
ordine 1. La forma canonica della matrice é
 
110000
0 1 1 0 0 0
   
0
0 0 1 1 0 0 J5 (1) 0
A = 
=
 .
0 0 0 1 1 0 0 J1 (1)
0 0 0 0 1 0
000001

Esercizio 72. Determinare una forma canonica della seguente matrice A ∈ M8 (R):
 
14500000
0 1 3 0 0 0 0 0
 
0 0 1 0 0 0 0 0
 
0 0 0 2 3 0 0 0
A=  .
0 0 0 0 2 0 0 0

0 0 0 0 0 2 2 1
 
0 0 0 0 0 0 1 3
00000001

Svolgimento: L’equazione caratteristica é det(A − hI) = (1 − h)5 (2 − h)3 = 0, da


cui otteniamo gli autovalori h1 = 1 con molteplicitá algebrica 5, h2 = 2 con molte-
plicitá algebrica 3.
L’autospazio relativo all’autovalore h1 = 1 ha dimensione 2, quindi vi sono 2 blocchi
di Jordan relativi a tale autovalore. Inoltre:
8.4 Esercizi svolti. 221
 
0 4 5 0 0 0 0 0
0 0 3 0 0 0 0 0
 
0 0 0 0 0 0 0 0
 
0 0 0 1 3 0 0 0
A − h1 I = 
0

 0 0 0 1 0 0 0
0 0 0 0 0 1 2 1
 
0 0 0 0 0 0 0 3
0 0 0 0 0 0 0 0

che ha rango 6, quindi r1 = 6.


 
0 0 12 0 0 0 0 0
0 0 0 0 0 0 0 0
 
0 0 0 0 0 0 0 0
 
2
0 0 0 1 6 0 0 0
(A − h1 I) = 
0

 0 0 0 1 0 0 0
0 0 0 0 0 1 2 7
 
0 0 0 0 0 0 0 0
0 0 0 00000
che ha rango 4, quindi r2 = 4.
 
0 0 0 0 0 0 0 0
0 0 0 0 0 0 0 0
 
0 0 0 0 0 0 0 0
 
0 0 0 1 9 0 0 0
(A − h1 I)3 = 
0

 0 0 0 1 0 0 0
0 0 0 0 0 1 2 7
 
0 0 0 0 0 0 0 0
0 0 0 0 0 0 0 0
che ha rango 3, quindi r3 = 3.
 
0 0 0 0 0 000
0 0 0 0 0 0 0 0
 
0 0 0 0 0 0 0 0
 
4
0 0 0 1 12 0 0 0 
(A − h1 I) = 
0

 0 0 0 1 0 0 0
0 0 0 0 0 1 2 7
 
0 0 0 0 0 0 0 0
0 0 0 0 0 000
che ha rango 3, quindi r4 = 3. Quindi

ORDINE DEI BLOCCHI NUMERO DEI BLOCCHI


1 0
2 1
3 1
222 8 La forma canonica di un operatore lineare.

c’é un blocco di ordine 2 ed uno di ordine 3 relativi all’autovalore h1 = 1.


Passiamo all’autovalore h2 = 2.
 
−1 4 5 0 0 0 0 0
 0 −1 3 0 0 0 0 0 
 
 0 0 −1 0 0 0 0 0 
 
 0 0 0 030 0 0 
A − h2 I = 
 
 0 0 0 000 0 0 

 0 0 0 000 2 1 
 
 0 0 0 0 0 0 −1 3 
0 0 0 0 0 0 0 −1
che ha rango 6, quindi r1 = 6 e vi sono in totale 2 blocchi relativi a tale autovalore.
 
1 −8 2 0 0 0 0 0
 0 1 −6 0 0 0 0 0 
 
0 0 1 0 0 0 0 0 
 
0 0 0 0 0 0 0 0 
(A − h2 I)2 = 0 0 0 0 0 0 0 0 

 
 0 0 0 0 0 0 −2 5 
 
 0 0 0 0 0 0 1 −6 
0 0 0 000 0 1
che ha rango 5, quindi r2 = 5.
 
−1 12 −21 0 0 0 0 0
 0 −1 9 0 0 0 0 0 
 
 0 0 −1 0 0 0 0 0 
 
3
 0 0 0 000 0 0 
(A − h2 I) = 
 0 0 0 000

 0 0  
 0 0 0 000
 2 −11 

 0 0 0 000 −1 10 
0 0 0 000 0 −1
che ha rango 5, quindi r3 = 5. Quindi

ORDINE DEI BLOCCHI NUMERO DEI BLOCCHI


1 1
2 1
c’é un blocco di ordine 1 ed uno di ordine 2 relativi all’autovalore h2 = 2.
Quindi una forma canonica di Jordan della matrice di partenza é la seguente:
8.4 Esercizi svolti. 223
 
1 1 0 0 0 0 0 0
0 1 0 0 0 0 0 0
 
0 0 1 1 0 0 0 0
 
0
0 0 0 1 1 0 0 0
A =
0
.
 0 0 0 1 0 0 0
0 0 0 0 0 2 0 0
 
0 0 0 0 0 0 2 1
0 0 0 0 0 0 0 2
Capitolo 9
Le forme bilineari e le forme quadratiche reali.

9.1 Introduzione.

Siano K un campo e V , W due spazi vettoriali su K, rispettivamente di dimensione


n e m. La applicazione f : V × W −→ K che fa corrispondere ad una coppia di
vettori (v, w) ∈ V ×W uno scalare f (v, w) ∈ K, é detta forma bilineare se valgono le
seguenti proprietá: per ogni v, v1 , v2 ∈ V , w, w1 , w2 ∈ W , α, β ∈ K

f (αv1 + β v2 , w) = α f (v1 , w) + β f (v2 , w)

f (v, αw1 + β w2 ) = α f (v, w1 ) + β f (v, w2 ).


In sostanza la f é lineare sia in V che in W .
Osservazione 9.1. Il prodotto interno (o scalare) definito da f : Rn × Rn −→ R, tale
che, per ogni coppia di vettori
   
x1 y1
 x2   y2 
   
 ... 
, Y =  ... 
 
X = ...   ... 
   
 ...   ... 
xn yn

si abbia f (X,Y ) = ∑ni=1 xi yi , é una forma bilineare reale.


Esempio 126. Dimostrare che l’applicazione f : R2 ×R3 −→ R, definita da f ((x1 , x2 ), (y1 , y2 , y3 )) =
x1 (y1 + y2 ) + x2 (y1 − y3 ), é una forma bilineare.

Svolg. Per dimostrarlo é sufficiente verificare che


–per ogni a, b ∈ R
–per ogni (x1 , x2 ), (x10 , x20 ) ∈ R2
–per ogni (y1 , y2 , y3 ), (y01 , y02 , y03 ) ∈ R3

225
226 9 Le forme bilineari e le forme quadratiche reali.

si abbia
f (a(x1 , x2 ) + b(x10 , x20 ), (y1 , y2 , y3 )) = a f ((x1 , x2 ), (y1 , y2 , y3 ))+
b f ((x10 , x20 ), (y1 , y2 , y3 ))
e
f ((x1 , x2 ), a(y1 , y2 , y3 ) + b(y01 , y02 , y03 )) = a f ((x1 , x2 ), (y1 , y2 , y3 ))+
+b f ((x1 , x2 ), (y01 , y02 , y03 )).
t
u
Osservazione 9.2. Siano 0V e 0W rispettivamente il vettore nullo in V e quello nullo
in W . Allora f (0V , w) = 0 e f (v, 0W ) = 0, per ogni v ∈ V , w ∈ W .

9.2 Forme bilineari e matrici.

Siano V e W due spazi vettoriali su di un campo K, rispettivamente di dimen-


sioni n e m. Sia f : V × W −→ K una forma bilineare e siano rispettivamente
B = {b1 , b2 , . . . , bn } e C = {c1 , c2 , . . . , cm } una base di V ed una di W . Scegliamo
una qualsiasi coppia di vettori X ∈ V e Y ∈ W , ciascuno dei quali sia espresso co-
me combinazione lineare dei vettori delle basi scelte, ciascuno tramite le proprie
componenti:
X = x1 b1 + x2 b2 + x3 b3 + . . . . . . + xn bn
Y = y1 c1 + y2 c2 + y3 c3 + . . . . . . + ym cm .
Possiamo quindi rappresentare X e Y tramite i vettori colonna composti dalle
rispettive componenti:
   
x1 y1
 x2   y2 
   
 x3   y3 
   
 
 . . .  ∈ W.
X =  . . .  ∈ V, Y =  
 ...   ... 
   
 ...   ... 
xn B ym C

Allora, sfruttando le proprietá di bilinearitá della f si ha:


 
n m
f (X,Y ) = f ∑i=1 xi bi , ∑ j=1 y j c j
(9.1)
= ∑ni=1 ∑mj=1 xi y j f (bi , c j ).

Nella relazione (9.1) poniamo ai j = f (bi , c j ), al variare di tutti gli elementi bi e c j


delle due basi. Con tali scalari costruiamo una matrice A = (ai j )n×m ∈ Mnm (K).
Possiamo adesso osservare che la relazione (9.1) equivale a f (X,Y ) = X T AY . Al
variare dei vettori X ∈ V e Y ∈ W , ciascuno espresso tramite le proprie componenti
9.2 Forme bilineari e matrici. 227

rispetto alle basi scelte, possiamo allora rappresentare la forma f attraverso la ma-
trice A, la quale viene fissata una volta per tutte, in dipendenza delle basi scelte per
gli spazi vettoriali. Quindi ad ogni forma bilineare f : V × W → K é associata una
matrice, che dipende esclusivamente dalla scelta iniziale delle basi per V e W .
Viceversa, fissata una matrice A = (ai j )n×m ∈ Mnm (K), consideriamo la corri-
spondenza f : V × W −→ K, definita da f (X,Y ) = X T AY , per ogni scelta dei
vettori    
x1 y1
 x2   y2 
   
 x3   y3 
   
X = . . .  ∈ V, Y =  . . .  ∈ W
   
 ...   ... 
   
 ...   ... 
xn B ym C
espressi per componenti rispetto alle basi B di V e C di W . Tale corrispondenza é
una forma bilineare su V ×W , la cui espressione é esplicitamente la (9.1), ovvero
n m
f (X,Y ) = X T AY = ∑ ∑ ai j xi y j .
i=1 j=1

In definitiva, una volta assegnate le basi per due qualsiasi spazi vettoriali V e W sul
campo K, esiste una corrispondenza biunivoca tra l’insieme delle matrici Mn,m (K)
e le forme bilineari f : V ×W → K.
Esempio 127. Sia f : R2 × R3 −→ R, definita da f ((x1 , x2 ), (y1 , y2 , y3 )) = x1 (y1 +
y2 ) + x2 (y1 − y3 ), determiniamo la matrice associata a f rispetto alle basi canoniche

B = {b1 , b2 } = {(1, 0), (0, 1)} di R2

e
C = {c1 , c2 , c3 } = {(1, 0, 0), (0, 1, 0), (0, 0, 1)} di R3 .

Svolg.
f (b1 , c1 ) = 1, f (b1 , c2 ) = 1, f (b1 , c3 ) = 0
f (b2 , c1 ) = 1, f (b2 , c2 ) = 0, f (b2 , c3 ) = −1
allora la f si puó rappresentare, rispetto alle basi canoniche, nel modo seguente:
 
  y1
  11 0
f (X,Y ) = x1 x2  y2  .
1 0 −1
y3
t
u
228 9 Le forme bilineari e le forme quadratiche reali.

9.3 Cambiamento di basi.

Il computo degli scalari tramite i quali costruire la matrice A associata ad una forma
bilineare f : V ×W → K, dipende esclusivamente dalla scelta iniziale delle basi per
i due spazi vettoriali V e W . É evidente allora che, la scelta di basi differenti in V e
W comporta che la matrice associata a f sia, in generale, differente dalla A.
A tal proposito, riprendiamo l’esempio 127:
Esempio 128. Sia f : R2 × R3 −→ R, definita da f ((x1 , x2 ), (y1 , y2 , y3 )) = x1 (y1 +
y2 ) + x2 (y1 − y3 ), e siano B = {b1 , b2 } = {(1, 0), (0, 1)} base canonica di R2 e C =
{c1 , c2 , c3 } = {(1, 0, 0), (0, 1, 0), (0, 0, 1)} base canonica di R3 . Abbiamo visto che
la matrice associata a f rispetto alle basi B e C é data da
 
11 0
.
1 0 −1

Consideriamo ora due basi diverse da quelle canoniche:

D = {d1 , d2 } = {(1, 1), (2, 1)} di R2

e
E = {e1 , e2 , e3 } = {(1, 1, 0), (0, 0, 1), (2, 0, 1)} di R3 .
Calcoliamo gli elementi della matrice associata in tali basi:
f ((x1 , x2 ), (y1 , y2 , y3 )) = x1 (y1 + y2 ) + x2 (y1 − y3 ),

f (d1 , e1 ) = 3, f (d1 , e2 ) = −1, f (d1 , e3 ) = 3


f (d2 , e1 ) = 5, f (d2 , e2 ) = −1, f (d2 , e3 ) = 5
allora la f si puó rappresentare, rispetto alle basi C e D, nel modo seguente:

 
  y1
  3 −1 3
f (X,Y ) = x1 x2  y2  = x1 (3y1 − y2 + 3y3 ) + x2 (5y1 − y2 + 5y3 ).
5 −1 5
y3
3 2
Esempio 129.  Sia  f : R × R −→ R una forma bilineare, alla quale sia associata
11
la matrice  1 0  rispetto alle basi B = {b1 , b2 , b3 } = {(1, 1, 0), (0, 0, 1), (0, 1, 1)}
21
di R3 e C = {c1 , c2 } = {(1, 1), (0, 1)} di R2 . Determiniamo la matrice associata
a f rispetto alle basi D = {d1 , d2 , d3 } = {(0, 1, 0), (0, 1, 1), (2, 0, 1)} di R3 e E =
{e1 , e2 } = {(1, 1), (2, 1)} di R2 .

Svolg. L’espressione della f rispetto alle basi B e C é data da:


9.3 Cambiamento di basi. 229
 
 1 1  
y
0  1 = x1 (y1 + y2 ) + x2 y1 + x3 (2y1 + y2 ).

f (X,Y ) = x1 x2 x3  1
y2
2 1

Per poter determinare la matrice rispetto alle nuove basi, dovremmo calcolare i coef-
ficienti f (di , e j ). Poiché l’unico strumento che abbiamo é la matrice data rispetto
alle basi B e C, siamo costretti inizialmente ad esprimere tutti i vettori di e e j per
componenti rispetto alle basi B e C, e solo allora potremo effettuare il calcolo degli
f (di , e j ). Avremo:

d1 = (0, 1, 0) = (0, −1, 1)B , d2 = (0, 1, 1) = (0, 0, 1)B


d3 = (2, 0, 1) = (2, 3, −2)B
e1 = (1, 1) = (1, 0)C , e2 = (2, 1) = (2, −1)C
da cui
f (d1 , e1 ) = 1, f (d1 , e2 ) = 1
f (d2 , e1 ) = 2, f (d2 , e2 ) = 3
f (d3 , e1 ) = 1, f (d3 , e2 ) = 2.
Allora la f si puó rappresentare, rispetto alle basi D e E, nel modo seguente:
 
 11
 
y
f (X,Y ) = x1 x2 x3  2 3  1 = x1 (y1 + y2 ) + x2 (2y1 + 3y2 ) + x3 (y1 + 2y2 ).

y2
12

t
u
Nota 9.3. Anche in questo Capitolo, per uniformitá con quanto fatto nei precedenti,
rivolgiamo la nostra attenzione esclusivamente al caso di forme definite su spazi
vettoriali reali.
Generalizziamo adesso quanto visto nei precedenti esempi.
Siano V e W due spazi vettoriali reali di dimensioni rispettivamente n e m, f :
V × W → R una forma bilineare, B = {b1 , . . . , bn } e B0 = {b01 , . . . , b0n } due distin-
te basi di V , E = {e1 , . . . , em } e E 0 = {e01 , . . . , e0m } due distinte basi di W . Sia inoltre
A ∈ Mn,m (R) la matrice associata a f , rispetto alle basi B e E, e sia A0 quella associa-
ta a f relativamente alla scelta delle basi B0 e E 0 . Ci proponiamo adesso di descrivere
la relazione che intercorre tra le due matrici A e A0 .
Sia C ∈ Mn (R) la matrice di transizione dalla base B0 alla B, ovvero la matrice
tramite la quale relazioniamo le componenti X 0 di un vettore v ∈ V relativamente
alla base B0 con le sua componenti X rispetto alla base B. É ben noto che tale re-
lazione si esprime attraverso l’uguaglianza vettoriale X = CX 0 . Analogamente, se
D ∈ Mm (R) é la matrice di transizione dalla base E 0 alla base E, allora le compo-
nenti Y 0 di un vettore w ∈ W rispetto alla base E 0 e le componenti Y dello stesso
vettore relativamente alla base E sono tali che Y = DY 0 . Quindi f (v, w) = X 0T A0Y 0
ma anche
230 9 Le forme bilineari e le forme quadratiche reali.

f (v, w) = X T · A ·Y
T
= C · X 0 · A · DY 0


= X 0T · CT · A · D ·Y 0 .


Dall’unicitá della matrice associata ad una forma bilineare f , una volta fissate le
basi rispetto alle quali f é definita, segue che A0 = CT AD.
Esempio 130. Sia f : R2 × R3 → R la forma bilineare associata alla matrice
 
111
A=
102

relativamente alle basi

B = {b1 , b2 } = {(1, 1), (0, 1)} per R2

e
E = {e1 , e2 , e3 } = {(1, 1, 0), (0, 0, 1), (0, 1, 1)} per R3 .
Ció vuol dire che, dati X = (x1 , x2 ) ∈ R2 , Y = (y1 , y2 , y3 ) ∈ R3 due qualsiasi vettori
espressi per componenti rispettivamente in termini delle basi B e E, l’espressione
della f é data da

f (X,Y ) = X T · A ·Y

= x1 y1 + x1 y2 + x1 y3 + x2 y1 + 2x2 y3 .

Siano adesso B0 = {b01 , b02 } = {(1, 1), (2, 1)} una nuova base per R2 e E 0 = {e01 , e02 , e03 } =
{(0, 1, 0), (0, 1, 1), (2, 0, 1)} una per R3 .
Le matrici di transizione, C ∈ M2 (R) da B0 a B, e D ∈ M3 (R) da E 0 a E, sono
rispettivamente  
1 2
C=
0 −1
e  
0 0 2
D =  −1 0 3  .
1 1 −2
Quindi, la matrice A0 di f , in termini delle basi B0 di R2 e E 0 di R3 , é

A0 = CT · A · D
 
    0 0 2
1 0 111 
= · · −1 0 3 
2 −1 102
1 1 −2
 
0 13
= .
−2 0 8
9.4 Forme bilineari simmetriche. 231

L’espressione della f in relazione alle nuove basi scelte sará

f (X 0 ,Y 0 ) = X 0T · A0 ·Y 0

= x10 y02 + 3x10 y03 − 2x20 y01 + 8x20 y03

nella quale abbiamo indicato con X 0 = (x10 , x20 )R2 e Y 0 = (y01 , y02 , y03 ) ∈ R3 i generici
vettori di R2 e R3 rispettivamente in termini delle basi B0 e E 0 .
Vogliamo ora soffermarci sul caso particolare in cui V = W , ovvero f : V ×V → R.
In tal caso é lecito considerare una unica base B = {e1 , . . . , en } per entrambi gli
spazi V , inoltre i coefficienti della matrice A = ai j associata a f rispetto a B sono
esattamente ai j = f (ei , e j ), per ogni i, j = 1, . . . , n.
Quindi, se B e B0 sono due differenti basi di V , f puó essere rappresentato da due
diverse matrici A ed A0 , relativamente alla base rispetto alla quale la matrice viene
costruita. Se indichiamo con C ∈ Mn (R) la matrice di transizione da B0 a B, per via
delle precedenti argomentazioni abbiamo che A0 = CT AC. Le matrici A ed A0 sono
quindi correlate in base alla seguente:
Definizione 9.4. Siano A, A0 due n × n matrici reali. Esse sono dette matrici con-
gruenti se esiste una matrice C ∈ Mn (R) invertibile tale che A0 = CT AC. La rela-
zione intercorrente tra le matrici A ed A0 é detta congruenza ed é una relazione di
equivalenza.
Quindi, in base a tale definizione, vale il seguente:
Teorema 9.5. Sia f : V ×V → R una forma bilineare reale su V ×V . Due matrici
A, A0 rappresentano f rispetto a due differenti basi di V se e solo se A ed A0 sono
congruenti.

9.4 Forme bilineari simmetriche.

Sia f : V ×V −→ R una forma bilineare reale definita in V ×V , dove V sia uno spa-
zio vettoriale di dimensione n sul campo R. Sia B = {e1 , e2 , . . . , en } una base di V e
sia A la matrice associata a f rispetto a B. Per quanto detto fin’ora, una rappresenta-
zione della f é la seguente: f (X,Y ) = X T AY , per ogni X,Y vettori di V espressi per
componenti rispetto alla base B. Denotiamo A = (ai j ) ∈ Mn (R).
Diciamo che la forma bilineare é simmetrica se accade che f (X,Y ) = f (Y, X) per
ogni X,Y ∈ V . Poiché, da tale definizione, in particolare dovrá essere f (ei , e j ) =
f (e j , ei ), per ogni coppia di vettori ei , e j della base B, allora ai j = a ji , per ogni i 6= j,
quindi la matrice A, associata alla f , é una matrice simmetrica.
Viceversa, data una matrice A simmetrica reale di ordine n, si puó definire la forma
bilineare f (X,Y ) = X T AY , definita in V × V , dove V sia uno spazio vettoriale di
dimensione n su un campo R. Si osserva facilmente che tale f (X,Y ) = f (Y, X) per
ogni X,Y ∈ V , ovvero f é una forma bilineare simmetrica.
232 9 Le forme bilineari e le forme quadratiche reali.

In definitiva tutte le forme bilineari simmetriche sono rappresentate da matrici sim-


metriche ed inoltre, per ogni matrice simmetrica si puó definire una forma bilineare
simmetrica, relativamente ad una base B dello spazio vettoriale.
3 3
 Siaf : R × R −→ R una forma bilineare, alla quale sia associata
Esempio 131.
110
la matrice  1 0 2  rispetto alla base canonica di R3 .
021
La rappresentazione della f é allora la seguente

  
  110 y1
f (X,Y ) = x1 x2 x3  1 0 2   y2  = x1 y1 + x2 y1 + x1 y2 + 2x3 y2 + 2x2 y3 + x3 y3 .
021 y3

Essa é simmetrica, poiché associata ad una matrice simmetrica.


Osservazione 9.6. Si noti che per determinare la simmetricitá della f é sufficiente
constatare che, nell’espressione f (X,Y ) = ∑i, j αi, j xi y j , le coppie dei termini misti
xi y j e x j yi abbiano lo stesso coefficiente, per ogni scelta di i e j.
Definizione 9.7. Sia f : V × V → R  una forma bilineare simmetrica sullo spazio
vettoriale reale V . Diremo che V, f é uno spazio vettoriale simmetrico.
Poiché una forma bilineare f : V ×V → R é in qualche modo la generalizzazione del
prodotto scalare interno in uno spazio vettoriale reale V , la condizione f (u, v) = 0
che talvolta potrebbe verificarsi per opportuni vettori u, v ∈ V , puó essere vista come
una generalizzazione del concetto di ortogonalitá tra i vettori u, v. In tal senso diamo
la seguente:
Definizione 9.8. Sia f : V → V una forma bilineare sullo spazio vettoriale reale V .
Diremo che u ∈ V é f -ortogonale a v ∈ V se f (u, v) = 0, e scriveremo u ⊥ v.
In generale potremmo avere che u ⊥ v ( f (u, v) = 0) ma v 6⊥ u ( f (v, u) 6= 0):
Esempio 132. Sia f : R2 × R2 → R la forma bilineare definita da
   
 10 y
· 1
 
f ( x1 , x2 ), (y1 , y2 ) = x1 x2 ·
11 y2

= x1 y1 + x2 y1 + x2 y2 .

Per X = [1, −1]T e Y = [−1, 0]T abbiamo f (X,Y ) = 0 ma f (Y, X) = −1.


Ma nel caso f sia simmetrica, la relazione di ortogonalitá ⊥ é anche essa simmetrica.
Infatti, se f (u, v) = f (v, u), per ogni u, v ∈ V , allora u ⊥ v se e solo se v ⊥ u.
Definizione 9.9. Se f : V × V → R é simmetrica e W é un sottospazio di V ,
definiamo lo spazio f -ortogonale di W il seguente

W ⊥ = {v ∈ V | f (v, w) = 0, ∀w ∈ W }.
9.4 Forme bilineari simmetriche. 233

Osserviamo che la precedente definizione é analoga a quella di complemento or-


togonale in uno spazio dotato di prodotto scalare interno. Ma vi é una sostanziale
differenza: essa consta nel fatto che, nel caso in cui W ⊥ sia il complemento orto-
gonale di W , allora W ⊕ W ⊥ = V . Al contrario, nel caso in cui W ⊥ sia lo spazio
f -ortogonale di W , non é garantita la precedente proprietá. Il seguente esempio puó
aiutare a comprendere tale situazione:
Esempio 133. Sia f : R3 × R3 → R la forma bilineare simmetrica definita da
   
   111 y1
f ( x1 , x2 , x3 ), (y1 , y2 , y3 ) = x1 x2 x3 ·  1 0 0  ·  y2 
100 y3

= x1 y1 + x1 y2 + x1 y3 + x2 y1 + x3 y1 .

Sia W =< (0, 1, −1) >. Allora W ⊥ = R3 e W ∩W ⊥ = W . Per cui W e W ⊥ non sono
in somma diretta.
Analogamente, sia U =< (0, 1, 0) >. Allora U ⊥ = {(0, α, β ) : ∀α, β ∈ R}, da cui
U ⊥ =< (0, 1, 0), (0, 0, 1) > ed ancora U ∩ U ⊥ = U. Quindi U e U ⊥ non sono in
somma diretta.

9.4.1 La restrizione di una forma bilineare simmetrica

Sia (V, f ) uno spazio simmetrico e sia W un sottospazio di V . In ogni caso si puó
introdurre la restrizione f|W : W ×W → R di f a W . Poiché appare evidente che f|W
debba essere simmetrica su W , possiamo parlare di spazio simmetrico (W, f|W ).
Esempio 134. Sia f : R3 × R3 → R la forma bilineare simmetrica definita da
   
   100 y1
f ( x1 , x2 , x3 ), (y1 , y2 , y3 ) = x1 x2 x3 ·  0 1 1  ·  y2 
010 y3

= x1 y1 + x2 y2 + x2 y3 + x3 y2 .

Sia W =< (1, 0, 0), (1, 0, 1) >, per cui ogni vettore di W si esprime come una
combinazione lineare dei vettori generatori di W , ovvero

∀X ∈ W ∃α1 , α2 ∈ R : X = α1 (1, 0, 0) + α2 (1, 0, 1).

Scegliamo X,Y due vettori in W e scriviamoli come appena fatto:

X = α1 (1, 0, 0) + α2 (1, 0, 1), Y = β1 (1, 0, 0) + β2 (1, 0, 1).

Avremo che
234 9 Le forme bilineari e le forme quadratiche reali.
 
f (X,Y ) = α1 β1 f (1, 0, 0), (1, 0, 0) + α1 β2 f (1, 0, 0), (1, 0, 1) +
 
α2 β1 f (1, 0, 1), (1, 0, 0) + α2 β2 f (1, 0, 1), (1, 0, 1) =

= α1 β1 + α1 β2 + α2 β1 + α2 β2 =
   
 11 β
· 1 .

= α1 α2 ·
11 β2
 
11
Quindi é la matrice della restrizione di f in W .
11

9.4.2 Forme bilineari simmetriche degeneri e non degeneri

Definizione 9.10. Sia f : V × V → R una forma bilineare simmetrica sullo spazio


vettoriale reale V . Il radicale di f é il sottospazio

Rad( f ) = V ⊥ = {u ∈ V | f (u, v) = 0, ∀v ∈ V }.

La forma f é detta non degenere se V ⊥ = {0}. Quindi f é non degenere se f (v, u) =


0, per ogni v ∈ V , solo se u = 0. Diremo che f é degenere nel caso V ⊥ 6= {0}.
Analogamente possiamo definire il radicale di una restrizione di f ad un sottospazio
di V :
Definizione 9.11. Sia f : V × V → R una forma bilineare simmetrica sullo spazio
vettoriale reale V e W un sottospazio di V . Il radicale della restrizione f|W , é il
sottospazio di W definito da

Rad( f|W ) = W ∩W ⊥ = {u ∈ W | f (u, w) = 0, ∀w ∈ W }.

Diremo che f|W é non degenere su W se W ∩ W ⊥ = {0}, in caso contrario f|W é


detta degenere su W .
Lemma 9.12. Una forma bilineare simmetrica f : V ×V → R é non degenere se e
solo se la matrice di f é invertibile.
Osservazione 9.13. Se f é una forma non degenere su V , la sua restrizione su
un sottospazio W di V potrebbe comunque essere degenere su W . Come caso
esemplificativo, si prenda l’Esempio 134, nel quale la matrice A di f é
 
100
A = 0 1 1
010

quindi é invertibile, per cui f é non degenere su V .


Ciononostante, la restrizione al sottospazio W =< (1, 0, 0), (1, 0, 1) > é rappresen-
9.4 Forme bilineari simmetriche. 235

tata dalla matrice singolare  


11
11

quindi f|W é degenere su W . Infatti X ∈ Rad f|W solo se ha componenti (x1 , x2 )
rispetto alla base {(1, 0, 0), (1, 0, 1)} tali che
     
11 x 0
· 1 =
11 x2 0

ovvero x1 + x2 = 0. Quindi se X ∈ Rad f|W allora X = α(1, 0, 0) − α(1, 0, 1) =
(0, 0, −α), per un opportuno α ∈ R, da cui concludiamo che

Rad f|W =< (0, 0, 1) > .

Per le forme simmetriche non degeneri valgono i seguenti risultati:


Teorema 9.14. Sia f : V ×V → R una forma bilineare simmetrica, W un sottospazio
di V . Allora f|W é non degenere su W se e solo se V = W ⊕W ⊥ .
Teorema 9.15. Se f : V × V → R é simmetrica e non degenere, allora, dato un
qualsiasi suo sottospazio W , vale la relazione dim(W ) + dim(W ⊥ ) = dim(V ).
Inoltre vale il seguente:
Corollario 9.16. Siano f : V ×V → R simmetrica e non degenere, W un sottospazio
di V . La restrizione f|W é non degenere su W se e solo se f|W ⊥ é non degenere su
W ⊥.
Definizione 9.17. Siano f : V × V → R simmetrica e B = {e1 , . . . , en } una base di
V . Diremo che B é una base f -ortogonale se f (ei , e j ) = 0, per ogni i 6= j.
Definizione 9.18. Siano f : V ×V → R simmetrica e B = {e1 , . . . , en } una base f -
ortogonale di V . Diremo che B é una base f -ortonormale se f (ei , ei ) = 1, per ogni
i = 1, . . . , n.
Esempio 135. Sia f : R3 × R3 → R la forma bilineare simmetrca definita da

f (X,Y ) = x1 y1 + 2x1 y2 + x1 y3 + 2x2 y1

+x2 y2 + x2 y3 + x3 y1 + x3 y2 .

In base alla Definizione 9.17,


 
1 1
B = (1, 0, 0), (−2, 1, 0), (− , − , 1)
3 3

é una base f -ortogonale di R3 .


Definizione 9.19. Sia f : V ×V → R una qualsiasi forma bilineare.
Un vettore v ∈ V é detto f -isotropo se f (v, v) = 0; in caso contrario v é detto f -
anisotropo.
236 9 Le forme bilineari e le forme quadratiche reali.

Lo spazio V é detto f -isotropo se contiene almeno un vettore f -isotropo; in caso


contrario V é detto f -anisotropo.
V é detto totalmente f -isotropo (od anche f -simplettico) se ogni vettore di V é
f -isotropo.
Esempio 136. Sia f : R3 × R3 → R la forma bilineare simmetrica non degenere
associata alla matrice  
1 0 1
A =  0 −1 1 
1 1 −2
rispetto alla base canonica di R3 .
Il vettore X = [1, 1, 0]T é f -isotropo, infatti f (X, X) = 0. Quindi R3 is f -isotropo.
Il vettore Y = [2, 1, 0]T is f -anisotropo, infatti f (Y,Y ) = 3.
Esempio 137. Sia f : R3 × R3 → R la forma simmetrica bilineare associata alla
matrice  
101
A = 0 2 1
112
rispetto alla base canonica di R3 . Se X = [x1 , x2 , x3 ]T é un qualsiasi vettore di R3
espresso per componenti rispetto alla base canonica, allora

f (X, X) =
x12 + 2x1 x3 + 2x22 + 2x2 x3 + 2x32 =
(x1 + x3 )2 + x22 + (x2 + x3 )2 > 0, ∀X ∈ R3 .

Quindi non esiste alcun vettore non nullo di R3 che sia f -isotropo. In base alla
Definizione 9.19, R3 é uno spazio vettoriale f -anisotropo.
Esempio 138. Sia f : R3 × R3 → R la forma bilineare associata alla matrice
 
0 3 1
A =  −3 0 2 
−1 −2 0

rispetto alla base canonica di R3 . In questo caso é banale constatare che f (X, X) = 0,
per ogni vettore X = [x1 , x2 , x3 ]T ∈ R3 espresso per componenti rispetto alla base
canonica. Ovvero R3 é uno spazio vettoriale totalmente f -isotropo.

9.5 Diagonalizzazione delle forme simmetriche reali.



Sia V, f uno spazio simmetrico reale. Come fatto nel caso dello studio di endo-
morfismi, anche adesso ci proponiamo di determinare una base di V rispetto alla
quale la marice associata a f sia espressa nella forma piú semplice possibile. Vedre-
9.6 Metodo per la diagonalizzazione delle forme simmetriche reali. 237

mo che tale forma (canonica) é esattamente una forma diagonale e che la base di V ,
in riferimento alla quale essa é determinata, é una base f -ortogonale. In taluni casi,
che sono quelli piú rilevanti al fine di costruire una metrica nello spazio V , la base é
precisamente f -ortonormale.
Lemma 9.20. Sia (V, f ) uno spazio simmetrico reale di dimensione n. Se f 6= 0 (cioé
esistono v, w ∈ V tali che f (v, w) 6= 0), allora esiste w0 ∈ V tale che f (w0 , w0 ) 6= 0.
Teorema 9.21. Sia (V, f ) uno spazio simmetrico reale di dimensione n. Allora esiste
una base f -ortogonale di V .

Dim. Proveremo il Teorema per induzione sulla dimensione di V . Dapprima os-


serviamo che se n = 1 oppure se f (v, w) = 0 per ogni v, w ∈ V , allora ogni base di V
é banalmente f -ortogonale.
Assumiamo quindi n ≥ 2 ed esistano x0 , y0 ∈ V tali che f (x0 , y0 ) 6= 0.
Supponiamo inoltre che l’asserto sia vero per ogni spazio (W, f ) simmetrico reale
di dimensione m < n.
Dal Lemma 9.20, esiste w0 ∈ V tale che f (w0 , w0 ) 6= 0.
Quindi la restrizione di f al sottospazio W =< w0 > di V é non degenere. Dal Teo-
rema 9.14, V = W ⊕W ⊥ . Inoltre W ⊥ é uno spazio simmetrico reale di dimensione
n − 1.
Per l’ipotesi induttiva, esiste una base f -ortogonale {e1 , . . . , en−1 } di W ⊥ . Da ció
deriva che {e1 , . . . , en−1 , w0 } é una base f -ortogonale di V . t
u

Osservazione 9.22. La matrice A = ai j n×n che rappresenta la forma f rispetto ad
una base f -ortogonale di V , é una matrice diagonale. Infatti ai j = f (ei , e j ) = 0, per
ogni i 6= j.
Possiamo quindi riformulare il Teorema 9.21, dandone una versione matriciale,
come segue: ogni matrice simmetrica reale é congruente ad una matrice diagonale.

9.6 Metodo per la diagonalizzazione delle forme simmetriche


reali.

Sia f (X,Y ) = ∑ ai j xi y j la forma bilineare associata alla matrice simmetrica A ∈


Mn (R) rispetto alla base B = {e1 , . . . , en } di Rn .
Supponiamo che la matrice A = (ai j )n×n non sia diagonale.
Vogliamo adesso fornire un algoritmo che consenta di determinare una base di
Rn rispetto alla quale la matrice, associata alla forma bilineare simmetrica f , sia
diagonale.
Passo 1
Supponiamo che f (e1 , e1 ) = a11 6= 0. Calcoliamo i seguenti coefficienti:

f (e1 , e2 ) f (e1 , e3 ) f (e1 , ei )


c2 = c3 = ... ... ci = .
f (e1 , e1 ) f (e1 , e1 ) f (e1 , e1 )
238 9 Le forme bilineari e le forme quadratiche reali.

Determiniamo il seguente cambiamento di base

e01 = e1 , e02 = e2 − c2 e1 , e03 = e3 − c3 e1 . . . . . . e0n = en − cn e1 .

Diciamo C la matrice di transizione dalla base B0 = {e01 , . . . , e0n } alla base B =


{e1 , . . . , en }, per cui X = CX 0 , per ogni vettore v ∈ V , le cui componenti rispet-
to alle basi B e B0 siano espresse rispettivamente dalle n-uple X = (x1 , . . . , xn ) e
X 0 = (x10 , . . . , xn0 ). A questo punto sostituiamo X = CX 0 e Y = CY 0 nell’espressione
della forma bilineare. La matrice A0 = (a0i j )n×n associata a f (X 0 ,Y 0 ) dopo aver ef-
fettuato tali sostituzioni, avrá la prima riga e la prima colonna tutte nulle, eccetto
l’elemento a011 . Tale matrice, congruente alla matrice A, potrá essere ottenuta dalla
relazione A0 = CT AC.
Passo 2
Supponiamo ancora f (e02 , e02 ) = a022 6= 0. Calcoliamo i seguenti coefficienti:

f (e02 , e03 ) f (e02 , e04 ) f (e02 , e0i )


c03 = c04 = ... ... c0i = .
f (e02 , e02 ) f (e02 , e02 ) f (e02 , e02 )

Determiniamo il seguente cambiamento di base

e001 = e01 , e002 = e02 , e003 = e03 − c03 e02 . . . . . . e00n = e0n − c0n e02 .

Diciamo C0 la matrice di transizione dalla base B00 = {e001 , . . . , e00n } alla base B0 =
{e01 , . . . , e0n }, per cui X 0 = C0 X 00 , per ogni vettore v ∈ V , le cui componenti rispet-
to alle basi B0 e B00 siano espresse rispettivamente dalle n-uple X 0 = (x10 , . . . , xn0 ) e
X 00 = (x100 , . . . , xn00 ). Come prima, sostituiamo X 0 = C0 X 00 e Y 0 = C0Y 00 nell’espressio-
ne f (X 0 ,Y 0 ) della forma La matrice A00 = (a00i j )n×n associata a f (X 00 ,Y 00 ) dopo aver
effettuato tali sostituzioni, avrá la seconda riga e la seconda colonna tutte nulle,
eccetto l’elemento a0022 . Tale matrice, congruente alle matrici A e A0 , potrá essere
ottenuta dalla relazione A00 = C0T A0C0 ovvero

A00 = C0T A0C0 = C0T CT ACC0 = (CC0 )T A(CC0 ).

Ripetiamo tale iter per ogni riga, ogni volta che f (ei , ei ) 6= 0. Alla fine otterremo la
forma diagonale della matrice e quindi della forma bilineare associata.
Quindi, dopo un certo numero di passi, diciamo r passi, avremo riportato la ma-
trice A associata inizialmente a f in forma diagonale D. Per farlo avremo quindi
effettuato r cambiamenti di base per lo spazio V , a ciascuno dei quali corrisponde
una matrice di transizione. Siano C1 , . . . ,Cr le matrici di transizione che compaiono
nell’algoritmo, in modo tale che Ci sia la matrice di transizione al passo i. Allora

D = (C1 ·C2 · · · · · ·Cr )T · A · (C1 ·C2 · · · · · ·Cr ).

In particolare C = (C1 ·C2 · · · · · ·Cr ) é la matrice di transizione dalla base finale B,


rispetto alla quale la matrice di f é diagonale, alla base inizialmente scelta.
Le colonne di C sono quindi i vettori f -ortogonali che costituiscono la base B.
9.6 Metodo per la diagonalizzazione delle forme simmetriche reali. 239

Osservazione 9.23. Il metodo precedentemente esposto non é altro che il metodo


per la costruzione di basi ortogonali (Gram-Schmidt), con la differenza che il pro-
dotto scalare interno viene sostituito dalla forma bilineare simmetrica Osserviamo
che tale algoritmo é certamente valido nel caso di una forma simmetrica reale non
isotropa su V , per la quale f (v, v) 6= 0, per ogni v ∈ V . In tal caso infatti non vi
é dubbio che f (ei , ei ) 6= 0 in qualunque passo dell’algoritmo, per cui i coefficienti
ck = ff (ei ,ek )
(ei ,ei ) possono sempre essere calcolati ed utilizzati per procedere alla costru-
zione delle basi f -ortogonali.
É facile notare che, ad ogni passo, la matrice di transizione ha la seguente forma:

1 α12 α13 · · ·
 
α1n
 0 1 α23 · · · α2n 
.. 
 
 0 0 ..
 .
. 
..  , per opportuni αi j ∈ R. (9.2)
 
 .. ..
.
 . . 

. ..
 ..

. αn−1,n 
0 0 0 1

Una tale matrice é usualmente detta unitriangolare superiore, nel senso che é trian-
golare superiore ed inoltre ha tutti gli elementi della diagonale principale pari a 1.
Inoltre, segue da semplici calcoli che il prodotto di matrici unitriangolari é ancora
una matrice unitriangolare. Ció implica che la matrice di transizione finale, la qua-
le é prodotto di tutte le matrici di transizione intermedie, é ancora unitriangolare
superiore.
Dobbiamo qundi porci il quesito di come procedere nel caso di spazi simmetri-
ci isotropi, per i quali potrebbe accadere che, ad un certo passo dell’algoritmo,
f (ei , ei ) = 0. In tale eventualitá inseriremo un passo intermedio nell’algoritmo pre-
cedentemente descritto (ogni qual volta sia necessario), al fine di costruire una base
di vettori nella quale quantomeno ei sia non isotropo.
Passo Intermedio
Supponiamo quindi di voler effettuare un cambiamento di base, al fine di annullare
gli elementi delle riga e colonna i-esime della matrice associata a f , ma che la base
B = {e1 , . . . , en }, dalla quale partiamo, sia tale che f (ei , ei ) = 0.
Per poter procedere come sopra esposto, dovremo allora dapprima effettuare un
cambiamento di base da {e1 , e2 , . . . , en } a {e01 , e02 , . . . , e0n } che garantisca la condi-
zione f (e0i , e0i ) 6= 0. Per farlo, sará sufficiente individuare il primo indice di colonna
j per il quale f (ei , e j ) 6= 0. Quindi si effettua il cambiamento di base seguente:

e0i = ei + αe j , e0k = ek per ogni altro indice k

in modo che la scelta di α ∈ R garantisca

f (e0i , e0i ) = f (ei + αe j , ei + αe j ) = 2α f (ei , e j ) + α 2 f (e j , e j ) 6= 0.


240 9 Le forme bilineari e le forme quadratiche reali.

A questo punto si potrá riprendere come nei passi 1 e 2.

Esempio 139. Sia f : R3 × R3 −→ R la forma bilineare simmetrica definita da


f (X,Y ) = 21 x1 y2 + 12 x1 y3 + 12 x2 y3 + 12 x2 y1 + 12 x3 y1 + 12 x3 y2 cioé
 1 1
 
  01 2 2
1
y1
f (X,Y ) = x1 x2 x3  2 0 2
  y2  .
1 1 y3
2 2 0

Si proceda con la diagonalizzazione.

Svolg. Passo 1.
f (e1 , e1 ) = 0 f (e1 , e2 ) 6= 0
e01 = e1 + e2 , e02 = e2 , e03 = e3
La matrice di cambiamento di base é
 
100
C1 =  1 1 0 
001

da cui
x1 = x10 , x2 = x10 + x20 , x3 = x30
y1 = y01 , y2 = y01 + y02 , y3 = y03
e sostituendo la nuova espressione del vettore X otteniamo la forma bilineare
espressa nella nuova base

f (X 0 ,Y 0 ) = x10 y01 + x10 y02 + 2x10 y03 + x20 y03 + x20 y01 + 2x30 y01 + x30 y02 =
 1  0 
 0 0 0  11 2 11 y1
x1 x2 x3  2 0 2   y02  .
1 21 0 y03
Passo 2.
f (e01 , e02 ) 1 f (e01 , e03 )
f (e01 , e01 ) 6= 0, c02 = = , c03 = =1
f (e01 , e01 ) 2 f (e01 , e01 )

1
e002 = e02 − e01 , e003 = e03 − e01
e001 = e01 ,
2
La matrice di cambiamento di base é

1 − 21 −1
 

C2 =  0 1 0 
0 0 1
9.6 Metodo per la diagonalizzazione delle forme simmetriche reali. 241

da cui
1
x10 = x100 − x200 − x300 , x20 = x200 , x30 = x300
2
1
y01 = y001 − y002 − y003 , y02 = y002 , y03 = y003
2
e sostituendo otteniamo la forma bilineare espressa nella nuova base
1
f (X 00 ,Y 00 ) = x100 y001 − x200 y002 − x300 y003 =
4
   00 
 00 00 00  1 01 0 y1
x1 x2 x3  0 − 4 0   y002  .
0 0 −1 y003
La matrice finale del cambiamento di base é

1 − 21 −1
 

C = C1 ·C2 =  1 12 −1 
0 0 1

ed infatti si ha che
  1 1 
1 − 21 −1
   
1 1 0 0 2 2 1 0 0
 − 1 1 0  ·  1 0 1  ·  1 1 −1  =  0 − 1 0  .
2 2 2 2 2 4
−1 −1 1 1 1 0 0 1 0 0 −1
2 2 0

t
u
Esempio 140. Si diagonalizzi la forma bilineare simmetrica f : R3 × R3 −→ R
definita da f (X,Y ) = 5x1 y1 + 3x2 y2 + 21 x1 y3 + 12 x3 y1 cioé
1
  
  50 2 y1
f (X,Y ) = x1 x2 x3  0 3 0   y2  .
1 y3
2 0 0

Svolg. Passo 1.

f (e1 , e2 ) f (e1 , e3 ) 1
f (e1 , e1 ) 6= 0, c2 = = 0, c3 = =
f (e1 , e1 ) f (e1 , e1 ) 10

1
e01 = e1 , e02 = e2 , e03 = e3 − e1
10
La matrice di cambiamento di base é
1
 
1 0 − 10
C = 0 1 0 
00 1
242 9 Le forme bilineari e le forme quadratiche reali.

da cui
1 0
x1 = x10 −x , x2 = x20 , x3 = x30
10 3
1
y1 = y01 − y03 , y2 = y02 , y3 = y03
10
e sostituendo la nuova espressione del vettore X otteniamo la forma bilineare
espressa nella nuova base
1 0 0
f (X 0 ,Y 0 ) = 5x10 y01 + 3x20 y02 − x y =
20 3 3
  0 
5 0 0 y1
x10 x20 x30  0 3 0   y02  .
 

0 1
0 − 20 y03
La matrice finale del cambiamento di base é data dalla sola C ed infatti si ha che

5 0 21 1
       
1 00 1 0 − 10 50 0
 0 1 0 · 0 3 0 · 0 1 0  =  0 3 0 .
1 1 1
− 10 01 2 0 0 00 1 0 0 − 20

t
u

Il metodo precedentemente esposto permette quindi di individuare una base B


rispetto alla quale la forma bilineare simmetrica assume una forma

f (X,Y ) = a11 x1 y1 + a22 x2 y2 + . . . + arr xr yr

con aii ∈ R, dove r é il rango della matrice associata alla forma bilineare f , detto
anche rango di f . Tale metodo per la diagonalizzazione é valido in qualsiasi caso,
indipendentemente dal campo K sul quale V é spazio vettoriale, quindi non neces-
sariamente per K = R.
Ma per spazi vettoriali reali possiamo ottenere risultati piú precisi:
Teorema 9.24. Sia V uno spazio vettoriale reale, dim(V ) = n ≥ 1, f : V ×V → R
una forma bilineare simmetrica reale su V . Esistono una base B di V ed un numero
intero 0 ≤ p ≤ r, dove r é il rango di f , tali che la matrice associata alla f rispetto
alla base B sia  
Ip 0 0
 0 −Ir−p 0  (9.3)
0 0 0
dove I p é il blocco costituito dalla matrice identica di ordine p, Ir−p é il blocco
costituito dalla matrice identica di ordine r − p e gli altri sono tutti blocchi nulli. In
altre parole, in riferimento alla base B, la forma bilineare simmetrica f si presenta
nella forma

f (X,Y ) = x1 y1 + x2 y2 + . . . + x p y p − x p+1 y p+1 − . . . − xr yr .


9.6 Metodo per la diagonalizzazione delle forme simmetriche reali. 243

Si noti che in modo equivalente possiamo dire che ogni matrice simmetrica A ∈
Mn (R) é congruente ad una matrice diagonale della forma (9.3).

Dim. Per quanto fin qui detto, sappiamo che esiste una base ortogonale di V ri-
spetto alla quale la matrice di f é espressa in forma diagonale. Supponiamo che r
sia il rango di tale matrice, p sia il numero di elementi diagonali positivi, r − p il nu-
mero di quelli negativi e n − r il numero di zeri sulla diagonale principale. Pensiamo
adesso di riordinare opportunamente i vettori di tale base, ottenendo una nuova base
di V ed indicandola B = {b1 , . . . , bn }, in modo tale che
– f (bi , bi ) > 0, per ogni i = 1, . . . , p
– f (bi , bi ) < 0, per ogni i = p + 1, . . . , r
– f (bi , bi ) = 0, per ogni i = r + 1, . . . , n.
La matrice di f rispetto a B sará quindi espressa nella forma diagonale

α12
 
..
.
 
 
α p2
 
 
2
 

 −α p+1 

D=
 .. 
 . 


 −αr2 


 0 

 .. 
 . 
0

dove αi ∈ R per ogni i = 1, . . . , r, ovvero


– f (bi , bi ) = αi2 , per ogni i = 1, . . . , p
– f (bi , bi ) = −αi2 , per ogni i = p + 1, . . . , r
– f (bi , bi ) = 0, per ogni i = r + 1, . . . , n.
A questo punto, costruiamo una ulteriore base B = {b01 , . . . , b0n }, partendo da B, nel
modo seguente:
–b0i = αbii , per ogni i = 1, . . . , r
–b0i = bi , per ogni i = r + 1, . . . , n.
Osserviamo infine che
– f (b0i , b0i ) = 1
αi2
f (bi , bi ) = 1, per ogni i = 1, . . . , p
– f (b0i , b0i ) = 1
αi2
f (bi , bi ) = −1, per ogni i = p + 1, . . . , r
– f (b0i , b0i ) = f (bi , bi ) = 0, per ogni i = r + 1, . . . , n
per cui la matrice associata a f rispetto alla base B é
244 9 Le forme bilineari e le forme quadratiche reali.
 
1
 .. 

 . 


 1 


 −1 

0
D =
 .. 
 . 


 −1 


 0 

 .. 
 . 
0

con p termini diagonali positivi, r − p negativi e n − r nulli. t


u
Il Teorema 9.24 ci consente di introdurre la seguente:
Definizione 9.25. Sia f : V × V → R una forma bilineare simmetrica reale sullo
spazio vettoriale reale V di dimensione n. Siano 0 ≤ p ≤ r ≤ n tali che esiste una
base di V rispetto alla quale la matrice associata a f sia
 
Ip 0 0
A =  0 −Ir−p 0  (9.4)
0 0 0

dove I p é il blocco costituito dalla matrice identica di ordine p, Ir−p é il blocco


costituito dalla matrice identica di ordine r − p e gli altri sono tutti blocchi nulli.
Allora r é detto indice di positivitá di f (o equivalentemente di A), e r − p é detto
indice di negativitá di f (ovvero di A).
Esempio 141. Rispetto ad una base inizale, la forma bilineare simmetrica in R3 ,
f (X,Y ) = 12 x1 y2 + 21 x1 y3 + 12 x2 y3 + 21 x2 y1 + 21 x3 y1 + 12 x3 y2 , ha una matrice associata
1 1
 
0 2 2
A =  12 0 12  .
1 1
2 2 0

Nell’Esempio 139, abbiamo visto che f ha una forma diagonale espressa da f (X 0 ,Y 0 ) =


x10 y01 − 14 x20 y02 − x30 y03 , rispetto alla base

1 1
B = {(1, 1, 0), (− , , 0), (−1, −1, 1)}.
2 2
Per determinare la forma di f del tipo (9.3), partiamo dalla matrice diagonale
congruente ad A, ovvero la matrice di f rispetto alla base B:
 
1 0 0
A0 =  0 − 14 0  .
0 0 −1
9.7 Segnatura di una forma bilineare simmetrica 245

Poniamo allora α12 = 1, α22 = 14 , α32 = 1 e scegliamo

(1, 1, 0) (− 12 , 12 , 0) (−1, −1, 1)


e01 = , e02 = , e03 =
α1 α2 α3
ovvero

(− 12 , 21 , 0)
e01 = (1, 1, 0), e02 = 1
= (−1, 1, 0), e03 = (−1, −1, 1).
2

Sia B0 = {e01 , e02 , e03 } ed esprimiamo la forma bilineare rispetto a tale base. Poiché la
matrice di transizione é fornita dalle componenti dei vettori della base B0 , essa é
 
1 −1 −1
C =  1 1 −1  .
0 0 1

Allora la matrice di f associata a B0 é

A00 = CT · A ·C
   1 1  
1 1 0 0 2 2 1 −1 −1
=  −1 1 0  ·  12 0 21  ·  1 1 −1 
−1 −1 1 1 1 0 0 1
2 2 0
 
1 0 0
=  0 −1 0  .
0 0 −1

Concludiamo che l’indice di positivitá di f é pari a 1, mentre l’indice di negativitá


é pari a 2.

9.7 Segnatura di una forma bilineare simmetrica

Per poter discutere degli indici di positivitá e negativitá di una forma bilineare
simmetrica é necessario fissare alcuni semplici risultati relativi alla teoria delle
matrici.
Lemma 9.26. Siano A, P ∈ Mn (R) matrici a coefficienti reali e sia P invertibile.
Allora le matrici A e PA hanno lo stesso rango.
Corollario 9.27. Siano A, P ∈ Mn (R) matrici a coefficienti reali e sia P invertibile.
Allora le matrici A e AP hanno lo stesso rango.
Corollario 9.28. Siano A, P ∈ Mn (R) matrici a coefficienti reali e sia P invertibile.
Allora le matrici A e PT AP hanno lo stesso rango.
246 9 Le forme bilineari e le forme quadratiche reali.

Teorema 9.29. Sia f : V ×V → R una forma bilineare simmetrica reale sullo spazio
vettoriale reale V di dimensione n. Gli indici di positivitá e negativitá di f non
dipendono dalla base di V rispetto alla quale si determina la matrice di f .
Teorema 9.30. Sia A ∈ Mn (R) una matrice simmetrica e siano p, r − p rispet-
tivamente i suoi indici di positivitá e di negativitá. Allora, il numero di autovalori
positivi di A é esattamente p, quello dei suoi autovalori negativi é esattamente r − p.
L’autovalore 0 di A possiede una molteplicitá algebrica pari a n − r.
Definizione 9.31. Sia f : V × V → R una forma bilineare simmetrica reale sullo
spazio V di dimensione n. Siano p il suo indice di positivitá e r − p quello di nega-
tivitá.
La coppia (p, r − p) é detta segnatura di f .
Essa é anche la segnatura di una qualsiasi matrice simmetrica che rappresenti f in
una data base di V .
Definizione 9.32. Sia (p, r − p) la segnatura della forma bilineare simmetrica f :
V ×V → R sullo spazio vettoriale reale V di dimensione n.
1. f é detta definita positiva se f (X, X) > 0, per ogni 0 6= X ∈ V . In tal caso la sua
segnatura é (n, 0);
2. f é detta definita negativa se f (X, X) < 0, per ogni 0 6= X ∈ V , e la sua segnatura
é (0, n);
3. f é detta semidefinita positiva se f (X, X) ≥ 0, per ogni 0 6= X ∈ V . Affinché non
sia esattamente definita positiva, dovrá esistere 0 6= X ∈ V tale che f (X, X) = 0.
In tal caso la sua segnatura é (r, 0) con r < n;
4. f é detta semidefinita negativa se f (X, X) ≤ 0, per ogni 0 6= X ∈ V . Affinché non
sia esattamente definita negativa, dovrá esistere 0 6= X ∈ V tale che f (X, X) = 0,
e la sua segnatura é (0, r) con r < n.
5. f é detta indefinita nel caso abbia segnatura (p, r − p) con 0 < p < r ≤ n. In tal
caso esisteranno vettori 0 6= X ∈ V e 0 6= Y ∈ V tali che f (X, X) > 0 e f (Y,Y ) < 0.
In modo del tutto analogo introduciamo la medesima classificazione per le matrici
reali simmetriche:
Definizione 9.33. Sia (p, r − p) la segnatura della di una matrice simmetrica A ∈
Mn (R).
1. A é detta definita positiva se X T AX > 0, per ogni 0 6= X ∈ Rn . In tal caso la sua
segnatura é (n, 0);
2. A é detta definita negativa se X T AX < 0, per ogni 0 6= X ∈ Rn , e la sua segnatura
é (0, n);
3. A é detta semidefinita positiva se X T AX ≥ 0, per ogni 0 6= X ∈ Rn . Affinché non
sia esattamente definita positiva, dovrá esistere 0 6= X ∈ Rn tale che X T AX = 0.
In tal caso la sua segnatura é (r, 0) con r < n;
4. A é detta semidefinita negativa se X T AX ≤ 0, per ogni 0 6= X ∈ Rn . Affinché non
sia esattamente definita negativa, dovrá esistere 0 6= X ∈ Rn tale che X T AX = 0,
e la sua segnatura é (0, r) con r < n.
9.8 Forme quadratiche reali. 247

5. A é detta indefinita nel caso abbia segnatura (p, r − p) con 0 < p < r ≤ n. In tal
caso esisteranno vettori 0 6= X ∈ Rn e 0 6= Y ∈ Rn tali che X T AX > 0 e Y T AY < 0.

9.8 Forme quadratiche reali.

Sia V uno spazio vettoriale reale di dimensione n, B = {e1 , . . . , en } una base di V e


denotiamo con X = [x1 , . . . , xn ]T il vettore delle componenti di X ∈ V rispetto alla
base B.
Una forma quadratica reale su V é una qualsiasi q : V → R tale che q(X) =
∑ni, j=1 αi j xi x j , cioé q(X) é un polinomio omogeneo di grado 2 nelle indetermi-
nate {x1 , . . . , xn }. Rimarchiamo che tale definizione é strettamente correlata alle
componenti x1 , . . . , xn di X ∈ V , ovvero alla base scelta per lo spazio vettoriale.
Diamo una definizione di forma quadratica che sia libera dal vincolo della scelta
delle componenti per un vettore:
Definizione 9.34. Sia q : V → R una qualsiasi corrispondenza e definiamo

f : V ×V → R
(9.5)
 
1
f (u, v) = q(u + v) − q(u) − q(v)
2

Diremo che q é una forma quadratica reale su V se valgono entrambe le seguenti:


1. la f definita in (9.5) é bilineare;
2. per ogni α ∈ R e X ∈ V , q(αX) = α 2 q(X).
Dalla sua definizione, é evidente che f sia simmetrica. Essa é detta forma bilineare
simmetrica associata a q.
Teorema 9.35. Una funzione q : V → R é una forma quadratica reale su V se e solo
se esiste una forma bilineare ϕ : V ×V → R tale che q(v) = ϕ(v, v), per ogni v ∈ V .
Osservazione 9.36. Il Teorema precedente evidenzia il fatto che una forma qua-
dratica q puó essere espressa sia attraverso una qualsiasi forma bilineare ϕ tale che
ϕ(v, v) = q(v), per ogni v ∈ V , ed anche tramite la forma bilineare simmetrica asso-
ciata a q.
Non é quindi necessario che la forma bilineare scelta per costruire la forma quadra-
tica sia simmetrica. Ma se lo é, allora essa deve essere esattamente la ϕ del Teorema
9.35.
In altre parole, esiste una sola forma bilineare simmetrica associata ad una forma
quadratica reale.
Definizione 9.37. Sia q : V → R una forma quadratica realeV , B = {e1 , . . . , en } una
base di V e ϕ una qualsiasi forma bilineare associata a q, cioé ϕ(v, v) = q(v), per
ogni v ∈ V . Se C é la matrice di ϕ ripsetto ad una base B di V , allora q(v) = ϕ(v, v) =
vT Cv. Diremo che C é una matrice associata a q rispetto alla base B di V .
248 9 Le forme bilineari e le forme quadratiche reali.

Alla luce dell’Osservazione 9.36, é chiaro che esiste una sola matrice simmetrica
associata ad una forma quadratica in termini di una fissata base di V .
La questione piú rilevante é quindi quella di individuare un modo per determinare
la forma simmetrica associata ad una forma quadratica.
Teorema 9.38. Sia q : V → R una forma quadratica reale sullo spazio vettoriale
V , B = {e1 , . . . , en } una base di V e A ∈ Mn (R) la matrice di q rispetto alla base B.
Allora l’unica forma bilineare  associata a q é rappresentata, rispetto alla base B di
V , dalla matrice 12 C + CT , dove C é la matrice di una qualsiasi forma bilineare
associata a q.
Quindi la matrice di una forma quadratica q coincide con la matrice della corri-
spondente forma bilineare simmetrica e puó essere determinata come segue. Sia
B una base di V , f : V × V → R una forma bilineare simmetrica e q : V → R la
corrispondente forma quadratica, tali che
n n
q(X) = ∑ αi xi2 + ∑ αi j xi x j , f (X,Y ) = ∑ βi j xi y j
i=1 i< j i, j=1

La matrice associata a f e q é data da:


α1n
α1 α212 · · ·
 
2
 α12 α2 · · · α2n 
 2 2
A= . . .

.. 
 .. .. . . . 
α1n α2n
2 2 ··· αn
 
β11 β12 ··· β1n
 β12 β22 ··· β2n 
= .
 
.. .. .. 
 .. . . . 
β1n β2n · · · βnn
αi j
ovvero 2 = βi j per ogni i 6= j e αi = βii per ogni i = 1, . . . , n.
Esempio 142. Sia q : R3 → R la forma quadratica definita da

q (x1 , x2 , x3 ) = x12 + 6x1 x2 + 4x1 x3 + x22 + 8x2 x3 + 2x32




rispetto alla base canonica di R3 . In base a quanto precedentemente detto, ciascuna


delle seguenti forme bilineari puó essere associata a q:
– f : R3 × R3 → R tale che

f (x1 , x2 , x3 ), (y1 , y2 , y3 ) = x1 y1 + 4x1 y2 + x1 y3 + 2x2 y1 + x2 y2

= +6x2 y3 + 3x3 y1 + 2x3 y2 + 2x3 y3

–g : R3 × R3 → R tale che
9.8 Forme quadratiche reali. 249

g (x1 , x2 , x3 ), (y1 , y2 , y3 ) = x1 y1 + x1 y2 + 2x1 y3 + 5x2 y1 + x2 y2

= +5x2 y3 + 2x3 y1 + 3x3 y2 + 2x3 y3

–h : R3 × R3 → R tale che

h (x1 , x2 , x3 ), (y1 , y2 , y3 ) = x1 y1 + 5x1 y2 + 3x1 y3 + x2 y1 + x2 y2
.
= +7x2 y3 + x3 y1 + x3 y2 + 2x3 y3

Nessuna delle precedenti forme bilineari é simmetrica. Per ottenere l’unica forma
bilineare simmetrica associata a q, é sufficiente scegliere una qualsiasi delle prece-
denti forme f , g oppure h, individuare la matrice A ad essa relativa, ed effettuare il
calcolo 21 (A + AT ).
Ad esempio, la matrice associata a f é
 
141
A = 2 1 6
322

per cui  
132
1
(A + AT ) =  3 1 4 
2
242
che é la matrice rappresentativa (rispetto alla base canonica) sia di q che dell’unica
forma bilineare associata a q.
Lo stesso otterremmo se scegliessimo di utilizzare le matrici rappresentative di g
oppure di h.
Esempio 143. Sia f : R3 × R3 −→ R la forma bilineare simmetrica definita da
  
  110 y1
f (X,Y ) = x1 x2 x3  1 0 2   y2  = x1 y1 + x2 y1 + x1 y2 + 2x3 y2 + 2x2 y3 + x3 y3
021 y3

rispetto alla base canonica di R3 . Allora la forma quadratica associata a f é la q :


R3 −→ R definita da
  
 110 x1
q(X) = f (X, X) = x1 x2 x3  1 0 2   x2  = x12 + 2x1 x2 + 4x2 x3 + x32 .


021 x3

Esempio 144. Sia q : R3 −→ R la forma quadratica definita da q(X) = 3x12 +2x1 x2 −


4x2 x3 + 7x22 − x32 cioé
250 9 Le forme bilineari e le forme quadratiche reali.
  
 3 1 0 x1
q(X) = x1 x2 x3  1 7 −2   x2  = 3x12 + 2x1 x2 − 4x2 x3 + 7x22 − x32


0 −2 −1 x3

rispetto alla base canonica di R3 . Allora la forma bilineare simmetrica associata a q


é la f : R3 × R3 −→ R definita da
  
  3 1 0 y1
f (X,Y ) = x1 x2 x3  1 7 −2   y2  =
0 −2 −1 y3

3x1 y1 + x1 y2 + 7x2 y2 + x2 y1 − 2x2 y3 − x3 y3 − 2x3 y2 .


Per quanto visto fin’ora, potremo da ora in poi confondere le proprietá relative ad
una forma bilineare simmetrica con quelle della forma quadratica ad essa associata.
Infatti tali proprietá sono semplicemente caratteristiche della matrice che rappresen-
ta entrambe le f e q.
Alla luce dell’equivalenza tra forme quadratiche a forme bilineari simmetriche in
termini della matrice simmetrica che le rappresenta simultaneamente, possiamo
riformulare tutti i Teoremi giá citati per le forme bilineari simmetriche.
Cambiamento di basi

Sia q : V → R una forma quadratica, B = {e1 , . . . , en } una base di V ed A = ai j
la matrice di q rispetto a B. Per ogni X ∈ V , espresso per componenti rispetto a B,
sappiamo che q(X) = X T AX.
Se B0 é un’altra base di V , allora la matrice A0 che rappresenta q rispetto a B0 é in
relazione con la matrice A tramite A0 = PT AP, dove P é la matrice di transizione
da B0 a B. Per cui q(X 0 ) = X 0T A0 X 0 , per ogni vettore X 0 espresso per componenti
rispetto a B0 . Quindi due matrici A, A0 rappresentano q, rispetto a due differenti basi
di V , se e solo se esse sono congruenti.
9.8 Forme quadratiche reali. 251

Diagonalizzazione di una forma quadratica reale


Teorema 9.39. Sia q : V → R una forma quadratica reale, associata alla forma
bilineare simmetrica f . Esiste una base f -ortogonale B = {e1 , . . . , en } di V tale che
la matrice di q rispetto a B ha la forma seguente:
 
Ip 0 0
 0 −Ir−p 0 
0 0 0

dove I p é il blocco costituito dalla matrice identica di ordine p, Ir−p é il blocco


costituito dalla matrice identica di ordine r − p e gli altri sono tutti blocchi nulli.
Quindi, per ogni vettore X = [x1 , . . . , xn ]T ∈ V espresso per componenti rispetto alla
base B:
q(X) = X T AX
  x 
Ip 0 0 1
= [x1 , . . . , xn ] ·  0 −Ir−p 0  ·  ... 
 
0 0 0 xn

= x12 + · · · + x2p − x2p+1 − · · · − xr2 .

Segnatura di una forma quadratica reale


Sia q : V → R una forma quadratica reale, associata alla forma bilineare simmetrica
f , e sia B = {e1 , . . . , en } la base f -ortogonale rispetto alla quale
 
Ip 0 0
 0 −Ir−p 0 
0 0 0

é la matrice associata a q. La coppia (p, r − p) é detta segnatura della forma


quadratica q. Inoltre:
1. q é detta definita positiva se q(X) > 0, per ogni 0 6= X ∈ V . In tal caso la sua
segnatura é (n, 0);
2. q é detta definita negativa se q(X) < 0, per ogni 0 6= X ∈ V , e la sua segnatura é
(0, n);
3. q é detta semidefinita positiva se q(X) ≥ 0, per ogni 0 6= X ∈ V . Affinché non sia
esattamente definita positiva, dovrá esistere 0 6= X ∈ V tale che q(X) = 0. In tal
caso la sua segnatura é (r, 0) con r < n;
4. q é detta semidefinita negativa se q(X) ≤ 0, per ogni 0 6= X ∈ V . Affinché non
sia esattamente definita negativa, dovrá esistere 0 6= X ∈ V tale che q(X) = 0, e
la sua segnatura é (0, r) con r < n.
5. q é detta indefinita nel caso abbia segnatura (p, r − p) con 0 < p < r ≤ n. In tal
caso esisteranno vettori 0 6= X ∈ V e 0 6= Y ∈ V tali che q(X) > 0 e q(Y ) < 0.
252 9 Le forme bilineari e le forme quadratiche reali.

9.9 Criteri di positivitá.

Siano f : Rn × Rn → R una forma bilineare simmetrica e q : Rn → R la forma


quadratica associata. La f é definita positiva, quando lo é q (e viceversa). Una forma
bilineare definita positiva é anche chiamata prodotto scalare. Per quanto detto in
precedenza, per ogni forma quadratica q definita positiva esiste una base B di Rn
rispetto alla quale q sia esprimibile

q(X) = x12 + x22 + . . . + xn2

e quindi
f (X,Y ) = x1 y1 + x2 y2 + . . . + xn yn
il quale é ben noto come prodotto scalare standard. Uno spazio vettoriale V in cui
sia introdotto un prodotto scalare é detto spazio vettoriale euclideo.
Dedichiamo quest’ultima Sezione proprio all’esposizione di alcuni metodi utili
per stabilire se una forma bilineare simmetrica (o equivalentemente una forma
quadratica) reale sia definita positiva, cioé quando essa é un prodotto scalare.
Teorema 9.40. Sia f : Rn × Rn → R una forma bilineare simmetrica e sia A la
matrice che la rappresenta rispetto ad una base B = {e1 , . . . , en } di Rn . Allora f é
definita positiva (é un prodotto scalare) se e solo se tutti gli autovalori di A sono
positivi.

Dim. É una conseguenza del Teorema 9.30. Infatti, f é definita positiva se e solo
se la sua segnatura é (n, 0). Ció accade se e solo se la matrice associata a f possiede
solo autovalori positivi. t
u
Il seguente Teorema, noto come criterio di Cartesio, é utile per determinare il segno
degli autovalori di una matrice, come radici del suo polinomio caratteristico. Puó
quindi essere uno strumento per determinare la segnatura di una forma quadratica o
bilineare simmetrica:
Teorema 9.41. Sia p(x) = an xn + an−1 xn−1 + . . . + ar xr un polinomio di grado n
a coefficienti ai reali, con 0 ≤ r ≤ n e ar 6= 0. Supponiamo che p(x) abbia tutte le
radici reali. Allora
1. 0 é radice di p(x) se e solo se r ≥ 1 (in tale caso la molteplicitá dello 0 come
radice del polinomio é esattamente r);
2. p(x) ha tante radici positive quante sono le variazioni di segno nella successione
dei suoi coefficienti non nulli.
Terminiamo osservando come l’analisi delle sottomatrici principali alte della matri-
ce associata ad una forma bilineare simmetrica possa aiutarci nel comprendere se la
forma sia definita positiva.
Di seguito, data la matrice A = Mn (R), indicheremo col termine Ak la sottomatrice
principale alta di ordine k, ovvero quella ottenuta prendendo ordinatamente la prime
k righe e k colonne di A. Inoltre diremo minore complementare principale di ordine
k il determinante della sottomatrice principale alta Ak .
9.10 Esercizi svolti. 253

Teorema 9.42. Siano q : V → R una forma quadratica reale ed A ∈ Mn (R) una


matrice simmetrica associata a q in una qualche base di V . Allora q é definita
positiva se e solo se ciascuno dei minori complementari principali di A é positivo.

9.10 Esercizi svolti.

Esercizio 73. Sia q : R3 −→ R la forma quadratica definita da q(X) = x12 − 2x1 x2 +


x1 x3 cioé
1
  
  1 −1 2 x1
q(X) = x1 x2 x3  −1 0 0   x2  .
1 x3
2 0 0
Determinarne una forma diagonale.

Svolg.
La matrice associata alla forma quadratica é

1 −1 12
 

A =  −1 0 0 
1
2 0 0

Poiché q(e1 ) 6= 0 allora possiamo effettuare il seguente cambiamento di base:


1
−1 1
e01 = e1 , e02 = e2 − e1 = e2 + e1 , e03 = e3 − 2 e1 = e3 − e1
1 1 2
la cui matrice associata é
1 1 − 21
 

C1 =  0 1 0 
00 1
da cui
1
x1 = x10 + x20 − x30 , x2 = x20 , x3 = x30 .
2
L’espressione della forma quadratica nelle nuove coordinate é
1
q(x10 , x20 , x30 ) = x10 2 − x20 2 − x30 2 + x20 x30
4
con matrice associata  
1 0 0
A0 =  0 −1 21  .
0 12 − 14
Poiché q(e02 ) 6= 0 allora possiamo effettuare il seguente cambiamento di base:
254 9 Le forme bilineari e le forme quadratiche reali.
1
e001 = e01 , e002 = e02 , e003 = e03 − 2
e02 = e03 + e02
−1
la cui matrice associata é  
100
C2 =  0 1 12 
001
da cui
1
x10 = x100 ,
x20 = x200 + x300 , x30 = x300 .
2
L’espressione della forma quadratica nelle nuove coordinate é

q(x100 , x200 , x300 ) = x100 2 − x200 2

con matrice associata  


1 0 0
A00 =  0 −1 0 .
0 0 0
La base ortogonale B rispetto a cui la forma quadratica si esprime in forma diagonale
é data dalle colonne della matrice C = C1 ×C2 :
 
110
C =  0 1 21 
001

per cui B = {(1, 0, 0), (1, 1, 0), (0, 12 , 1)} e la matrice associata alla forma quadratica
in base B é A00 = CT AC. tu
Esercizio 74. Sia q : R3 −→ R la forma quadratica definita da q(X) = 2x12 − x1 x2 +
2x1 x3 cioé
1
  
  21 − 2 1 x1
q(X) = x1 x2 x3  − 2 0 0   x2  .
1 0 0 x3
Determinarne una forma diagonale.

Svolg. La matrice associata alla forma quadratica é

2 − 12 1
 

A =  − 12 0 0 
1 0 0

Poiché q(e1 ) 6= 0 allora possiamo effettuare il seguente cambiamento di base:

− 21 1 1
e01 = e1 , e02 = e2 − e1 = e2 + e1 , e03 = e3 − e1
2 4 2
9.10 Esercizi svolti. 255

la cui matrice associata é


1 41 − 12
 

C1 =  0 1 0 
00 1
da cui
1 1
x1 = x10 + x20 − x30 , x2 = x20 , x3 = x30 .
4 2
L’espressione della forma quadratica nelle nuove coordinate é
1 1 1
q(x10 , x20 , x30 ) = 2x10 2 − x20 2 − x30 2 + x20 x30
8 2 2
con matrice associata  
2 0 0
A0 =  0 − 18 14  .
0 14 − 12
Poiché q(e02 ) 6= 0 allora possiamo effettuare il seguente cambiamento di base:
1
e001 = e01 , e002 = e02 , e003 = e03 − 4
e02 = e03 + 2e02
− 81

la cui matrice associata é  


100
C2 =  0 1 2 
001
da cui
x10 = x100 , x20 = x200 + 2x300 , x30 = x300 .
L’espressione della forma quadratica nelle nuove coordinate é
1
q(x100 , x200 , x300 ) = 2x100 2 − x200 2
8
con matrice associata  
2 0 0
A00 =  0 − 81 0  .
0 0 0
La base ortogonale B rispetto a cui la forma quadratica si esprime in forma diagonale
é data dalle colonne della matrice C = C1 ×C2 :
 1 
1 4 0
C = 0 1 2
001

per cui B = {(1, 0, 0), ( 14 , 1, 0), (0, 2, 1)} e la matrice associata alla forma quadratica
in tale base é A00 = CT AC.
256 9 Le forme bilineari e le forme quadratiche reali.

Per ottenere una forma diagonale in cui compaiano come coefficienti non nulli so-
lamente +1 e −1 dobbiamo esprimere la forma quadratica rispetto ad una base
ortonormale B0 . Questa si ottiene dalla base ortogonale B, dividendo il vettore che
occupa la colonna i nella matrice C, per la radice quadrata del valore assoluto dello
scalare (se é non nullo) presente nella colonna i della matrice A00 :

(1, 0, 0) ( 1 , 1, 0)
B0 = { √ , 4q , (0, 2, 1)} =
2 1
8

1 1 √
{( √ , 0, 0), ( √ , 2 2, 0), (0, 2, 1)}.
2 2
Quindi la matrice ortonormale di cambiamento di base
 1 1 
√ √ 0
2 √2
C0 =  0 2 2 2 
0 0 1

ci permette di ottenere la forma diagonale


 
1 0 0
A000 = C0T AC0 =  0 −1 0  .
0 0 0

t
u
Esercizio 75. Sia q : R3 −→ R la forma quadratica definita da q(X) = 2x1 x2 +
2x1 x3 − 2x32 cioé   
  01 1 x1
q(X) = x1 x2 x3  1 0 0   x2  .
1 0 −2 x3
Determinarne una forma diagonale.

Svolg.
La matrice associata alla forma quadratica é
 
01 1
A = 1 0 0 
1 0 −2

Poiché q(e1 ) = 0 allora dobbiamo effettuare il seguente cambiamento di base:

e01 = e1 + e2 , e02 = e2 , e03 = e3

la cui matrice associata é


9.10 Esercizi svolti. 257
 
100
C1 =  1 1 0 
001
da cui
x1 = x10 , x2 = x10 + x20 , x3 = x30 .
L’espressione della forma quadratica nelle nuove coordinate é

q(x10 , x20 , x30 ) = 2x10 2 + 2x10 x20 + 2x10 x30 − 2x30 2

con matrice associata  


21 1
A0 =  1 0 0  .
1 0 −2
Poiché q(e01 ) 6= 0 allora possiamo effettuare il seguente cambiamento di base:

1 1
e001 = e01 , e002 = e02 − e01 , e003 = e03 − e01
2 2
la cui matrice associata é
1 − 21 − 12
 

C2 =  0 1 0 
0 0 1
da cui
1 1
x10 = x100 − x200 − x300 , x20 = x200 , x30 = x300 .
2 2
L’espressione della forma quadratica nelle nuove coordinate é
1 5
q(x100 , x200 , x300 ) = 2x100 2 − x200 2 − x300 2 − x200 x300
2 2
con matrice associata  
2 0 0
A00 =  0 − 21 − 12  .
0 − 12 − 52
Poiché q(e002 ) 6= 0 allora possiamo effettuare il seguente cambiamento di base:

e000 00
1 = e1 , e000 00
2 = e2 , e000 00 00
3 = e3 − e2

la cui matrice associata é  


10 0
C3 =  0 1 −1 
00 1
da cui
x100 = x1000 , x200 = x2000 − x3000 , x300 = x3000 .
258 9 Le forme bilineari e le forme quadratiche reali.

L’espressione della forma quadratica nelle nuove coordinate é


1
q(x1000 , x2000 , x3000 ) = 2x1000 2 − x2000 2 − 2x3000 2
2
con matrice associata  
2 0 0
A000 =  0 − 21 0  .
0 0 −2
La base ortogonale B rispetto a cui la forma quadratica si esprime in forma diagonale
é data dalle colonne della matrice C = C1 ×C2 ×C3 :

1 − 12 0
 

C =  1 12 −1 
0 0 1

per cui B = {(1, 1, 0), (− 21 , 21 , 0), (0, −1, 1)} e la matrice associata alla forma qua-
dratica in tale base é A000 = CT AC.
Per ottenere una forma diagonale in cui compaiano come coefficienti non nulli so-
lamente +1 e −1 dobbiamo esprimere la forma quadratica rispetto ad una base
ortonormale B0 . Questa si ottiene dalla base ortogonale B, dividendo il vettore che
occupa la colonna i nella matrice C, per la radice quadrata del valore assoluto dello
scalare (se é non nullo) presente nella colonna i della matrice A000 :

(1, 1, 0) (− 21 , 12 , 0) (0, −1, 1)


B0 = { √ , q , √ }=
2 1 2
2

1 1 1 1 1 1
{( √ , √ , 0), (− √ , √ , 0), (0, − √ , √ )}.
2 2 2 2 2 2
Quindi la matrice ortonormale di cambiamento di base
 1
√ − √1

2 2
0
C0 =  √12 √12 − √12 
 
0 0 √1
2

ci permette di ottenere la forma diagonale




1 0 0
Aiv = C0T AC0 =  0 −1 0  .
0 0 −1

t
u
Esercizio 76. Sia q : R3 −→ R la forma quadratica definita da q(X) = x12 + 2x22 +
2x1 x3 + x32 cioé
9.10 Esercizi svolti. 259
  
  101 x1
q(X) = x1 x2 x3  0 2 0   x2  .
101 x3
Determinarne una forma diagonale.

Svolg. La matrice associata alla forma quadratica é


 
101
A = 0 2 0
101

Poiché q(e1 ) 6= 0 allora possiamo effettuare il seguente cambiamento di base:

e01 = e1 , e02 = e2 , e03 = e3 − e1

la cui matrice associata é  


1 0 −1
C = 0 1 0 
00 1
da cui
x1 = x10 − x30 , x2 = x20 , x3 = x30 .
L’espressione della forma quadratica nelle nuove coordinate é

q(x10 , x20 , x30 ) = x10 2 + 2x20 2

con matrice associata  


100
A0 =  0 2 0  .
000
La base ortogonale B rispetto a cui la forma quadratica si esprime in forma diagonale
é data dalle colonne della matrice C, cioé:

B = {(1, 0, 0), (0, 1, 0), (−1, 0, 1)}

e la matrice associata alla forma quadratica in tale base é A0 = CT AC.


Per ottenere una forma diagonale in cui compaiano come coefficienti non nulli so-
lamente +1 e −1 dobbiamo esprimere la forma quadratica rispetto ad una base
ortonormale B0 . Questa si ottiene dalla base ortogonale B, dividendo il vettore che
occupa la colonna i nella matrice C, per la radice quadrata del valore assoluto dello
scalare (se é non nullo) presente nella colonna i della matrice A0 :

(0, 1, 0)
B0 = {(1, 0, 0), √ , (−1, 0, 1)}.
2
Quindi la matrice ortonormale di cambiamento di base
260 9 Le forme bilineari e le forme quadratiche reali.

1 0 −1
 

C0 =  0 √12 0 
0 0 1

ci permette di ottenere la forma diagonale


 
100
A00 = C0T AC0 =  0 1 0 
000

che individua una forma quadratica semidefinita positiva di segnatura (2, 0).

Risolviamo adesso lo stesso esercizio con un metodo alternativo, tramite l’utilizzo


degli autovalori della matrice A.
Dopo aver risolto l’equazione caratteristica associata alla matrice A, si ottengono co-
me autovalori λ1 = 0 con molteplicitá algebrica 1, λ2 = 2 con molteplicitá algebrica
2.
L’autospazio relativo a λ1 é generato dall’autovettore (1, 0, −1). Normalizzando tale
vettore, otteniamo ( √12 , 0, − √12 ).
L’autospazio relativo a λ2 é generato dagli autovettori (1, 0, 1), (0, 1, 0). Normaliz-
zando tali vettori, otteniamo la coppia ( √12 , 0, √12 ), (0, 1, 0) (il secondo resta invariato
avendo norma pari ad 1).
La base rispetto a cui la forma quadratica é diagonalizzabile, con gli autovalori
sulla diagonale principale della matrice ad essa associata, é formata proprio da tali
autovettori, :
1 1 1 1
B = {( √ , 0, − √ ), ( √ , 0, √ ), (0, 1, 0)}.
2 2 2 2
La matrice di cambiamento di base é allora
 1 
√ √1 0
2 2
C= 0 0 1
 
− √12 √1 0
2

tale che  
000
CT AC =  0 2 0  .
002
Al solito, per ottenere una forma diagonale in cui compaiano come coefficienti non
nulli solamente +1 e −1 dobbiamo esprimere la forma quadratica rispetto ad una
base ortonormale B0 che si ottiene dalla base B, dividendo ogni vettore per la radice
quadrata del valore assoluto dell’autovalore (se é non nullo) ad esso relativo:
1 1 1 1 1
B0 = {( √ , 0, − √ ), ( , 0, ), (0, √ , 0)}.
2 2 2 2 2
La matrice di cambiamento di base é allora
9.10 Esercizi svolti. 261

√1 1
 
2 2 0
C0 =  0 0 √12 
 
− √12 12 0

tale che  
000
C0T AC0 =  0 1 0  .
001
t
u
Esercizio 77. Sia q : R2 −→ R la forma quadratica definita da q(X) = 3x12 + 3x22 +
4x1 x2 cioé   
  3 2 x1
q(X) = x1 x2 .
2 3 x2
Determinarne una forma diagonale.

Svolg. La matrice associata alla forma quadratica é


 
32
A=
23

Poiché q(e1 ) 6= 0 allora possiamo effettuare il seguente cambiamento di base:

2
e01 = e1 , e02 = e2 − e1
3
la cui matrice associata é
1 − 23
 
C=
0 1
da cui
2
x1 = x10 − x20 , x2 = x20 .
3
L’espressione della forma quadratica nelle nuove coordinate é
5
q(x10 , x20 ) = 3x10 2 + x20 2
3
con matrice associata  
0 30
A = .
0 53
La base ortogonale B rispetto a cui la forma quadratica si esprime in forma diagonale
é data dalle colonne della matrice C, cioé:
2
B = {(1, 0), (− , 1)}
3
e la matrice associata alla forma quadratica in tale base é A0 = CT AC.
262 9 Le forme bilineari e le forme quadratiche reali.

Per ottenere una forma diagonale in cui compaiano come coefficienti non nulli so-
lamente +1 e −1 dobbiamo esprimere la forma quadratica rispetto ad una base
ortonormale B0 . Questa si ottiene dalla base ortogonale B, dividendo il vettore che
occupa la colonna i nella matrice C, per la radice quadrata del valore assoluto dello
scalare (se é non nullo) presente nella colonna i della matrice A0 :

(1, 0) (− 2 , 1)
B0 = { √ , q3 }=
3 5
3

1 2 3
{( √ , 0), (− √ , √ )}.
3 15 5
Quindi la matrice ortonormale di cambiamento di base
" 1
√ − √2
#
C0 = 3 √ 15
0 √35

ci permette di ottenere la forma diagonale


 
10
A00 = C0T AC0 =
01

che individua una forma quadratica definita positiva.

Risolviamo adesso lo stesso esercizio tramite l’utilizzo degli autovalori della matri-
ce A.
Dopo aver risolto l’equazione caratteristica associata alla matrice A, si ottengono
come autovalori λ1 = 5 e λ2 = 1, entrambi con molteplicitá algebrica 1.
L’autospazio relativo a λ1 é generato dall’autovettore (1, 1), il quale diventa ( √12 , √12 ),
una volta normalizzato.
L’autospazio relativo a λ2 é generato dall’autovettore (1, −1). Da quest’ultimo, una
volta normalizzato, otteniamo il vettore di componenti ( √12 , − √12 ).
La base rispetto a cui la forma quadratica é diagonalizzabile, con gli autovalori sul-
la diagonale principale della matrice ad essa associata, é formata proprio da tali
autovettori normalizzati:
1 1 1 1
B = {( √ , √ ), ( √ , − √ )}.
2 2 2 2
La matrice di cambiamento di base é allora
" 1 #
√ √1
C= 2 2
√1 − √12
2

tale che
9.10 Esercizi svolti. 263
 
50
CT AC = .
01
Al solito, per ottenere una forma diagonale in cui compaiano come coefficienti non
nulli solamente +1 e −1 dobbiamo esprimere la forma quadratica rispetto ad una
base ortonormale B0 che si ottiene dalla base B, dividendo ogni vettore per la radice
quadrata del valore assoluto dell’autovalore (se é non nullo) ad esso relativo:
1 1 1 1
B0 = {( √ , √ ), ( √ , − √ )}.
10 10 2 2
La matrice di cambiamento di base é allora
" 1 #
√ √1
0 10 2
C =
√1 − √12
10

tale che  
0T 100
C AC = .
01
t
u
Esercizio 78. Sia q : R3 −→ R la forma quadratica definita da q(X) = x12 + 6x22 +
4x1 x2 + x32 cioé   
  120 x1
q(X) = x1 x2 x3  2 6 0   x2  .
001 x3
Determinarne una forma diagonale.

Svolg. La matrice associata alla forma quadratica é


 
120
A = 2 6 0
001

Poiché q(e1 ) 6= 0 allora possiamo effettuare il seguente cambiamento di base:

e01 = e1 , e02 = e2 − 2e1 , e03 = e3

la cui matrice associata é  


1 −2 0
C = 0 1 0
0 0 1
da cui
x1 = x10 − 2x20 , x2 = x20 , x3 = x30 .
L’espressione della forma quadratica nelle nuove coordinate é

q(x10 , x20 , x30 ) = x10 2 + 2x20 2 + x30 2


264 9 Le forme bilineari e le forme quadratiche reali.

con matrice associata  


100
A0 =  0 2 0  .
001
La base ortogonale B rispetto a cui la forma quadratica si esprime in forma diagonale
é data dalle colonne della matrice C, cioé:

B = {(1, 0, 0), (−2, 1, 0), (0, 0, 1)}

e la matrice associata alla forma quadratica in tale base é A0 = CT AC.


Per ottenere una forma diagonale in cui compaiano come coefficienti non nulli so-
lamente +1 e −1 dobbiamo esprimere la forma quadratica rispetto ad una base
ortonormale B0 . Questa si ottiene dalla base ortogonale B, dividendo il vettore che
occupa la colonna i nella matrice C per la radice quadrata del valore assoluto dello
scalare (se é non nullo) presente nella colonna i della matrice A0 :

(−2, 1, 0)
B0 = {(1, 0, 0), √ , (0, 0, 1)}.
2
Quindi la matrice ortonormale di cambiamento di base
 
1 − √22 0
C0 =  0 √1 0 
 
2
0 0 1

ci permette di ottenere la forma diagonale


 
100
A00 = C0T AC0 =  0 1 0 
001

che individua una forma quadratica definita positiva di segnatura (3, 0), che in base
B0 si presenta in forma di prodotto scalare standard. t
u
Esercizio 79. Sia q : R3 −→ R la forma quadratica definita da q(X) = 4x1 x2 − x12 −
5x22 − 4x32 cioé   
  −1 2 0 x1
q(X) = x1 x2 x3  2 −5 0   x2  .
0 0 −4 x3
Determinarne una forma diagonale.

Svolg. La matrice associata alla forma quadratica é


 
−1 2 0
A =  2 −5 0 
0 0 −4
9.10 Esercizi svolti. 265

Poiché q(e1 ) 6= 0 allora possiamo effettuare il seguente cambiamento di base:

e01 = e1 , e02 = e2 + 2e1 , e03 = e3

la cui matrice associata é  


120
C = 0 1 0
001
da cui
x1 = x10 + 2x20 , x2 = x20 , x3 = x30 .
L’espressione della forma quadratica nelle nuove coordinate é

q(x10 , x20 , x30 ) = −x10 2 − x20 2 − 4x30 2

con matrice associata  


−1 0 0
A0 =  0 −1 0  .
0 0 −4
La base ortogonale B rispetto a cui la forma quadratica si esprime in forma diagonale
é data dalle colonne della matrice C, cioé:

B = {(1, 0, 0), (2, 1, 0), (0, 0, 1)}

e la matrice associata alla forma quadratica in tale base é A0 = CT AC.


Per ottenere una forma diagonale in cui compaiano come coefficienti non nulli so-
lamente +1 e −1 dobbiamo esprimere la forma quadratica rispetto ad una base
ortonormale B0 . Questa si ottiene dalla base ortogonale B, dividendo il vettore che
occupa la colonna i nella matrice C per la radice quadrata del valore assoluto dello
scalare (se é non nullo) presente nella colonna i della matrice A0 :

(0, 0, 1)
B0 = {(1, 0, 0), (2, 1, 0), √ }=
4
(0, 0, 1)
{(1, 0, 0), (2, 1, 0), }.
2
Quindi la matrice ortonormale di cambiamento di base
 
120
C0 =  0 1 0 
0 0 12

ci permette di ottenere la forma diagonale


 
−1 0 0
A00 = C0T AC0 =  0 −1 0 
0 0 −1
266 9 Le forme bilineari e le forme quadratiche reali.

che individua una forma quadratica definita negativa di segnatura (0, 3).
t
u
Capitolo 10
I sistemi di riferimento in uno spazio affine ed
euclideo.

Lo scopo della seconda parte della presente trattazione é quello di descrivere al-
cune applicazioni geometriche delle strutture algebriche analizzate nei precedenti
Capitoli. Per farlo é necessario introdurre un modello geometrico che aderisca alle
caratteristiche fondamentali dell’ambiente reale in cui le nostre esperienze sensoriali
si sviluppano, ovvero un ambiente a 3 dimensioni.

10.1 Sistemi di riferimento nel piano e nello spazio.

Inizialmente preferiamo introdurre un metodo intuitivo per individuare le coordinate


di un punto nello spazio 1-dimensionale (una retta), in quello 2-dimensionale (un
piano) ed in quello 3-dimensionale.
Coordinate su di una retta.
Sia r una retta nel piano o nello spazio. Essa puó essere percorsa in due versi opposti.
Quando uno dei versi di percorrenza é scelto come positivo e l’altro come negativo,
si dice che la retta é orientata.
Sia adesso r una retta orientata e fissiamo un punto O ∈ r. Scegliamo un qualsiasi
altro punto P ∈ r.
Il numero reale che esprime la distanza di P da O é detta coordinata di P. Il punto
O é detto origine delle coordinate e divide la retta in due semirette:
–una semiretta é quella percorsa, a partire da O, nel verso positivo della retta ed é
detta semiretta positiva;
–l’altra é quella percorsa, a partire da O, nel verso negativo della retta ed é detta
semiretta negativa.
La coordinata di un qualsiasi punto P ∈ r é accompagnata dal segno positivo o ne-
gativo, in accordo col fatto che il punto P si trovi nella semiretta positiva o negativa
della retta r.
Coordinate nel piano.

267
268 10 I sistemi di riferimento in uno spazio affine ed euclideo.

Fissiamo nel piano un punto O e due rette X ed Y orientate ed incidenti in O. Sia


O l’origine per entrambe le rette X ed Y e scegliamo un qualsiasi altro punto P del
piano. Costruiamo le rette passanti per P e parallele ad X ed Y . Tali rette sono uniche
ed incontrano X ed Y in due punti, che indichiamo rispettivamente PX e PY .
Siano α ∈ R la coordinata di PX sulla retta X e β ∈ R la coordinata di PY sulla retta
Y.
I numeri reali (α, β ) individuano quindi la posizione del punto P nel piano, in rife-
rimento alla scelta del punto O e delle rette X ed Y .
Diremo che OXY rappresenta un sistema di riferimento nel piano e che (α, β ) sono
le coordinate del punto P, rispetto al sistema di riferimento OXY . Il punto O é detto
origine del sistema di riferimento.
Viceversa, scelti arbitrariamente due numeri reali (α, β ), esiste ed é unico il punto
del piano che abbia tali coordinate rispetto ad un riferimento precedentemente asse-
gnato.
Possiamo quindi concludere che, una volta assegnato un sistema di riferimento OXY
nel piano, esiste una corrispondenza biunivoca fra i punti del piano e le coppie ordi-
nate di numeri reali dell’insieme R2 .
In particolare, un sistema di riferimento OXY nel piano é detto cartesiano (ortogo-
nale) se le rette X ed Y sono fra loro perpendicolari.
Coordinate nello spazio.

Fissiamo nello spazio un punto O e tre rette X, Y e Z orientate ed incidenti in O. Sia


O l’origine delle rette X, Y , Z e scegliamo un qualsiasi altro punto P dello spazio.
Costruiamo i piani passanti per P e paralleli ai piani XY , XZ ed Y Z. Tali piani sono
unici ed incontrano X, Y e Z in tre punti, che indichiamo rispettivamente PX , PY e
PZ .
Siano α ∈ R la coordinata di PX sulla retta X, β ∈ R la coordinata di PY sulla retta
Y e γ la coordinata di PZ sulla retta Z.
I numeri reali (α, β , γ) individuano quindi la posizione del punto P nello spazio, in
riferimento alla scelta del punto O e delle rette X, Y e Z.
Come sopra, diremo che OXY Z rappresenta un sistema di riferimento nello spazio
e che (α, β , γ) sono le coordinate del punto P, rispetto al sistema di riferimento
OXY Z. Il punto O é detto origine del sistema di riferimento.
Viceversa, scelti arbitrariamente tre numeri reali (α, β , γ), esiste ed é unico il punto
dello spazio che abbia tali coordinate rispetto ad un riferimento precedentemente
assegnato.
Concludiamo quindi che, una volta assegnato un sistema di riferimento OXY Z nello
spazio, esiste una corrispondenza biunivoca fra i punti dello spazio e le terne ordi-
nate di numeri reali dell’insieme R3 .
In particolare, un sistema di riferimento OXY Z nello spazio é detto cartesiano (or-
togonale) se le rette X, Y e Z sono a due a due fra loro perpendicolari.
Inoltre, un sistema di riferimento OXY Z é detto destrorso o standard se un os-
servatore solidale con l’asse Z vede la rotazione dell’asse X sull’asse Y in senso
antiorario.
10.2 Lo spazio affine e lo spazio euclideo. 269

10.2 Lo spazio affine e lo spazio euclideo.

A questo punto vogliamo costruire un modello generale seguendo la via assiomatica,


del quale i precedenti saranno un caso particolare.
Definizione 10.1. Siano K un campo, V uno spazio vettoriale di dimensione n su
K e S un insieme non vuoto. L’insieme S é detto spazio affine associato allo spazio
vettoriale V se esiste una applicazione

ϕ : S×S →V

che verifica i seguenti assiomi:


1. Fissato un qualsiasi O ∈ S e per ogni v ∈ V , esiste un unico P ∈ S tale che
ϕ(O, P) = v.
2. Per ogni P1 , P2 , P3 ∈ S, ϕ(P1 , P3 ) = ϕ(P1 , P2 ) + ϕ(P2 , P3 ).
Gli elementi di S sono detti punti.
Se dim(V ) = n ≥ 1 allora diremo che lo spazio affine S ha dimensione n. Indichere-
mo uno spazio affine di dimensione n col termine S = An .
Un sistema di riferimento affine di An é costituito da un punto fissato O ∈ An , detto
origine del sistema di riferimento, e da una base {e1 , . . . , en } di V . In un tale riferi-
mento, si chiamano coordinate di un punto P ∈ An le componenti del vettore ϕ(O, P)
rispetto alla base {e1 , . . . , en }. Quindi se

ϕ(O, P) = x1 e1 + x2 e2 + · · · + xn en

allora x1 , . . . , xn sono le coordinate di P e scriveremo P = (x1 , . . . , xn ).


Definizione 10.2. Sia S uno spazio affine di dimensione n, associato allo spazio
vettoriale V su di un campo K. Se V é uno spazio vettoriale euclideo, ovvero é
dotato di un prodotto interno, allora S é detto spazio euclideo associato allo spazio
vettoriale V .
Indicheremo uno spazio euclideo di dimensione n col termine S = E n .
Un sistema di riferimento di E n é costituito da un punto fissato O ∈ E n , detto origine
del sistema di riferimento, e da una base {e1 , . . . , en } ortonormale di V . In un tale
riferimento, si chiamano coordinate ortonormali o cartesiane di un punto P ∈ E n
le componenti del vettore ϕ(O, P) rispetto alla base {e1 , . . . , en }. Quindi se

ϕ(O, P) = x1 e1 + x2 e2 + · · · + xn en

allora x1 , . . . , xn sono le coordinate cartesiane di P e scriveremo P = (x1 , . . . , xn ).


In particolare svilupperemo le applicazioni geometriche nei successivi Capitoli nei
casi in cui:
–n = 2 e V = R2 : piano affine A2 e piano euclideo E 2 ;
–n = 3 e V = R3 : spazio affine 3-dimensionale A3 e spazio euclideo 3-dimensionale
E 3.
Capitolo 11
I vettori geometrici.

11.1 Definizione di vettore geometrico.

Siano A, B due punti del piano (o dello spazio). Il segmento avente per estremi i
punti A, B é detto orientato, se su di esso si individua un verso di percorrenza posi-
tivo. Per convenzione, indicheremo AB il segmento orientato nel caso in cui il verso
dipercorrenza positivo sia da A verso B.
Un segmento orientato AB si dice anche vettore applicato in A ed é individuato da:
1. una direzione, che é la retta su cui giace il segmento AB;
2. un verso, che é quello che si osserva percorrendo il segmento AB andando da A
verso B;
3. un modulo, che é il numero reale non negativo che misura la lunghezza del
segmento AB.
Un vettore é detto versore di una retta, se giace su tale retta ed é ha modulo pari a 1.
Due segmenti orientati AB e A0 B0 sono equipollenti se hanno la stessa direzione,
lo stesso verso e lo stesso modulo. Tale relazione tra segmenti é una equivalenza
nell’insieme di tutti i segmenti orientati del piano (o dello spazio). Per cui tutti i
segmenti orientati sono ripartiti in classi di equivalenza. Un classe di equivalenza é
quindi rappresentata da un qualsiasi segmento orientato AB che vi appartiene, ed é
composta da tutti i segmenti orientati equipollenti al segmento AB. In generale, una
tale classe di equivalenza é indicata con il simbolo [AB]. Identifichiamo tra loro tutti
i segmenti appartenenti ad una stessa classe di equivalenza. Ciascuna classe é detta
vettore geometrico o vettore libero. Ció vuol dire che, fissato un punto del piano
(o dello spazio) O, ciascuna classe di equivalenza possiede un rappresentante che
sia applicato in O. Pertanto, da ora in avanti, tutti i vettori geometrici da noi intro-
dotti, saranno applicati in un medesimo punto, mantenendo la propria direzione ed
il proprio verso. In particolare, é utile rappresentare ciascuna classe di equivalenza
attraverso l’elemento della classe applicato nell’origine del sistema di riferimento
OXY nel piano, ovvero OXY Z nello spazio.

271
272 11 I vettori geometrici.

11.2 Somma e prodotto per uno scalare.

Siano OP e OP1 due vettori aventi la stessa direzione e stesso verso. Definiamo
somma dei vettori OP + OP1 , il vettore avente per direzione e verso quelli dei vetto-
ri addendi, e per modulo |OP + OP1 | la somma dei moduli |OP| + |OP1 |.
Siano ora OP e OP1 due vettori aventi stessa direzione ma verso opposto. Definiamo
somma dei vettori OP + OP1 il vettore avente direzione dei vettori addendi, verso
coincidente con quello del vettore addendo di modulo maggiore, e modulo pari al
valore assoluto della differenza dei moduli dei vettori addendi.
Siano infine OP e OP1 due vettori con direzioni differenti. Si costruisca il parallelo-
gramma di lati OP e OP1 e si indichi con Q il quarto vertice del parallelogramma.
Definiamo somma OP + OP1 il vettore avente per direzione quella della retta su cui
giace la diagonale OQ, per verso quello di percorrenza da O a Q e per modulo la
lunghezza della diagonale OQ del parallelogramma.
Notiamo anche che dalla definizione di somma OP+OP1 = OQ si ricava al contrario
anche quella di differenza tra due vettori OQ − OP = OP1 , come lato del parallelo-
gramma avente per diagonale OQ e per secondo lato OP.
Indichiamo piú semplicemente v, v1 , v2 , v3 vettori del piano (o dello spazio). La
somma tra vettori gode delle seguenti proprietá:
1. Commutativa: v1 + v2 = v2 + v1 ;
2. Associativa : (v1 + v2 ) + v3 = v1 + (v2 + v3 );
3. Esistenza dell’elemento neutro : v + 0 = 0 + v = v, dove per vettore 0 si intende
il segmento OO;
4. Esistenza dell’elemento opposto : v + (−v) = 0.
Tali proprietá rendono l’insieme dei vettori geometrici del piano (o dello spazio),
rispetto all’operazione di somma, un gruppo commutativo.
Siano v un vettore e α ∈ R. Definiamo il vettore αv come quello avente la direzione
del vettore v, il modulo pari a |α||v| e verso coincidente con quello di v, se α > 0,
opposto a quello di v, se α < 0.
Il prodotto cosı́ definito gode delle seguenti proprietá:
1. (αβ )v = α(β v), per α, β ∈ R;
2. 1 · v = v, dove 1 é l’unitá in R;
3. (α + β )v = αv + β v;
4. α(v1 + v2 ) = αv1 + αv2 .
L’insieme di tutti i vettori geometrici, rispetto alle operazioni di somma e prodotto
per uno scalare da noi definite, viene detto spazio vettoriale geometrico.
11.3 Combinazioni lineari e componenti in un riferimento ortogonale. 273

11.3 Combinazioni lineari e componenti in un riferimento


ortogonale.

Siano v1 , v2 , . . . , vr vettori e α1 , . . . , αr ∈ R. Diciamo combinazione lineare dei vet-


tori v1 , . . . , vr a coefficienti reali α1 , . . . , αr il vettore v risultante dalla seguente
somma:
v = α1 v1 + α2 v2 + α3 v3 + . . . . + αr vr .
Diremo anche che il vettore v é scomposto nella somma dei vettori α1 v1 , α2 v2 ,. . . ,αr vr
e che α1 , . . . , αr sono le componenti del vettore v rispetto ai vettori v1 , . . . , vr .
Consideriamo un sistema di riferimento cartesiano ortogonale nello spazio, di ori-
gine O e assi coordinati X,Y, Z. Un qualsiasi punto dello spazio sará individuato
da un terna di numeri reali (a, b, c), che sono le sue coordinate rispetto al sistema
di riferimento. Il vettore v = OP ha come componenti rispetto ai tre assi coordi-
nati, rispettivamente vx = a, vy = b, vz = c, cioé il vettore v si puó esprimere come
combinazione lineare dei tre versori i, j, k dei tre assi coordinati nel modo seguente:
v = vx i + vy j + vz k.
Esempio 145. Consideriamo i vettori v1 = 2i − 2 j + k, v2 = i + 3 j + 2k, v3 = 2i +
j + 4k e gli scalari α1 = 3, α2 = −2, α3 = 2. La combinazione lineare α1 v1 + α2 v2 +
α3 v3 genera il vettore

v = 3(2i − 2 j + k) − 2(i + 3 j + 2k) + 2(2i + j + 4k) = 8i − 10 j + 7k.

Diremo quindi che il vettore v ha componenti (3, −2, 2) rispetto alla terna di vettori
v1 , v2 , v3 , mentre ha componenti (8, −10, 7) rispetto all aterna i, j, k dei versori degli
assi coordinati.
Diremo che due vettori v = (vx , vy , vz ) e w = (wx , wy , wz ) sono uguali se e solo se
hanno le stesse componenti rispetto ai versori degli assi coordinati.
Dalla definizione di componenti appena data, si puó facilmente q dedurre che il
modulo di un vettore v di componenti (vx , vy , vz ) é dato da |v| = v2x + v2y + v2z .

Osservazione 11.1. In accordo con quanto detto in relazione alla normalizzazione


dei vettori in Rn , é sempre possibile ottenere un versore di una retta, a partire da un
qualsiasi vettore v che abbia:
1. stessa direzione della retta (cioé parallelo alla retta stessa);
2. stesso verso.
É sufficiente infatti dividere ciascuna delle componenti di v per il modulo di v.
Esempio 146. Consideriamo il vettore v = 2i − 3 j + k. Per determinare il versore u
avente uguale direzione e verso di v é sufficiente calcolare il modulo di v
√ √
|v| = 4 + 9 + 1 = 14

e quindi dividere ciascuna componente di v per il suo modulo:


274 11 I vettori geometrici.

v 2 3 1
u= = i− j + k.
|v| 14 14 14

Siano ora v = (vx , vy , vz ) e w = (wx , wy , wz ) due distinti vettori. Costruiamo la loro


somma v + w = (vx i + vy j + vz k) + (wx i + wy j + wz k) = (vx + wx )i + (vy + wy ) j +
(vz + wz )k cioé v + w ha componenti (vx + wx , vy + wy , vz + wz ).
In modo del tutto analogo si osservi che se α ∈ R, allora il vettore αv ha per
componenti la terna (αvx , αvy , αvz ).

11.4 Prodotto scalare e proiezione su una retta.

Siano v = OP e w = OQ. Per convenzione si definisce angolo ϕ compreso tra i due


vettori quello interno al triangolo POQ.
Si definisce prodotto scalare il seguente

v × w = |v| · |w| · cos(ϕ).

Si noti che tale prodotto ha come risultato uno scalare (un numero reale), inoltre
esso é commutativo, cioé v × w = w × v ed infine esso é nullo solo quando uno dei
due vettori é nullo oppure se i due vettori sono tra loro ortogonali.
Nel caso i due vettori siano considerati in uno spazio dotato di riferimento cartesia-
no, allora v = (vx , vy , vz ) e w = (wx , wy , wz ), ed il prodotto scalare si presenta nella
seguente forma:

v × w = (vx i + vy j + vz k) × (wx i + wy j + wz k) = vx wx + vy wy + vz wz .

Da quest’ultima ricaviamo anche che:


vx wx + vy wy + vz wz
cos(ϕ) = q q .
v2x + v2y + v2z w2x + w2y + w2z

Applicando tale formula si ricavano anche i coseni direttori di un vettore v, cioé i


coseni degli angoli che il vettore forma con i tre assi coordinati, o meglio con i tre
versori dei suddetti assi:
vx vy vz
cos(vX) = q , cos(vY ) = q , cos(vZ) = q .
v2x + v2y + v2z v2x + v2y + v2z v2x + v2y + v2z

Esempio 147. Siano v = 2i − j + k e w = i + j − 2k. Il loro prodotto scalare é

v × w = 2 − 1 − 2 = −1
√ √
ed i rispettivi moduli sono |v| = 6, |w| = 6. Per cui, detto l’angolo tra di essi
compreso,
11.4 Prodotto scalare e proiezione su una retta. 275

−1 1
cos(ϕ) = √ √ = − .
6 6 6
Inoltre i coseni direttori di v e w sono rispettivamente:
2 1 1
cos(vX) = √ , cos(vY ) = − √ , cos(vZ) = √
6 6 6
1 1 2
cos(wX) = √ , cos(wY ) = √ , cos(wZ) = − √ .
6 6 6

Siano ora v = OQ e r una retta passante per O di versore u, tale che la direzione del
vettore e quella della retta formino un angolo ϕ. Diciamo componente di v rispetto
alla retta r, la lunghezza del segmento OQ0 , dove Q0 é la proiezione ortogonale di
Q su r. Applicando ora le regole trigonometriche sulla risoluzione dei triangoli, si
osserva facilmente che

|OQ0 | = |OQ| · cos(ϕ) = v × u.

Diremo invece vettore proiezione v0 di v su r, quello avente per modulo |OQ0 | e per
direzione e verso quelli di u, cioé:

v0 = |OQ0 | · u = (v × u) · u.

Si noti che pur non conoscendo il versore della retta r, esso si puó ricavare se é noto
w
un vettore w parallelo a r. In tal caso u = |w| ed otteniamo

w w w
v0 = (v × )· = (v × w) · 2 .
|w| |w| |w|

Esempio 148. Proiettiamo il vettore v = i − 2 j + 3k sulla retta r orientata dal vettore


w = i + j + k.
Il versore concorde alla retta é
w 1 1 1
u= = √ i + √ j + √ k.
|w| 3 3 3
La componente di v lungo la retta r é
1 2 3 2
v×u = √ − √ + √ = √ .
3 3 3 3
Infine il vettore v0 proiezione di v su r é
276 11 I vettori geometrici.

v0 = (v × u)u
= √23 u
 
2 1 1 1
= 3
√ √
3
i+ 3 j + 3k
√ √
 
2
= 3 i+ j+k .

11.5 Prodotto vettoriale e componente rispetto ad un piano.

Siano v, w vettori nello spazio affine A3 . Definiamo prodotto vettoriale v ∧ w quel


vettore che abbia:
–direzione ortogonale al piano individuato dalle direzioni dei vettori v, w;
–modulo pari a |v||w|sen(ϕ), dove ϕ é l’angolo compreso tra i due vettori;
–verso tale che un osservatore posto dalla parte in cui é rivolto il vettore v ∧ w,
veda il vettore v sovrapporsi al vettore w in senso antiorario.
Il prodotto vettoriale é nullo solo quando uno dei due vettori é nullo oppure i due
vettori sono tra loro paralleli.
Notiamo che in base alla definizione, al contrario di quanto avviene per il prodotto
scalare, il prodotto vettoriale é anticommutativo cioé: v ∧ w = −w ∧ v. Inoltre il pro-
dotto vettoriale é distributivo rispetto alla somma tra vettori, cioé: v ∧ (w1 + w2 ) =
(v ∧ w1 ) + (v ∧ w2 ).
Consideriamo ora un parallelogramma che abbia come lati i vettori v, w. L’area di
tale parallelogramma é A = b · h, dove b é la base da noi scelta e h é l’altezza rela-
tiva a tale base. Scegliamo ad esempio il vettore v come base. L’altezza relativa a v
forma con v e w un triangolo rettangolo, per cui h = |w|sen(ϕ), quindi

A = |v||w|sen(ϕ) = |v ∧ w|

concludendo che l’area del parallelogramma avente per lati v, w é pari al modulo del
prodotto vettoriale v ∧ w.
Poniamoci ora nel caso in cui i due vettori appartengano allo spazio cartesiano, per
cui v = (vx , vy , vz ) e w = (wx , wy , wz ). Calcoliamo ora il loro prodotto vettoriale:

v ∧ w = (vx i + vy j + vz k) ∧ (wx i + wy j + wz k) =

i(vy wz − vz wy ) + j(vz wx − vx wz ) + k(vx wy − vy wx ) =



i j k

vx vy vz .

wx wy wz
Osserviamo che lo sviluppo della formula precedente dipende dall’applicazione del-
la proprietá distributiva del prodotto vettoriale rispetto alla somma tra vettori ed an-
che dalle seguenti identitá che derivano direttamente dalla definizione di prodotto
11.5 Prodotto vettoriale e componente rispetto ad un piano. 277

vettoriale:

i ∧ j = − j ∧ i = k, j ∧ k = −k ∧ j = i, i ∧ k = −k ∧ i = − j

i ∧ i = 0, j ∧ j = 0, k ∧ k = 0.
Dal controesempio che segue si deduce inoltre che per il prodotto vettoriale non vale
la proprietá associativa:
i ∧ (i ∧ k) = i ∧ (− j) = k
ma
(i ∧ i) ∧ k = 0.
Esempio 149. Il prodotto vettoriale tra v = 2i − 3 j + k e w = i + 2 j + k nello spazio
cartesiano ortogonale é dato dal seguente determinante:

i j k

v ∧ w = 2 −3 1 = −5i − j + 7k.
1 2 1

Invertendo l’ordine dei fattori nel prodotto vettoriale otteniamo



i j k

w ∧ v = 1 2 1 = 5i + j − 7k.
2 −3 1

Sia ora v = OP e π un piano passante per O, al quale appartengano i due vetto-


ri u1 , u2 . La proiezione di OP su π é il vettore v0 = OP0 , dove P0 é la proiezione
ortogonale del punto P sul piano. Se indichiamo w il vettore PP0 allora otteniamo
v0 + w = v, da cui ovviamente v0 = v − w. Per ottenere la proiezione v0 é allora suffi-
ciente determinare il vettore w. Tale vettore non é altro che la proiezione di v lungo
la retta normale al piano e passante per O. Per ottenere tale proiezione é necessario
conoscere il versore u normale al piano e, poiché sappiamo che il vettore u1 ∧ u2 é
∧u2
normale al piano, deduciamo che u = |uu1 ∧u | . Infine otteniamo:
1 2

u1 ∧ u2
w = (v × u) · u = [v × (u1 ∧ u2 )] · .
|u1 ∧ u2 |2

Esempio 150. Proiettiamo il vettore v = i + j − 2k sul piano contenente i vettori


u1 = 2i + k e u2 = i − j + k.
Dapprima calcoliamo il vettore u1 ∧ u2 :

i j k

u1 ∧ u2 = 2 0 1 = i − j − 2k.
1 −1 1

Quindi il versore ortogonale al piano é


278 11 I vettori geometrici.

u1 ∧ u2 1 1 2
u= = √ i − √ j − √ k.
|u1 ∧ u2 | 6 6 6
Determiniamo adesso la proiezione w0 di v lungo la retta contenente u:

v0 = (v
 × u)u   
1 1 4 1 1 2
= √
6
− 6+ 6 ·
√ √ √
6
i− 6 j − 6k
√ √
 
= √46 · √16 i − √16 j − √26 k

2 2 4
= 3i− 3 j − 3k .

11.6 Prodotto misto.

Siano u, v, w tre vettori nello spazio affine A3 . Definiamo prodotto misto quello sca-
lare che si ottiene dall’esecuzione, rispettando l’associazione tramite parentesi, dei
prodotti u × (v ∧ w).
Nel caso i tre vettori siano dati in uno spazio cartesiano ortogonale, allora u =
(ux , uy , uz ), v = (vx , vy , vz ), w = (wx , wy , xz ) e svolgendo il prodotto si ottiene:

u × (v ∧ w) = ux (v ∧ w)x + uy (v ∧ w)y + uz (v ∧ w)z =



v v v v v v
ux y z + uy z x + uz x y =
wy wz wz wx wx wy

ux uy uz

vx vy vz .

wx wy wz
Si noti che il prodotto misto si annulla solo quando uno dei tre vettori é nullo oppure
quando i tre vettori sono complanari, cioé quando il vettore u é ortogonale al vettore
v ∧ w.
Consideriamo ora il parallelepipedo avente per spigoli i tre vettori u, v, w. Il volume
del parallelepipedo é V0 = b · h, dove indichiamo con b l’area di una base e con h
l’altezza relativa a tale base scelta.
Scegliamo come base quella avente per lati i vettori v, w e sappiamo per quanto detto
in precedenza che l’area cercata é pari al modulo |v ∧ w|.
L’altezza h é la componente ortogonale del terzo spigolo del parallelepipedo lungo
la normale al piano individuato da v e w, per cui
v∧w
h = u×
|v ∧ w|

da cui
11.7 Esercizi svolti. 279

v∧w
V0 = |v ∧ w| · u × = u × (v ∧ w).
|v ∧ w|
Esempio 151. Determiniamo i valori reali dei parametri α, β per i quali is eguenti
vettori siano complanari:

v = αi + j − 2β k, w = αi − j − k, u = 2i − 2 j − β k.

É sufficiente determinare i valori di α, β ∈ R tali che il prodotto misto u × (v ∧ w)


si annulli, ovvero
2 −2 −β

α 1 −2β = 0.

α −1 −1
Dal calcolo del precedente determinante segue che

u × (v ∧ w) = 3αβ − 2β − α − 1. (11.1)

Quindi
u × (v ∧ w) = 0 ⇐⇒ 3αβ − 2β − α − 1 = 0.
Riscrivendo la precedente relazione, abbiamo che

u × (v ∧ w) = 0 ⇐⇒ α(3β − 1) = 1 + 2β .
1+2β
Per ogni valore β 6= 31 , e per α = 3β −1 , i tre vettori sono complanari.
1
Nel caso di β = 3, la (11.1) si scrive

2
u × (v ∧ w) = − − 1 6= 0
3
per cui i tre vettori non sono complanari.

11.7 Esercizi svolti.

Esercizio 80. Esprimere il vettore v = 2i − j + k come somma di due vettori v1 e v2


tali che
–v1 é un vettore parallelo a j + k;
–v2 é un vettore complanare con i + 2 j e 2i + k.

Svolgimento: Il vettore v1 é pari a α1 ( j + k) per qualche α1 ∈ R.


Il vettore v2 é invece una combinazione lineare α2 (i + 2 j) + α3 (2i + k). Quindi
dobbiamo imporre che

2i − j + k = α1 ( j + k) + α2 (i + 2 j) + α3 (2i + k)
280 11 I vettori geometrici.

ovvero
2i − j + k = (α2 + 2α3 )i + (α1 + 2α2 ) j + (α1 + α3 )k.
Uguagliando le componenti omologhe dei due vettori, abbiamo

 α2 + 2α3 = 2
α1 + 2α2 = −1
α1 + α3 = 1

da cui
1 2 6
α1 = − , α2 = − , α3 = .
5 5 5
Infine
1
v1 = − ( j + k)
5
e
2 6 4 6
v2 = − (i + 2 j) + (2i + k) = 2i − j + k.
5 5 5 5

Esercizio 81. Siano u = i − 2 j + 3k e v = −3 j vettori nello spazio cartesiano.


Determinare:
1. i moduli dei vettori;
2. il loro prodotto scalare;
3. il coseno dell’angolo da essi formato;
4. i coseni direttori dei due vettori.

Svolgimento:
√ √ √
1. |u| = 1 + 4 + 9 = 14, |v| = 9 = 3.
2. u × v= (-2)(-3)=6; q
6 2
3. cos(ϕ) = √
3· 14
= 7;
√1 , −2 √3 ,
4. u × i = 14
u× j = √
14
, u×k = 14
v × i = 0, v × j = −1, v × k = 0.

Esercizio 82. Determinare a ∈ R tale che i vettori u = (1, −2, 1) e v = (a +


1, −a, −1) formino un angolo di π3 .

Svolgimento:
1 u×v
cos( π3 ) = 2 = |u|·|v|
a+1+2a−1
= √ √
6 a2 +1+2a+a2 +1
3a
= √ √
6 2a2 +2a+2

da cui √ p
6 2a2 + 2a + 2 = 6a.
Poiché 2a2 + 2a + 2 > 0 per ogni a ∈ R, avremo allora a > 0. Risolvendo allora
11.7 Esercizi svolti. 281

12a2 + 12a + 12 = 36a2 (a > 0)

otteniamo infine 2a2 − a − 1 = 0, cioé a = 1 (scartando quindi a = − 21 ).


Esercizio 83. Determinare a ∈ R tale che i vettori u = (2, a, 1) e v = (1, 0, 2) formino
un angolo di π4 .

2 u×v
cos( π4 ) = 2 = |u|·|v|
Svolgimento:
= √ 42 .
5a +25

Poiché 5a2 + 25 > 0 per ogni a ∈ R, non abbiamo alcun vincolo su a ∈ R. Quindi

2 4
=√
2 2
5a + 25
√ q
da cui 10a2 + 50 = 8 e quindi 10a2 + 50 = 64. Infine otteniamo a = ± 75 .

Esercizio 84. Siano u = (1, 2, −2) e v = (3, 0, 1). Determinare la proiezione orto-
gonale di v rispetto alla retta r contenente u.

Svolgimento: La proiezione di v su r é data da


u 1
v× = .
|u| 3

Allora il vettore proiezione é dato da


 
u u 1
v× × = (i + 2 j − 2k).
|u| |u| 9

Esercizio 85. Determinare il vettore v0 proiezione di v = 2i − j + 3k sul piano XY .

Svolgimento: Proiettando v su XY si avrebbe v = v0 + w, dove w é il vettore


(perpendicolare al piano XY ) che é proiezione ortogonale di v rispetto alla retta
contenente i ∧ j, cioé
i∧ j
w = v × (i ∧ j) · = 3k
|i ∧ j|2
e v0 = 2i − j.
Esercizio 86. Determinare il vettore v0 proiezione di v = i + 2 j + k sul piano π
contenente i vettori u1 = 2i + j e u2 = i − k.

Svolgimento: Proiettando v su π si avrebbe v = v0 + w, dove w é il vettore (perpen-


dicolare al piano π) che é proiezione ortogonale di v rispetto alla retta contenente
282 11 I vettori geometrici.

u1 ∧ u2 . Dapprima determiniamo il versore u perpendicolare a π. Esso é dato da

u1 ∧ u2 −i + 2 j − k
u= = √ .
|u1 ∧ u2 | 6
Quindi
2 −i + 2 j − k
w = (v × u)u = √ u =
6 3
ed infine
−i + 2 j − k 4 4 4
v0 = v − w = (i + 2 j + k) − ( ) = i+ j+ k
3 3 3 3

Esercizio 87. Determinare il volume del parallelepipedo avente per spigoli i vettori
i + j, k, −i + k.

Svolgimento: Il volume é dato dal valore assoluto del determinante della matrice
 
1 10
 0 0 1 .
−1 0 1

Tale determinante é pari a −1, quindi il volume é 1.


Capitolo 12
Geometria nel piano affine e nel piano euclideo.

12.1 Equazioni di una retta.

Ogni retta del piano affine A2 puó essere individuata da un suo punto P0 = (x0 , y0 )
e da un vettore v = (l, m) ad essa parallelo. Se P = (x, y) é un qualsiasi punto della
retta, ne segue che il vettore P0 P sará proporzionale al vettore v, in quanto paralleli.
In altri termini avremo che P0 P = tv, per un opportuno scalare t, dipendente dalla
scelta di P sulla retta. Al variare di t ∈ R, si ottiene la proporzionalitá di v con ogni
vettore scelto sulla retta, cioé si ottengono tutti i punti della retta:

x − x0 = tl
y − y0 = tm

note come le equazioni parameriche di una retta e generalmente espresse da:



x = x0 + tl
.
y = y0 + tm

Gli elementi della coppia (l, m) sono detti parametri direttori della retta. In partico-
lare se si conoscono due punti della retta P1 = (x1 , y1 ) e P2 = (x2 , y2 ), il vettore P1 P2
é parallelo alla retta e quindi (l, m) = (x2 − x1 , y2 − y1 ) e l’equazione si puó ottenere
nel modo seguente: 
x = x1 + t(x2 − x1 )
y = y1 + t(y2 − y1 )
da cui
x − x1 y − y1
t= =
x2 − x1 y2 − y1
x − x1 y − y1
t= =
l m
che é detta equazione a catena di una retta.
Da tali espressioni, eliminando il parametro t, otteniamo

283
284 12 Geometria nel piano affine e nel piano euclideo.

mx − mx0 − ly + ly0 = 0

che possiamo riscrivere


ax + by + c = 0
che é l’equazione lineare (implicita) che rappresenta la retta in coordinate affini.
Esempio 152. Consideriamo il punto P = (1, 3) ed il vettore v = 2i − 5 j. La retta
contenente il punto P e parallela al vettore v ha equazioni parametriche

x = 2t + 1
y = −5t + 3

da cui
x−1 y−3
t= = .
2 −5
L’equazione cartesiana in forma implicita della retta é quindi

5x − 2y − 11 = 0.

Diremo che due rette sono parallele se esse hanno i parametri direttori proporzionali
(in particolare identici). Si noti che quando la retta é espressa in forma implicita, i
suoi parametri direttori sono dati dalla coppia (l, m) = (b, −a).
Dalla forma implicita ax + by + c = 0 di una retta, ricaviamo la forma detta esplicita:
y = mx + q, per m = − ab e q = − bc .
Abbiamo visto come l’equazione della retta passante per i due punti P1 = (x1 , y1 ) e

P2 = (x2 , y2 ) si scriva
x − x1 y − y1
=
x2 − x1 y2 − y1
che equivale alla

(x − x1 )(y2 − y1 ) − (y − y1 )(x2 − x1 ) = 0

cioé
x − x1 y − y1
x2 − x1 y2 − y1 = 0.

Quest’ultima si puó riscrivere anche nel modo seguente:



x y 1

x1 y1 1 = 0.

x2 y2 1

Diremo allora che il punto P3 = (x3 , y3 ) é allineato con i punti P1 e P2 se



x3 y3 1

x1 y1 1 = 0.

x2 y2 1
12.2 Reciproca posizione di due rette. 285

Esempio 153. Scegliamo due punti nel piano: P1 = (1, 3) e P2 = (2, −2). Per de-
terminare la retta contenente i due punti, inizialmente esprimiamo le componenti
del vettore v = P1 P2 , attraverso la differenza delle coordinate omologhe dei punti:
v = (1, −5). La retta richiesta é quindi quella contenente il punto P1 e parallela al
vettore v. Essa ha equazioni parametriche

x = t +1
y = −5t + 3

da cui
y−3
t = x−1 = .
−5
L’equazione cartesiana in forma implicita della retta é quindi

5x + y − 8 = 0.

Se ne deduce che, tutti e soli i punti allineati con P1 e P2 , ovvero i punti appartenenti
alla retta, hanno coordinate (α, 8 − 5α), al variare di α ∈ R. Infatti

1 3 1

2 −2 1 = 0.

α 8 − 5α 1

12.2 Reciproca posizione di due rette.

Siano r : ax + by + c = 0 e r0 : a0 x + b0 y + c0 = 0 due rette. I punti in comune alle due


rette sono le soluzioni del sistema lineare

ax + by + c = 0
a0 x + b0 y + c0 = 0

nelle incognite x, y. Le matrici associate al sistema sono


   
a b a b −c
A= 0 0 , C= 0 0 .
a b a b −c0
Se rango(A) = rango(C) = 1, allora le due rette sono coincidenti poiché ax + by +
c = α(a0 x + b0 y + c0 ), per un opportuno α ∈ R.
Se rango(A) = rango(C) = 2, allora il sistema ammette una sola soluzione cioé le
due rette sono incidenti.
Se rango(A) = 1 e rango(C) = 2, allora il sistema é incompatibile e le due rette non
hanno punti in comune, cioé sono parallele. Ció si verifica quando

a b c
= 0 6= 0 .
a0 b c
286 12 Geometria nel piano affine e nel piano euclideo.
0
Allora possiamo dire che le due rette sono parallele quando ab = ab0 . Il rapporto − ab
é detto coefficiente direttore (o angolare) della retta, quindi due rette sono parallele
se hanno lo stesso coefficiente direttore.
Esempio 154. Le rette
r1 : 2x − 3y + 1 = 0
e
r2 : 4x − 6y + 7 = 0
sono parallele. Invece le equazioni lineari

x − 3y + 4 = 0, 2x − 6y + 8 = 0

rappresentano la medesima retta.


Infine, le rette
t1 : x − 3y + 3 = 0
e
t2 : 2x − 5y + 6 = 0
son incidenti.

12.3 Fascio di rette.

Siano r : ax + by + c = 0 e r0 : a0 x + b0 y + c0 = 0 due rette distinte. La totalitá delle


rette di equazione

λ (ax + by + c) + ρ(a0 x + b0 y + c0 ) = 0

al variare dei parametri reali λ e ρ, é detta fascio di rette. Quindi, assegnando ai


parametri λ , ρ ∈ R un qualsiasi valore, si ottiene l’equazione di una qualsiasi retta
del fascio. Si possono verificare due casi:
1. r e r0 sono incidenti in un punto, detto centro del fascio. Tutte le rette del fascio
hanno in comune quel punto e si parla di fascio proprio di rette.
2. Oppure r e r0 sono tra loro parallele, ed allora tutte le rette del fascio sono tra
loro parallele e si parla di fascio improprio di rette.
Supponiamo allora di avere una terza retta r00 : a00 x + b00 y + c00 = 0 ed analizziamo in
quale casi essa appartiene al fascio individuato da r e r0 . In pratica si deve studiare
il sistema

 ax + by + c = 0
a0 x + b0 y + c0 = 0
 00
a x + b00 y + c00 = 0
nelle incognite x, y. Le matrici associate al sistema sono
12.4 Angoli nel piano euclideo. 287

   
a b a b c
A =  a0 b0  , C =  a0 b0 c0  .
a00 b00 a00 b00 c00
Se rango(A) = rango(C) = 2, allora il sistema ammette una sola soluzione, cioé le
tre rette hanno un punto in comune: esse appartengono ad un fascio proprio.
Se rango(A) = 1 e rango(C) = 2, allora il sistema é incompatibile ed le tre rette
sono parallele: esse appartengono ad un fascio improprio.
Possiamo concludere allora che la condizione necessaria e sufficiente affinché le tre
rette appartengano allo stesso fascio (proprio o improprio che sia) é che la matrice
 
a b c
 a0 b0 c0 
a00 b00 c00

abbia rango ≤ 2.
Esempio 155. Riprendiamo l’Esempio 154 e consideriamo le rette parallele r1 , r2 .
Esse possono essere utilizzate per generare un fascio improprio di rette, la cui
equazione é

λ (2x − 3y + 1) + µ(4x − 6y + 7) = 0, λ , µ ∈ R.

Se invece consideriamo le rette tra loro incidenti t1 ,t2 , queste possono essere
utilizzate per generare un fascio proprio di rette, la cui equazione é

λ (x − 3y + 3) + µ(2x − 5y + 6) = 0, λ , µ ∈ R.

Il centro di tale fascio si ottiene intersecando due qualsiasi rette del fascio. Le sue
coordinate sono (−3, 0).

12.4 Angoli nel piano euclideo.

Fissiamo nello piano euclideo E 2 un riferimento cartesiano ortogonale OXY di cen-


tro O e versori i, j, rispettivamente per gli assi X,Y .
Chiameremo coseni direttori di una retta r, i coseni degli angoli che la retta forma
con gli assi coordinati. Se la retta é individuata dai parametri direttori (l, m), i suoi
coseni direttori saranno:
l l
α = cos(r, X) ∈ {+ √ ,−√ }
2
l +m 2 l + m2
2

m m
β = cos(r,Y ) ∈ {+ √ ,−√ }
2
l +m 2 l + m2
2

Consideriamo ora due rette ed individuiamole tramite i rispettivi parametri direttori:


288 12 Geometria nel piano affine e nel piano euclideo.

r = (l, m) e r0 = (l 0 , m0 ). Indichiamo con v e v0 due vettori paralleli rispettivamente a


r e r0 , uno di componenti (l, m) e l’altro (l 0 , m0 ). L’angolo tra le due rette é lo stesso
formato dai due vettori:
ll 0 + mm0 ll 0 + mm0
cos(r, r0 ) = cos(v, v0 ) ∈ {+ √ √ ,−√ √ }.
l 2 + m2 · l 02 + m02 l 2 + m2 · l 02 + m02
Quindi le due rette sono ortogonali se ll 0 + mm0 = 0.
Esempio 156. Consideriamo le rette

r : x + 3y − 1 = 0

s : 2x − y + 3 = 0.
I parametri direttori delle rette sono, rispettivamente, (3, −1)r e (1, 2)s . I loro coseni
direttori sono allora:
3 1
cos(r, X) = ± √ , cos(r,Y ) = ± √
10 10
1 2
cos(s, X) = ± √ , cos(s,Y ) = ± √ .
5 5
Il coseno dell’angolo compreso fra le due rette é:
1 1
cos(r, r0 ) = cos(v, v0 ) = ± √ √ =± √ .
10 · 5 5 2

12.5 Distanze nel piano euclideo.

Siano P1 = (x1 , y1 ) e P2 = (x2 , y2 ) due punti del piano euclideo E 2 . La distanza tra i
punti P1 e P2 é il modulo del vettore P1 P2 :
q
δ (P1 , P2 ) = (x2 − x1 )2 + (y2 − y1 )2 .

Esempio 157. Dati i punti P1 = (1, 2) e P2 = (−1, −4), la loro distanza é pari a
p √ √
22 + 62 = 40 = 2 10.

Consideriamo ora il punto P0 = (x0 , y0 ) e la retta r : ax + by + c = 0. Ci proponiamo


di calcolare la distanza del punto dalla retta (disegno 9). Se P0 ∈ r chiaramente
diremo che essa é nulla. poniamoci allora nel caso in cui P0 ∈ / r. La distanza di
P1 da r é pari alla lunghezza δ (P0 H) del segmento P0 H, dove H é la proiezione
ortogonale di P0 su r. Scegliamo un qualsiasi punto Q1 = (x1 , y1 ) ∈ r. Allora δ (P0 H)
é la proiezione ortogonale del vettore P0 Q1 lungo la direzione del vettore P0 H. Per
determinare tale proiezione abbiamo bisogno del versore u di P0 H, che é il versore
12.6 Simmetrie. 289

normale alla retta r:


a b
u = (√ ,√ ).
2
a +b 2 a + b2
2

Quindi:
a b
δ (P0 H) = |(P0 Q1 ) × ( √ i+ √ j)| =
2
a +b 2 a + b2
2

a b
|((x1 − x0 )i + (y1 − y0 ) j) × ( √ i+ √ j)| =
2
a +b 2 a + b2
2

|ax1 − ax0 + by1 − by0 |


√ .
a2 + b2
Dal fatto che Q1 ∈ r segue che ax1 + by1 = −c, per cui

|ax0 + by0 + c|
δ (P0 H) = √ .
a2 + b2
Esempio 158. Dati un punto P = (1, 2) ed una retta r di equazione 3x − 2y + 4 = 0,
la distanza del punto dalla retta é pari a

|3 − 4 + 4| 3
√ =√ .
9+4 13

12.6 Simmetrie.

Due punti P1 = (x1 , y1 ) e P2 = (x2 , y2 ) sono simmetrici rispetto al punto Q = (a, b),
se Q é il punto medio del segmento P1 P2 , cioé se
x1 + x2 y1 + y2
a= , b= .
2 2
Quindi, dato un punto P1 = (x1 , y1 ), per determinare le coordinate (x2 , y2 ) del suo
simmetrico P2 rispetto al punto Q = (a, b), é sufficiente calcolare

x2 = 2a − x1 , y2 = 2b − y1 .

Consideriamo ora il punto P1 = (x1 , y1 ) e la retta r : y = mx + q. Consideriamo una


qualsiasi retta del fascio di centro P1 :

y − y1 = k(x − x1 )

al variare del parametro reale k otteniamo tutte le rette passanti per P1 . Fissiamo una
retta r0 di tale fascio e sia Q = r ∩ r0 . Il punto P2 , simmetrico di P1 rispetto a Q, é
detto il simmetrico di P1 rispetto alla retta r, lungo la direzione individuata dalla
290 12 Geometria nel piano affine e nel piano euclideo.

retta r0 . Quindi uno stesso punto puó avere infiniti simmetrici rispetto ed una retta
che non lo contenga.
Esempio 159. Siano P = (1, 1) e r : x − y = 4. Determiniamo il simmetrico di P
rispetto a r lungo la direzione di coefficiente k = 3.

Svolg. La retta r0 del fascio di centro P e coefficiente angolare 3 é

r0 : y − 1 = 3(x − 1) → y − 3x + 2 = 0.

Il punto Q = r ∩ r0 é dato dalle soluzioni del sistema



y − 3x + 2 = 0
x−y−4 = 0

dal quale otteniamo Q = (−1, −5). Il simmetrico di P rispetto a Q é



0 x = −2 − 1 = −3
P = (x, y) → → P0 = (−3, −11).
y = −10 − 1 = −11

t
u
Concludiamo con la definizione di asse di un segmento AB: esso é la retta perpendi-
colare ad AB e passante per il suo punto medio:
Esempio 160. Siano A = (1, −1) e B = (2, 3). Calcoliamo l’asse del segmento AB.

Svolg. Il vettore AB ha componenti (−1, −4), quindi una retta ad esso ortogonale
ha come parametri direttori, la coppia (b, −a) = (−4, 1).
Il punto medio di AB é Q = ( 32 , 1). Quindi l’asse del segmento AB ha equazione:

1 3
y − 1 = − (x − )
4 2
cioé
2x + 8y + 11 = 0.
t
u

12.7 Coordinate omogenee nel piano.

Sia P = (x, y) un punto del piano. Diremo che (x1 , x2 , x3 ) sono le coordinate omo-
genee di P se xx31 = x e xx23 = y.
Nel caso x3 6= 0 le precedenti scritture hanno evidentemente un senso, e diremo che
il punto é proprio. Ogni punto proprio (x, y) puó banalmente essere individuato da
una terna di coordinate omogenee (x, y, 1). Inoltre due terne tra loro proporzionali
individuano lo stesso punto nel piano.
Nel caso il punto P sia individuato dalla terna (x, y, 0) esso verrá detto improprio.
12.7 Coordinate omogenee nel piano. 291

L’insieme dei punti impropri del piano forma una retta detta retta impropria, la cui
equazione é x3 = 0.

Consideriamo ora la retta r : ax + by + c = 0, nel passaggio alle coordinate omoge-


nee, la sua equazione diventa
x1 x2
a +b +c = 0
x3 x3
cioé
ax1 + bx2 + cx3 = 0.
Tale retta avrá uno ed un solo punto di intersezione con la retta impropria, e sará
detto il punto improprio di r.
É noto che tutti e soli i punti di r sono quelli le cui coordinate soddisfino l’equazione
ax1 + bx2 + cx3 = 0. Tra questi punti c’é anche il punto improprio di coordinate
omogenee (b, −a, 0). Ma similmente ogni punto improprio del tipo (αb, −αa, 0)
soddisfa la precedente equazione, quindi un punto improprio di una retta é unico a
meno di un fattore di proporzionalitá. In generale, direttamente dalla definizione di
coordinate omogenee, é di facile verifica che due qualsiasi terne (x, y, 0) e (z,t, 0)
tra loro proporzionali, individuino il medesimo punto improprio.
In particolare il punto improprio (b, −a, 0) di una retta r non parallela all’asse delle
Y (cioé con b 6= 0) si puó riscrivere come (1, − ba , 0) (dividendo la terna per b).
Mentre il punto improprio di una retta parallela all’asse Y é (0, 1, 0).
Consideriamo ora l’equazione in coordinate omogenee di due rette tra loro parallele:

r : ax1 + bx2 + cx3 = 0 s : ax1 + bx2 + c0 x3 = 0.

Certamente esse non hanno punti propri in comune. Osserviamo cosa accade alla
loro intersezione: il sistema formato dalle loro equazioni

ax1 + bx2 + cx3 = 0
ax1 + bx2 + c0 x3 = 0

é omogeneo, con 3 incognite ma di rango 2. Una soluzione é quindi una qualsiasi


terna proporzionale ai minori della matrice associata, cioé la seguente:
     
b c a c ab
α , − , =
b c0 a c0 ab

α b(c0 − c), −a(c0 − c), 0 = α(c0 − c) (b, −a, 0) ∼



= (l, m, 0)
dove (l, m, 0) indicano al solito i parametri direttori che le rette (in quanto paralle-
le) hanno in comune. Concludiamo allora che rette parallele hanno lo stesso punto
improprio.
292 12 Geometria nel piano affine e nel piano euclideo.

12.8 La circonferenza.

Sia C = (α, β ) un punto del piano euclideo E 2 e sia r un numero reale positivo.
Diciamo circonferenza di centro C e raggio r, il luogo dei punti del piano la cui
distanza da C é pari a r. Sia P = (x, y) un punto qualsiasi della circonferenza:
q
δ (P,C) = (x − α)2 + (y − β )2 = r

da cui otteniamo (quadrando e riordinando):

(x − α)2 + (y − β )2 = r2

x2 + y2 + ax + by + c = 0.
In quest’ultima equazione compaiono i coefficienti legati alle coordinate del centro
e alla lunghezza del raggio r:

a b 1p 2
α =− , β =− , r= a + b2 − 4c
2 2 2
Esempio 161. Determinare la circonferenza di centro C = (1, 1) e raggio r = 3.

Svolg. Applicando la definizione, l’equazione é

(x − 1)2 + (y − 1)2 = 9

x2 + y2 − 2x − 2y − 7 = 0
t
u
Esempio 162. Determinare centro e raggio della circonferenza x2 + y2 − 4x − 2y +
2 = 0.

Svolg. Il centro C ha coordinate α = − −4


2 , β = − −3
2 , da cui C = (2, 1). Il raggio
√ √ √
é r = 12 16 + 4 − 8 = 21 12 = 3. t
u
Osservazione 12.1. La circonferenza é l’unica curva di secondo grado nel piano
che contenga i due seguenti punti impropri: (1, i, 0) e (1, −i, 0) detti punti ciclici.
Circonferenza per tre punti.
Consideriamo tre punti P1 = (x1 , y1 ), P2 = (x2 , y2 ) e P3 = (x3 , y3 ) non allineati cioé

x1 y1 1

x2 y2 1 6= 0.

x3 y3 1

Per ottenere la circonferenza passante per i tre punti dobbiamo sostituire le coor-
dinate dei punti nella generica equazione di una circonferenza. In tale caso la cir-
12.9 Trasformazioni nel piano euclideo. 293

conferenza contenente i tre punti é unica. Infatti il sistema lineare che dobbiamo
risolvere  2
 x1 + y21 + ax1 + by1 + c = 0
x2 + y22 + ax2 + by2 + c = 0
 22
x3 + y23 + ax3 + by3 + c = 0
nelle incognite (a, b, c) ha rango 3.
Esempio 163. Determinare la circonferenza contenente i punti (1, 2), (1, 8), (5, 0).

Svolg. Il sistema lineare da risolvere é



 1 + 4 + a + 2b + c = 0
1 + 64 + a + 8b + c = 0
25 + 5a + c = 0


 a + 2b + c = −5
a + 8b + c = −65
5a + c = −25

le cui soluzioni sono a = −10, b = −10, c = 25 e l’equazione della circonferenza é

x2 + y2 − 10x − 10y + 25 = 0.

t
u

12.9 Trasformazioni nel piano euclideo.

Traslazioni.
In un riferimento cartesiano OXY del piano euclideo, siano (x, y) le coordinate del
punto P. Supponiamo di considerare un secondo riferimento O0 X 0Y 0 in cui gli assi
X 0 , Y 0 siano paralleli rispettivamente a X e Y ed il punto O0 abbia coordinate (a, b)
rispetto a OXY .
Denotiamo (x0 , y0 ) le coordinate di P rispetto al riferimento O0 X 0Y 0 . La relazione che
intercorre tra le due coppie di coordinate di P é la seguente:
 0
x = x−a
y0 = y − b

e le formule inverse sono


x = x0 + a

.
y = y0 + b
Rotazioni.
Siano (x, y) le coordinate del punto P nel riferimento OXY . Consideriamo ora un
secondo riferimento OX 0Y 0 in cui gli assi X 0 e Y 0 siano ruotati in senso antiorario
294 12 Geometria nel piano affine e nel piano euclideo.

di un angolo φ rispetto agli assi X e Y . Per determinare le coordinate (x0 , y0 ) di P


nel secondo riferimento, ricordiamo che esse sono le componenti del vettore OP.
Esprimiamo tali componenti rispetto alle due coppie di versori, i, j in OXY , i0 , j0 in
OX 0Y 0 :
OP = x0 i0 + y0 j0 OP = xi + y j.
Quindi possiamo richiamare quanto detto in relazione al cambiamento di base in
uno spazio vettoriale:    0
x x
= A· 0
y y
dove A é la matrice di cambiamento di base. Nel nostro caso abbiamo che
 
cos(φ ) −sen(φ )
A=
sen(φ ) cos(φ )

cioé      0
x cos(φ ) −sen(φ ) x
= · 0
y sen(φ ) cos(φ ) y
e le formule inverse sono
 0    
x cos(φ ) sen(φ ) x
= ·
y0 −sen(φ ) cos(φ ) y

da cui
x0 = xcos(φ ) + ysen(φ )

.
y0 = −xsen(φ ) + ycos(φ )
Si noti che le due matrici usate per il cambiamento di base sono l’una la tra-
sposta dell’altra, ma anche l’una l’inversa dell’altra, infatti sono entrambe matrici
ortogonali.
Rototraslazioni.
Siano (x, y) le coordinate di P in OXY e consideriamo un riferimento O00 X 00Y 00 in cui
O00 abbia coordinate (a, b) rispetto a OXY e tale che gli assi X 00 e Y 00 siano ruotati in
senso antiorario di un angolo φ rispetto a X e Y . Determiniamo le coordinate (x00 , y00 )
di P nel secondo riferimento.
Effettuiamo prima una traslazione

OXY −→ O00 X 0Y 0
 0
x = x−a
.
y0 = y − b
Successivamente operiamo con una rotazione

O00 X 0Y 0 −→ O00 X 00Y 00


12.10 Un breve cenno sulle curve algebriche piane. 295

x00
     
cos(φ ) sen(φ ) x−a
= ·
y00 −sen(φ ) cos(φ ) y−b
ed inversamente abbiamo che:
     00 
x−a cos(φ ) −sen(φ ) x
= · 00
y−b sen(φ ) cos(φ ) y

cioé
     00   
x cos(φ ) −sen(φ ) x a
= · 00 + .
y sen(φ ) cos(φ ) y b

12.10 Un breve cenno sulle curve algebriche piane.

Due polinomi f (x, y) e g(x, y) nelle variabili x, y a coefficienti reali, sono detti pro-
porzionali se esiste α ∈ R tale che f (x, y) = αg(x, y). É facile verificare che tale
proporzionalitá é una relazione di equivalenza. In tal modo l’insieme R[x, y] di tutti
i polinomi nelle variabili x, y a coefficienti reali é suddiviso in classi di equivalenza,
ciascuna rappresentata da un qualsiasi polinomio ad essa appartenente. Una curva
algebrica piana γ é esattamente una qualsiasi classe di equivalenza di polinomi in
R[x, y]. Quindi, se f (x, y) é un polinomio rappresentante della classe, allora la curva
γ é rappresentata dall’equazione f (x, y) = 0.
In altre parole: sia f (x, y) = 0 una equazione in cui f (x, y) é un polinomio di grado
n nelle variabili x, y. Il luogo geometrico dei punti del piano le cui coordinate soddi-
sfano all’equazione f (x, y) = 0 si chiama curva algebrica di ordine n (o di grado n).
Ovvero, il grado di una curva é il grado del polinomio che la rappresenta. Le curve
algebriche di ordine 1 sono le rette, quelle di ordine 2 sono dette coniche, quelle di
ordine 3 e 4 sono dette rispettivamente cubiche e quartiche etc. etc.
Se il polinomio f (x, y) di grado n non si puó decomporre nel prodotto di due altri
polinomi di gradi inferioni a n, allora la curva γ : f (x, y) = 0 é detta irriducibile. In
caso contrario, ovvero se f (x, y) = g(x, y) · h(x, y), dove g(x, y) e h(x, y) sono due
polinomi di gradi rispettivamente r,t tali che r + t = n, allora la curva γ : f (x, y) = 0
si dice riducibile. In tale situazione, i punti della curva γ sono tutti e soli i punti della
curva γ1 : g(x, y) = 0 e quelli della curva γ2 : h(x, y) = 0. Quindi, geometricamente
parlando, la curva γ é l’unione delle due curve γ1 e γ2 . Algebricamente parlando, il
polinomio che rappresenta γ é il prodotto dei polinomi che rappresentano rispetti-
vamente γ1 e γ2 . Diremo inoltre che le curve γ1 e γ2 sono componenti della curva γ,
oppure anche che γ si spezza nelle sue componenti γ1 e γ2 .
296 12 Geometria nel piano affine e nel piano euclideo.

12.10.1 Intersezione di due curve algebriche.

Adesso chiediamoci in quanti punti possano intersecarsi due distinte curve algebri-
che. La risposta a tale quesito é data dal seguente Teorema (di Bezout), del quale
forniamo semplicemente l’enunciato:
Teorema 12.2. Siano γ : f (x, y) = 0 e δ : g(x, y) = 0 due curve algebriche rispettiva-
mente di gradi n e m. Allora esse hanno esattamente n · m punti in comune (distinti
od anche alcuni di essi coincidenti), eccetto il caso in cui le due curve abbiano
infiniti punti in comune.
L’eventualitá che le due curve γ : f (x, y) = 0 e δ : g(x, y) = 0 abbiano infiniti punti
in comune si verifica nei seguenti casi:
1. sono entrambe riducibili e, nella loro decomposizione, presentano una compo-
nente η : h(x, y) = 0 in comune. I punti di intersezione sono in tal caso quelli
della curva η.
2. una di esse é riducibile e, nella sua decomposizione, ammette l’altra come
componente. Ad esempio, se γ ammette come compnente δ , allora f (x, y) =
g(x, y) · h(x, y), dove h(x, y) é un polinomio di grado m − n. In tal caso, i punti di
intersezione sono tutti e soli quelli della curva δ : g(x, y) = 0.
Esempio 164. Siano γ : x4 + 3xy2 = 0 e δ : x2 − y2 = 0. Dalla loro intersezione
otteniamo
 4
x + 3xy2 = 0
 3  
x (x + 3) = 0 x = 0 3 volte x = −3 1 volta
=⇒ =⇒ ∪ =⇒ .
x 2 = y2 x2 = y2 y2 = 0 y2 = 9

Da qui abbiamo che (0, 0) appare come punto di intersezione con molteplicitá 6,
mentre i punti (−3, 3) e (−3, −3) entrambi con moteplicitá 1, per un totale di 8
punti di intersezione.
Esempio 165. Siano

γ : x5 − x3 y − x2 y − 2xy2 + x2 − y2 + y = 0

e
δ : x3 − x2 y + xy − y2 = 0.
Osservando che

x5 − x3 y − x2 y − 2xy2 + x2 − y2 + y = (x3 − 2xy − y + 1)(x2 + y)

e
x3 − x2 y + xy − y2 = (x − y)(x2 + y)
concludiamo che γ e δ hanno in comune tutti gli infiniti punti della curva η : x2 +y =
0.
Come caso particolare consideriamo quello in cui γ : f (x, y) = 0 sia una curva di
grado n e δ : g(x, y) = 0 sia una curva di grado 1, cioé una retta. Allora γ e δ
12.10 Un breve cenno sulle curve algebriche piane. 297

hanno n punti in comune eccetto quando δ sia una componente di γ cioé f (x, y) =
g(x, y) · h(x, y), dove h(x, y) é un polinomio di grado n − 1.
Esempio 166. Le curve γ : x3 − 3xy2 = 0 e δ : y − 1 = 0 hanno in comune i seguenti
3 punti : √ √
(0, 1), ( 3, 1), (− 3, 1).
Esempio 167. Le curve γ : x3 + x2 y − xy + x − y2 + y = 0 e δ : x + y = 0 hanno in
comune infiniti punti, cioé tutti quelli di δ , quindi la retta δ é una componente di γ,
infatti x3 + x2 y − xy + x − y2 + y = (x + y) · (x2 − y + 1).

12.10.2 Molteplicitá di un punto.

Sia γ : f (x, y) = 0 una curva algebrica di grado n e sia P0 = (x0 , y0 ) un punto di γ.


Consideriamo una generica retta r : y − y0 = k(x − x0 ) appartenente al fascio di cen-
tro P0 . La retta r e la curva γ hanno in comune n punti, non necessariamente tutti
distinti, almeno uno dei quali é proprio P0 . Diciamo che r e γ hanno molteplicitá di
intersezione m0 nel punto P0 = (x0 , y0 ) se (x0 , y0 ) é una soluzione di molteplicitá m0
per il sistema 
f (x, y) = 0
.
y − y0 = k(x − x0 )
Esempio 168. Sia γ : x3 − y2 = 0 e siano P0 = (0, 0) e r : x − y = 0. Determiniamo
la molteplicitá di intersezione tra γ e r nel punto P0 .
Il sistema  3
x − y2 = 0
x−y = 0
ha tre soluzioni:
P1 = (1, 1) con molteplicitá pari a 1
P0 = (0, 0) con molteplicitá pari a 2.
Quindi nel punto P0 le due curve hanno molteplicitá di intersezione m0 = 2 (e nel
punto P1 = (1, 1) hanno molteplicitá di intersezione m1 = 1).
Sia ora data una curva algebrica γ : f (x, y) = 0 di grado n e sia ancora P0 = (x0 , y0 ) un
punto di γ. Consideriamo ora una generica retta ri del fascio di centro P0 . Ciascuna
delle rette ri ha una molteplicitá di intersezione mi con γ in P0 . Diremo molteplicitá
di P0 per la curva γ, il minimo di tali mi .
Il punto P0 é detto semplice se min{mi } = 1, é detto punto multiplo se min{mi } ≥ 2,
in particolare é detto doppio se min{mi } = 2, triplo se min{mi } = 3, quadruplo se
min{mi } = 4, etc. etc.
Esempio 169. Sia γ : x3 − x2 + y2 = 0 una cubica in OXY . Determiniamo la
molteplicitá di P(0, 0) ∈ γ.
Sia r la generica retta passante per P : y = mx.
298 12 Geometria nel piano affine e nel piano euclideo.

Per ottenere la molteplicitá di P dobbiamo intersecare r con γ.


 3
x − x 2 + y2 = 0
 3
x − x 2 + m2 x 2 = 0

y = mx y = mx

in cui la soluzione (0, 0) é doppia, quindi P é un punto doppio per γ.


Esempio 170. Sia γ : (x2 + y2 )2 + 3x2 y − y3 = 0 una quartica in OXY . Determi-
niamo la molteplicitá di P(0, 0) ∈ γ.
Sia r la generica retta passante per P : y = mx.
Per ottenere la molteplicitá di P dobbiamo intersecare r con γ.
 2
(x + y2 )2 + 3x2 y − y3 = 0
 2
(x + m2 x2 )2 + 3mx3 − m3 x3 = 0
→ →
y = mx y = mx

x3 (x + m4 x + 2m2 x + 3m − m3 ) = 0


y = mx
in cui la soluzione (0, 0) é tripla, quindi P é un punto triplo per γ.
Esempio 171. Sia la curva di sesto grado γ : (x2 + y2 )3 − 4x2 y2 = 0 in OXY .
Determiniamo la molteplicitá di P(0, 0) ∈ γ.
Sia r la generica retta passante per P : y = mx.
Per ottenere la molteplicitá di P dobbiamo intersecare r con γ.
 2
(x + y2 )3 − 4x2 y2 = 0
 2
(x + m2 x2 )3 − 4m2 x4 = 0
→ →
y = mx y = mx

x4 (x2 + m6 x2 + 3m2 x2 + 3m4 x2 − 4m2 ) = 0




y = mx
in cui la soluzione (0, 0) é quadrupla, quindi P é un punto quadruplo per γ.

12.10.3 Calcolo dei punti doppi di una curva.

Per terminare questo capitolo, esponiamo a brevi linee (e certamente non nella sua
forma completa) il metodo generalmente utilizzato per una prima analisi degli even-
tuali punti doppi di una curva algebrica. Esso prevede la conoscenza da parte degli
studenti della definizione e del metodo di calcolo delle derivate parziali di funzioni
a due variabili, ed é qui espresso solo per un senso di completezza.
Sia P0 = (x0 , y0 ) un punto della curva rappresentata dal polinomio f (x, y), per cui
avremo che f (x0 , y0 ) = 0. Diremo che P0 é un punto doppio della curva se la coppia
di coordinate (x0 , y0 ) é una soluzione del sistema formato dalle equazioni:
12.11 Esercizi svolti. 299

 f (x, y) = 0
f 0 (x, y) = 0
 x0
fy (x, y) = 0
00 (x, y), f 00 (x, y), f 00 (x, y) non si annulla,
ed almeno una delle derivate seconde fxx yy xy
quando é calcolata in (x0 , y0 ).
Esempio 172. Sia γ : 2x4 − 3x2 y + y2 − 2y3 + y4 = 0. Determiniamo gli eventuali
punti doppi.
Risolviamo dapprima il sistema

 f (x, y) = 2x4 − 3x2 y + y2 − 2y3 + y4 = 0


fx0 (x, y) = 8x3 − 6xy = 0


 0
fy (x, y) = −3x2 + 2y − 6y2 + 4y3 = 0

Esso é soddisfatto dalla coppia (0, 0). Quindi il punto P0 = (0, 0) é il candidato ad
essere un punto doppio per la curva. In effetti la derivata seconda fyy 00 = 2 − 12y +
2
12y non si annulla in P0 , per cui esso é effettivamente un punto doppio.

12.11 Esercizi svolti.

Esercizio 88. Nel piano euclideo determinare il punto simmetrico di P = (1, 1)


rispetto alla retta r : x − 2y + 4 = 0 lungo la direzione ortogonale a r.

Svolgimento: Una qualsiasi retta ortogonale alla retta r ha equazione 2x + y + c =


0, al variare di c ∈ R. Imponendo il passaggio per il punto P, abbiamo 2 + 1 + c = 0,
da cui c = −3. Quindi la retta s, ortogonale a r e passante per P, ha equazione
2x+y−3 = 0. Intersecando le due rette otteniamo il centro della simmetria richiesta.
Si tratta di risolvere il sistema

x − 2y + 4 = 0
.
2x + y − 3 = 0

Svolgendo, otteniamo r ∩ s = C = ( 25 , 115 ).


Detto P0 = (x0 , y0 ) il punto simmetrico di P rispetto a C, avremo

1 + x0 2 1 + y0 11
= , =
2 5 2 5
da cui
1 17 
P0 = − , .
5 5

Esercizio 89. Siano A = (2, −1) e B = (3, 2) punti del piano euclideo. Determinare
l’asse del segmento AB.
300 12 Geometria nel piano affine e nel piano euclideo.

Svolgimento: Il punto medio M del segmento AB ha coordinate ( 52 , 12 ), mentre


le componenti del vettore AB sono (1, 3). L’asse del segmento é quindi la retta r
passante per M e di parametri direttori (l, m), tali che l + 3m = 0. Possiamo quindi
assumere (l, m) = (−3, 1), per cui

x = −3t + 52

r ::
y = t + 12
ovvero
x − 52 1
t= = y−
−3 2
cioé
r : x + 3y − 4 = 0.

Esercizio 90. Determinare la circonferenza passante per i punti (0, 2), (1, 1),
(2, −1).

Svolgimento: Verifichiamo inizialmente che i tre punti non siano allineati:



0 2 1

1 1 1 6= 0.

23 −1 1

Consideriamo adesso l’equazione di una generica circonferenza nel piano: x2 + y2 +


ax + by + c = 0.
Per ottenere la circonferenza passante per i tre punti dobbiamo sostituire le coordi-
nate dei punti nell’equazione:

 2b + c + 4 = 0
a+b+c+2 = 0
2a − b + c + 5 = 0

le cui soluzioni sono a = 7, b = 5, c = −14. Quindi l’equazione della circonferenza


richiesta é
x2 + y2 + 7x + 5y − 14 = 0.

Esercizio 91. Siano r : x = 3t − 1, y = 2t + 5 l’equazione in forma parametrica


di una retta nel piano e P(1, 1) un punto esterno ad essa. Determinare:
1. L’equazione in forma implicita della retta.
2. La distanza tra il punto P e la retta r.
3. La retta passante per P ed ortogonale a r.
4. La retta passante per P e parallela a r.

Svolgimento:
12.11 Esercizi svolti. 301

1. Dall’equazione in forma cartesiana deriva che


x+1 y−5
t= =
3 2
da cui r : 2x − 3y + 17 = 0.
2. La distanza di P da r é pari a

|2 − 3 + 17| 16
√ =√
4+9 13
3. Una qualsiasi retta ortogonale a r ha equazione 3x + 2y + c = 0, al variare di
c ∈ R. Imponendo il passaggio per il punto P, otteniamo 3 + 2 + c = 0, da cui
c = −5. L’equazione richiesta é quindi 3x + 2y − 5 = 0.
4. Una qualsiasi retta parallela a r ha equazione 2x − 3y + c = 0, al variare di c ∈ R.
Imponendo il passaggio per P, abbiamo 2 − 3 + c = 0, ovvero c = 1. Per cui
2x − 3y + 1 = 0 é l’equazione richiesta.
Esercizio 92. Siano γ : x2 + y2 − 3x + 2y − 1 = 0 una circonferenza e P0 = (1, 1) un
suo punto. Determinare l’equazione della retta tangente a γ in P0 .

Svolgimento: Determiniamo inizialmente le coordinate del centro C della circon-


ferenza: C = ( 23 , −1). Sia P = (x, y) un qualsiasi punto della retta t, tangente a γ in
P0 .
Le componenti del vettore P0 P sono (x − 1, y − 1), quelle del vettore P0C sono
( 21 , −2). Poiché P0 , P ∈ t, allora i vettori P0 P e P0C dovranno essere ortogonali.
Quindi il loro prodotto scalare dovrá annullarsi:
1
(x − 1) − 2(y − 1) = 0.
2
Quest’ultima é la relazione lineare che intercorre tra le coordinate dei punti che
giacciono sulla retta tangente richiesta:

x − 4y + 3 = 0.

Esercizio 93. Siano γ : x2 + y2 − 3x + 2y − 1 = 0 una circonferenza e P0 = (1, 1)


un suo punto. Determinare l’equazione della circonferenza γ 0 tangente a γ in P0 e
passante per il punto Q di coordinate (3, −1).

Svolgimento: Dall’esercizio precedente abbiamo l’equazione della retta tangente


a γ in P0
t : x − 4y + 3 = 0.
Tale retta sará quindi anche tangente a γ 0 in P0 .
Diremo fascio di circonferenze tangenti in P0 la totalitá delle circonferenze che sia-
no tutte tra loro tangenti in P0 . Inoltre la retta t é tangente in P0 ad ogni circonferenza
302 12 Geometria nel piano affine e nel piano euclideo.

del fascio .
Tale insieme di circonferenze puó essere rappresentato tramite una equazione f (x, y) =
0. Il polinomio f (x, y) si ottiene tramite una combinazione lineare di due polinomi:
uno rappresentativo di una circonferenza del fascio e l’altro rappresentativo della
retta tangente a tutte le circonferenze del fascio nel punto P0 . Costruiamo il fascio
di circonferenze tangenti alla retta t nel punto P0 :

x2 + y2 − 3x + 2y − 1 + λ (x − 4y + 3) = 0 λ ∈ R.

Imponiamo il passaggio di una circonferenza del fascio per il punto Q:


1
9 + 1 − 9 − 2 − 1 + λ (3 + 4 + 3) = 0 =⇒ λ = .
5
Sostituendo nell’equazione del fascio, otteniamo l’equazione della circonferenza
richiesta:
1
x2 + y2 − 3x + 2y − 1 + (x − 4y + 3) = 0
5
ovvero
5x2 + 5y2 − 14x + 6y − 2 = 0.

Esercizio 94. Siano γ : x2 + y2 − 2x − 2y + 1 = 0 una circonferenza e P1 = (1, 0),


P2 = (0, 1) due suoi punti. Determinare l’equazione della circonferenza γ 0 secante γ
in P1 , P2 e passante per il punto Q di coordinate (−2, −1).

Svolgimento: Inizialmente determiniamo l’equazione della retta passante per P1 e


P2 :
r : x + y − 1 = 0.
Tale retta sará quindi anche secante γ 0 in P1 e P2 .
Diremo fascio di circonferenze secanti in P1 e P2 la totalitá delle circonferenze che
siano tutte tra loro secanti in P1 e P2 . Inoltre la retta r é interseca ogni circonferenza
del fascio in P1 e P2 .
Tale insieme di circonferenze puó essere rappresentato tramite una equazione f (x, y) =
0. Il polinomio f (x, y) si ottiene tramite una combinazione lineare di due polinomi:
uno rappresentativo di una circonferenza del fascio e l’altro rappresentativo della
retta che interseca ogni circonferenza del fascio nei punti P1 e P2 .
Costruiamo il fascio di circonferenze secanti la retta r in P1 e P2 :

x2 + y2 − 2x − 2y + 1 + λ (x + y − 1) = 0 λ ∈ R.

Imponiamo il passaggio di una circonferenza del fascio per il punto Q:

4 + 1 + 4 + 2 + 1 + λ (−2 − 1 − 1) = 0 =⇒ λ = 3.

Sostituendo nell’equazione del fascio, otteniamo l’equazione della circonferenza


richiesta:
12.11 Esercizi svolti. 303

x2 + y2 − 2x − 2y + 1 + 3(x + y − 1) = 0
ovvero
x2 + y2 + x + y − 2 = 0.

Esercizio 95. Sia P(−1, 3) nel sistema di riferimento cartesiano OXY , O0 (1, 2) e
X 0 ,Y 0 assi paralleli rispettivamente a X e Y e passanti per O0 . Sia inoltre r : 3x +
y−1 = 0 rispetto a OXY . Determiniamo le coordinate di P e l’equazione di r rispetto
al riferimento O0 X 0Y 0 .

Svolgimento: Le equazioni di traslazione sono


 0
x = x−a
y0 = y − b

dove (a, b) sono le coordinate del nuovo centro del riferimento, ed applicandole nel
nostro caso otteniamo le coordinate di P0
 0
x = −1 − 1 = −2
.
y0 = 3 − 2 = 1

Le formule inverse sono


x = x0 + a


y = y0 + b
da cui otteniamo l’equazione della retta

r: 3(x0 + 1) + (y0 + 2) − 1 = 0

3x0 + y0 + 4 = 0.

Esercizio 96. Siano P(1, −1) in OXY e X 0 ,Y 0 assi passanti per O e ruotati di π4
in senso antiorario rispetto a X,Y . Sia inoltre r : 2x + 3y − 2 = 0 rispetto al
riferimento OXY . Determiniamo le coordinate di P e l’equazione di r rispetto a
OX 0Y 0 .

Svolgimento: Le equazioni di rotazione sono


 0    
x cos(φ ) sen(φ ) x
= ·
y0 −sen(φ ) cos(φ ) y

da cui
x0 = xcos(φ ) + ysen(φ )

.
y0 = −xsen(φ ) + ycos(φ )
Le formule inverse sono
304 12 Geometria nel piano affine e nel piano euclideo.
     0
x cos(φ ) −sen(φ ) x
= · 0
y sen(φ ) cos(φ ) y

da cui
x = x0 cos(φ ) − y0 sen(φ )

.
y = x0 sen(φ ) + y0 cos(φ )
Nel nostro caso le coordinate di P0 sono
( √ √
x0√= 22 (x + y) =√ 22 (1 − 1) = 0

y0 = 22 (−x + y) = 22 (−1 − 1) = − 2

e l’equazione della retta é


√ √
2 0 0 2 0
r: 2 (x − y ) + 3 (x + y0 ) − 2 = 0
2 2
√ √
r : 5 2x0 + 2y0 − 4 = 0.

Esercizio 97. Siano P(3, −1) e O0 (1, 2) in OXY e X 0 ,Y 0 assi passanti per O0 e ruotati
di π4 in senso antiorario rispetto a X,Y . Sia inoltre r : 2x + y − 3 = 0 rispetto al
riferimento OXY . Determiniamo le coordinate di P e l’equazione di r rispetto a
O0 X 0Y 0 .

Svolgimento: Le equazioni di rototraslazione sono


 0    
x cos(φ ) sen(φ ) x−a
= ·
y0 −sen(φ ) cos(φ ) y−b

da cui
x0 = (x − a)cos(φ ) + (y − b)sen(φ )

.
y0 = −(x − a)sen(φ ) + (y − b)cos(φ )
Le formule inverse sono
     0  
x cos(φ ) −sen(φ ) x a
= · 0 +
y sen(φ ) cos(φ ) y b

da cui
x = x0 cos(φ ) − y0 sen(φ ) + a

.
y = x0 sen(φ ) + y0 cos(φ ) + b
Nel nostro caso le coordinate di P0 sono
( √ √ √
x0 =
√ 2
2
(x − a + y − b) = √2
2
(2 − 3) = − 2
2 √
0
y = 2 (−x + a + y − b) = 2 (−2 − 3) = −5 22
2 2

e l’equazione della retta é


12.11 Esercizi svolti. 305
√ √
2 0 2 0
r: 2( (x − y0 ) + 1) + ( (x + y0 ) + 2) − 3 = 0
2 2
√ √
r : 3 2x0 − 2y0 + 2 = 0.

Esercizio 98. Siano OXY un sistema di riferimento ortogonale nel piano euclideo,
P un punto di coordinate (−1, 3) rispetto a OXY e r una retta di equazione 2x −
3y + 1 = 0 in OXY . Determinare le coordinate di P e l’equazione di r in un secondo
sistema di riferimento O0 X 0Y 0 nei seguenti casi:
1. O0 ha coordinate (1, 2) rispetto a OXY e gli assi X 0 ,Y 0 sono paralleli agli assi X,Y
(traslazione).
2. O0 = O e gli assi X 0 ,Y 0 sono ruotati di un angolo α = π3 in senso antiorario
rispetto a X,Y (rotazione).
3. O0 ha coordinate (1, −1) rispetto a OXY e gli assi X 0 ,Y 0 sono ruotati di un angolo
α = π6 in senso antiorario rispetto a X,Y (rototraslazione).
Esercizio 99. Siano OXY un sistema di riferimento ortogonale nel piano euclideo e
γ : x2 + y2 − 3x + y − 1 = 0 una circonferenza la cui equazione é espressa rispetto
a OXY . Determinare l’equazione di γ in un secondo sistema di riferimento O0 X 0Y 0
nel caso i cui O0 abbia coordinate (2, 1) rispetto a OXY e gli assi X 0 ,Y 0 siano ruotati
di un angolo α = π4 in senso antiorario rispetto a X,Y .
Capitolo 13
Le Coniche.

Definizione 13.1. Consideriamo una curva algebrica γ nel piano, rappresentata da


un polinomio f (x, y) di secondo grado nelle variabili x, y. L’insieme dei punti del
piano che, con le loro coordinate, soddisfano all’equazione f (x, y) = 0 é detto coni-
ca. Nel caso piú generale, un polinomio di secondo grado nelle variabili x, y assume
la seguente forma:

f (x, y) = ax2 + bxy + cy2 + dx + ey + f .

Quindi una conica si puó rappresentare tramite l’equazione:

ax2 + bxy + cy2 + dx + ey + f = 0

dove a, b, c, d, e, f ∈ R. In coordinate omogenee la curva é rappresentata da un


polinomio omogeneo f (x1 , x2 , x3 ) di secondo grado e, nella forma piú generale,
l’equazione diventa:

a11 x12 + 2a12 x1 x2 + a22 x22 + 2a13 x1 x3 + 2a23 x2 x3 + a33 x32 = 0

dove ai j ∈ R, per ogni i = 1, 2, 3 e j = 1, 2, 3.


Si noti che anche il prodotto di due polinomi di primo grado determina un polino-
mio rappresentante di una conica. In tale caso la conica é l’unione delle due rette
rappresentate dai due polinomi di primo grado.
Consideriamo ora la matrice simmetrica di ordine 3, formata con i coefficienti
dell’equazione della conica in coordinate omogenee:
 
a11 a12 a13
A =  a12 a22 a23 
a13 a23 a33
 
x1
ed indichiamo X =  x2  il vettore delle coordinate omogenee. Allora l’equazione
x3

307
308 13 Le Coniche.

della conica puó essere riscritta in forma compatta:

f (x1 , x2 , x3 ) = X T · A · X = 0

ed il polinomio f (x1 , x2 , x3 ) é una forma quadratica in R3 , di matrice associata A.


Esempio 173. L’equazione x2 + 2y2 − xy + x − 1 = 0 rappresenta una conica nel
piano. La matrice associata alla conica é

1 − 12 12
 

A =  − 12 2 0  .
1
2 0 −1

Definizione 13.2. Diremo che una conica γ : f (x, y) = 0 é riducibile se essa é com-
posta dall’unione di due rette cioé se esistono due polinomi di primo grado g(x, y) e
h(x, y) tali che f (x, y) = g(x, y) · h(x, y).
Vale il seguente:
Teorema 13.3. Sia γ una conica con matrice associata A. Le seguenti affermazioni
sono equivalenti:
1. La conica γ é riducibile.
2. La conica γ ha almeno un punto doppio.
3. det(A) = 0.

13.1 Intersezione di una conica ed una retta.

Siano γ : X T AX = 0 l’equazione di una conica e r una retta passante per i punti


P = Y e P0 = Y 0 . Un generico punto della retta é allora rappresentabile tramite il
suo vettore di coordinate X, come combinazione lineare dei vettori di coordinate
Y e Y 0 : X = λY + µY 0 é quindi l’equazione della retta. Se vogliamo determinare
l’intersezione tra γ e r, sostituiamo X nell’equazione della conica:

(λY + µY 0 )T A(λY + µY 0 ) = 0

da cui
µλ (Y 0T AY ) + µ 2 (Y 0T AY 0 ) + λ 2 (Y T AY ) + λ µ(Y T AY 0 ) = 0.
Osserviamo che, poiché Y 0T AY é un numero reale, Y 0T AY = (Y 0T AY )T = Y T AY 0 ,
quindi segue che

λ 2 (Y T AY ) + 2µλ (Y T AY 0 ) + µ 2 (Y 0T AY 0 ) = 0

che é una equazione di secondo grado nelle incognite (λ , µ). Le due soluzioni for-
niscono le coppie di valori (λ1 , µ1 ), (λ2 , µ2 ) che, sostituiti alternativamente nel-
l’equazione della retta, individuano i due punti di intersezione P1 = λ1Y + µ1Y 0 ,
13.1 Intersezione di una conica ed una retta. 309

P2 = λ2Y + µ2Y 0 . Il discriminante dell’equazione di secondo grado é

(Y T AY 0 )2 − (Y T AY )(Y 0T AY 0 ) = ∆ .

Si puó concludere allora che la retta r é esterna, tangente o secante la conica, in


relazione al valore del ∆ (rispettivamente < 0, = 0, > 0.)
Esempio 174. Siano γ : x2 − 4xy + y2 − 2x − 2y − 1 = 0 e r : x + y − 1 = 0. Per de-
terminare la loro reciproca posizione, inizialmente costruiamo la matrice associata
alla conica γ. Essa é  
1 −2 −1
A =  −2 1 −1  .
−1 −1 −1
Scegliamo adesso due punti sulla retta r:

P1 = (2, −1), P2 = (−1, 2)

che in coordinate omogenee esprimiamo


   
2 −1
P1 → Y =  −1  P2 → Y 0 =  2  .
1 1

La retta r puó quindi essere espressa tramite la relazione


     
x 2 −1
 x2  = λ  −1  + µ  2  , λ , µ ∈ R.
x3 1 1

Calcoliamo adesso il discriminante dell’intersezione

∆ = (Y T AY 0 )2 − (Y T AY )(Y 0T AY 0 )

ovvero
  
 1 −2 −1 −1 2
∆ = [2, −1, 1]  −2 1 −1   2 
−1 −1 −1 1
     
 1 −2 −1 2   1 −2 −1 −1 
− [2, −1, 1]  −2 1 −1   −1  · [−1, 2, 1]  −2 1 −1   2 
−1 −1 −1 1 −1 −1 −1 1

= (−17)2 − 100 > 0.

Quindi retta e conica sono secanti in due punti reali e distinti.


310 13 Le Coniche.

13.2 Polaritá rispetto ad una conica.

Sia γ : X T · A · X = 0 una conica non riducibile. Definiamo polaritá rispetto a γ la


corrispondenza biunivoca che associa ad ogni punto X 0 = (x10 , x20 , x30 ) del piano, una
retta r0 la cui equazione é data da X 0T · A · X = 0. La retta r0 é detta retta polare di X 0
rispetto a γ, il punto X 0 é detto polo della retta r0 rispetto a γ.
Consideriamo r0 la retta polare del punto X 0 rispetto alla conica γ. La sua equazione é
data in forma compatta da X 0T AX = 0. Sia X 00 un qualsiasi punto di r0 . Quindi le sue
coordinate soddisfano l’equazione algebrica che individua la retta r0 , cioé X 0T AX 00 =
0. Trasponendo ambo i membri di tale uguaglianza, otteniamo X 00T AX 0 = 0, che ha
un significato ben preciso: il punto di coordinate X 0 appartiene alla retta r00 polare di
X 00 (tale proprietá é detta reciprocitá). Riassumendo potremmo dire che se un punto
X 00 appartiene alla polare di un altro punto X 0 , allora X 0 appartiene alla polare di X 00 .
Diremo che i punti X 0 e X 00 sono tra loro coniugati, ed analogamente che le rette r0
e r00 sono tra loro coniugate.
Un punto é detto autoconiugato se appartiene alla propria polare.
Consideriamo ora un punto X 0 appartenente alla conica. Sia t la retta tangente alla
conica in X 0 . Se scegliamo un qualsiasi altro punto X 00 ∈ t, l’equazione della tangente
si puó esprimere in forma compatta con X = αX 0 + β X 00 . Se ora intersechiamo la
retta t con la conica, ovviamente ci aspettiamo un discriminante del sistema pari a
zero, vista la tangenza in X 0 . Come giá visto nel paragrafo relativo all’intersezione
di una retta con una conica, il discriminante di tale sistema sará:

∆ = (X 00T AX 0 )2 − (X 00T AX 00 )(X 0T AX 0 ) = 0

e poiché X 0 ∈ γ, segue che X 0T AX 0 = 0 e quindi ∆ = (X 00T AX 0 )2 = 0. Inoltre


(X 00T AX 0 ) é uno scalare, il suo quadrato é nullo solo se esso stesso é nullo ed il
suo valore trasposto deve essere ancora nullo; in definitiva X 0T AX 00 = 0, cioé X 00
(generico punto della tangente t) é un punto della retta polare di X 0 . Questo é suffi-
ciente per affermare che se un punto appartiene alla conica, allora la sua retta polare
coincide con la tangente in esso alla conica. Tale punto é autoconiugato, piú in gene-
rale, tutti e soli i punti autoconiugati rispetto ad una conica sono quelli della conica
stessa. Ed ancora, una retta é detta autoconiugata se contiene il proprio polo. Tutte e
sole le rette autoconiugate rispetto ad una conica, sono le tangenti alla conica stessa.
Concludiamo fornendo un significato geometrico della retta polare di un punto ester-
no alla conica. Sia P0 = X 0 il punto scelto. Da esso si possono condurre due rette
tangenti alla conica, siano esse r1 e r2 . Indichiamo con P1 il punto di tangenza per la
retta r1 , con P2 il punto di tangenza per la retta r2 . Per quanto detto sopra, tali rette
tangenti alla conica sono proprio le rispettive polari di P1 e P2 , per cui il punto P0 ap-
partiene sia alla polare di P1 , che alla polare di P2 . Grazie alla reciprocitá, entrambi
P1 e P2 devono appartenere alla retta r polare di P0 . Tale retta r é quindi esattamente
la retta congiungente i punti P1 e P2 .
Esempio 175. Sia γ : x2 + 2y2 − 4x + 2y − 1 = 0. La matrice associata alla conica γ

13.3 Classificazione di una conica. 311
 
1 0 −2
A =  0 2 1 .
−2 1 −1
La matrice a é non singolare, per cui é possibile definire la corrispondenza di pola-
ritá tra i punti e le rette del piano rispetto a γ.
Scegliamo adesso un qualsiasi punto del piano P0 = (1, 2) ed esprimiamolo in coor-
dinate omogenee P0 → X0 = (1, 2, 1). Determiniamo la retta polare di P0 rispetto a
γ:   
1 0 −2 x1
r0 : [1, 2, 1]  0 2 1   x2  = 0
−2 1 −1 x3
cioé x1 − 5x2 + x3 = 0, ovvero x − 5y + 1 = 0. Consideriamo adesso un punto di
r0 , scegliamo ad esempio P1 = (−1, 0) ed esprimiamolo in coordinate omogenee
P1 → X1 = (−1, 0, 1). La retta polare di P1 rispetto a γ é quindi
  
1 0 −2 x1
r1 : [−1, 0, 1]  0 2 1   x2  = 0
−2 1 −1 x3

cioé 3x1 − x2 − x3 = 0 ovvero 3x − y − 1 = 0. A questo punto é facile osservare che


P0 ∈ r1 . Valgono entrambe le seguenti relazioni:

X0T AX1 = 0, X1T AX0 = 0.

Diremo quindi che i punti P0 e P1 sono tra loro coniugati (rispetto a γ), poiché l’uno
appartiene alla polare dell’altro. Diremo inotre che le due rette r0 e r1 sono tra loro
coniugate (rispetto a γ), poiché l’una contiene il polo dell’altra.

13.3 Classificazione di una conica.

Sia γ : X T · A · X = 0 una conica nel piano. Se intersechiamo γ con la retta impropria


x3 = 0 otteniamo ovviamente 2 soluzioni, cioé i due punti impropri della conica.
Una conica con due punti impropri reali e distinti é detta Iperbole. Una conica con
due punti impropri reali e coincidenti é detta Parabola. Una conica con due punti im-
propri complessi coniugati é detta ellisse. In altre parole la retta impropria é secante
all’iperbole, tangente alla parabola, esterna all’ellisse. Consideriamo il sistema
 T
a11 x12 + 2a12 x1 x2 + a22 x22 = 0

X ·A·X = 0
→ .
x3 = 0 x3 = 0

Il discriminante del sistema é ∆ = a212 − a11 a22 , da cui


312 13 Le Coniche.

 ∆ > 0 → Iperbole
∆ = 0 → Parabola .
∆ < 0 → Ellisse

In particolare se indichiamo con A33 il complemento algebrico dell’elemento a33


della matrice A, notiamo che A33 = −∆ , quindi

 A33 > 0 → Ellisse
A33 = 0 → Parabola .
A33 < 0 → Iperbole

Esempio 176. Sia γ : 3x2 − y2 + 2xy − 3x + 2y − 2 = 0. La matrice associata alla


conica γ é
3 1 − 23
 

A =  1 −1 1  .
− 32 1 −2
La matrice non é singolare, per cui la conica non é ridotta nell’unione di rette. Il
complemento A33 = −3 − 1 < 0 indica che la conica é una iperbole.
Esempio 177. Sia γ : 3x2 + 2y2 − xy − 6x − 4y + 1 = 0. La matrice associata alla
conica γ é
3 − 12 −3
 

A =  − 12 2 −2  .
−3 −2 1
La matrice non é singolare, per cui la conica non é ridotta nell’unione di rette. Il
complemento A33 = 6 − 14 > 0 indica che la conica é una ellisse.
Esempio 178. Sia γ : 4x2 + y2 − 4xy − 2x + 3y − 2 = 0. La matrice associata alla
conica γ é  
4 −2 −1
A =  −2 1 32  .
−1 23 −2
La matrice non é singolare, per cui la conica non é ridotta nell’unione di rette. Il
complemento A33 = 4 − 4 = 0 indica che la conica é una parabola.
Osservazione 13.4. Tutte e sole le coniche che contengono i punti ciclici (1, i, 0) e
(1, −i, 0) sono le circonferenze (che sono particolari ellissi).

13.4 Intersezione fra due coniche.

Siano γ1 : f1 (x1 , x2 , x3 ) = 0 e γ2 : f2 (x1 , x2 , x3 ) = 0 due coniche distinte. Poiché sono


curve di secondo grado, escludendo il caso in cui siano entrambe riducibili con una
retta in comune, esse hanno 4 punti A, B,C, D in comune (distinti o no, reali o no).
Il caso generale é quello in cui A, B,C, D sono tutti tra loro distinti.
13.4 Intersezione fra due coniche. 313

Nel caso in cui si abbia A = B e C, D distinti, allora le due coniche hanno una
tangente comune ad entrambe (γ1 e γ2 sono chiamate coniche tangenti), ed é la retta
tangente nel punto A = B.
Nel caso si abbia A = B e C = D allora le due coniche hanno due rette tangenti
in comune (γ1 e γ2 sono chiamate coniche bitangenti), e sono le tangenti nel punto
A = B ed in C = D.
Nel caso si abbia A = B = C allora le due coniche hanno una retta tangente in
comune (γ1 e γ2 sono chiamate coniche osculatrici), ed é la tangente nel punto
A = B = C.
Nel caso si abbia A = B = C = D allora le due coniche hanno una retta tangente in
comune (γ1 e γ2 sono chiamate coniche iperosculatrici), ed é la tangente nel punto
A = B = C = D.

Diciamo fascio di coniche la totalitá delle coniche che si ottengono da

a1 · f1 (x1 , x2 , x3 ) + a2 · f2 (x1 , x2 , x3 ) = 0

al variare di a1 , a2 ∈ R. Tutte le coniche del fascio hanno in comune gli stessi 4 punti
A, B,C, D, i quali sono chiamati punti base del fascio. Inoltre per individuare un
fascio di coniche é sufficiente conoscere due qualsiasi coniche di esso (ad esempio
anche due coniche riducibili che appartengono al fascio).

Il caso generale é quello in cui A, B,C, D sono tutti tra loro distinti, si parla quindi
di fascio generale.
Nel caso in cui si abbia A = B e C, D distinti, allora le coniche del fascio hanno una
tangente comune (si parla di fascio di coniche tangenti), ed é la retta tangente nel
punto A = B.
Nel caso si abbia A = B e C = D allora le coniche del fascio hanno due rette tangenti
in comune (si parla di fascio di coniche bitangenti), e sono le tangenti nel punto
A = B ed in C = D.
Nel caso si abbia A = B = C allora le coniche del fascio hanno una retta tangente
in comune (si parla di fascio di coniche osculatrici), ed é la tangente nel punto
A = B = C.
Nel caso si abbia A = B = C = D allora le coniche del fascio hanno una retta tangente
in comune (si parla di fascio di coniche iperosculatrici), ed é la tangente nel punto
A = B = C = D.

Ogni fascio di coniche contiene 3 coniche riducibili (distinte o no a seconda del tipo
di fascio). Nel seguente prospetto indichiamo con tA e tC rispettivamente le tangenti
comuni a tutte le coniche di un fascio nei punti A e C:
314 13 Le Coniche.

Tipo di fascio Coniche riducibili


generale AB ∪CD
AD ∪ BC
AC ∪ BD
tangenti A = B tA ∪CD
AD ∪ AC contata due volte
bitangenti A = B, C = D tA ∪ tC
AC ∪ AC contata due volte
osculatrici A = B = C tA ∪ AD contata tre volte
iperosculatrici A = B = C = D tA ∪ tA contata tre volte
Esempio 179. Consideriamo il fascio di coniche di equazione F : x2 + (λ − 1)y = 0,
che in forma omogenea esprimiamo F : x12 + (λ − 1)x2 x3 = 0.
Scegliamo le due coniche del fascio x12 = 0 e x2 x3 = 0. Intersecandole avremo le
coordinate dei 4 punti base del fascio.
La soluzione x1 = 0 si presenta 2 volte, ed in corrispondenza di ciascuna di esse
avremo x2 = 0 oppure x3 = 0.
Per cui, in coordinate omogenee, i punti P1 = (0, 0, 1) e P2 = (0, 1, 0) sono gli unici
punti base del fascio, ciscuno dei quali si presenta 2 volte. Siamo di fronte ad un
fascio di coniche bitangenti nei punti P1 e P2 .
La matrice associata alla generica conica del fascio é
 
1 0 0
Aλ =  0 0 λ −1 2
.
λ −1
0 2 0
2
Quindi |A| = − (λ −1) 4 e possiede una radice doppia in λ = 1. Nel panorama delle
3 coniche riducibili del fascio, quella in corrispondenza di λ = 1, cioé x12 = 0, si
presenta 2 volte. La terza conica riducibile é ovviamente x2 x3 = 0, che si palesava
inizialemnte anch’essa nell’equazione del fascio.
Osserviamo che il complemento algebrico dell’elemento di posto (3, 3) é sempre
nullo, per cui tutte le coniche del fascio sono parabole, con l’escusione della conica
riducibile x2 x3 = 0.
Determiniamo infine le tangenti comuni a tutte le coniche del fascio. Per farlo po-
tremmo individuare una conica non ridotta del fascio e calcolare le tangenti ad essa
nei punti P1 , P2 . Scegliamo ad esempio λ = 0, ed otteniamo x2 − y = 0. La matrice
associata a tale conica é  
1 0 0
A =  0 0 − 12 
0 − 12 0
e le due tangenti sono
  
1 0 0 x1
[0, 0, 1]  0 0 − 12   x2  = 0 ⇒ x2 = 0
0 − 12 0 x3
13.4 Intersezione fra due coniche. 315

e   
1 0 0 x1
[0, 1, 0]  0 0 − 12   x2  = 0 ⇒ x3 = 0.
0 − 21 0 x3
Ma avremmo potuto ottenere tali rette anche semplicemente analizzando le informa-
zioni precedentemente ottenute sul fascio. Infatti, poiché x1 = 0 é la retta congiun-
gente i due punti base del fascio, allora x12 = 0 é la conica riducibile che li contiene
e che ha valenza doppia nel conteggio delle coniche riducibili del fascio. Quindi le
due rette tangenti si ottengono decomponendo la restante conica riducibile, ovvero
quella che ha valenza 1 nel computo delle 3 coniche riducibili del fascio (appunto
x2 x3 = 0).
Esempio 180. Consideriamo il fascio di coniche di equazione F : xy + λ (x2 − y2 −
2y) = 0, che in forma omogenea esprimiamo

F : x1 x2 + λ (x12 − x22 − 2x2 x3 ) = 0.

Scegliamo le due coniche del fascio x1 x2 = 0 e x12 − x22 − 2x2 x3 = 0. Intersecandole


avremo le coordinate dei 4 punti base del fascio:

x1 x2 = 0
.
x12 − x22 − 2x2 x3 = 0

Dalla prima avremo x1 = 0 oppure x2 = 0.


Nel caso x1 = 0 la seconda si riduce a x22 + 2x2 x3 = 0, da cui x2 = 0 oppure x2 =
−2x3 . I punti ottenuti sono quindi P1 = (0, 0, 1) e P2 = (0, −2, 1).
Nel caso scegliessimo nella prima equazione x2 = 0, la seconda si ridurrebbe a x12 =
0, con radice doppia x1 = 0. Avremmo allora un solo punto (contato 2 volte) P3 =
(0, 0, 1).
Osserviamo quindi che il punto P1 = P3 = (0, 0, 1) compare 3 volte nel panorama
dei punti base del fascio: si tratta di un fascio di coniche osculatrici.
La soluzione x1 = 0 si presenta 2 volte, ed in corrispondenza di ciascuna di esse
avremo x2 = 0 oppure x3 = 0.
La matrice associata alla generica conica del fascio é

λ 21 0
 

Aλ =  12 −λ −λ  .
0 −λ 0

Quindi |A| = −λ 3 e possiede una radice tripla in λ = 0. Nel panorama delle 3 coni-
che riducibili del fascio, quella in corrispondenza di λ = 0, cioé x1 x2 = 0, si presenta
3 volte.
Il complemento algebrico dell’elemento di posto (3, 3) é pari a −λ 2 − 14 , per cui
sempre negativo. Tutte le coniche del fascio sono iperboli.
Determiniamo infine la tangente comune a tutte le coniche del fascio. Per farlo po-
316 13 Le Coniche.

tremmo individuare una conica non ridotta del fascio e calcolare la tangenti ad es-
sa in P1 . Ma, come precedentemente osservato, potremmo ottenere tale retta anche
semplicemente analizzando le informazioni precedentemente ottenute sul fascio. In-
fatti, poiché x1 = 0 é la retta congiungente i due punti base del fascio, e x1 x2 = 0 é
l’unica conica riducibile, possiamo concludere che x2 = 0 é la retta tangente a tutte
le coniche del fascio in (0, 0, 1).
Esempio 181. Determiniamo adesso l’equazione di un fascio di coniche a partire
dai suoi punti base: scegliamo i punti P1 = (−1, −2), P2 = (3, −2) ed il punto impro-
prio P3 = (1, −1, 0). Vogliamo ottenere l’equazione del fascio di coniche contenenti
P1 e P2 e tangenti in P3 alla retta di equazione x + y = 0.
Per farlo osserviamo inizialmente che trattasi di un fascio di coniche tangenti. Volen-
do esprimere la sua equazione, é sufficiente conoscere le equazioni di due qualsiasi
coniche del fascio. La via piú facile é quella di determinare le 2 coniche riducibili
del fascio.
Una conica riducibile γ1 é data dall’unione della tangente comune in P3 e della retta
P1 P2 . Quest’ultima ha equazione y + 2 = 0, quindi

γ1 : (x + y)(y + 2) = 0.

Ricordiamo che la precedente conica ha valenza 1 nel computo totale delle coniche
riducibili del fascio.
L’altra conica riducibile del fascio γ2 (la quale ha valenza doppia nel computo delle 3
coniche riducibili) é data dall’unione delle rette P1 P3 e P2 P3 . Poiché P1 P3 : x+y+3 =
0 e P2 P3 : x + y − 1 = 0, avremo che

γ2 : (x + y + 3)(x + y − 1) = 0.

Da ció segue che l’equazione del fascio richiesta é

(x + y)(y + 2) + λ (x + y + 3)(x + y − 1) = 0.

13.5 Diametri e centro di una conica.

Definiamo diametro di una conica γ non riducibile, la retta polare, rispetto alla co-
nica, di un qualsiasi punto improprio (h, k, 0). Sia A la matrice associata alla conica,
allora un diametro é dato da
   
  a11 a12 a13 x1
h k 0 ·  a12 a22 a23  ·  x2  = 0
a13 a23 a33 x3

(a11 h + a12 k)x1 + (a12 h + a22 k)x2 + (a13 h + a23 k)x3 = 0


o meglio
13.5 Diametri e centro di una conica. 317

h(a11 x1 + a12 x2 + a13 x3 ) + k(a12 x1 + a22 x2 + a23 x3 ) = 0.

Al variare del punto (h, k, 0) si ottiene un diverso diametro, e quindi al variare dei
parametri h, k si ottengono tutte le rette di un fascio. Il centro di tale fascio é detto
centro della conica (il quale, per la reciprocitá, é il polo della retta impropria).
Le coordinate del centro sono allora la soluzione del sistema

a11 x1 + a12 x2 + a13 x3 = 0
.
a12 x1 + a22 x2 + a23 x3 = 0

Per quanto visto nel Capitolo deidicato ai sitemi lineari,le soluzioni sono date dai
complementi algebrici A13 , A23 , A33 della matrice A associata alla conica.
Il centro C di una conica é un centro di simmetria: infatti, per ogni punto P della
curva, il suo simmetrico P0 , rispetto a C, é ancora un punto della curva.
Il caso in cui A33 = 0 é quello della parabola, la quale é detta conica a centro impro-
prio (o conica senza centro). Il fascio dei diametri é un fascio improprio, cioé tutti i
diametri sono tra loro paralleli ed hanno la direzione data dal punto improprio della
parabola.
Iperbole ed ellisse sono dette coniche a centro (proprio). Tutti i diametri sono tra
loro a due a due coniugati.
In particolare i diametri tra loro coniugati ed ortogonali sono detti assi della conica
(2 assi per iperbole ed ellisse, 1 asse per la parabola). I diametri che siano autoconiu-
gati (cioé tangenti alla conica) sono detti asintoti della conica (2 reali per l’iperbole,
2 immaginari per l’ellisse, 1 improprio per la parabola).
Per determinare assi e asintoti é sufficiente operare come segue: sappiamo bene che
il generico diametro della conica é dato da:
   
  a11 a12 a13 x1
h k 0 ·  a12 a22 a23  ·  x2  = 0. (13.1)
a13 a23 a33 x3

In definitiva il nostro compito é quello di determinare le coppie di valori (h, k) da


utilizzare nella (13.1), sia per individuare gli asintoti che gli assi.
Affinché (h, k, 0) sia esattamente il polo di un asintoto, dovrá accadere che esso
sia un punto autoconiugato (l’asintoto sará tangente in esso alla conica), quindi
imporremo che    
  a11 a12 a13 h
h k 0 ·  a12 a22 a23  ·  k  = 0.
a13 a23 a33 0
Svolgendo, si ottiene a11 h2 + a22 k2 + 2a12 hk = 0. Le soluzioni di tale equazione
biquadraica forniscono le due coppie di valori (h, k) e (h0 , k0 ) che, sostituite nella
(13.1) serviranno per il calcolo effettivo dei due asintoti (stiamo ovviamente parlan-
do di una conica che sia una iperbole).
Analogamente, affinché (h, k, 0) sia esattamente il polo di un asse, dovrá accadere
che sia un punto coniugato con la direzione ad esso ortogonale, quindi imporremo
che
318 13 Le Coniche.
   
  a11 a12 a13 k
h k 0 ·  a12 a22 a23  ·  −h  = 0.
a13 a23 a33 0
In questo caso si ottiene a12 (k2 − h2 ) + (a11 − a22 )hk = 0. Le soluzioni di tale equa-
zione biquadraica forniscono le due coppie di valori (h, k) e (h0 , k0 ) che, sostituite
nella (13.1) serviranno per il calcolo dei due assi.
Nel caso della parabola la ricerca dell’asse é facilitata. Infatti il polo dell’asse della
parabola é dato dalla direzione ortogonale a quella individuata dal punto improprio
della parabola stessa.
Gli assi di una conica sono degli assi di simmetria: ogni punto P della conica ha un
simmetrico P0 rispetto a ciascun asse della conica, lungo la direzione ad esso orto-
gonale, che é ancora un punto della conica.
Osserviamo infine che le direzioni dei due asintoti di un’iperbole si possono ottenere
ricordando che esse non sono altro che i punti impropri dell’iperbole stessa.
Esempio 182. Sia γ : x2 + 4xy − y2 + 2x − 1 = 0 con matrice associata
 
1 2 1
A =  2 −1 0  .
1 0 −1

La matrice é non singolare e A33 = −5 < 0, per cui γ é una iperbole non ridotta.
Per determinarne gli assi, ricerchiamo le direzioni ortogonali e tra loro coniugate
rispetto all’iperbole. Per farlo imponiamo quindi
  
  1 2 1 k
h k 0 ·  2 −1 0   −h  = 0.
1 0 −1 0

Da ció segue 2h2 − 2hk − 2k2 = 0 e, ponendo t = hk , t 2 − t − 1 = 0, le cui soluzioni


sono √
h 1± 5
t= = .
k 2
I poli dei due assi sono allora
√ √
(1 + 5, 2, 0) e (1 − 5, 2, 0).

Le equazione degli assi sono:


  
 √  1 2 1 x1 √ √ √
1 + 5 2 0 ·  2 −1 0   x2  = 0 ⇒ (5 + 5)x + 2 5y + (1 + 5) = 0
1 0 −1 x3
e
13.5 Diametri e centro di una conica. 319
  
 √  1 2 1 x1 √ √ √
1 − 5 2 0 ·  2 −1 0   x2  = 0 ⇒ (5 − 5)x − 2 5y + (1 − 5) = 0.
1 0 −1 x3

Per determinarne gli asintoti, ricerchiamo le direzioni autoconiugate rispetto all’i-


perbole. Per farlo imponiamo quindi
  
  1 2 1 h
h k 0 ·  2 −1 0   k  = 0.
1 0 −1 0

Da ció segue h2 + 4hk − k2 = 0 e, ponendo t = hk , t 2 + 4t − 1 = 0, le cui soluzioni


sono
h √
t = = −2 ± 5.
k
I poli dei due asintoti sono allora
√ √
(−2 + 5, 1, 0) e (−2 − 5, 1, 0).

Le equazione degli asintoti sono:


  
 √  1 2 1 x1 √ √ √
−2 + 5 1 0 ·  2 −1 0   x2  = 0 ⇒ 5x + (2 5 − 5)y + ( 5 − 2) = 0
1 0 −1 x3
e
  
 √  1 2 1 x1 √ √ √
−2 − 5 1 0 ·  2 −1 0   x2  = 0 ⇒ 5x + (2 5 + 5)y + ( 5 + 2) = 0.
1 0 −1 x3

Esempio 183. Sia γ : 4x2 + 4xy + y2 − 2x + 1 = 0 con matrice associata


 
4 2 −1
A =  2 1 0 .
−1 0 1

La matrice é non singolare e A33 = 0, per cui γ é una parabola non ridotta.
Per determinarne l’asse, ricerchiamo inizialmente la sua direzione, ovvero le coordi-
nate del punto impropio della parabola. Lo facciamo intersecando la retta impropria
e la parabola, esprimendo l’equazione di quest’ultima in forma omogenea:

x3 = 0
4x12 + 4x1 x2 + x22 − 2x1 x3 + x32 = 0
ovvero 
x3 = 0
4x12 + 4x1 x2 + x22 = 0.
320 13 Le Coniche.

Ponendo t = xx21 , ci riduciamo a risolvere l’equazione 4t 2 + 4t + 1 = 0, da cui t =


− 12 (come radice doppia). Quindi il punto improprio della parabola ha coordinate
(−1, 2, 0). Il polo P∞ dell’asse individua la direzione ortogonale all’asse stesso, per
cui P∞ = (2, 1, 0) . Infine, l’equazione dell’asse é
  
  4 2 −1 x1
2 1 0 ·  2 1 0   x2  = 0 ⇒ 10x + 5y − 2 = 0.
−1 0 1 x3

13.6 Riduzione delle coniche in forma canonica.

Consideriamo ora l’equazione di una conica

γ : f (x, y) = a11 x2 + 2a12 xy + a22 y2 + 2a13 x + 2a23 y + a33 = 0

dove ai j ∈ R, per ogni i = 1, 2, 3 e j = 1, 2, 3.


Ci proponiamo di determinare un opportuno cambiamento del sistema di riferimen-
to, al fine di poter ottenere la piú semplice espressione possibile dell’equazione che
rappresenta γ. Tale forma é detta canonica e si riferisce al caso in cui:
–gli assi di simmetria della conica (o l’asse, nel caso particolare della parabola)
coincidono con gli assi coordinati;
–il centro della conica (o il vertice, nel caso particolare della parabola) coincide
con il centro del sistema di riferimento.

É infati ben noto che, in tali casi le equazioni di una conica non degenere si riducono
come segue:
2 2
– ax2 + by2 = c, con a, b, c 6= 0, nel caso di una ellisse non degenere;
2 2
– ax2 − by2 = c, con a, b, c 6= 0, nel caso di una iperbole non degenere;
–y2 − ax = 0 oppure x2 − ay = 0, nel caso di una parabola non degenere.

Se indichiamo q(x, y) = a11 x2 +2a12 xy+a22 y2 la forma quadratica in R2 , con matri-


ce associata A33 , allora sappiamo che, per diagonalizzare la forma quadratica q(x, y)
(ovvero per diagonalizzare la matrice simmetrica A33 ad essa associata), dovremo
effettuare un cambiamento del sistema di riferimento individuato da una matrice or-
togonale C. La trasformazione di q(x, y) fornisce una nuova equazione della conica,
in riferimento ad un nuovo sistema di coordinate. Tale nuovo sistema di riferimento
é ottenuto in seguito ad una rotazione degli assi, rappresentata appunto dalla matrice
ortogonale C.
Sia quindi    
  a a x
q(x, y) = x y · 11 12 ·
a12 a22 y
13.6 Riduzione delle coniche in forma canonica. 321

e siano λ1 e λ2 gli autovalori di A33 . Indichiamo con w1 = (b11 , b21 ) l’autovettore


che genera l’autospazio relativo a λ1 e w2 = (b12 , b22 ) quello
 che genera l’autospazio
b11 b12
relativo a λ2 . Allora la matrice che diagonalizza A33 é con
b21 b22
 
λ 0
CT · A33 ·C = C−1 · A33 ·C = 1 .
0 λ2

Il cambiamento di variabili che permette tale diagonalizazione é


     0
x b11 b12 x
= · 0
y b21 b22 y
dopo il quale la conica si presenta nella seguente forma:

f (x0 , y0 ) = λ1 x2 + λ2 y2 + 2a013 x + 2a023 y + a033 = 0

ottenuta in seguito ad una rotazione degli assi della conica.


Per completare la trasformazione dell’equazione della conica nella forma piú sem-
plice possibile, resta ancora da effettuare una traslazione che riporti il centro della
conica (nel caso di ellisse o iperbole) oppure il vertice (nel caso della parabola)
nell’origine degli assi coordinati.

Il caso delle coniche con centro proprio.


Supponiamo che la conica sia un’iperbole o una ellisse.
Determiniamo una traslazione che riporti il centro della conica nel centro degli assi
coordinati:  0
x = x00 − c
y0 = y00 − d
λ1 (x00 − c)2 + λ2 (y00 − d)2 + 2a013 (x00 − c) + 2a023 (y00 − d) + a033 = 0
cioé

λ1 x002 + λ2 y002 + (−2λ1 c + 2a013 )x00 +


(13.2)
(−2λ2 d + 2a023 )y00 + (λ1 c2 + λ2 d 2 + 2a013 c + 2a023 d + a033 ) = 0.

Poiché dopo la rototraslazione scompaiono i termini in x00 e y00 , allora imponiamo


che
−2λ1 c + 2a013 = 0
−2λ2 d + 2a023 = 0.
I valori di c e d che risolvono le precedenti equazioni sono i valori che determinano
la traslazione. É sufficiente sostituirli nell’equazione (13.2) per ottenere la conica in
forma ridotta:
λ1 x002 + λ2 y002 + λ3 = 0
con λ3 = λ1 c2 + λ2 d 2 + 2a013 c + 2a023 d + a033 .
322 13 Le Coniche.

Esempio 184. Sia γ : 2x2 + 2y2 − 6xy + 2x − 1 = 0 con marice associata


 
2 −3 1
A =  −3 2 0  .
1 0 −1

La matrice é non singolare, inoltre A33 = −5 < 0, per cui γ é un’iperbole non
degenere. Gli autovalori della sottomatrice
 
2 −3
−3 2

sono {−1, 5}. Gli autospazi associati sono

1 1
λ1 = −1 ⇒ V1 =< (1, 1) >=< √ , √ >
2 2
1 1
λ2 = 5 ⇒ V2 =< (−1, 1) >=< − √ , √ > .
2 2
Determiniamo adesso una trasfomazione delle coordinate, al fine di eliminare il
termine in xy (é una rotazione degli assi):
  " √1 #  
x 2
− √12 x0
= 1 · 0
y √ √1 y
2 2

ovvero (
x= √1 (x0 − y0 )
2 .
y= √1 (x0 + y0 )
2

Sostituendo nell’equazione iniziale otterremo la nuova equazione


√ √
5y02 − x02 + 2x0 − 2y0 − 1 = 0.

Imponiamo adesso una traslazione

x0 = x00 − c


y0 = y00 − d

al fine di eliminare i termini di primo grado. Sostituendo avremo


√ √ √ √
5y002 − x002 + (2c + 2)x00 − (10d + 2)y00 + (5d 2 − c2 − 2c + 2d − 1) = 0.

Annullando i termini in x e y, segue che



√ 2
2c + 2 = 0 ⇒ c = −
2
13.6 Riduzione delle coniche in forma canonica. 323

√ 2
10d + 2 = 0 ⇒ d = −
10
ed il termine noto dell’equazione diventa
√ √ 3
5d 2 − c2 − 2c + 2d − 1 = − .
5

Concludiamo che l’equazione dell’iperbole in forma ridotta é 5Y 2 − X 2 − 35 = 0.

Il caso della parabola.


Consideriamo ora il caso in cui la conica sia una parabola.
Per prima cosa osserviamo che la prima rotazione viene effettuata utilizzando co-
me riferimento quello formato dall’asse della parabola e dalla retta tangente nel suo
vertice.
Inoltre uno dei due autovalori λ1 , λ2 di A33 é nullo, per cui, dopo il primo cambia-
mento di riferimento (rotazione), l’equazione della conica si presenta in una delle
due seguenti forme:

λ1 x02 + 2a023 y0 + a033 = 0 (13.3)


oppure
λ2 y02 + 2a 130 x0 + a033 = 0. (13.4)
Cominciamo con il caso (13.3). Determiniamo una traslazione che riporti il vertice
della parabola nel centro degli assi coordinati:
 0
x = x00 − c
y0 = y00 − d

l’equazione diventa

λ1 (x00 − c)2 + 2a023 (y00 − d) + a033 = 0 (13.5)

Poiché dopo la rototraslazione scompaiono il termine noto ed il termine in x00 ,


dobbiamo imporre che
2λ1 c = 0
λ1 c2 − 2a023 d + a033 = 0.
I valori di c e d che risolvono le precedenti equazioni sono i valori che determinano
la traslazione. É sufficiente sostituirli nell’equazione (13.5) per ottenere la conica in
forma ridotta:
λ1 x002 + 2a023 y00 = 0.
Passiamo ora al caso (13.4). Dopo la traslazione, l’equazione della parabola é

λ2 (y00 − c)2 + 2a013 (x00 − d) + a033 = 0. (13.6)


324 13 Le Coniche.

Poiché dopo la rototraslazione scompaiono il termine noto ed il termine in y00 ,


dobbiamo imporre che
2λ2 d = 0
λ2 d 2 − 2a013 c + a033 = 0.
I valori di c e d che risolvono le precedenti equazioni sono i valori che determinano
la traslazione. É sufficiente sostituirli nell’equazione (13.6) per ottenere la conica in
forma ridotta:
λ2 y002 + 2a013 x00 = 0.
Esempio 185. Sia γ : 4x2 + y2 + 4xy + 4x − 1 = 0 con marice associata
 
42 2
A =  2 1 0 .
2 0 −1

La matrice é non singolare, inoltre A33 = 0, per cui γ é una parabola non degenere.
Gli autovalori della sottomatrice  
42
21
sono {0, 5}. Gli autospazi associati sono

1 2
λ1 = 0 ⇒ V1 =< (1, −2) >=< √ , − √ >
5 5
2 1
λ2 = 5 ⇒ V2 =< (2, 1) >=< √ , √ > .
5 5
Determiniamo adesso una trasfomazione delle coordinate, al fine di eliminare il
termine in x2 (é una rotazione degli assi):
  " √1 √2 #  0 
x 5 5 · x
=
y − √25 √15 y0

ovvero (
x = √15 (x0 + 2y0 )
.
y = √15 (−2x0 + y0 )
Sostituendo nell’equazione iniziale otterremo la nuova equazione
4 8
5y02 + √ x0 + √ y0 − 1 = 0.
5 5
Imponiamo adesso una traslazione

x0 = x00 − c


y0 = y00 − d
13.6 Riduzione delle coniche in forma canonica. 325

al fine di eliminare il termine di primo grado in y00 ed il termine noto. Sostituendo


avremo
4 8 4 8
5y002 + √ x00 + ( √ − 10d)y00 + (5d 2 − √ c − √ d − 1) = 0.
5 5 5 5
Annullando i termini in y00 ed il termine noto, segue che
8 4
√ − 10d = 0 ⇒ d = √
5 5 5
4 8 41
5d 2 − √ c − √ d − 1 = 0 ⇒ c = − √ .
5 5 20 5
Quindi, la traslazione finale (
x0 = x00 + 2041√5
y0 = y00 − 5√4 5

determina l’equazione della parabola in forma ridotta 5Y 2 + 5√4 5 X = 0.


Quanto detto fin’ora si puó riassumere nel seguente modo (di seguito elenchiamo
anche i casi relativi alle coniche degeneri):
Teorema 13.5. Ogni conica del piano euclideo reale é congruente ad una delle
seguenti forme, dette forme canoniche:
2
x2
1. a2
+ by2 = 1, ellisse;
2
x2
2. a2
+ by2 = −1, ellisse a punti non reali;
2
x2
3. a2
+ by2 = 0, ellisse degenere;
2
x2
4. a2
− by2 = 1, iperbole;
2
x2
5. a2
− by2
= 0, iperbole degenere;
6. y2 − ax
= 0, parabola;
7. y2 − a2 = 0, parabola degenere;
8. y2 + a2 = 0, parabola degenere a punti non reali;
9. y2 = 0, conica doppiamente degenere.

Riassumendo, abbiamo visto che una rototraslazione degli assi di una conica (del-
l’asse e della retta tangente al vertice, nel caso della parabola) ci permette di ottenere
una ulteriore e piú semplice forma della conica stessa, riferita ad un sistema di rife-
rimento opportuno. Tali forme sono dette ’ridotte’ o ’canoniche’ .
Nel cambiamento del sistema di riferimento ortogonale, vi sono alcune quantitá
(reali) che non mutano, cioé si mantengono invarianti nel passaggio da una forma
della conica all’altra. Tali quantitá vengono dette invarianti ortogonali:

Teorema 13.6. Sia γ : X T · A · X = 0 una conica del piano euclideo e sia X 0T · A0 ·


X 0 = 0 la sua equazione in forma ridotta, cioé dopo una cambiamento ortonormale
326 13 Le Coniche.

del sistema di riferimento. Siano ai j gli elementi della matrice A e a0i j quelli della
matrice A0 . Allora valgono le seguenti:
1. det(A) = det(A0 ).
2. A33 = A033 .
3. a11 + a22 = a011 + a022 .
Possiamo sfruttare il precedente Teorema per ottenere la forma ridotta di una conica.
Sia A la matrice associata alla conica, operiamo nel modo seguente:
Coniche a centro.

La forma canonica alla quale si vuole arrivare é la seguente

a011 x12 + a022 x22 + a033 x32 = 0

la cui matrice associata é


a011 0 0
 
 0 a022 0  .
0 0 a033
Per ottenere i valori di a011 , a022 , a033 é sufficiente applicare i tre punti del teorema e
risolvere le equazioni:
det(A) = a011 · a022 · a033
A33 = a011 · a022
a11 + a22 = a011 + a022 .
Parabola.

Una forma canonica alla quale si puó arrivare é la seguente

a011 x12 + 2a023 x2 x3 = 0

la cui matrice associata é


a011 0 0
 
 0 0 a023  .
0 a023 0
Per ottenere i valori di a011 , a023 é sufficiente applicare i tre punti del teorema e
risolvere le equazioni:
det(A) = −a011 · a02
23

a11 + a22 = a011 .


L’altra forma canonica puó essere

a022 x22 + 2a013 x1 x3 = 0

la cui matrice associata é


13.6 Riduzione delle coniche in forma canonica. 327

0 0 a013
 
 0 a022 0  .
a013 0 0
Per ottenere i valori di a022 , a023 é sufficiente applicare i tre punti del teorema e
risolvere le equazioni:
det(A) = −a022 · a02
13

a11 + a22 = a022 .


Esempio 186. Ripetiamo l’Esempio 184, utilizzando il metodo degli invarianti or-
togonali.
Sia γ : 2x2 + 2y2 − 6xy + 2x − 1 = 0 con marice associata
 
2 −3 1
A =  −3 2 0  .
1 0 −1

La matrice é non singolare, inoltre

|A| = 3, A33 = −5, a11 + a22 = 4.

Poiché γ é un’iperbole non degenere, una espressione della sua equazione in forma
canonica ( o ridotta) é
a011 x2 + a022 y2 + a033 = 0
con matrice associata
a011 0 0
 

A0 =  0 a022 0  .
0 0 a033
In tale matrice dovremo avere

|A0 | = 3, A033 = −5, a011 + a022 = 4

da cui
a011 a022 a033 = 3, a011 a022 = −5, a011 + a022 = 4.
Segue che
3
a033 = − , a011 = −1, a022 = 5
5
oppure
3
a033 = − , a011 = 5, a022 = −1.
5
Le possibili forme canoniche (in dipendenza della rototraslazione effettuata) sono
quindi
3
5Y 2 − X 2 − = 0
5
e
328 13 Le Coniche.

3
5X 2 −Y 2 − = 0.
5
Esempio 187. Riprendiamo l’Esempio 185, utilizzando ancora il metodo degli
invarianti ortogonali. Sia γ : 4x2 + y2 + 4xy + 4x − 1 = 0 con marice associata
 
42 2
A =  2 1 0 .
2 0 −1

La matrice é non singolare, inoltre

|A| = −4, A33 = 0, a11 + a22 = 5.

Poiché la conica é unaparabola non degenere, una espressione della sua equazione
in forma canonica é
a022 y2 + 2a013 x = 0
con matrice associata
0 0 a013
 

A0 =  0 a022 0  .
a013 0 0
In tale matrice dovremo avere

|A0 | = −4, a011 + a022 = 5

da cui
−a02 0
13 a22 = −4, a022 = 5.
Segue che
2
a013 = √
5
oppure
2
a013 = − √
5
Le possibili forme canoniche (in dipendenza della rototraslazione effettuata) sono
quindi
4
5Y 2 + √ X = 0
5
e
4
5Y 2 − √ X = 0.
5
Osserviamo infine che, se inizialmente avessimo scelto di considerare una forma
canonica della parabola del tipo

a011 x2 + 2a023 y = 0
13.8 Esercizi svolti. 329

con matrice associata


a011 0 0
 

A0 =  0 0 a023 
0 a023 0
avremmo ottenuto i medesimi risultati, ottenendo le espressioni finali
4
5X 2 + √ Y = 0
5
e
4
5X 2 − √ Y = 0.
5

13.7 Fuochi di una conica.

Siano C1 = (1, i, 0) e C2 = (1, −i, 0) i punti ciclici della retta impropria. Le rette
appartenenti ai due fasci di centri C1 e C2 , sono dette rette isotrope. Quindi una
qualsiasi retta isotropa avrá equazione y = ix + q oppure y = −ix + p, per opportuni
q, p elementi del campo dei numeri complessi.
Indichiamo con f1 : y = ix + q il fascio delle rette isotrope di centro C1 e con f2 : y =
−ix + p il fascio di centro C2 . Osserviamo che se una retta r appartiene al fascio f1
allora la sua coniugata r appartiene al fascio f2 . Inoltre r ∩ r = P é un punto reale.
Consideriamo quindi una conica non riducibile γ. Dal fascio f1 si possono condurre
due rette r,t che siano tangenti a γ. Analogamente dal fascio f2 si possono condurre
due rette che siano tangenti a γ: esse sono esattamente le coniugate r e t delle rette
r,t.
Definizione 13.7. I fuochi di una conica non riducibile γ sono i 4 punti (di cui 2
reali) che scaturiscono dall’intersezione delle rette r,t con le rette r e t. In particolare
i due fuochi reali sono F1 = r ∩ r e F2 = t ∩ t.
Osservazione 13.8. La parabola ha un solo fuoco reale, poiché da ciascun punto
ciclico si puó condurre una sola retta tangente alla curva.
Anche nel caso della circonferena vi é un solo fuoco, esattamente il centro della
circonferenza: il motivo risiede nel fatto che, poiché i punti ciclici appartengono
alla circonferenza, anche in questo caso da ciascuno di essi si puó condurre una sola
tangente alla curva.

13.8 Esercizi svolti.

Esercizio 100. Classificare la conica x2 + 4xy − y2 + x − y + 1 = 0, determinarne


eventuali assi e asintoti, una sua forma canonica ed il polo della retta 2x − y + 1 = 0
rispetto ad essa.
330 13 Le Coniche.

Svolgimento: La matrice associata alla conica é

1 2 12
 

A =  2 −1 − 12 
1 1
2 −2 1

con det(A) = −6, A33 = −5, I = a11 + a22 = 0, quindi é una iperbole equilatera.
Determiniamo il generico diametro:
1
   
  1 2 21 x1
1 h 0 ·  2 −1 − 2  ·  x2  = 0.
1 1
2 −2 1
x3

Il generico diametro é

1 h
(1 + 2h)x1 + (2 − h)x2 + ( − )x3 = 0
2 2

il cui coefficiente angolare é h0 = 1+2h


h−2 . √ √
Per ottenere gli asintoti imponiamo h = h0 , da cui h ∈ {2 + 5, 2 − 5} e quindi gli
asintoti sono √ √ √
(10 + 4 5)x − 2 5y + (− 5 − 1) = 0
e √ √ √
(10 − 4 5)x + 2 5y + ( 5 − 1) = 0.
√ √
Per ottenere gli assi imponiamo hh0 = −1, da cui h ∈ { −1+2 5 , −1−2 5
} e quindi gli
assi sono √ √ √
4 5x + (10 − 2 5)y + (3 − 5) = 0
e √ √ √
−4 5x + (10 + 2 5)y + (3 + 5) = 0.
Una forma canonica é data da a11 x12 + a22 x22 + a33 x32 = 0, con matrice associata
 
a11 0 0
A0 =  0 a22 0  .
0 0 a33

I suoi invarianti ortogonali sono allora

−6 = det(A0 ) = a11 a22 a33 , −5 = A033 = a11 a22 , 0 = I = a11 + a22

dai quali otteniamo


√ √ 6
a11 = 5, a22 = − 5, a33 =
5
oppure
13.8 Esercizi svolti. 331
√ √ 6
a11 = − 5, a22 = 5, a33 =
5
e le due forme canoniche ottenute sono:
√ 2 √ 2 6
5x − 5y + = 0
5
e
√ √ 6
− 5x2 + 5y2 + = 0.
5
Infine determiniamo il polo della retta 2x1 − x2 + x3 = 0 rispetto alla conica. Esso
avrá coordinate (a, b, c) tali che
1
   
  1 2 21 x1
a b c ·  2 −1 − 2  ·  x2  = α(2x1 − x2 + x3 ).
1 1
2 −2 1
x3

Quindi dobbiamo risolvere il sistema



 2a + 4b + c = 4α
4a − 2b − c = −2α
a − b + 2c = 2α

le cui soluzioni sono


1 5 5
(α , α , α )
8 8 4
che é una terna proporzionale a (1, 5, 10) = (a, b, c).
Esercizio 101. Determinare l’asse ed una forma canonica della parabola 2x2 +2y2 −
4xy + 2x − 1 = 0.

Svolgimento: La matrice associeginata alla conica é


 
2 −2 1
A =  −2 2 0 
1 0 −1

con det(A) = −2, A33 = 0, I = a11 + a22 = 4.


Determiniamo il punto improprio della parabola:
 2
2x1 + 2x22 − 4x1 x2 = 0
x3 = 0

da cui x1 = 1, x2 = 1, x3 = 0 sono le coordinate di tale punto. La direzione ad esso


ortogonale é Q = (1, −1, 0). Tale punto Q é il polo dell’asse:
332 13 Le Coniche.
   
  2 −2 1 x1
1 −1 0 ·  −2 2 0  ·  x2  = 0
1 0 −1 x3

4x1 − 4x2 + x3 = 0.
Una forma canonica é data da a11 x12 + 2a23 x2 x3 = 0, con matrice associata
 
a11 0 0
A0 =  0 0 a23  .
0 a23 0

I suoi invarianti ortogonali sono allora

−2 = det(A0 ) = −a11 a223 , 4 = I = a11 + a22

dai quali otteniamo


1
a11 = 4, a23 = √
2
oppure
1
a11 = 4, a23 = − √
2
e le due forme canoniche ottenute sono:
2
4x12 + √ x2 x3 = 0
2
e
2
4x12 − √ x2 x3 = 0
2
cioé √ √
y = −2 2x2 e y = 2 2x2 .

Esercizio 102. Determinare una forma canonica dell’ellisse x2 −xy+y2 −5x +7y+
1 = 0.

Svolgimento: Utilizziamo ilmetodo degli  autovalori. Quindi determiniamo gli au-


1 − 12
tovalori della matrice A33 = . Essi sono h1 = 21 e h2 = 32 . L’autospazio
− 12 1
relativo all’autovalore h1 é generato dall’autovettore v = (1, 1) che ha come versore
( √12 , √12 ).
L’autospazio relativo all’autovalore h2 é generato dall’autovettore v = (−1, 1) che
ha come versore (− √12 , √12 ).
Allora il primo cambiamento di variabili, con il quale annulliamo il termine in xy é
dato da
13.8 Esercizi svolti. 333
  " √1 #  
x 2
− √12 x0
= 1 1 · 0
y √ √ y
2 2

da cui (
x= √1 (x0 − y0 )
2
y= √1 (x0 + y0 )
2

e l’equazione della conica diventa


1 0 1 1 5 7
(x − y0 )2 − (x0 − y0 )(x0 + y0 ) + (x0 + y0 )2 − √ (x0 − y0 ) + √ (x0 + y0 ) + 1 = 0
2 2 2 2 2
cioé
1 02 3 02 2 12
x + y + √ x0 + √ y0 + 1 = 0.
2 2 2 2
Per annullare i termini in x0 e y0 operiamo la seguente traslazione:

x0 = x00 − a y0 = y00 − b
1 00 3 2 12
(x − a)2 + (y00 − b)2 + √ (x00 − a) + √ (y00 − b) + 1 = 0
2 2 2 2
da cui
1 002 3 2 2 12 12
(x + a2 − 2ax00 ) + (y002 + b2 − 2by00 ) + √ x00 − √ a + √ y00 − √ b + 1 = 0.
2 2 2 2 2 2

I coefficienti di x00 e y00 si annullano per a = √2


2
eb= √4
2
e l’equazione della conica
diventa:
1 00 2 3 4 2 2 12 4
(x − √ )2 + (y00 − √ )2 + √ (x00 − √ ) + √ (y00 − √ ) + 1 = 0
2 2 2 2 2 2 2 2
cioé
1 002 3 002
x + y = 12
2 2
x002 y002
+ = 1.
24 8
Vogliamo ora ripetere l’esercizio utilizzando il metodo di rototraslazione degli assi:
gli assi dell’ellisse sono

a1 : x−y−4 = 0 che scegliamo come X0

a2 : x+y+2 = 0 che scegliamo come Y 0 .


I versori degli assi sono
1 1 1 1
a1 = ( √ , √ ) a2 = (− √ , √ )
2 2 2 2
334 13 Le Coniche.

ed il centro dell’ellisse é C = (1, −3). Allora le formule del cambiamento del


riferimento (rototraslazione) sono
  " √1 √1
#   
− x0

x 1
= 12 1 2 · 0 +
y √ √ y −3
2 2

da cui (
x= √1 (x0 − y0 ) + 1
2
y= √1 (x0 + y0 ) − 3
2

e l’equazione della conica diventa


1 1 1 1
( √ (x0 − y0 ) + 1)2 − ( √ (x0 − y0 ) + 1)( √ (x0 + y0 ) − 3) + ( √ (x0 + y0 ) − 3)2
2 2 2 2
1 1
−5( √ (x0 − y0 ) + 1) + 7( √ (x0 + y0 ) − 3) + 1 = 0
2 2
cioé
1 02 3 02
x + y − 12 = 0
2 2
x02 y02
+ = 1.
24 8
Esercizio 103. Determinare una forma canonica della parabola x2 − 4xy + 4y2 +
2x + y − 5 = 0.

Svolgimento: Utilizziamo il metodo degli autovalori. Quindi determiniamo gli au-


1 −2
tovalori della matrice A33 = . Essi sono h1 = 0 e h2 = 5. L’autospazio
−2 4
relativo all’autovalore h1 é generato dall’autovettore v = (2, 1) che ha come versore
( √25 , √15 ).
L’autospazio relativo all’autovalore h2 é generato dall’autovettore v = (1, −2) che
ha come versore ( √15 , − √25 ).
Allora il primo cambiamento di variabili, con il quale annulliamo i termini in xy e y
é dato da   " √2 #  
x 5
− √15 x0
= 1 · 0
y √ √2 y
5 5

da cui (
x= √1 (2x0 − y0 )
5
y= √1 (x0 + 2y0 )
5

e l’equazione della conica diventa


1 0 0 2 4 0 0 0 4 1 1
(2x −y ) − (2x −y )(x +2y0 )+ (x0 +2y0 )2 + √ (2x0 −y0 )+ √ (x0 +2y0 )−5 = 0
5 5 5 5 5
13.8 Esercizi svolti. 335

cioé
5
5y02 + √ x0 − 5 = 0
5
1
y02 + √ x0 − 1 = 0.
5
Per annullare il termine noto operiamo la seguente traslazione:

x0 = x00 − a y0 = y00 − b
1
(y00 − b)2 + √ (x00 − a) − 1 = 0
5
1 1
y002 + b2 − 2by00 + √ x00 − √ a − 1 = 0.
5 5
Per avere la forma canonica dovremmo avere

b=0 a=− 5

da cui
1 1 √
(y00 − b)2 + √ (x00 − a) − 1 = y002 + √ (x00 + 5) − 1 =
5 5
1
y002 + √ x00 = 0.
5
Esercizio 104. Sia dato il fascio di coniche x2 + kxy − 1 = 0, al variare di k ∈ R.
Determinare i punti base, le coniche riducibili e le tangenti comuni a tutte le coniche
del fascio.

Svolgimento: In coordinate omogenee l’equazione del fascio é data da

x12 + kx1 x2 − x32 = 0.

Per determinare i punti base del fascio scegliamo due coniche qualsiasi, per esempio
x12 − x32 = 0 e x1 x2 = 0 ed intersechiamole:

x1 x2 = 0
x12 − x32 = 0

che ha come soluzione i punti (0, 1, 0) con molteplicitá 2, (1, 0, 1) e (1, 0, −1) en-
trambi con molteplicitá 1. Siamo in presenza di un fascio di coniche tangenti, cioé vi
é una tangente comune a tutte le coniche del fascio. Per determinarla, scegliamo una
conica irriducibile del fascio, per esempio per k = 2, e calcoliamo la tangente a tale
conica nel punto (0, 1, 0): per il valore k = 2, la conica ottenuta é x2 + 2xy − 1 = 0.
La tangente ad essa in (0, 1, 0) é
336 13 Le Coniche.
   
  11 0 x1
0 1 0 ·  1 0 0  ·  x2  = 0 cioé x1 = 0.
0 0 −1 x3
Infine calcoliamo le coniche riducibili del fascio: esse si ottengono annullando il
determinante della matrice associata alla generica conica del fascio:
k
1
2 0 k2

0 = 2k 0 0 =
0 0 −1 4

quindi per k = 0, con molteplicitá 2, si ottiene la conica riducibile del fascio x2 −1 =


0. La terza conica riducibile si ottiene osservando che il parametro k é associato ad
essa nell’espressione del fascio: xy = 0, con molteplicitá 1.
Esercizio 105. Sia dato il fascio di coniche kx2 + 2xy − k = 0, al variare di k ∈ R.
Determinare i punti base, le coniche riducibili e le tangenti comuni a tutte le coniche
del fascio.

Svolgimento: In coordinate omogenee l’equazione del fascio é data da

kx12 + 2x1 x2 − kx32 = 0.

Per determinare i punti base del fascio scegliamo due coniche qualsiasi, per esempio
x12 − x32 = 0 e 2x1 x2 = 0 ed intersechiamole:

2x1 x2 = 0
x12 − x32 = 0

che ha come soluzione i punti (0, 1, 0) con molteplicitá 2, (1, 0, 1) e (1, 0, −1) en-
trambi con molteplicitá 1. Siamo in presenza di un fascio di coniche tangenti, cioé vi
é una tangente comune a tutte le coniche del fascio. Per determinarla, scegliamo una
conica irriducibile del fascio, per esempio per k = 2, e calcoliamo la tangente a tale
conica nel punto (0, 1, 0): per il valore k = 2, la conica ottenuta é 2x2 + 2xy − 2 = 0.
La tangente ad essa in (0, 1, 0) é
   
  21 0 x1
0 1 0 ·  1 0 0  ·  x2  = 0 cioé x1 = 0.
0 0 −2 x3
Infine calcoliamo le coniche riducibili del fascio: esse si ottengono annullando il
determinante della matrice associata alla generica conica del fascio:

k 1 0

0 = 1 0 0 = k
0 0 −k

quindi per k = 0, con molteplicitá 1, si ottiene la conica riducibile del fascio x1 x2 =


13.8 Esercizi svolti. 337

0. L’altra conica riducibile (il fascio ne contiene solo 2 distinte) si ottiene osservando
che il parametro k é associato ad essa nell’espressione del fascio: x2 − 1 = 0, con
molteplicitá 2.
Esercizio 106. Sia dato il fascio di coniche x2 +ky2 +xy −1 = 0, al variare di k ∈ R.
Determinare i punti base, le coniche riducibili e le tangenti comuni a tutte le coniche
del fascio.

Svolgimento: In coordinate omogenee l’equazione del fascio é data da

x12 + kx22 + x1 x2 − x32 = 0.

Per determinare i punti base del fascio scegliamo due coniche qualsiasi, per esempio
x12 + x1 x2 − x32 = 0 e x22 = 0 ed intersechiamole:

x22 = 0


x1 + x1 x2 − x32 = 0
2

che ha come soluzione i punti (1, 0, 1) e (1, 0, −1) entrambi con molteplicitá 2. Sia-
mo in presenza di un fascio di coniche bitangenti, cioé vi sono due tangenti comuni
a tutte le coniche del fascio. Per determinarle, scegliamo una conica irriducibile
del fascio, per esempio per k = 1, e calcoliamo la tangente a tale conica nei punti
(1, 0, 1) e (1, 0, −1): per il valore k = 1, la conica ottenuta é x2 + y2 + xy − 1 = 0. La
tangente ad essa in (1, 0, 1) é
 1   
  11 2 0 x1
1
1 0 1 ·  2 1 0  ·  x2  = 0 cioé x1 + x2 − x3 = 0.
2
0 0 −1 x3
La tangente in (1, 0, −1) é
 1   
  11 2 0 x1
1
1 0 −1 ·  2 1 0  ·  x2  = 0 cioé x1 + x2 + x3 = 0.
2
0 0 −1 x3
Infine calcoliamo le coniche riducibili del fascio: esse si ottengono annullando il
determinante della matrice associata alla generica conica del fascio:
1
1
1 2 0

1
0 = 2 k 0 = −k +

0 0 −1 4

quindi per k = 14 , con molteplicitá 1, si ottiene la conica riducibile del fascio


x1 + 41 x22 + x1 x2 − x32 = 0. Le altre coniche riducibili si ottengono osservando che il
parametro k é associato ad esse nell’espressione del fascio: y2 = 0, con molteplicitá
2 (infatti la retta y = 0 passa per i due base che hanno molteplicitá 2).
Esercizio 107. Determinare il fuoco della parabola γ : 2x2 − y + x + 1 = 0.
338 13 Le Coniche.

Svolgimento: Consideriamo il fascio di rette isotrope di centro C1 = (1, i, 0): f1 :


y = ix+q. Costruiamo il sistema che determini l’intersezione tra il fascio e la conica:
 2
2x − y + x + 1 = 0
y = ix + q

2x2 − ix − q + x + 1 = 0


y = ix + q

2x2 + (1 − i)x + (1 − q) = 0

.
y = ix + q
Poiché siamo interessati alla retta del fascio che sia tangente alla parabola, imponia-
mo che il discriminante del sistema sia nullo:

(1 − i)2 − 8(1 − q) = 0

la cui soluzione é q = 41 i + 1. Quindi la retta del fascio f1 che sia tangente alla para-
bola ha equazione y = ix + 14 i + 1. Inoltre la retta che appartenga al fascio f2 di cen-
tro C2 = (1, −i, 0) e che sia tangente alla parabola, é la coniugata della precedente :
y = −ix − 14 i + 1. Il fuoco della parabola é l’intersezione di tali due rette:

r : y = ix + 14 i + 1


r : y = −ix − 14 i + 1

da cui F = r ∩ r = (− 14 , 1).
Esercizio 108. Determinare i fuochi dell’iperbole γ : xy + x − y + 1 = 0.

Svolgimento: Consideriamo il fascio di rette isotrope di centro C1 = (1, i, 0): f1 :


y = ix+q. Costruiamo il sistema che determini l’intersezione tra il fascio e la conica:

xy + x − y + 1 = 0
y = ix + q

x(ix + q) + x − (ix + q) + 1 = 0
y = ix + q
 2
ix + (q − i + 1)x + (1 − q) = 0
.
y = ix + q
Poiché siamo interessati alle rette del fascio che siano tangenti all’iperbole, impo-
niamo che il discriminante del sistema sia nullo:

(q − i + 1)2 − 4i(1 − q) = 0

le cui soluzioni sono


13.8 Esercizi svolti. 339
√ √
q = −1 − i + 2 2i q = −1 − i − 2 2i.

Quindi le rette del fascio f1 che siano tangenti all’iperbole hanno equazioni:
√ √
y = ix − 1 − i + 2 2i y = ix − 1 − i − 2 2i.
√ p
Osserviamo che 2i = (1 + i)2 ∈ {+(1 + i), −(1 + i)} per cui possiamo conside-
rare le tangenti

r : y = ix − 1 − i + 2(1 + i) t : y = ix − 1 − i − 2(1 + i).

Inoltre le rette appartenenti al fascio f2 di centro C2 = (1, −i, 0) e che siano tangenti
all’iperbole, sono le coniugate delle precedenti :

r : y = −ix − 1 + i + 2(1 − i) t : y = −ix − 1 + i − 2(1 − i).

I fuochi sono le intersezioni di tali coppie di rette, in particolare i fuochi reali


scaturiscono da: 
r : y = ix − 1 − i + 2(1 + i)
r : y = −ix − 1 + i + 2(1 − i)
da cui F1 = r ∩ r = (−1, 1) e da

t : y = ix − 1 − i − 2(1 + i)
t : y = −ix − 1 + i − 2(1 − i)

da cui F2 = t ∩ t = (3, −3).


Capitolo 14
Rette e piani nello spazio affine ed euclideo.

In tutto ció che segue con il termine sistema di riferimento nello spazio 3-dimensionale
intenderemo sempre un sistema di riferimento destrorso (standard).

14.1 Equazioni di un piano.

Sia π un piano nello spazio affine A3 . Tale piano é univocamente determinato da


un punto P0 = (x0 , y0 , z0 ) e da un vettore v0 = (l0 , m0 , n0 ) (non contenente P0 ) che
appartengano al piano stesso. Per descrivere il piano π, ci proponiamo quindi di
stabilire le relazioni che debbano essere soddisfatte dalle coordinate di un qualsiasi
punto P = (x, y, x) del piano, in funzione della scelta di P0 e v0 .
Consideriamo inizialmente due vettori v1 = (l1 , m1 , n1 ) e v2 = (l2 , m2 , n2 ), tra loro
linearmente indipendenti ed appartenenti al piano. Tali vettori generano ogni altro
vettore del piano, ed in particolare esistono t,t 0 ∈ R tali che v0 = tv1 +t 0 v2 . Al variare
di P ∈ π, il vettore P0 P = (x − x0 , y − y0 , z − z0 ) dipende allora da P0 e da v0 , ovvero
dipende da P0 , v1 e v2 :
 x − x0 = tl1 + t 0 l2

y − y0 = tm1 + t 0 m2
z − z0 = tn1 + t 0 n2

da cui
 x = x0 + tl1 + t 0 l2

y = y0 + tm1 + t 0 m2 .
z = z0 + tn1 + t 0 n2

Queste ultime relazioni descrivono quindi in modo univoco in quale modo ottenere
le coordinate di un qualsiasi punto del piano, al variare dei parametri reali t,t 0 . Esse
sono dette equazioni parametriche del piano. Dalla dipendenza lineare tra i vettori
(x − x0 , y − y0 , z − z0 ), (l1 , m1 , n1 ) e (l2 , m2 , n2 ) segue che

341
342 14 Rette e piani nello spazio affine ed euclideo.

x − x0 y − y0 z − z0

l1
m1 n1 = 0.
l2 m2 n2

Svolgendo il precedente determinante si ottiene una equazione lineare nelle indeter-


minate x, y, z, riordinando la quale avremo

ax + by + cz + d = 0

che é detta equazione cartesiana del piano.

Supponiamo ora di avere tre punti non allineati nello spazio P0 = (x0 , y0 , z0 ), P1 =
(x1 , y1 , z1 ), P2 = (x2 , y2 , z2 ). Essi individuano un piano, in particolare i vettori P0 P1
e P0 P2 sono tra loro indipendenti ed hanno componenti

P0 P1 = (x1 − x0 , y1 − y0 , z1 − z0 )

P0 P2 = (x2 − x0 , y2 − y0 , z2 − z0 ).
Per ogni altro punto P = (x, y, z) del piano, il vettore P0 P = (x − x0 , y − y0 , z − z0 ) é
dipendente dai precedenti due vettori, cioé

x − x0 y − y0 z − z0

x1 − x0 y1 − y0 z1 − z0 = 0

x2 − x0 y2 − y0 z2 − z0

che é equivalente alla


x
y z 1
x0 y0 z0 1
=0
x1
y1 z1 1
x2 y2 z2 1
che ci riporta ancora alla forma affine dell’equazione del piano.
Esempio 188. Se consideriamo il punto P0 = (1, 2, 1) ed i vettori v1 = (1, 1, 0), v2 =
(−1, 0, −2) avremo le seguenti equazioni in forma parametrica del piano contenente
P0 , v1 , v2 :
 x = 1 + t − t0

y = 2+t
z = 1 − 2t 0

ed in forma cartesiana:
x−1 y−2 z−1

1
1 0 = 0
−1 0 −2
ovvero 2x − 2y − z + 3 = 0.
Esempio 189. Se consideriamo i punti P0 = (1, 2, 1), P1 = (−1, −1, 2) e P2 =
(3, 1, 0) otteniamo l’equazione in forma cartesiana del piano contenente i tre punti
14.3 Fascio di piani. 343

svolgendo:
x y
z 1
1 2 1 1

−1 −1 =0
2 1
3 1 0 1
ovvero x + 2z − 3 = 0.

14.2 Reciproca posizione di due piani.

Siano π : ax + by + cz + d = 0 e π 0 : a0 x + b0 y + c0 z + d 0 = 0 due piani. I punti in


comune ai due piani sono le soluzioni del sistema lineare

ax + by + cz + d = 0
a0 x + b0 y + c0 z + d 0 = 0

nelle incognite x, y, z. Le matrici associate al sistema sono


   
a b c a b c d
A= 0 0 0 , C= 0 0 0 0 .
a b c a b c d
Se rango(A) = rango(C) = 1, allora i due piani sono coincidenti poiché ax + by +
cz + d = α(a0 x + b0 y + c0 z + d 0 ), per un opportuno α ∈ R.
Se rango(A) = rango(C) = 2, allora il sistema ammette ∞1 soluzioni, cioé i due
piani hanno ∞1 punti in comune e quindi sono incidenti in una retta.
Se rango(A) = 1 e rango(C) = 2, allora il sistema é incompatibile ed i due piani
non hanno punti in comune, cioé sono paralleli. Ció si verifica quando

a b c d
= 0 = 0 6= 0
a0 b c d

14.3 Fascio di piani.

Siano π : ax + by + cz + d = 0 e π 0 : a0 x + b0 y + c0 z + d 0 = 0 due piani distinti. La


totalitá dei piani di equazione

λ (ax + by + cz + d) + ρ(a0 x + b0 y + c0 z + d 0 ) = 0

al variare dei parametri reali λ e ρ, é detta fascio di piani. Si possono verificare due
casi: π e π 0 sono incidenti in una retta, ed allora tutti i piani del fascio hanno in
comune quella retta, si parla di fascio proprio. Oppure π e π 0 sono tra loro paralleli,
ed allora tutti i piani del fascio sono tra loro paralleli, si parla di fascio improprio.
Si noti che che un fascio di piani puó essere generato da una qualsiasi coppia di
piani ad esso appartenenti.
344 14 Rette e piani nello spazio affine ed euclideo.

Supponiamo allora di avere un terzo piano π 00 : a00 x + b00 y + c00 z + d 00 = 0 ed analiz-


ziamo in quale casi esso appartiene al fascio individuato da π e π 0 . In pratica si deve
studiare il sistema

 ax + by + cz + d = 0
a0 x + b0 y + c0 z + d 0 = 0
 00
a x + b00 y + c00 z + d 00 = 0
nelle incognite x, y, z. Le matrici associate al sistema sono
   
a b c a b c d
A =  a0 b0 c0  , C =  a0 b0 c0 d 0  .
00
a b c00 00 a00 b00 c00 d 00
Se rango(A) = rango(C) = 2, allora il sistema ammette ∞1 soluzioni, cioé i tre piani
hanno ∞1 punti in comune e quindi sono incidenti in una retta, detta asse del fascio:
essi appartengono ad un fascio proprio.
Se rango(A) = 1 e rango(C) = 2, allora il sistema é incompatibile ed i tre piani
non hanno punti in comune, cioé sono paralleli: essi appartengono ad un fascio
improprio.
Esempio 190. I piani di equazioni 2x + 3y − z + 1 = 0 e 4x + 6y − 2z + 5 = 0 sono
paralleli. L’equazione del fascio improprio da essi generato é

α(2x + 3y − z + 1) + β (4x + 6y − 2z + 5) = 0, α, β ∈ R.

Il piano 10x + 15y − 5z + 11 = 0 appartiene al fascio, infatti le matrici


   
2 3 −1 2 3 −1 1
A =  4 6 −2  , C =  4 6 −2 5 
10 15 −5 10 15 −5 11

hanno rispettivamente rango 1 e rango 2.


Il piano 6x + 9y + 3z − 1 = 0 non appartiene al fascio, infatti la matrice
 
2 3 −1 1
C =  4 6 −2 5 
6 9 3 −1

ha rango 3.
Esempio 191. I piani di equazioni x − y + 2z + 1 = 0 e 2x − y + z − 2 = 0 non sono
paralleli. L’equazione del fascio proprio da essi generato é

α(x − y + 2z + 1) + β (2x − y + z − 2) = 0, α, β ∈ R.

Il piano 4x − 3y + 5z = 0 appartiene al fascio, infatti le matrici


14.4 Stella di piani. 345
   
1 −1 2 1 −1 2 1
A =  2 −1 1 , C =  2 −1 1 −2 
4 −3 5 4 −3 5 0

hanno entrambe rango 2.


Il piano 3x − 2y + 5z + 2 = 0 non appartiene al fascio, infatti la matrice
 
1 −1 2 1
C =  2 −1 1 −2 
3 −2 5 2

ha rango 3.

14.4 Stella di piani.

Siano π : ax + by + cz + d = 0, π 0 : a0 x + b0 y + c0 z + d 0 = 0 e π 00 : a00 x + b00 y + c00 z +


d 00 = 0 tre piani. Consideriamo il sistema

 ax + by + cz + d = 0
a0 x + b0 y + c0 z + d 0 = 0 (14.1)
 00
a x + b00 y + c00 z + d 00 = 0

nelle incognite x, y, z. Le matrici associate al sistema sono


   
a b c a b c d
A =  a0 b0 c0  , C =  a0 b0 c0 d 0  .
00
a b c00 00 a00 b00 c00 d 00

Abbiamo giá visto che:


1. se rango(A) = rango(C) = 1 allora i tre piani coincidono tra loro;
2. se rango(A) = rango(C) = 2 allora i tre piani formano un fascio proprio;
3. se rango(A) = 1 e rango(C) = 2 allora i tre piani formano un fascio improprio.
Poniamoci ora nel caso in cui i tre piani siano distinti e non appartengano ad uno
stesso fascio. Quindi dovremo avere necessariamente rango(C) = 3. Consideriamo
l’equazione ottenuta combinando linearmente i polinomi che rappresentano i tre
piani:

α(ax + by + cz + d) + β (a0 x + b0 y + c0 z + d 0 ) + γ(a00 x + b00 y + c00 z + d 00 ) = 0 (14.2)

al variare dei parametri α, β , γ ∈ R.


Analizziamo adesso le varie possibilitá per i ranghi delle sopraccitate matrici:
–Se anche rango(A) = 3, allora il sistema (14.1) ammette una sola soluzione, cioé i
tre piani hanno in comune un punto P0 . In tal caso l’equazione (14.2) rappresenta
346 14 Rette e piani nello spazio affine ed euclideo.

un insieme infinito di piani che hanno tutti in comune il solo punto P0 . Tale
insieme é detto stella propria di piani, generata dai piani π, π 0 , π 00 .
–Se rango(A) = 2, allora il sistema (14.1) é incompatibile, i tre piani non hanno
punti in comune. Ció accade banalmente se uno dei tre piani é parallelo alla retta
che gli altri due hanno in comune. In tal caso l’equazione (14.2) rappresenta un
insieme infinito di piani che sono tutti paralleli ad una stessa retta. Tale insieme
é detto stella impropria di piani, generata dai piani π, π 0 , π 00 . La direzione a cui
sono paralleli i piani si puó individuare intersecando due qualsiasi piani della
stella impropria.
É bene sottolineare che una stella di piani puó essere generata da una qualsiasi terna
di piani ad essa appartenenti.
Supponiamo di avere un quarto piano π 000 : a000 x + b000 y + c000 z + d 000 = 0. Esso ap-
partiene alla stella formata dai piani π, π 0 e π 00 (propria o impropria che sia) se la
matrice  
a b c d
 a0 b0 c0 d 0 
 00 00 00 00 
a b c d 
a000 b000 c000 d 000
ha rango 3.
Esempio 192. I piani di equazioni x − y + z − 1 = 0, 2x + z − 2 = 0 e x − 2y − z = 0
generano una stella propria, infatti le matrici
   
1 −1 1 1 −1 1 −1
A =  2 0 1 , C =  2 0 1 −2 
1 −2 −1 1 −2 −1 0

hanno entrambe rango 3. L’equazione che rappresenta la stella é

α(x − y + z − 1) + β (2x + z − 2) + γ(x − 2y − z) = 0, α, β , γ ∈ R.

Per ottenere le coordinate del centro della stella é sufficiente risolvere il sistema
lineare 
x−y+z−1 = 0
2x + z − 2 = 0
x − 2y − z = 0

la cui soluzione é C = ( 54 , 51 , 25 ).
Esempio 193. I piani di equazioni x + y + z + 1 = 0, 2x − y + z + 2 = 0 e x + 4y +
2z − 1 = 0 generano una stella impropria, infatti le matrici
   
1 1 1 1 1 1 1
A =  2 −1 1  , C =  2 −1 1 2 
1 4 2 1 4 2 −1
14.5 Equazioni di una retta. 347

hanno rispettivamente rango 2 erango 3. entrambe rango 3. L’equazione che rappre-


senta la stella é

α(x + y + z + 1) + β (2x − y + z + 2) + γ(x + 4y + 2z − 1) = 0, α, β , γ ∈ R.

14.5 Equazioni di una retta.

Definiamo retta nello spazio affine A3 , l’insieme di tutti i punti comuni a due piani
non paralleli, quindi la rappresentiamo con le equazioni:

ax + by + cz + d = 0
.
a0 x + b0 y + c0 z + d 0 = 0

Ogni retta dello spazio puó essere individuata da un suo punto P0 = (x0 , y0 , z0 ) e da
un vettore v = (l, m, n) ad essa parallelo. Quindi ogni punto P = (x, y, z) della retta é
tale che il vettore P0 P sia proporzionale al vettore v cioé P0 P = tv, per un opportuno
parametro t dipendente dalla scelta di P sulla retta. Al variare del parametro t si
ottengono tutti i punti della retta:

 x = x0 + tl
y = y0 + tm
z = z0 + tn

che sono dette equazioni parametriche della retta. La terna (l, m, n) é detta terna di
parametri direttori della retta. In particolare se si conoscono due punti della ret-
ta P1 = (x1 , y1 , z1 ) e P2 = (x2 , y2 , z2 ), il vettore P1 P2 é parallelo alla retta e quindi
(l, m, n) = (x2 − x1 , y2 − y1 , z2 − z1 ) e l’equazione si puó ottenere nel modo seguente:

 x = x1 + t(x2 − x1 )
y = y1 + t(y2 − y1 )
z = z1 + t(z2 − z1 )

da cui
x − x1 y − y1 z − z1
t= = =
x2 − x1 y2 − y1 z2 − z1
che é detta equazione a catena di una retta.
Due rette sono parallele se e solo se esse hanno i tre parametri direttori proporzionali
(in particolare identici).
Esempio 194. Dati il punto P0 di coordinate (3, 1, 2) ed il vettore v = (−1, 3, −1),
possiamo descrivere immediatamente le equazioni parametriche della retta conte-
nente P0 e parallela a v: 
 x = 3−t
y = 1 + 3t
z = 2−t

348 14 Rette e piani nello spazio affine ed euclideo.

da cui
y−1
t = 3−x = = 2 − z.
3
Una espressione cartesiana della retta é allora la seguente:

x−z−1 = 0
.
3x + y − 10 = 0

La retta é quindi l’asse del fascio di piani rappresentato dall’equazione

α(x − z − 1) + β (3x + y − 10) = 0, α, β ∈ R.

14.6 Rette complanari.

Siano date le due rette



ax + by + cz + d = 0
r:
a0 x + b0 y + c0 z + d 0 = 0

di parametri direttori (l, m, n) e


 00
a x + b00 y + c00 z + d 00 = 0
r0 :
a000 x + b000 y + c000 z + d 000 = 0

di parametri direttori (l 0 , m0 , n0 ).
Diremo che r e r0 sono complanari se esiste un piano che le contenga entrambe. In
tal caso, le due rette possono essere incidenti in un punto, cioé i quattro piani che le
formano appartengono ad un stella propria, oppure le due rette sono parallele, cioé i
quattro piani che le formano appartengono ad una stella impropria. Possiamo quindi
dedurre che due rette sono complanari se la matrice
 
a b c d
 a0 b0 c0 d 0 
 00 00 00 00 
a b c d 
a000 b000 c000 d 000

ha rango ≤ 3, cioé quando



a b c d
0
a b0 c0 d 0
00 = 0.
a
000 b00 c00 d 00
a b000 c000 000
d

Analogamente possiamo determinare una ulteriore condizione di complanaritá tra le


due rette: siano scelti due punti P = (x, y, z) ∈ r e P0 = (x0 , y0 , z0 ) ∈ r0 . Se le due rette
14.6 Rette complanari. 349

fossero complanari, i vettori (l, m, n), (l 0 , m0 , n0 ) e (x − x0 , y − y0 , z − z0 ) sarebbero tra


loro dipendenti cioé
x − x0 y − y0 z − z0


l
0 m n = 0.
l m0 n0
Se due rette non sono complanari sono dette sghembe.
Esempio 195. Consideriamo le rette r, s di equazioni cartesiane
 
x−y+z = 0 x−y−z+2 = 0
r: , s:
2x + y + 1 = 0 3y − 1 = 0

ed osserviamo che
1 −1 1 0

2 1 0 1
1 −1 −1 2 = 0.


0 3 0 −1
Quindi le due rette sono complanari. Per determinare il piano al quale appartengono
entrambe, inizialmente costuiamo il fascio di piani il cui asse é la retta r:

Fr : α(x − y + z) + β (2x + y + 1) = 0.

Adesso scegliamo un punto di s che non appartenga alla retta r, ad esempio Hs =


(0, 31 , 53 ). Infine imponiamo che il passaggio di un piano di Fr per il punto Hs :

1 5 1
α(− + ) + β (0 + + 1) = 0 ⇒ α = −β .
3 3 3
Concludiamo che l’equazione del piano richiesta é

α(x − y + z) − α(2x + y + 1) = 0, α 6= 0

ovvero
x + 2y − z + 1 = 0.
Esempio 196. Consideriamo le rette r, s di equazioni cartesiane
 
x+y−z+1 = 0 y−z = 0
r: , s:
2x + y + 1 = 0 x + 2y − 2z − 3 = 0

ed osserviamo che
1
1 −1 1
2 1 0 1
6= 0.
0
1 −1 0
1 2 −2 −3
Quindi le due rette non sono complanari (sono sghembe).
350 14 Rette e piani nello spazio affine ed euclideo.

14.7 Reciproca posizione tra una retta ed un piano.

Siano dati il piano


π: ax + by + cz + d = 0
e la retta
π 0 : a0 x + b0 y + c0 z + d 0 = 0

r: .
π 00 : a00 x + b00 y + c00 z + d 00 = 0
Se i tre piani appartengono allo stesso fascio, allora la retta r appartiene al piano π
(essa é l’asse del fascio).
Se i tre piani appartengono ad una stella propria allora la retta ed il piano π hanno
un punto in comune (che é il centro della stella).
Se i tre piani appartengono ad una stella impropria allora la retta ed il piano non
hanno punti in comune, cioé la retta é parallela al piano.
Poniamoci in quest’ultimo caso, retta e piano siano paralleli. Siano P1 = (x1 , y1 , z1 )
e P2 = (x2 , y2 , z2 ) due punti della retta. Allora il vettore (l, m, n) = (x2 − x1 , y2 −
y1 , z2 − z1 ) é parallelo a π. In altre parole esiste un piano σ : ax + by + cz + d iv ,
parallelo a π, che contiene il vettore P1 P2 . Quindi le coordinate dei punti P1 e P2
soddisfano all’equazione di σ :

ax2 + by2 + cz2 + d iv = 0




ax1 + by1 + cz1 + d iv = 0

e sottraendo le due equazioni otteniamo

a(x2 − x1 ) + b(y2 − y1 ) + c(z2 − z1 ) = 0

al + bm + cn = 0
che é la condizione di parallelismo tra una retta ed un piano.
Esempio 197. Consideriamo la retta

 x = 2t + 3
r: y = t −1
z = −t + 4

ed il piano π : x − y + z − 1 = 0. Diciamo (l, m, n) = (2, 1, −1) i parametri direttori


della retta, (a, b, c) = (1, −1, 1) i coefficienti delle indeterminate nell’equazione del
piano. Poiché si verifica che al + bm + cn = 0, allora la retta ed il piano sono paral-
leli. Inoltre é sufficiente individuare un punto di r che non appartenga al piano π,
per concludere che la retta non appartiene completamente al piano. Ad esempio il
punto di coordinate (3, −1, 4) appartiene alla retta ma non al piano.
Esempio 198. Consideriamo adesso la retta
14.8 Calcolo dei parametri direttori di una retta. 351

 x = −t + 2
r: y=t
z = t −2

ed il piano π : x + 3y − 2z + 2 = 0. Diciamo (l, m, n) = (−1, 1, −1) i parametri diret-


tori della retta, (a, b, c) = (1, 3, −2) i coefficienti delle indeterminate nell’equazione
del piano. Poiché si verifica che al + bm + cn = 0, allora la retta ed il piano sono
paralleli. Inoltre é sufficiente individuare un punto di r che appartenga al piano π,
per concludere che la retta appartiene completamente al piano. Ad esempio il punto
di coordinate (2, 0, −2) appartiene sia alla retta che al piano.
Esempio 199. Siano 
 x = t +1
r: y = 2t − 1
z = t +2

e π : 2x − 2y + z − 1 = 0. Diciamo (l, m, n) = (1, 2, 1) i parametri direttori della


retta, (a, b, c) = (2, −2, 1) i coefficienti delle indeterminate nell’equazione del piano.
Poiché si verifica che al + bm + cn 6= 0, allora la retta ed il piano non sono paralleli.
Vogliamo quindi individuare le coordinate del punto di intersezione tra la retta ed il
piano. É sufficiente sostituire la generica espressione delle coordinate di un punto
di r nell’equazione del piano, e verificare per quale valore del parametro variabile t
l’uguaglianza é verificata:

2(t + 1) − 2(2t − 1) + (t + 2) − 1 = 0 ⇒ t = 5

da cui P0 = r ∩ π = (6, 9, 7).

14.8 Calcolo dei parametri direttori di una retta.

Sia data la retta 


π : ax + by + cz + d = 0
r:
π 0 : a0 x + b0 y + c0 z + d 0 = 0
con parametri direttori (l, m, n). Essi sono le componenti di un vettore parallelo alla
retta. Tale vettore deve allora essere parallelo sia al piano π che a π 0 , ed applicando
la condizione di parallelismo otteniamo:

al + bm + cn = 0
.
a0 l + b0 m + c0 n = 0

Questo é un sistema omogeneo di rango 2, nelle tre incognite (l, m, n), le cui
soluzioni sono proporzionali ai minori
     
b c c a a b
, , .
b0 c0 c0 a0 a0 b0
352 14 Rette e piani nello spazio affine ed euclideo.

Esempio 200. Consideriamo la retta



2x − 3y + z − 1 = 0
r:
x + 2y − z + 2 = 0

e calcoliamone i parametri direttori. Costruiamo la matrice


 
2 −3 1
1 2 −1

e calcoliamone i minori che abbiamo sopra descritto. Avremo:

(l, m, n) = (1, 3, 7).

14.9 Coordinate omogenee nello spazio.

Sia P = (x, y, z) un punto nello spazio. Diremo che (x1 , x2 , x3 , x4 ) sono le coordinate
omogenee di P se
x1 x2 x3
x= , y= , z= .
x4 x4 x4
Se x4 6= 0, il punto é detto proprio e puó essere individuato dalla quaterna di
coordinate omogenee (x, y, z, 1).
In caso x4 = 0, il punto é detto improprio. L’insieme di tutti i punti impropri del-
lo spazio é individuato dall’equazione x4 = 0, che rappresenta un piano, il piano
improprio.
Tutte le quaterne di coordinate omogenee che siano tra loro proporzionali, indivi-
duano il medesimo punto.
L’equazione di un piano in coordinate omogenee é

π: ax1 + bx2 + cx3 + dx4 = 0

ed i suoi punti impropri si ottengono intersecandolo col piano improprio:



ax1 + bx2 + cx3 + dx4 = 0
x4 = 0

Questa é una retta, detta retta impropria (o giacitura) di π. Tutti i piani tra loro
paralleli hanno la stessa giacitura.

L’equazione di una retta in coordinate omogenee é data dall’intersezione dei due


piani 
ax1 + bx2 + cx3 + dx4 = 0
r:
a0 x1 + b0 x2 + c0 x3 + d 0 x4 = 0
I punti impropri di r si ottengono intersecandola col piano improprio:
14.10 Angoli nello spazio euclideo. 353

 ax1 + bx2 + cx3 + dx4 = 0
a0 x1 + b0 x2 + c0 x3 + d 0 x4 = 0
x4 = 0

Quindi ogni retta contiene un solo punto improprio (l, m, n, 0), dove (l, m, n) sono
esattamente i parametri direttori di r. Per cui rette tra loro parallele hanno lo stesso
punto improprio.
Osservazione 14.1. Se una retta ed un piano sono paralleli, allora il punto improprio
della retta appartiene alla giacitura del piano.
Esempio 201. Riprendiamo l’Esempio 197 e consideriamo la retta

 x = 2t + 3
r: y = t −1
z = −t + 4

ed il piano π : x − y + z − 1 = 0. Il punto improprio della retta é P∞ = (2, 1, −1, 0) e


la giacitura del piano é 
x1 − x2 + x3 = 0
t∞ : .
x4 = 0
É immediato osservare che P∞ ∈ t∞ , infatti la retta ed il piano sono paralleli.

14.10 Angoli nello spazio euclideo.

Fissiamo nello spazio euclideo E 3 un riferimento cartesiano ortogonale OXY Z di


centro O e versori i, j, k, rispettivamente per gli assi X,Y, Z.
Angolo compreso tra due rette.
Chiameremo coseni direttori di una retta r, i coseni degli angoli che la retta forma
con gli assi coordinati. Se la retta é individuata dai parametri direttori (l, m, n), i suoi
coseni direttori saranno:
l l
α = cos(r,ˆX) ∈ {+ √ ,−√ }
2 2
l +m +n2 l + m2 + n 2
2

ˆ ) ∈ {+ √ m m
β = cos(r,Y ,−√ }
2 2
l +m +n2 l + m2 + n2
2

n n
γ = cos(r,ˆZ) ∈ {+ √ ,−√ }
2 2
l +m +n 2 l + m2 + n 2
2

Consideriamo ora due rette ed individuiamole tramite i rispettivi parametri direttori:


r = (l, m, n) e r0 = (l 0 , m0 , n0 ). Indichiamo con v e v0 due vettori paralleli rispettiva-
mente a r e r0 , uno di componenti (l, m, n) e l’altro (l 0 , m0 , n0 ). L’angolo tra le due
rette é lo stesso formato dai due vettori:
354 14 Rette e piani nello spazio affine ed euclideo.

ll 0 + mm0 + nn0
cos(r,ˆr0 ) = cos(v,ˆv0 ) = ± √ √ .
l 2 + m2 + n2 · l 02 + m02 + n02
In particolare, le due rette sono ortogonali se ll 0 + mm0 + nn0 = 0.
Esempio 202. Consideriamo le rette
 
 x = t +1  x = 3t − 1
r: y = 2t − 1 , s: y = 2t + 2 .
z = t +2 z = −t + 1
 

Utilizzando le terne di parametri direttori delle rette, otteniamo



3+4−1 3
cos(r,ˆs) = ± √ √ = ± √ .
6 · 14 7
Angolo compreso tra due piani.
Inizialmente osserviamo che, dato il piano π : ax + by + cz + d = 0, un vettore v di
componenti (vx , vy , vz ) é parallelo a π solo se avx +bvy +cvz = 0. Ció vuol dire anche
che il vettore v é ortogonale ad ogni vettore w di componenti proporzionali alla terna
(a, b, c). In particolare questo implica che ogni vettore w = ρ(a, b, c) é ortogonale
anche al piano ax + by + cz + d = 0. Diciamo allora che una retta é ortogonale al
piano π se i suoi coseni direttori sono
a a
α = cos(r,ˆX) ∈ {+ √ ,−√ }
2 2
a +b +c2 a + b2 + c2
2

ˆ ) ∈ {+ √ b b
β = cos(r,Y ,−√ }
2
a +b +c2 2 a + b2 + c2
2

c c
γ = cos(r,ˆZ) ∈ {+ √ ,−√ }.
a2 + b2 + c2 a2 + b2 + c2

Siano ora π : ax + by + cz + d = 0 e π 0 : a0 x + b0 y + c0 z + d 0 = 0 due piani distinti.


L’angolo formato dai due piani é uguale a quello formato dai due versori normali ai
due piani:

aa0 + bb0 + cc0 aa0 + bb0 + cc0


cos(π,ˆπ 0 ) ∈ {+ √ √ ,−√ √ }.
a2 + b2 + c2 · a02 + b02 + c02 a2 + b2 + c2 · a02 + b02 + c02
In particolare, i due piani sono ortogonali se aa0 + bb0 + cc0 = 0.
Esempio 203. Considerando i piani

π : 3x − y + z − 1 = 0 π0 : x + y − z − 1 = 0

ed utilizzando i coefficienti delle indeterminate che compaiono nelle equazioni dei


piani abbiamo che
14.11 Distanze nello spazio euclideo. 355

3−1−1 1
cos(π,ˆπ 0 ) = ± √ √ = ±√ .
11 · 3 33
Angolo compreso tra una retta ed un piano.
Consideriamo infine una retta r di parametri direttori (l, m, n) ed un piano π : ax +
by + cz + d = 0. Il versore normale al piano é

a b c
n = (√ ,√ ,√ ).
2 2
a +b +c 2 2 2
a +b +c2 a + b2 + c2
2

π
Sia ϕ = (r, n) l’angolo formato dalla retta r e dal versore n. Definiamo 2 −ϕ
l’angolo formato dalla retta r e dal piano π. Quindi segue che:

al + bm + cn
sen(r, π) = cos(r, n) = ± √ √ .
a + b + c 2 · l 2 + m2 + n 2
2 2

In particolare, la retta ed il piano sono paralleli se al + bm + cn = 0.


Esempio 204. Consideriamo la retta ed il piano espressi come segue

 x = 3t − 1
r: y = 2t + 1 , π : x + y − x + 1 = 0.
z = t −1

Utilizzando le terne di parametri direttori della retta ed i coefficienti delle indeter-


minate nell’equazione del piano, otteniamo

3+2−1 2 2
sen(r, π) = ±
ˆ √ √ =± √ .
14 · 3 21

14.11 Distanze nello spazio euclideo.

Distanza tra due punti.


Siano P1 = (x1 , y1 , z1 ) e P2 = (x2 , y2 , z2 ) due punti dello spazio euclideo 3-dimensionale.
La distanza tra i punti P1 e P2 é il modulo del vettore P1 P2 :
q
δ (P1 , P2 ) = (x2 − x1 )2 + (y2 − y1 )2 + (z2 − z1 )2 .

Esempio 205. Dati i punti P1 = (−1, 2, −2) e P2 = (3, 1, −1), la loro distanza é data
da p √
δ (P1 , P2 ) = 42 + 12 + 12 = 3 2.
Distanza tra un punto ed un piano.
Consideriamo ora il punto P0 = (x0 , y0 , z0 ) ed il piano π : ax + by + cz + d = 0. Ci
proponiamo di calcolare la distanza del punto dal piano. Se P0 ∈ π chiaramente
356 14 Rette e piani nello spazio affine ed euclideo.

diremo che essa é nulla. Poniamoci allora nel caso in cui P0 ∈ / π. La distanza di
P0 da π é pari alla lunghezza δ (P0 , H) del segmento P0 H, dove H é la proiezione
ortogonale di P0 su π. Scegliamo un qualsiasi punto Q1 = (x1 , y1 , z1 ) ∈ π. Allora
δ (P0 , H) é la proiezione ortogonale del vettore P0 Q1 lungo la direzione del vettore
P0 H. Per determinare tale proiezione abbiamo bisogno del versore u di P0 H, che é
il versore normale al piano π:

a b c
u = (√ ,√ ,√ ).
2 2
a +b +c 2 2 2
a +b +c2 a + b2 + c2
2

Quindi:

a b c
δ (P0 , H) = (P0 Q1 ) × ( √ i+ √ j+ √ k) =
2 2
a +b +c 2 2
a +b +c 2 2 2
a +b +c 2 2


((x1 − x0 )i + (y1 − y0 ) j + (z1 − z0 )k) × √ai + b j + ck =

2 2
a +b +c 2

|ax1 − ax0 + by1 − by0 + cz1 − cz0 |


√ .
a2 + b2 + c2
Dal fatto che Q1 ∈ π segue che ax1 + by1 + cz1 = −d, per cui

|ax0 + by0 + cz0 + d|


δ (P0 , π) = δ (P0 , H) = √ .
a2 + b2 + c2
Esempio 206. Dati il punti P0 = (1, 2, −1) ed il piano π : x + y + 4z − 1 = 0, la loro
distanza é data da √
|1 + 2 − 4 − 1| 2
δ (P0 , π) = √ = .
18 3
Distanza tra un punto ed una retta.
Scegliamo ora una retta r di parametri direttori (l, m, n) ed il punto P1 = (x1 , y1 , z1 )
non appartenente alla retta. Costruiamo il piano π ortogonale alla retta e passante per
P1 : tale piano é unico. Indichiamo con Q1 = π ∩ r, esso é la proiezione ortogonale
di P1 su r. La distanza del punto P1 dalla retta r é la distanza δ (P1 , Q1 ).
Esiste comunque una formula che consente di calcolare direttamente la distanza tra
il punto P1 e la retta r. Indichiamo come sopra Q1 il punto che sia la proiezione di P1
su r. Scegliamo arbitrariamente un punto Q0 = (x0 , y0 , z0 ) ∈ r. Il segmento h = P1 Q1
é l’altezza del trapezio avente come lati l1 = Q1 Q0 e l2 = P1 Q0 , inoltre h é l’altezza
relativa al lato l1 . Per cui h = lA1 , dove A é l’area del trapezio. Quindi:

|Q1 Q0 ∧ P1 Q0 |
δ (P1 , r) = h = .
|Q1 Q0 |

In particolare, pur supponendo di non conoscere le coordinate di Q1 , é certo che


esiste un opportuno α ∈ R, tale che Q1 Q0 = α(l, m, n), da cui
14.11 Distanze nello spazio euclideo. 357


i j k

Q1 Q0 ∧ P1 Q0 = αl
αm αn .
(x1 − x0 ) (y1 − y0 ) (z1 − z0 )

Allora il modulo del vettore Q1 Q0 ∧ P1 Q0 é


s
x1 − x0 y1 − y0 2 x1 − x0 z1 − z0 2 y1 − y0 z1 − z0 2

α + +
l m l n m n

ed il modulo di Q1 Q0 é α l 2 + m2 + n2 . Concludiamo che

|Q1 Q0 ∧ P1 Q0 |
δ (P1 , r) = h = =
|Q1 Q0 |
v
u x1 − x0 y1 − y0 2 x1 − x0 z1 − z0 2 y1 − y0 z1 − z0 2
u
u + +
t l m l n m n
.
l 2 + m2 + n2
Esempio 207. Consideriamo il punto P = (1, 3, −1) e la retta

 x = t +1
r: y = 2t − 1 .
z = t +2

Scgliamo un punto sulla retta: banalmente prendiamo Q = (1, −1, 2), le cui coor-
dinate sono presenti nell’espressione parametrica della retta. Otteniamo quindi le
componenti del vettore QP = (0, 4, −3).
Tramite QP ed i parametri direttori della retta costruiamo la matrice
 
0 4 −3
12 1

dalla quale estrarre i minori per l’applicazione della formula sopra dimostrata.
Quindi: v
u 4 −3 2 0 −3 2 0 4 2
u

t 2 1 + 1 1 + 1 2
u
5 5
δ (P, r) = h = = √ .
1+4+1 6
Distanza tra due piani paralleli.

Siano ora π : ax + by + cz + d1 = 0 e π 0 : ax + by + cz + d2 = 0 due piani paralleli,


le cui equazioni sono riportate in modo tale che i coefficienti delle indeterminate
corrispondenti siano identici (e non semplicemente proporzionali). La distanza tra
di essi é pari alla distanza di un qualsiasi punto P0 = (x0 , y0 , z0 ) di π dall’altro piano
π 0:
|ax0 + by0 + cz0 + d2 |
δ (π, π 0 ) = √ =
a2 + b2 + c2
358 14 Rette e piani nello spazio affine ed euclideo.

|d − d1 |
√ 2
a2 + b2 + c2
poiché ax0 + by0 + cz0 = −d1 .
Esempio 208. Dati i piani

π : x + y − 2z + 1 = 0 π 0 : 2x + 2y − 4z − 3 = 0

e riportando l’equazione del secondo come segue:


3
π 0 : x + y − 2z − =0
2
la loro distanza é data da

| − 3 − 1| 5
δ (π, π 0 ) = √ 2 = √ .
1+1+4 2 6
Minima distanza tra due rette.
Osserviamo inizialmente che se due rette fossero incidenti, allora la loro minima
distanza é ovviamente pari a zero.
Inoltre, nel caso fossero parallele (quindi ancora complanari), la loro distanza é pari
alla distanza tra un qualsiasi punto di una di esse e l’altra.
Infine siano r = (l, m, n) e r0 = (l 0 , m0 , n0 ) due rette sghembe. Esisteranno quindi due
piani π e π 0 tra loro paralleli tali che r ∈ π e r0 ∈ π 0 . La minima distanza tra r e r0 é
pari alla distanza tra π e π 0 .
Un altro metodo per calcolare la minima distanza tra le rette sghembe r e r0 é quello
di sfruttare la proprietá per la quale esiste una ed una sola retta s incidente e perpen-
dicolare ad entrambe le rette r e r0 . Dopo aver individuato la retta s, si costruiscono
i punti P = r ∩ s e P0 = r0 ∩ s. La distanza tra le due rette sghembe é pari alla distan-
za tra P e P0 . Esponiamo nei particolari questo procedimento: Siano scelti i punti
Q = (x0 , y0 , z0 ) ∈ r e Q0 = (x00 , y00 , z00 ):

 x = x00 + t 0 l 0
 
 x = x0 + tl
0
r : y = y0 + tm , r : y = y00 + t 0 m0 .
z = z0 + tn z = z00 + t 0 n0
 

I generici punti P ∈ r e P0 ∈ r0 sulle due rette hanno coordinate

P = (lt + x0 , mt + y0 , nt + z0 ), P0 = (l 0t 0 + x00 , m0t 0 + y00 , n0t 0 + z00 ) t,t 0 ∈ R

ed il vettore w che li congiunge ha componenti

w = (lt − l 0t 0 + x0 − x00 , mt − m0t 0 + y0 − y00 , nt + n0t 0 + z0 − z00 ).

Al variare dei parametri t e t 0 dobbiamo determinare i punti P e P0 tali che il vettore


w sia ortogonale sia con r che con r0 , cioé devono annullarsi i seguenti prodotti
14.11 Distanze nello spazio euclideo. 359

scalari:

w × (l, m, n) = 0 e w × (l 0 , m0 , n0 ) = 0
ovvero, rispettivamente,

(lt − l 0t 0 + x0 − x00 ) · l + (mt − m0t 0 + y0 − y00 ) · m + (nt + n0t 0 + z0 − z00 ) · n = 0

(lt − l 0t 0 + x0 − x00 ) · l 0 + (mt − m0t 0 + y0 − y00 ) · m0 + (nt + n0t 0 + z0 − z00 ) · n0 = 0.

Le soluzioni t,t 0 , del precedente sistema, permettono di individuare i punti P, P0 sulle


rette r e r0 , tali che il vettore PP0 sia ortogonale ad entrambe le rette r e r0 , ed abbia
modulo pari alla minima distanza tra le rette stesse.
Esempio 209. Riprendiamo l’Esempio 196 e consideriamo le rette
 
x+y−z+1 = 0 y−z = 0
r: , s:
2x + y + 1 = 0 x + 2y − 2z − 3 = 0

le quali non sono complanari. Determinando le terne vr , vs dei parametri direttori


delle rette avremo che

vr = (1, −2, −1) e vs = (0, 1, 1).

Costruiamo il fascio di piani che abbia come asse la retta r:

Fr : α(x+y−z+1)+β (2x+y+1) = 0 −→ (α +2β )x+(α +β )y−αz+(α +β ) = 0.

Imponiamo il parallelismo con la retta s:

(α + β ) − α = 0 ⇒ β = 0.

Quindi il piano πr , contenente r e parallelo alla retta s, ha equazione x+y−z+1 = 0.


Analogamente, costruiamo il fascio di piani che abbia come asse la retta s:

Fs : γ(y − z) + δ (x + 2y − 2z − 3) = 0 −→ δ x + (γ + 2δ )y − (γ + 2δ )z − 3δ = 0.

Imponendo il parallelismo con la retta r avremo:

δ − (2γ + 4δ ) + (γ + 2δ ) = 0 ⇒ δ = −γ.

Quindi il piano πs , contenente r e parallelo alla retta s, ha equazione

γ(y − z) − γ(x + 2y − 2z − 3) = 0 γ 6= 0

ovvero x + y − z − 3 = 0. Concludiamo allora che la minima distanza fra le rette é


360 14 Rette e piani nello spazio affine ed euclideo.

pari a
| − 3 − 1| 4
δ (πr , πs ) = √ =√ .
1+1+1 3
Esempio 210. Vogliamo adesso ripetere l’Esempio 209, utilizzando l’altro me-
doto precedentemente esposto. Scegliamo il punto Pr = (0, −1, 0) sulla retta r ed
esprimiamo quest’ultima in forma parametrica:

 x=t
r: y = −2t − 1 .
z = −t

Analogamente, scelto Ps = (3, 0, 0) un punto della retta s, esprimiamo quest’ultima


in forma parametrica: 
x=3
s: y = t0 .
z = t0

I generici punti P ∈ r e P0 ∈ s sulle due rette hanno coordinate

P = (t, −2t − 1, −t), P0 = (3,t 0 ,t 0 ) t,t 0 ∈ R

ed il vettore w che li congiunge ha componenti

w = (t − 3, −2t − 1 − t 0 , −t − t 0 ).

Al variare dei parametri t e t 0 dobbiamo determinare i punti P e P0 tali che il vettore w


sia ortogonale sia con r che con s, cioé devono annullarsi i seguenti prodotti scalari:

w × (1, −2, −1) = 0 e w × (0, 1, 1) = 0


ovvero, rispettivamente,
6t + 3t 0 − 1 = 0
e
3t + 2t 0 + 1 = 0.
Risolvendo il precedente sistema, avremo t = 35 e t 0 = −3. Tali valori permettono di
individuare i punti P, P0 sulle rette r e s, tali che il vettore w abbia modulo pari alla
minima distanza tra le rette stesse:
5 10 5 4 4 4
w = (t − 3, −2t − 1 − t 0 , −t − t 0 ) = ( − 3, − + 3 − 1, − + 3) = (− , − , ).
3 3 3 3 3 3
Concludiamo allora che
r
0 16 16 16 4
δ (r, r ) = |w| = + + =√ .
9 9 9 3
14.12 Simmetrie nello spazio. 361

14.12 Simmetrie nello spazio.

Nello spazio euclideo, distinguiamo 3 diverse tipologie di simmetrie:


Simmetria centrale
É la simmetria rispetto ad un punto fissato C = (α, β , γ).
Il simmetrico di un punto P = (x, y, z) rispetto al punto C é lunico punto P0 =
(x0 , y0 , z0 ) tale che valga l’identitá vettoriale PC = CP0 . In altre parole, le coordinate
dei punti coinvolti nella precedente identitá devono soddisfare alle seguenti:

x + x0 y + y0 z + z0
α= , β= , γ= .
2 2 2
Simmetria assiale
É la simmetria rispetto ad una retta fissata r.
Il simmetrico di un punto P = (x, y, z) rispetto alla retta r é precisamente il simme-
trico di P rispetto alla propria proiezione ortogonale sulla retta r.
Ovvero, si determina dapprima il piano π, passante per P ed ortogonale alla retta r.
Quindi si calcolano le coordinate del punto C dato dall’intersezione π ∩ r. Il simme-
trico di P rispetto a C (simmetria centrale) é esattamente il simmetrico di P rispetto
alla retta r.
Simmetria planare
É la simmetria rispetto ad un piano fissato.
Il simmetrico di un punto P = (x, y, z) rispetto al piano π é precisamente il simme-
trico di P rispetto alla propria proiezione ortogonale sul piano π.
Si determina dapprima la retta r, passante per P ed ortogonale al piano π. Quindi si
calcolano le coordinate del punto C dato dall’intersezione π ∩ r. Il simmetrico di P
rispetto a C (simmetria centrale) é esattamente il simmetrico di P rispetto al piano
π.
Esempio 211. Siano C = (2, 1, 1), π : x − y + z − 1 = 0 e

x = t −1
r : y = t +2.
z = t +1

Dato il punto P di coordinate (3, 4, 1), determiniamone i simmetrici rispetto a C, π


e r.
Detto P0 = (x0 , y0 , z0 ) il simmetrico di P rispetto a C, avremo che

3 + x0 4 + y0 1 + z0
2= , 1= , 1=
2 2 2
da cui P0 = (1, −2, 1).
Consideriamo ora la retta ortogonale a π e passante per P:
362 14 Rette e piani nello spazio affine ed euclideo.

 x = t +3
s : y = −t + 4 .
z = t +1

Il punto di intersezione tra s e π é C1 = ( 10 11 4


3 , 3 , 3 ). Il simmetrico di P rispetto a C1
(ovvero il simmetrico di P rispetto a π) é P00 = ( 11 10 5
3 , 3 , 3 ).
Infine consideriamo il piano ortogonale a r e passante per P:

σ : x + y + z − 8 = 0.

Il punto di intersezione tra σ e r é C2 = (1, 4, 3). Il simmetrico di P rispetto a C2


(ovvero il simmetrico di P rispetto a r) é P000 = (−1, 4, 5).

14.13 Cambiamento di riferimento cartesiano ortonormale.

In relazione alla possibilitá di determinare un cambiamento del sistema di riferimen-


to, estendiamo quanto detto nello spazio affine euclideo 2-dimensionale allo spazio
3-dimensionale.
Osserviamo che, tutta la trattazione é analoga a quella giá svolta nel caso bi-
dimensionale, col solo accorgimento di individuare il versore concorde ad una retta,
tramite terne (e non coppie) di parametri direttori, in qualitá di componenti rispetto
al sistema di riferimento iniziale.
Siano quindi OXY Z e O0 X 0Y 0 Z 0 due distinti riferimenti cartesiani ortonormali ed in-
dichiamo con (i, j, k) e (i0 , j0 , k0 ) rispettivamente la terna dei versori degli assi X,Y, Z
e quella dei versori di X 0 ,Y 0 , Z 0 .
Traslazioni.
Siano (x, y, z) le coordinate del punto P, rispetto al riferimento cartesiano OXY Z
dello spazio euclideo. Supponiamo che gli assi X 0 ,Y 0 , Z 0 del secondo riferimento
siano paralleli e concordi rispettivamente agli assi X,Y, Z e che il punto O0 abbia
coordinate (a, b, c) rispetto a OXY Z.
Denotiamo (x0 , y0 , z0 ) le coordinate di P rispetto al riferimento O0 X 0Y 0 Z 0 . La relazio-
ne che intercorre tra le due terne di coordinate di P é la seguente:
 0
x = x−a
y0 = y − b
 0
z = z−c

e le formule inverse sono


 x = x0 + a

y = y0 + b .
z = z0 + c

Rotazioni.
14.13 Cambiamento di riferimento cartesiano ortonormale. 363

Siano ancora (x, y, z) le coordinate di un punto P nel riferimento cartesiano orto-


gonale OXY Z. Supponiamo ora che l’origine del secondo sistema di riferimento
coincida con quella del sistema iniziale, ma almeno uno degli assi X 0 , Y 0 e Z 0 non
sia parallelo e concorde ad alcuno degli assi del sistema di riferimento iniziale. Stia-
mo supponendo che
Indichiamo come segue le terne delle componenti dei versori i0 , j0 , k0 :
q
i0 = (l1 , m1 , n1 ) : l12 + m21 + n21 = 1
q
j0 = (l2 , m2 , n2 ) : l22 + m22 + n22 = 1
q
k0 = (l3 , m3 , n3 ) : l32 + m23 + n23 = 1.
Ricordiamo che
ˆ 0 ),
l1 = cos(XX m1 = cos(Y ˆX 0 ), ˆ 0)
n1 = cos(ZX
ˆ 0 ),
l2 = cos(XY ˆ 0 ),
m2 = cos(YY ˆ 0)
n2 = cos(ZY
ˆ 0 ),
l3 = cos(XZ m3 = cos(Y ˆZ 0 ), ˆ 0 ).
n3 = cos(ZZ
Per determinare le coordinate (x0 , y0 , z0 ) di P nel secondo riferimento, ricordiamo
inizialmente che esse sono le componenti del vettore OP. Esprimiamo quindi il
vettore OP rispetto ad entrambe le terne di versori; i, j, k in OXY Z ed anche i0 , j0 , k0
in OX 0Y 0 Z 0 :
OP = x0 i0 + y0 j0 + z0 k0 OP = xi + y j + zk.
Ancora una volta possiamo richiamare quanto detto in relazione al cambiamento di
base in uno spazio vettoriale:
   0
x x
 y  = A ·  y0 
z z0

dove A é la matrice di cambiamento di base. Nel nostro caso abbiamo che


 
l1 l2 l3
A =  m1 m2 m3 
n1 n2 n3

cioé      0
x l1 l2 l3 x
 y  =  m1 m2 m3  ·  y0 
z n1 n2 n3 z0
e le formule inverse sono
364 14 Rette e piani nello spazio affine ed euclideo.
 0    
x l1 m1 n1 x
 y0  =  l2 m2 n2  ·  y 
z0 l3 m3 n3 z

poiché le due matrici usate sono matrici ortogonali, ovvero l’una la trasposta
dell’altra, ma anche l’una l’inversa dell’altra.
Rototraslazioni.
Consideriamo infine il caso piú generale: i due sistemi OXY Z e O0 X 0Y 0 Z 0 hanno
origini differenti ed almeno uno degli assi del sistema O0 X 0Y 0 Z 0 non sia parallelo e
concorde ad alcuno degli assi del sistema di riferimento iniziale. In tale caso, per
ottenere le coordinate (x0 , y0 , z0 ) di un punto P rispetto al sistema O0 X 0Y 0 Z 0 , saranno
necessarie sia una traslazione che una rotazione degli assi. Quindi, indicando le
componenti dei versori i0 , j0 , k0 come segue

i0 = (l1 , m1 , n1 ), j0 = (l2 , m2 , n2 ), k0 = (l3 , m3 , n3 )

ed assegnando le coordinate (a, b, c) al punto O0 rispetto al sistema OXY Z, avremo


che
     0  
x l1 l2 l3 x a
 y  =  m1 m2 m3  ·  y0  +  b 
z n1 n2 n3 z0 c
ed inversamente
 0    
x l1 m1 n1 x−a
 y0  =  l2 m2 n2  ·  y − b  .
z0 l3 m3 n3 z−c
Osservazione 14.2. A partire dal sistema di riferimento ortonormale originario
OXY Z, nel passaggio ad un nuovo sistema ortonormale O0 X 0Y 0 Z 0 standard, possia-
mo ottenere 4 diversi cambiamenti di coordinate. In realtá i possibili cambiamente
di coordinate sono pari a 6, rispettivamente cambiando i segni ai versori i0 , j0 , k0 .
Infatti per ogni direzione di i0 , j0 , k0 vi sono 2 possibili versi. Ma solo 4 delle pos-
sibili 6 scelte ci porteranno ad un sistema di riferimento standard. In sostanza, una
volta scelti arbitrariamente i versi per i0 e j0 , per ottenere un sistema di riferimento
standard, il verso di k0 sará univocamente determinato.
Esempio 212. Nello spazio eucliedo con riferimento ortonormale OXY Z, conside-
riamo un nuovo riferimento ortonormale individuato dalle rette
  
x−y = 0 y−1 = 0 x+z−2 = 0
r1 : , r2 : , r3 : .
x+z−2 = 0 x−z = 0 y − 2z + 1 = 0

Osserviamo dapprima che le rette sono ortogonali, infatti le loro direzioni, ottenute
calcolandone i rispettivi parametri direttori sono

r1 = (1, 1, −1), r2 = (1, 0, 1), r3 = (−1, 2, 1).


14.13 Cambiamento di riferimento cartesiano ortonormale. 365

Le rette si intersecano nel punto O0 di coordinate (1, 1, 1). Tale punto sará l’origine
del nuovo sistema di riferimento.
Per procedere con il cambiamento del sistema di riferimento dobbiamo a questo
punto effettuare delle scelte: quali siano i nuovi assi X 0 ,Y 0 , Z 0 ed inoltre quali siano
i loro versi.
Considereremo
r1 = X 0 , r2 = Y 0 , r3 = Z 0
e scegliamo i seguenti versi
   
1 1 1 1 1
i0 = √ , √ , − √ , j0 = √ , 0, √
3 3 3 2 2
da cui  
0 1 1 1
k = −λ √ , 2λ √ , λ √
6 6 6
dove λ = ±1, in accordo col fatto che (i0 , j0 , k0 ) debba rappresentare una terna di
riferimento standard. Ció avverrá nel caso in cui la matrice ortogonale di rotazione
 1
√1 −λ √1


3 2 6
C =  √13 0 2λ √16 
 
− √13 √12 λ √16

avrá determinante pari a 1. Questo si verifica per λ = −1, per cui


 1 1 1 
√ √ √
3 2 6
1
C=
 √
3
0 − √26 .

− √13 √12 − √16

Avremo allora le formule


   √1 √1 √1
    
x 3 2 6 x0 1
1
 √3
y =  0 − √26   0  
 · y + 1
z − √1 √1 − √1 z0 1
3 2 6

e
 0   √1 √1 − √1   
x 3 3 3 x−1
 y0  =  1 √1 
 √2 0 2 
·y−1
z0 1 2
√ −√ −√ 1 z−1
6 6 6
ovvero
366 14 Rette e piani nello spazio affine ed euclideo.

1 1 1
x = √ x0 + √ y0 + √ z0 + 1
3 2 6
1 0 2 0
y = √ x − √ z +1 (14.3)
3 6
1 0 1 1
z = − √ x + √ y0 − √ z0 + 1
3 2 6
e
1 1 1
x0 = √ (x − 1) + √ (y − 1) − √ (z − 1)
3 3 3
1 1
y0 = √ (x − 1) + √ (z − 1) (14.4)
2 2
0 1 2 1
z = √ (x − 1) − √ (y − 1) − √ (z − 1).
6 6 6
Ad esempio, dato in punto P di coordinate (2, 1, −1) in OXY Z, otterremo le sue
coordinate in O0 X 0Y 0 Z 0 applicando le (14.4), per cui
1 1 1 3
x0 = √ (2 − 1) + √ (2 − 1) − √ (−1 − 1) = √
3 3 3 3
0 1 1 1
y = √ (2 − 1) + √ (−1 − 1) = − √
2 2 3
1 2 1 3
z0 = √ (2 − 1) − √ (1 − 1) − √ (−1 − 1) = √ .
6 6 6 6
Analogamente, dato il piano π, la cui equazione in OXY Z é x + 3y − 2z + 1 = 0,
otterremo la sua equazione rappresentativa in O0 X 0Y 0 Z 0 tramite le relazioni (14.3):
 
1 0 1 0 1 0
√ x + √ y + √ z +1
3 2 6
 
1 0 2 0
+3 √ x − √ z +1
3 6
 
1 0 1 0 1 0
−2 −√ x + √ y − √ z +1 +1 = 0
3 2 6
ovvero
√ 1 √
3 2x0 − √ y0 − z0 + 6 = 0.
3

14.14 Esercizi svolti.

Esercizio 109. Determinare l’equazione del piano contenente i punti (1, 1, 0),
(1, 0, 1), (3, −1, 0).
14.14 Esercizi svolti. 367

Svolgimento: Sia (x, y, z) il generico punto del piano. L’equazione in forma carte-
siana si ottiene allora da:
x y z 1

1 1 0 1
1 0 1 1 = 0


3 −1 0 1
cioé x + y + z − 2 = 0.
Esercizio 110. Determinare l’equazione del piano contenente il punto (1, 1, 2) e
parallelo ai vettori di componenti (−1, 2, 3) e (1, 2, −1).

Svolgimento: In forma parametrica l’equazione del piano é

 x = −t + t 0 + 1

y = 2t + 2t 0 + 1
z = 3t − t 0 + 2

ed in forma cartesiana
x−1 y−1 z−2

−1 2 3 = 0

1 2 −1
cioé 4x − y + 2z − 7 = 0.
Esercizio 111. Sia dato il fascio di piani (3x + y − 2z) + a(x − 4y + 2z − 1) = 0.
Verificare se il piano π : 2x + 5y − 4z + 2 = 0 appartiene o no al fascio.

Svolgimento: Le matrici associate al fascio sono


   
3 1 −2 3 1 −2 0
A= , C= .
1 −4 2 1 −4 2 −1

Hanno entrambe rango 2, quindi il fascio é proprio. Consideriamo ora le matrici


associate ai tre piani (aggiungiamo una riga con i coefficienti di π):
   
3 1 −2 3 1 −2 0
A0 =  1 −4 2  , C0 =  1 −4 2 −1  .
2 5 −4 2 5 −4 2
La matrice A0 ha rango 2, ma la matrice C0 ha rango 3, quindi il piano π non
appartiene al fascio dato. In particolare i tre piani formano una stella impropria.
Esercizio 112. Sia dato il fascio di piani (x + y − 3z + 2) + a(2x + 2y − 6z + 1) = 0.
Verificare se il piano π : 5x + 5y − 15z + 4 = 0 appartiene o no al fascio.

Svolgimento: Le matrici associate al fascio sono


   
1 1 −3 1 1 −3 2
A= , C= .
2 2 −6 2 2 −6 1
368 14 Rette e piani nello spazio affine ed euclideo.

La matrice A ha rango 1, e la matrice C ha rango 2: é un fascio improprio. Conside-


riamo ora le matrici associate ai tre piani (aggiungiamo una riga con i coefficienti di
π):
   
1 1 −3 1 1 −3 2
A0 =  2 2 −6  , C0 =  2 2 −6 1  .
5 5 −15 5 5 −15 4
La matrice A0 ha rango 1 e la matrice C0 ha rango 2, quindi il piano π appartiene al
fascio dato.
Esercizio 113. Sia data la stella di piani (3x + y − 2z) + a(x − 4y + 2z − 1) + b(x −
1) = 0. Verificare se i piani π : 3x + 5y − 4z = 0 e σ : 2x + y − 1 = 0 appartengono o
no alla stella.

Svolgimento: Le matrici associate alla stella sono


   
3 1 −2 3 1 −2 0
A =  1 −4 2  , C =  1 −4 2 −1  .
1 0 0 1 0 0 −1

Hanno entrambe rango 3, quindi la stella é propria. Consideriamo ora le matrici


associate ai quattro piani (aggiungiamo una riga con i coefficienti di π):
   
3 1 −2 3 1 −2 0
 1 −4 2   1 −4 2 −1 
A0 = 
 1 0 0 ,
 C0 =  1 0 0 −1  .

3 5 −4 3 5 −4 0
Le matrici A0 e C0 hanno entrambe rango 3, quindi il piano π appartiene alla stella.
Consideriamo ora le matrici ottenute aggiungendo i coefficienti del piano σ :
   
3 1 −2 3 1 −2 0
 1 −4 2   1 −4 2 −1 
A00 = 
1 0 0 
, C 00
=  1 0 0 −1  .
 

2 1 0 2 1 0 −1

La matrice C00 ha rango 4, quindi il piano σ non appartiene alla stella.


Esercizio 114. Sia data la stella di piani (2x+y+z)+a(x+y+z)+b(3x+2y+2z−
1) = 0. Verificare se i piani π : 2x + 2y + 2z − 3 = 0 e σ : x + 2y − z = 0 appartengono
o no alla stella.

Svolgimento: Le matrici associate alla stella sono


   
211 211 0
A =  1 1 1 , C =  1 1 1 0 .
322 3 2 2 −1
14.14 Esercizi svolti. 369

La matrice A ha rango 2 e la matrice C ha rango 3, quindi la stella é impropria.


Consideriamo ora le matrici associate ai quattro piani (aggiungiamo una riga con i
coefficienti di π):
   
211 211 0
1 1 1 1 1 1 0 
A0 = 
 3 2 2 ,
 C0 = 
 3 2 2 −1  .

222 2 2 2 −3
Le matrici A0 e C0 hanno rispettivamente rango 2 e rango 3, quindi il piano π
appartiene alla stella.
Consideriamo ora le matrici ottenute aggiungendo i coefficienti del piano σ :
   
21 1 21 1 0
1 1 1  1 1 1 0 
A00 = 
 3 2 2 ,
 C00 = 
 3 2 2 −1  .

1 2 −1 1 2 −1 0

La matrice A00 ha rango 3, quindi il piano σ non appartiene alla stella impropria.
Esercizio 115. Determinare la retta passante per i punti (1, 0, 1) e (2, 3, 1).

Svolgimento: I parametri direttori della retta sono dati dalle differenze delle coor-
dinate omologhe dei due punti: (l, m, n) = (1, 3, 0), quindi una forma parametrica
dell’equazione della retta é: 
x = t +1
y = 3t .
z=1

In forma cartesiana, eliminando il parametro t dalle precedenti, otteniamo:



3x − y − 3 = 0
.
z=1

Esercizio 116. Determinare la retta passante per il punto (2, 1, 1) e parallela alla
retta s : 2x + y − z = x + y − 2z + 1 = 0.

Svolgimento: I parametri direttori della retta s sono (−1, 3, 1), e quindi essi pos-
sono essere considerati anche come i parametri direttori della retta da determinare.
Quindi una forma parametrica dell’equazione della retta é:

 x = −t + 2
y = 3t + 1 .
z = t +1

In forma cartesiana, eliminando il parametro t dalle precedenti, otteniamo:


370 14 Rette e piani nello spazio affine ed euclideo.

x+z−3 = 0
.
3x + y − 7 = 0

Esercizio 117. Determinare se le seguenti rette sono complanari:

r: x − 3y + 2 = x + y + z − 1 = 0

s: x = −t + 2, y = 5t + 3, z = −t.

Svolgimento: Ps = (2, 3, 0) é un punto di s. Qr = (−2, 0, 3) é un punto di r. Il


vettore che li congiunge ha componenti (4, 3, −3). Inoltre i parametri direttori di r
sono (−3, −1, 4) e quelli di s sono (−1, 5, −1). Calcoliamo il determinante della
matrice  
4 3 −3
A =  −3 −1 4  .
−1 5 −1
Poiché det(A) 6= 0, le due rette non sono complanari (si dice che sono sghembe).
Esercizio 118. Determinare se le seguenti rette sono complanari:

r: x−y = z−1 = 0

s: x − y + 2z − 3 = 2x − 2y + 3z − 4 = 0

Svolgimento: Per verificare se sono complanari consideriamo la matrice formata


dai coefficienti dei quattro piani che le compongono:
 
1 −1 0 0
 0 0 1 −1 
A=  1 −1 2 3  .

2 −2 3 −4

Poiché det(A) = 0, le due rette sono complanari. Calcoliamo allora il piano π che
la contiene entrambe: determiniamo dapprima il fascio di piani che abbia come
sostegno la retta s:

F: (x − y + 2z − 3) + a(2x − 2y + 3z − 4) = 0.

Il piano π appartiene a tale fascio. Inoltre π deve contenere ogni punto di r. Sceglia-
mo P = (0, 0, 1) ∈ r (la scelta di tale punto deve essere tale da essere certi che P non
sia il punto comune alle due rette). Imponiamo ora che il generico piano del fascio
contenga P:
2 − 3 + a(3 − 4) = 0 → a = −1
quindi
π: x − y + z − 1 = 0.
14.14 Esercizi svolti. 371

Esercizio 119. Si verifichi che il piano π : 2x − y − 4z + 2 = 0 e la retta

r: x = t, y = 1 − 2t, z=t

sono paralleli.

Svolgimento: I parametri direttori della retta sono (l, m, n) = (1, −2, 1) ed i coef-
ficienti del piano sono (a, b, c) = (2, −1, −4), per cui é verificata la condizione
al + bm + cn = 0.
Esercizio 120. Determinare il punto comune alla retta

r: 2x − y + 1 = x + y − z = 0

ed al piano π : 3x − y + 2z = 0.

Svolgimento: I parametri direttori di r sono (1, 2, 3) ed un suo punto é P0 =


(0, 1, 1). Una forma parametrica della retta é allora:

 x=t
y = 2t + 1 .
z = 3t + 1

Sostituiamo il generico punto di r, che ha coordinate (t, 2t + 1, 3t), nell’equazione


del piano. Otterremo il valore del parametro t per il quale il punto appartiene sia alla
retta che al piano:
1
3t − (2t + 1) + 2(3t + 1) = 0 → t = −
7
quindi il punto comune a retta e piano é dato da

 x = − 17

y = 57 .
z = 47

Esercizio 121. Determinare il piano passante per il punto (2, 1, 1) e parallelo al


piano π : 2x − y + 3z + 5 = 0.

Svolg. Il generico piano parallelo a π ha equazione

2x − y + 3z + k = 0

al variare di k ∈ R si ottengono tutti i piani paralleli a π. Imponiamo il passaggio per


il punto (2, 1, 1): → k = −6, cioé l’equazione del piano richiesto é 2x − y + 3z − 6 =
0.
Esercizio 122. Determinare il piano contenente la retta di equazioni r : 3x + 6y −
z − 2 = x + y − 1 = 0 e ortogonale alla retta s : x = t + 2, y = 4t, z = −t + 1.
372 14 Rette e piani nello spazio affine ed euclideo.

Svolgimento: Determiniamo il fascio di piani contenente r:

3x + 6y − z − 2 + h(x + y − 1) = 0

(3 + h)x + (6 + h)y − z − 2 − h = 0
ed imponiamo la ortogonalitá con s:

3 + h = 1, 6+h = 4

da cui h = −2 e quindi il piano é

x + 4y − z = 0.

Esercizio 123. Determiniamo il piano contenente i punti (1, 0, 1), (−1, 1, 1) e


parallelo alla retta r : 2x + y − 1 = x − y − 2 = 0.

Svolgimento: I parametri direttori della retta r sono (0, 0, 1), per cui il piano richie-
sto contiene i tre punti in coordinate omogenee (1, 0, 1, 1), (−1, 1, 1, 1), (0, 0, 1, 0):

x1 x2 x3 x4

1 0 1 1
π : =0
−1 1 1 1

0 0 1 0

π: x1 + 2x2 − x4 = 0
x + 2y − 1 = 0.
Esercizio 124. Determinare la distanza tra il punto P = (0, 2, 1) ed il piano π di
equazione 3x + 2y − z + 2 = 0.

Svolgimento:
|3 · 0 + 2 · 2 + (−1) · 1 + 2| 5
δ (P, π) = √ =√ .
9+4+1 14
Esercizio 125. Determinare la distanza tra i due piani π : 2x + y + 2z − 1 = 0 e
π 0 : 4x + 2y + 4z + 7 = 0.

Svolgimento: Riscriviamo π 0 : 2x + y + 2z + 72 = 0.

| 7 − (−1)| 9
3
δ (π, π 0 ) = √2 = √2 = .
4+1+4 9 2
Esercizio 126. Determinare la distanza tra le rette 2x + 3y − 2 = x − 1 = 0 e y =
x − z + 1 = 0.

Svolgimento: Verifichiamo se le rette sono complanari o sghembe:


14.14 Esercizi svolti. 373

2
3 0 −2
1 0 0 −1

0 =0
1 0 0
1 0 −1 1
quindi le rette sono complanari. Inoltre le terne di parametri direttori delle rette sono

(0, 0, 1) (−1, 0, −1)

per cui le rette sono incidenti e non parallele e la loro distanza é nulla.
Esercizio 127. Determinare la distanza tra le rette r : 2x + 3y − 1 = x − 1 = 0 e
s : y = x − z + 1 = 0.

Svolgimento: Verifichiamo se le rette sono complanari o sghembe:



2 3 0 −1

1 0 0 −1
0 1 0 0 6= 0


1 0 −1 1
quindi le rette sono sghembe. Inoltre le terne di parametri direttori delle rette sono

r = (0, 0, 1) s = (1, 0, 1).

Costruiamo il fascio di piani che ha come sostegno r

Fr : 2x + 3y − 1 + h(x − 1) = 0

Fr : (2 + h)x + 3y − 1 − h = 0
Il piano πr di Fr parallelo alla retta s deve contenere il punto improprio di s, s∞ =
(1, 0, 1, 0). Imponiamo il passaggio per s∞ : 2 + h = 0 ed il piano é πr : 3y + 1 = 0.
Analogamente costruiamo il fascio di piani che ha come sostegno s

Fs : y + k(x − z − 1) = 0

Fs : kx + y − kz − k = 0
Il piano πs di Fs parallelo alla retta r deve contenere il punto improprio di r, r∞ =
(0, 0, 1, 0). Imponiamo il passaggio per r∞ : k = 0 ed il piano é πs : y = 0. Allora

1
δ (r, s) = δ (πr , πs ) = .
3
Calcoliamo ora la retta di minima distanza, cioé quella ortogonale ed incidente ad
entrambe le rette date:
In coordinate parametriche il generico punto di r é Pr = (1, − 13 ,t) e quello di s é
Qs = (t 0 , 0,t 0 + 1). Il segmento Pr Qs ha componenti (t 0 − 1, 13 ,t 0 − t + 1). Inponiamo
che tale vettore sia ortogonale ad entrambe le rette:
374 14 Rette e piani nello spazio affine ed euclideo.

ortogonalitá con r : t0 − t + 1 = 0

ortogonalitá con s : t0 − 1 + t0 − t + 1 = 0
da cui t 0 = 1 e t = 2.
Quindi i punti contenuti nella retta di minima distanza sono
1
Pr = (1, , 2) Qs = (1, 0, 2)
3
e la retta di minima distanza é x − 1 = y − 2 = 0.
Esercizio 128. Determinare la distanza tra il punto P = (1, 1, 3) e la retta r di
equazioni x = 2t, y = 1 − 2t, z = t.

Svolgimento: Scegliamo Q = (0, 1, 0) sulla retta r. Il segmento PQ ha componenti


(1, 0, 3). Consideriamo la matrice
   
1 0 3 1 0 3
A= = .
l mn 2 −2 1

La distanza tra punto P e la retta r é


q
A211 + A212 + A213
δ (P, r) = √
l 2 + m2 + n 2
in cui Ai j sono i minori di ordine 2 di A.
√ √
4 + 25 + 36 65
δ (P, r) = √ = .
4+4+1 3

Un altro metodo potrebbe essere il seguente:


Determiniamo la stella di piani passante per P:

a(x − 1) + b(y − 1) + c(z − 3) = 0

e tra tutti questi piani scegliamo l’unico ortogonale alla retta r, cioé a = 2, b = −2,
c = 1:
π : 2x − 2y + z − 3 = 0.
Il punto di intersezione tra r e π é Q = ( 10 1 5
9 , − 9 , 9 ). La distanza tra r e P é pari alla
distanza tra P e Q:
r √
1 100 484 65
δ (P, r) = δ (P, Q) = + + = .
81 81 81 3
14.14 Esercizi svolti. 375

Esercizio 129. Determinare la distanza tra le rette r : x − 2z + 1 = y − 3z = 0 e


s : x − z − 2 = y − 2z + 3 = 0.

Svolgimento: Verifichiamo se le rette sono complanari o sghembe:



1 0 −2 −1

0 1 −3 0
1 0 −1 −2 6= 0


0 1 −2 3
quindi le rette sono sghembe. Inoltre la terna di parametri direttori della retta s é
(1, 2, 1). Costruiamo il fascio di piani che ha come sostegno r

Fr : x − 2z + 1 + h(y − 3z) = 0

Fr : x + hy + (−2 − 3h)z + 1 = 0.
Il piano πr di Fr parallelo alla retta s deve contenere il punto improprio di s, s∞ =
(1, 2, 1, 0). Imponiamo il passaggio per s∞ : 1 + h = 0 ed il piano é πr : x − y + z +
1 = 0.
Scegliamo ora un punto Q a piacere su s: Q = (2, −3, 0). La distanza tra s e r é pari
alla distanza tra Q e πr :

|2 + 3 + 1| 6 √
δ (r, s) = δ (Q, πr ) = √ = √ = 2 3.
3 3
Calcoliamo ora la retta di minima distanza, cioé quella ortogonale ed incidente ad
entrambe le rette date:
In coordinate parametriche il generico punto di r é Pr = (2t − 1, 3t,t) e quello di s é
Qs = (t 0 + 2, 2t 0 − 3,t 0 ). Il segmento Pr Qs ha componenti (2t −t 0 − 3, 3t − 2t 0 + 3,t −
t 0 ). Inponiamo che tale vettore sia ortogonale ad entrambe le rette:

ortogonalitá con r : 14t − 9t 0 + 3 = 0

ortogonalitá con s : 9t − 6t 0 + 3 = 0
da cui t = 3 e t 0 = 5.
Quindi i punti contenuti nella retta di minima distanza sono

Pr = (5, 9, 3) Qs = (7, 7, 5)

e la retta di minima distanza é



 x = t +5
y = −t + 9 .
z = t +3

376 14 Rette e piani nello spazio affine ed euclideo.

Esercizio 130. Determinare il piano passante per i punti P = (1, 0, 1), Q = (−1, 1, 1)
e parallelo alla retta r : 2x + y − 1 = x − y − 2 = 0.

Svolgimento: I parametri direttori della retta r sono (0, 0, 1) e quelli della retta
contenente il segmento PQ sono (2, −1, 0). Il piano richiesto é quello contenente i
vettori (0, 0, 1), (2, −1, 0) e passante per il punto P (oppure Q):

x−1 y z−1

0 0 1 =0

2 −1 0

oppure analogamente é il piano contenente i tre punti, in coordinate omogenee,


(0, 0, 1, 0), (2, −1, 0, 0), (1, 0, 1, 1):

x1 x2 x3 x4

0 0 1 0
2 −1 0 0 = 0


1 0 1 1

cioé x1 + 2x2 − x4 = 0, o equivalentemente x + 2y − 1 = 0.


Esercizio 131. Determinare il piano passante per il punto P = (0, 1, 1) e parallelo
alle rette r : x = 3t + 1, y = t − 2, z = 3 e s : x − 2z = y − 3z + 1 = 0.

Svolgimento: I parametri direttori delle due rette sono:

r : (3, 1, 0) s : (2, 3, 1).

Il piano richiesto é quello contenente i vettori (3, 1, 0), (2, 3, 1) e passante per il
punto P:
x y−1 z−1

3 1
0 = 0
2 3 1
oppure analogamente é il piano contenente i tre punti, in coordinate omogenee,
(3, 1, 0, 0), (2, 3, 1, 0), (0, 1, 1, 1):

x1 x2 x3 x4

3 1 0 0
2 3 1 0 =0


0 1 1 1

cioé x1 − 3x2 + 7x3 − 4x4 = 0, o equivalentemente x − 3y + 7z − 4 = 0.


Esercizio 132. Determinare la retta contenente il punto P = (0, 0, 3) e parallela alla
retta r : x = 1 − t, y = 2, z = 1 + 2t.

Svolgimento: I parametri direttori della retta sono identici a quelli della retta r,
14.14 Esercizi svolti. 377

quindi la sua equazione in forma parametrica é data da:



 x = −t
y=0
z = 2t + 3

ed in forma cartesiana, eliminando t dalle precedenti equazioni:



y=0
.
2x + z − 3 = 0

Esercizio 133. Determinare la retta passante per i punti P = (1, 1, 0, 0) e Q =


(0, 0, 1, 0).

Svolg. Sia (x1 , x2 , x3 , x4 ) il generico punto appartenente alla retta richiesta. La


condizione di allineamento di tale punto con i punti P e Q é che la matrice
 
x1 x2 x3 x4
1 1 0 0
0 0 1 0

abbia rango 2, cioé


x1 x2 x3

1 1 0 =0
x1 − x2 = 0.
0 0 1
Inoltre la retta certamente appartiene al piano improprio x4 = 0, quindi una sua
espressione cartesiana é la seguente:

x4 = 0
.
x1 − x2 = 0

Esercizio 134. Determinare la retta s passante per il punto P = (−1, 2, 3) e tale da


essere complanare ed ortogonale alla retta r : 2x + y = 2x − z − 1 = 0.

Svolgimento: Determiniamo la retta in forma cartesiana. Il primo piano é quello


appartenente al fascio di asse r e passante per P:

Fr : 2x + y + h(2x − z − 1) = 0

ed imponendo il passaggio per P si ottiene h = 0, quindi il primo piano é 2x + y = 0.


Il secondo piano é quello appartenente alla stella di centro P e ortogonale alla retta
r:
SP : a(x + 1) + b(y − 2) + c(z − 3) = 0
ed imponendo l’ortogonalitá con r, i cui parametri direttori sono (−1, 2, −1), si
ottiene a = −1, b = 2, c = −1, quindi il secondo piano é x − 2y + 2z − 1 = 0. La
retta s é allora:
378 14 Rette e piani nello spazio affine ed euclideo.

2x + y = 0
.
x − 2y + 2z − 1 = 0
Capitolo 15
Le superfici quadriche.

15.1 Definizione.

Una quadrica é una superficie nello spazio A3 , luogo dei punti P = (x, y, z) le cui
coordinate soddisfano ad un’equazione del tipo

a11 x2 + a22 y2 + a33 z2 + a44 + 2a12 xy + 2a13 xz + 2a23 yz+

+2a14 x + 2a24 y + 2a34 z = 0.


In coordinate omogenee l’equazione di una quadrica é data da

a11 x12 + a22 x22 + a33 x32 + a44 x42 + 2a12 x1 x2 + 2a13 x1 x3 + 2a23 x2 x3 +

+2a14 x1 x4 + 2a24 x2 x4 + 2a34 x3 x4 = 0.


Osserviamo immediatamente che se π : ax + by + cz + d = 0 e π 0 : a0 x + b0 y + c0 z +
d 0 = 0 sono due piani, allora

π ∪ π0 : (ax + by + cz + d) · (a0 x + b0 y + c0 z + d 0 ) = 0

é l’equazione di una quadrica, che é detta ridotta nell’unione dei due piani.
Indichiamo con  
x1
 x2 
X = 
 x3 
x4
il generico vettore delle coordinate omogenee e sia
 
a11 a12 a13 a14
 a12 a22 a23 a24 
A= 
 a13 a23 a33 a34 
a14 a24 a34 a44

379
380 15 Le superfici quadriche.

la matrice simmetrica formata dai coefficienti che compaiono nell’equazione della


quadrica. Allora l’equazione puó essere riscritta in forma compatta come segue:

X T · A · X = 0.

Si noti che ogni retta dello spazio interseca una superficie quadrica esattamente in
due punti, eccetto il caso in cui la retta appartenga completamente alla quadrica (nel
qual caso i punti di incontro tra la retta e la quadrica sono infiniti, ovvero tutti quelli
della retta). Analogamente a quanto detto per i punti di una curva algebrica nel piano
affine o euclideo, un punto P0 di una superficie quadrica é detto doppio se, per una
qualsiasi retta r passante per P0 , si verifica una delle seguenti:
1. r é completamente contenuta nella superficie;
2. r interseca la superficie solo in P0 , ovvero r ha una intersezione doppia con la
quadrica nel punto stesso.
Determiniamo una prima classificazione delle quadriche in base alla presenza di
punti doppi:
Teorema 15.1. Sia Q : X T · A · X = 0 una quadrica. Essa contiene almeno un punto
doppio se e solo se det(A) = 0.
In particolare diciamo quadriche generali quelle che non presentano punti doppi
(det(A) 6= 0); diciamo quadriche specializzate quelle che presentano un solo punto
doppio; diciamo quadriche riducibili quelle che presentano infiniti punti doppi.

Concludiamo questa prima sezione osservando che l’intersezione tra un piano π :


ax + by + cz + d = 0 ed una quadrica Q : X T · A · X = 0 determina una curva nello
spazio, piú precisamente una conica che giace sul piano π, la cui equazione si scrive

ax + by + cz + d = 0
X T · A · X = 0.

Tale conica é riducibile solo quando il piano π é tangente alla quadrica, oppure nel
caso la quadrica sia ridotta nell’unione di due piani.

15.2 Quadriche generali.

Queste sono le quadriche prive di punti doppi. In presenza di una quadrica generale
Q : X T · A · X = 0, si definisce una corrispondenza biunivoca tra l’insieme di tutti
i punti dello spazio e l’insieme di tutti i piani dello spazio, tale che ad ogni punto
P = (a1 , a2 , a3 , a4 ) corrisponda il piano di equazione
   
a11 a12 a13 a14 x1
   a12 a22 a23 a24   x2 
π: a1 a2 a3 a4 ·  a13 a23 a33 a34  ·  x3  = 0.
  

a14 a24 a34 a44 x4


15.2 Quadriche generali. 381

Il punto P é detto polo del piano π rispetto alla quadrica Q, il piano π é detto piano
polare di P rispetto alla quadrica Q.
In particolare, se P ∈ Q é un punto della quadrica, il suo piano polare, rispetto a Q,
coincide col piano tangente in P alla quadrica.

Fissiamo ora una quadrica Q : X T · A · X = 0 ed un punto P ∈ Q. Tramite la polaritá


appena definita, é possibile costruire il piano tangente π in P alla quadrica. L’inter-
sezione tra Q e π genera una conica γ riducibile. Se γ é ridotta in due rette reali e
distinte, il punto P é detto iperbolico. Se γ é ridotta in due rette reali e coincidenti,
il punto P é detto parabolico. Se γ é ridotta in due rette immaginarie e coniugate, il
punto P é detto ellittico.
Una quadrica generale non possiede punti parabolici, inoltre vale il seguente:
Teorema 15.2. Sia Q una quadrica generale. Se essa possiede un punto iper-
bolico (rispettivamente ellittico) allora ogni punto della quadrica é iperbolico
(rispettivamente ellittico).
Le quadriche generali a loro volta si suddividono in tre classi. Tale classificazione
viene fatta in base allo studio della conica che scaturisce dall’intersezione tra la
quadrica ed il piano improprio x4 = 0.
Iperboloide.

É una quadrica generale che interseca il piano improprio in una conica irriducibile
e reale (si dice che il piano improprio e l’iperboloide sono secanti l’un l’altro).
Se i punti dell’iperboloide sono tutti iperbolici, esso é detto iperboloide iperbolico,
se i punti sono tutti ellittici esso é detto iperboloide ellittico.
Teorema 15.3. Un iperboloide é iperbolico se e solo se det(A) > 0, é ellittico se e
solo se det(A) < 0.
L’iperboloide possiede tutte le sezioni piane, cioé si possono generare sia iperboli
che parabole che ellissi, intersecando un iperboloide con i piani dello spazio.

Paraboloide.

É una quadrica generale che interseca il piano improprio in una conica riducibile (si
dice che il piano improprio é tangente al paraboloide).
Teorema 15.4. Una quadrica generale é tangente al piano improprio (quindi é un
paraboloide) se e solo se il complemento algebrico A44 , dell’elemento a44 della
matrice A, é nullo.
Se i punti del paraboloide sono tutti iperbolici, esso é detto paraboloide iperbolico,
se i punti sono tutti ellittici esso é detto paraboloide ellittico.
Teorema 15.5. Un paraboloide é iperbolico se e solo se det(A) > 0, é ellittico se e
solo se det(A) < 0.
382 15 Le superfici quadriche.

Teorema 15.6. Un paraboloide iperbolico possiede iperboli o parabole come se-


zioni piane, cioé si possono generare sia iperboli che parabole, intersecando un
paraboloide iperbolico con i piani dello spazio.
Teorema 15.7. Un paraboloide ellittico possiede ellissi o parabole come sezioni
piane, cioé si possono generare sia ellissi che parabole, intersecando un paraboloi-
de ellittico con i piani dello spazio.

Ellissoide.

É una quadrica generale che interseca il piano improprio in una conica irriducibile
immaginaria (si dice che il piano improprio é esterno all’ ellissoide).
I punti di un ellissoide sono tutti ellittici.
Teorema 15.8. Un ellissoide possiede punti tutti reali se e solo se det(A) < 0, ed é
detto ellissodie reale, in caso contrario é detto ellissoide immaginario.
Teorema 15.9. Un ellissoide possiede solamente ellissi come sezioni piane, cioé
si possono generare solamente ellissi, intersecando un ellissoide con i piani dello
spazio.
Esempio 213. Classificare la quadrica di equazione x2 + y2 − z2 + xy + x − 3 = 0.

Svolg. La matrice associata alla quadrica é


 1
1 2 0 12

1 1 0 0 
A= 2
 0 0 −1 0  .

1
2 0 0 −3

Otteniamo che det(A) > 0 con A44 6= 0, dove A44 é il complemento algebrico
dell’elemento a44 della matrice A.
Inoltre la conica impropria della quadrica é data da:
 2
x1 + x22 − x32 + x1 x2 = 0
x4 = 0

ed é a punti reali. Allora la superficie é un iperboloide a punti iperbolici. t


u
Esempio 214. Classificare la quadrica di equazione 2x2 + y2 + z2 − x − 2y + 1 = 0.

Svolg. La matrice associata alla quadrica é

2 0 0 − 21
 
 0 1 0 −1 
A=  0 0 1 0 .

− 12 −1 0 1
15.3 Quadriche specializzate. 383

Otteniamo che det(A) < 0 con A44 6= 0.


Inoltre la conica impropria della quadrica é data da:
 2
2x1 + x22 + x32 = 0
x4 = 0

ed é a punti immaginari. Allora la superficie é un ellissoide a punti reali. t


u
Esempio 215. Classificare la quadrica di equazione x2 + y2 + 2xy + 2xz + 2yz − 2x +
2 = 0.

Svolg. La matrice associata alla quadrica é


 
1 11 −1
 1 11 0 
A=  1 10
.
0 
−1 0 0 2

Otteniamo che det(A) > 0 con A44 = 0.


Allora la superficie é un paraboloide a punti iperbolici. t
u

15.3 Quadriche specializzate.

Sono le quadriche con un solo punto doppio, il quale é detto vertice. Per quanto
riguarda i punti delle superfici specializzate, vale il seguente:
Teorema 15.10. Se Q é una quadrica specializzata, allora tutti i punti della
superficie Q sono parabolici.

Anche le quadriche specializzate si suddividono in sottoclassi, e tale classificazione


si riferisce alla conica impropria della quadrica, cioé la conica che scaturisce con
l’intersezione col piano improprio.

15.3.1 Cono.

Il cono é la quadrica specializzata la cui conica impropria é irriducibile (il cono ed


il piano improprio sono secanti). Il vertice V del cono é un punto proprio e le sue
coordinate (x1 , x2 , x3 , x4 ) sono la soluzione del sistema lineare
   
a11 a12 a13 a14 x1
 a12 a22 a23 a24   x2 
 a13 a23 a33 a34  ·  x3  = 0.
   

a14 a24 a34 a44 x4


384 15 Le superfici quadriche.

Si riconosce che la superficie é un cono utilizzando il seguente risultato:


Teorema 15.11. Sia A la matrice associata alla quadrica Q. La superficie é un cono
se e solo se det(A) = 0, rango(A) = 3 e A44 6= 0.

Un cono possiede tutte le sezioni piane, cioé si possono generare sia iperboli che
parabole che ellissi, intersecando un cono con i piani dello spazio.
Inoltre tutti i piani tangenti alla superficie di un cono devono contenerne il vertice.

15.3.2 Cilindro.

Il cilindro é la quadrica specializzata la cui conica impropria é riducibile (il cilindro


ed il piano improprio sono tangenti). Il vertice V del cilindro é un punto improprio
e le sue coordinate (x1 , x2 , x3 , 0) sono la soluzione del sistema lineare
   
a11 a12 a13 a14 x1
 a12 a22 a23 a24   x2 
 a13 a23 a33 a34  ·  x3  = 0.
   

a14 a24 a34 a44 0

Si riconosce che la superficie é un cilindro utilizzando il seguente risultato:


Teorema 15.12. Sia A la matrice associata alla quadrica Q. La superficie é un
cilindro se e solo se det(A) = 0, rango(A) = 3 e A44 = 0.

Ogni cilindro possiede un solo tipo di sezione piana indipendentemente dal piano
col quale si effettua l’intersezione. Cioé, fissato il cilindro Q, si possono generare
solo iperboli o parabole o ellissi, intersecando il cilindro con i piani dello spazio.
In particolare, poiché il piano improprio é tangente al cilindro, la conica impropria
ridotta dovrá rispettare il seguente prospetto:

Conica impropria Generica sezione del cilindro


2 rette reali e distinte Iperbole
2 rette reali e coincidenti Parabola
2 rette immaginarie coniugate Ellisse

Inoltre tutti i piani tangenti alla superficie di un cilindro devono contenerne il vertice.
Esempio 216. Classificare la quadrica di equazione x2 + y2 + 2xy + 2yz − 2x + y +
1 = 0.

Svolg. La matrice associata alla quadrica é


15.4 Quadriche riducibili. 385
 
1 1 0 −1
 1 11 1 
A= 2 
 0 1 0 0 .
−1 12 0 1

Otteniamo che det(A) = 0 con A44 6= 0.


La superficie é un cono di vertice X = (x1 , x2 , x3 , x4 ) tale che AX = 0, da cui

 x1 + x2 − x4 x= 0

x1 + x2 + x3 + 24 = 0

 x2 = 0
−x1 + x22 + x4 = 0

e quindi X = (1, 0, − 32 , 1). t


u
Esempio 217. Classificare la quadrica di equazione x2 + y2 + 2xy − 2x + y + 1 = 0.

Svolg. La matrice associata alla quadrica é


 
1 1 0 −1
 1 10 1 
A= 2 
 0 0 0 0 .
−1 12 0 1

Otteniamo che det(A) = 0 con A44 = 0 e rango 3.


La superficie é un cilindro di vertice X = (x1 , x2 , x3 , x4 ) tale che AX = 0, da cui

x1 + x2 = 0
−x1 + x22 = 0

e quindi X = (0, 0, 1, 0).


Inoltre la conica impropria della quadrica é data da:
 2
x1 + x22 + 2x1 x2 = 0
x4 = 0

(x1 + x2 )2 = 0


x4 = 0
cioé sono due rette reali e coincidenti, quindi la conica generatrice del cilindro é una
parabola. t
u

15.4 Quadriche riducibili.

Sono le quadriche di equazione


386 15 Le superfici quadriche.

a11 x2 + a22 y2 + a33 z2 + a44 + 2a12 xy + 2a13 xz + 2a23 yz+

+2a14 x + 2a24 y + 2a34 z = 0


tali che
a11 x2 + a22 y2 + a33 z2 + a44 + 2a12 xy + 2a13 xz + 2a23 yz+
+2a14 x + 2a24 y + 2a34 z = (ax + by + cz + d) · (a0 x + b0 y + c0 z + d 0 )
cioé sono unione di due piani π e π 0 di equazioni:

π : ax + by + cz + d = 0 π 0 : a0 x + b0 y + c0 z + d 0 = 0.

Tali quadriche hanno infiniti punti doppi, sono i punti della retta comune ai due piani
π e π 0.
In particolare si possono verificare i seguenti 3 casi:
1. i due piani sono distinti ed incidenti, ed allora det(A) = 0, rango(A) = 2, vi sono
∞1 punti doppi propri;
2. i due piani sono distinti e paralleli, ed allora det(A) = 0, rango(A) = 2, vi sono
∞1 punti doppi impropri;
3. i due piani sono coincidenti, ed allora det(A) = 0, rango(A) = 1, vi sono ∞2 punti
doppi propri.
Aggiungiamo come caso ’molto particolare’ quello in cui la quadrica ha equazione
x42 = 0, cioé vi sono ∞2 punti doppi impropri.

Consideriamo π 00 : a00 x + b00 y + c00 z + d 00 = 0 un qualsiasi altro piano dello spazio.


L’intersezione della quadrica riducibile con π 00 determina una conica ridotta nelle
due rette:
 0
a x + b0 y + c0 z + d 0 = 0

ax + by + cz + d = 0
r1 : 00 00 00 00 r2 :
a x+b y+c z+d = 0 a00 x + b00 y + c00 z + d 00 = 0
Nota 15.13. In tutto ció che segue, i luoghi gemetrici analizzati saranno definiti
nello spazio euclideo E 3 .

15.5 Piani ed assi di simmetria delle quadriche generali.

Definizione 15.14. Sia A = (ai j ) ∈ M4 (R) la matrice non singolare associata all’e-
quazione di una quadrica generale. Si definisce centro di simmetria della quadrica
Q, il punto C ∈ Q tale che, ogni punto P della superficie ha un simmetrico rispetto a
C che é ancora un punto della superficie.
Analogamente a quanto visto per le coniche, le coordinate del centro di simme-
tria si ottengono direttamante dalla matrice A: C = (A41 , A42 , A43 , A44 ), dove ogni
Ai j é il complemento algebrico dell’elemento ai j ∈ A considerato con l’opportu-
no segno (derivante dalla posizione che esso occupa nella matrice). Nel caso di un
15.5 Piani ed assi di simmetria delle quadriche generali. 387

paraboloide, essendo A44 = 0, si parla di quadrica a centro improprio o priva di


centro.
Definizione 15.15. I piani di simmetria (anche detti piani principali) di una quadri-
ca generale sono quei piani dello spazio che, rispetto alla quadrica definita, hanno
come polo il punto improprio della direzione ad essi ortogonale. Gli assi di sim-
metria della quadrica sono le rette che si ottengono dall’intersezione dei piani di
simmetria.
Ogni punto P della superficie ha un simmetrico rispetto a ciascun piano di simme-
tria, e rispetto a ciascun asse di simmetria, che é ancora un punto della superficie.
Nel caso di iperboloide ed ellissoide (quadriche a centro) si determinano 3 assi di
simmetria (in corrispondenza di 3 piani di simmetria): tali assi si possono ottenere
anche congiungendo il centro di simmetria con ciascun polo di ogni piano di sim-
metria. Nel caso del paraboloide (senza centro di simmetria) si determina 1 asse di
simmetria (in corrispondenza di 2 piani di simmetria).
Teorema 15.16. Le prime 3 coordinate dei poli dei piani di simmetria sono le com-
ponenti degli autovettori della sottomatrice A44 della matrice A rappresentante la
quadrica generale. In particolare, nel caso di un paraboloide, i due piani di sim-
metria sono i piani polari dei punti impropri ottenuti dalle prime 3 coordinate degli
autovettori relativi agli autovalori non nulli della sottomatrice A44 .
Esempio 218. Consideriamo il paraboloide iperbolico Q di equazione 6xz + 8yz −
5x = 0. Determiniamo piani ed asse di simmetria.

Svolg. É sufficiente verificare che gli autovalori della matrice A44 sono esattamente
λ = 0, 5, −5 e che i rispettivi autovettori rispetto ad A44 sono: (− 34 , 1, 0) per λ = 0,
( 35 , 54 , 1) per λ = 5, (− 35 , − 54 , 1) per λ = −5. Quindi i poli dei piani di simmetria
sono esattamente ( 35 , 54 , 1, 0), (− 35 , − 45 , 1, 0).
I piani di simmetria sono i piani polari dei punti precedente determinati, ovvero
6x + 8y + 10z − 3 = 0. e 6x + 8y − 10z + 3 = 0.
L’asse di simmetria é quindi

6x + 8y + 10z − 3 = 0
6x + 8y − 10z + 3 = 0.

Inoltre possiamo calcolare il punto di sella del paraboloide iperbolico, intersecando


l’asse con la sua superficie:

 6x + 8y + 10z − 3 = 0
6x + 8y − 10z + 3 = 0
6xz + 8yz − 5x = 0

t
u
Osservazione 15.17. Il calcolo del punto di sella di un paraboloide iperboli-
co é analogo al calcolo del punto del parabolide ellittico che potremmo definire
impropriamente ’vertice del parabolide’.
388 15 Le superfici quadriche.

15.6 Forma canonica di una quadrica generale con centro di


simmetria.

Sia X t AX = 0 l’equazione di una quadrica generale con centro di simmetria (iperbo-


loide o ellissoide). Una rototraslazione degli assi di simmetria che riporti l’origine
del sistema di riferimento nel centro di simmetria e faccia coincidere gli assi coordi-
nati con gli assi di simmetria della quadrica, trasforma l’equazione della superficie
nella sua forma canonica:
Per quanto riguarda l’Ellissoide avremo:
2
x2 2
1. a2
+ by2 + cz2 = 1 ellissoide reale;
2
x2 2
2. a2
+ by2 + cz2 = −1 ellissoide immaginario.
Per quanto riguarda l’Iperboloide avremo:
2
x2 2
1. a2
+ by2 − cz2 = 1 iperboloide iperbolico;
2 2 2
2. ax2 − by2 + cz2 = 1 iperboloide iperbolico;
2 2 2
3. - ax2 + by2 + cz2 = 1 iperboloide iperbolico;
2 2 2
4. ax2 − by2 − cz2 = 1 iperboloide ellittico;
2 2 2
5. - ax2 + by2 − cz2 = 1 iperboloide ellittico;
2 2 2
6. - ax2 − by2 + cz2 = 1 iperboloide ellittico.
Diciamo X t BX = 0 l’equazione della quadrica nella sua forma ridotta. Come ac-
cadeva per le coniche, si puó dimostrare (non lo facciamo!) che esistono alcune
quantitá che non cambiano dopo aver effettuato un cambiamento di base tramite
vettori ortonormali: vengono detti invarianti ortogonali e sono

det(A) = det(B)

det(A44 ) = det(B44 )
la somma degli autovalori di A44 é pari alla somma degli autovalori di B44 .
In altre parole, se indichiamo con αx2 +β y2 +γz2 = δ l’equazione di una quadrica a
centro, é sufficiente calcolare gli autovalori di A44 per ottenere i coefficienti α, β , γ.
Esempio 219. Consideriamo l’iperboloide ellittico di equazione 2x2 − 2y2 − 2yz −
2z2 − 3 = 0 di matrice associata:
 
2 0 0 0
 0 −2 −1 0 
A= 0 −1 −2 0  .

0 0 0 3

Determiniamone una forma canonica.


15.7 Forma canonica di un paraboloide. 389

Svolg. Una forma canonica é ax2 + by2 + cz2 = d, dove a, b, c sono gli autovalori
della sottomatrice A44 . Eseguendo i calcoli, essi sono 2, −3, −1, quindi l’equazione
sará 2x2 − 3y2 − z2 = d con matrice associata
 
2 0 0 0
 0 −3 0 0 
B=  0 0 −1 0 

0 0 0 −d

di determinante −6d. Inoltre é facile calcolare che det(A) = −18 e grazie all’inva-
rianza ortogonale sappiamo che 6d = 18, quindi d = 3. Infine l’equazione in forma
ridotta sará 2x2 − 3y2 − z2 − 3 = 0. t
u

15.7 Forma canonica di un paraboloide.

Sia X t AX = 0 l’equazione di un paraboloide. Come in precedenza una trasforma-


zione ortonormale del sistema di riferimento trasforma l’equazione della superficie
nella sua forma canonica. Facciamo riferimento ai paraboloidi che abbiano nella
loro forma ridotta, l’asse di simmetria coincidente con l’asse Z, in modo del tutto
identico il discorso puó essere riportato agli altri 2 casi:

2
x2
1. a2
+ by2 = 2dz paraboloide ellittico;
2
x2
2. a2
− by2 = 2dz paraboloide iperbolico.
Diciamo X t BX = 0 l’equazione della quadrica nella sua forma ridotta. Come in
precedenza i seguenti sono invarianti ortogonali:

det(A) = det(B)

det(A44 ) = det(B44 )
la somma degli autovalori di A44 é pari alla somma degli autovalori di B44 .
Esempio 220. Consideriamo il paraboloide iperbolico di equazione 6xz + 8yz −
5x = 0 di matrice associata:

0 0 3 − 52
 
 0 04 0 
A=
 3 4 0 0 .

− 25 0 0 0

Determiniamone una forma canonica.

Svolg. Una forma canonica con asse di simmetria coincidente con l’asse Z é
ax2 + by2 + 2cz = 0, dove a, b sono gli autovalori non nulli della sottomatrice A44
390 15 Le superfici quadriche.

(ricordiamo che per un paraboloide l’autovalore nullo esiste sempre ma non forni-
sce indicazioni, poiché l’autovettore ad esso relativo é il polo del piano improprio).
Eseguendo i calcoli, gli autovalori non nulli sono 5, −5, quindi l’equazione sará
5x2 − 5y2 + 2cz = 0 con matrice associata
 
5 0 00
 0 −5 0 0 
B= 0 0 0 c

0 0 c0

di determinante 25c2 . Inoltre é facile calcolare che det(A) = 100 e grazie all’inva-
rianza ortogonale sappiamo che 25c2 = 100, quindi c ∈ {−2, 2}. Infine le equazioni
in forma ridotta saranno
5x2 − 5y2 − 4z = 0
oppure
5x2 − 5y2 + 4z = 0.
t
u

15.8 Forma canonica di un cono.

Sia X t AX = 0 l’equazione di un cono, con matrice associata A e vertice V di


coordinate XV = (xv , yv , zv ). Quanto detto sulle quadriche a centro proprio (iper-
boloidi ed ellessoidi) puó essere applicato anche per la riduzione a forma canonica
dell’equazione di un cono. L’equazione ridotta assume una delle seguenti forme:
1. a2 x2 + b2 y2 + c2 z2 = 0 nel caso di un cono immaginario (l’unico punto reale é il
vertice);
2. a2 x2 +b2 y2 −c2 z2 = 0 nel caso di un cono reale (con asse perpendicolare al piano
z = 0).
I coefficienti a2 , b2 , c2 sono gli autovalori della A44 .
Esempio 221. Consideriamo il cono immaginario di equazione 3x2 + 2y2 + 2xz +
3z2 = 0 di matrice associata:
 
3010
0 2 0 0
A= 1 0 3 0 .

0000

Determiniamone una forma canonica e le formule di rototraslazione che ci permet-


tono di ottenerla.
15.9 La sfera. 391

Svolg. Il vertice del cono ha coordinate (0, 0, 0). Gli autovalori della sottomatrice
A44 sono λ1 = 2, con molteplicitá algebrica 2, e λ2 = 4. Una forma canonica é
2x2 + 2y2 + 4z2 = 0. t
u
Nota 15.18. Per non appesantire oltremodo la discussione sulle superfici quadri-
che, tralasciamo volutamente l’analisi dei metodi necessari per ottenere una forma
canonica dell’equazione di un cilindro, poiché essa richiederebbe l’introduzione di
ulteriori nozioni e tecniche.

15.9 La sfera.

Siano C = (α, β , γ) un punto dello spazio Euclideo e r ∈ R con r > 0. La sfera di


centro C e raggio r é una superficie, luogo dei punti P = (x, y, z) dello spazio la cui
distanza da C é pari a r. Da tale definizione ricaviamo che la sua equazione é:
q
(x − α)2 + (y − β )2 + (z − γ)2 = r

da cui
(x − α)2 + (y − β )2 + (z − γ)2 = r2
x2 + y2 + z2 + ax + by + cz + d = 0
dove
a b c
C = (α, β , γ) = (− , − , − )
2 2 2
1p 2
r= a + b2 + c2 − 4d > 0.
2
Esempio 222. Determinare centro e raggio della sfera x2 + y2 + z2 − 3x + 5y − z +
1 = 0.

Svolg. Il centro della sfera data é:


3 5 1
C = ( ,− , )
2 2 2
ed il raggio: √ √
9 + 25 + 1 − 4 31
r= = .
2 2
L’equazione della sfera si puó infatti scrivere anche:
3 5 1 31
(x − )2 + (y + )2 + (z − )2 = .
2 2 2 4
t
u
Sezioni piane.
392 15 Le superfici quadriche.

Siano S : x2 + y2 + z2 + ax + by + cz + d = 0 una sfera di centro C e raggio r e


π : ex + f y + gz + h = 0 un piano. Indichiamo con δ = δ (C, π) la distanza tra il
centro della sfera ed il piano π.
Se δ > r allora il piano é esterno alla sfera. La loro intersezione genera una
circonferenza non riducibile ed immaginaria.

Se δ = r allora il piano é tangente alla sfera. La loro intersezione genera una


circonferenza ridotta in due rette immaginarie coniugate.

Se δ < r allora il piano é secante la sfera. La loro intersezione genera una circonfe-
renza reale, le cui equazioni sono date dal sistema:

S : x2 + y2 + z2 + ax + by + cz + d = 0


π : ex + f y + gz + h = 0.

Il centro C0 della circonferenza é dato dall’intersezione s ∩ π, dove s é la retta pas-


sante√per il centro della sfera ed ortogonale a π; il raggio della circonferenza é
r0 = r2 − δ 2 .

Poiché la sfera é un particolare ellissoide, possiamo utilizzare la polaritá definita per


le quadriche irriducibili per calcolare un qualsiasi piano tangente ad una sfera in un
suo punto.
Fascio di sfere.
Siano S : x2 + y2 + z2 + ax + by + cz + d = 0 una sfera di raggio r e centro C e
S0 : x2 + y2 + z2 + a0 x + b0 y + c0 z + d 0 = 0 una di raggio r0 e centro C0 .
Indichiamo con δ = δ (C,C0 ) la distanza tra i due centri. Supponiamo che

|r − r0 | < δ < r + r0

allora le due sfere si intersecano in una circonferenza reale. Tale circonferenza giace
su di un piano la cui equazione si ricava sottraendo membro a membro le equazioni
delle due sfere:  2
x + y2 + z2 + ax + by + cz + d = 0
x + y2 + z2 + a0 x + b0 y + c0 z + d 0 = 0.
2

cioé
π : (a − a0 )x + (b − b0 )y + (c − c0 )z + (d − d 0 ) = 0.
Da cui otteniamo l’equazione della circonferenza che le due sfere hanno in comune:

x2 + y2 + z2 + ax + by + cz + d = 0

γ:
(a − a )x + (b − b0 )y + (c − c0 )z + (d − d 0 ) = 0.
0

Definiamo fascio di sfere secanti in γ e di piano radicale π, tutte le sfere che si


ottengono, al variare del parametro reale λ , dall’equazione

(x2 + y2 + z2 + ax + by + cz + d) + λ (x2 + y2 + z2 + a0 x + b0 y + c0 z + d 0 ) = 0
15.9 La sfera. 393

o equivalentemente dall’equazione

(x2 + y2 + z2 + ax + by + cz + d) + λ ((a − a0 )x + (b − b0 )y + (c − c0 )z + (d − d 0 )) = 0.

Se P1 é un punto non appartenente a π allora esiste una ed una sola sfera apparte-
nente al fascio e passante per P1 .

Supponiamo ora che


|r − r0 | = δ
allora le due sfere sono tangenti internamente l’un l’altra. Viceversa, se r + r0 = δ
allora le due sfere sono tangenti esternamente. In entrambi i casi le due sfere si
intersecano in una circonferenza ridotta in due rette immaginarie coniugate, con un
unico punto reale P0 , che é quello di tangenza tra le due sfere.
Indichiamo con π : ex + f y + gz + h = 0 il piano tangente ad entrambe le sfere S e
S0 in P0 .
Definiamo fascio di sfere tangenti in P0 e di piano radicale π, tutte le sfere che si
ottengono, al variare del parametro reale λ , dall’equazione

(x2 + y2 + z2 + ax + by + cz + d) + λ (x2 + y2 + z2 + a0 x + b0 y + c0 z + d 0 ) = 0

o equivalentemente dall’equazione

(x2 + y2 + z2 + ax + by + cz + d) + λ (ex + f y + gz + h) = 0.

Anche in questo caso, se P1 é un punto non appartenente a π allora esiste una ed una
sola sfera appartenente al fascio e passante per P1 .
Esempio 223. Determinare il piano tangente alla sfera x2 + y2 + z2 − 2x + 2y − 2z −
1 = 0 nel punto P = (1, 1, 1).

Svolg. Il piano tangente puó essere visto come il piano polare del punto P rispetto
alla quadrica (in questo caso la sfera) data:
   
1 0 0 −1 x1
   0 1 0 1   x2 
1 1 1 1 ·  ·  =0
0 0 1 −1   x3 
−1 1 −1 −1 x4

che si riduce a 2x2 − 2x4 = 0, cioé y − 1 = 0.

Analogamente possiamo seguire il seguente procedimento:


Il centro della sfera data é C = (1, −1, 1), quindi il vettore CP ha componenti pro-
porzionali alla terna (0, 1, 0). Il piano tangente deve essere ortogonale a tale vettore
quindi la sua equazione é:

(0)(x − 1) + (1)(y − 1) + (0)(z − 1) = 0

cioé y − 1 = 0. t
u
394 15 Le superfici quadriche.

Esempio 224. Determinare la sfera contenente la circonferenza di equazioni

γ: S∩π : x2 + y2 + z2 − 2x + 2z = y + z = 0

ed il punto P = (1, 1, 1).

Svolg. Per prima cosa verifichiamo che √ la circonferenza γ é reale; infatti il centro
di S é C = (1, 0, −1), il suo raggio é 2, e la distanza δ (C, π) = √12 é minore del
raggio di S.
Costruiamo il fascio di sfere contenenti la circonferenza γ:

Fγ : x2 + y2 + z2 − 2x + 2z + h(y + z) = 0

ed imponiamo il passaggio di tali sfere per il punto P: otteniamo h = − 23 . Quindi la


sfera richiesta é ottenuta dal fascio per tale valore di h:

2x2 + 2y2 + 2z2 − 4x + z − 3y = 0.

t
u
Esempio 225. Determinare la sfera tangente al piano π : x + y − z − 1 = 0 nel
punto P = (1, 1, 1) e passante per il punto Q = (2, 1, 0).

Svolg. La retta ortogonale a π e passante per P ha equazioni:



 x = t +1
y = t +1 .
z = −t + 1

Su tale retta si trovano i centri di ogni sfera che sia tangente a π in P. Per esempio
scegliamo t = 1, il centro
√ della sfera S1 ottenuta é C = (2, 2, 0) ed il suo raggio é
la distanza δ (P,C) = 3. La sfera ha equazione (x − 2)2 + (y − 2)2 + z2 = 3 cioé
x2 + y2 + z2 − 4x − 4y + 5 = 0. Costruiamo il fascio di sfere tangenti a π in P:

F: x2 + y2 + z2 − 4x − 4y + 5 + h(x + y − z − 1) = 0.

Imponiamo il passaggio per il punto Q, sostituendo le sue coordinate nell’equazione


del fascio F : h = 1. Quindi la sfera richiesta ha equazione

x2 + y2 + z2 − 3x − 3y − z + 4 = 0.

t
u
15.10 Esercizi svolti. 395

15.10 Esercizi svolti.

Esercizio 135. Classificare la quadrica x2 + 2y2 + z2 + 2xy − 2x − 3 = 0 ed i suoi


punti. Determinare inoltre una forma canonica della quadrica.

Svolgimento: La matrice associata alla quadrica é


 
1 1 0 −1
 1 20 0 
A=  0 0 1 0 .

−1 0 0 −3

Il determinante della matrice é pari a |A| = −5, per cui la superficie é una quadrica
generale. Determiniamo gli autovalori della sottomatrice A44 :

1−λ 1 0
1 2 − λ 0 = 0 ⇒ (1 − λ )(λ 2 − 3λ + 1) = 0.


0 0 1−λ
√ √
La radici del precedente polinomio caratteristico sono quindi {1, 3+2 5 , 3+2 5 }, per
cui gli autovalori sono tutti positivi.
Dalle informazioni ottenute, possiamo concludere che la superficie é un ellissoide
reale. √ √
Una forma canonica dell’equazione di tale quadrica é x2 + 3+2 5 y2 + 3+2 5 z2 +a044 =
0, con matrice associata

1 0√ 0 0
 
3+ 5
0 0√ 0 
A0 = 

2 .
0 3− 5
0 2 0 
0 0 0 a044

Il determinante della matrice é un invariante ortogonale per la quadrica, per cui


|A0 | = −5, ovvero √ √
3+ 5 3− 5 0
· · a44 = −5
2 2
da cui a044 = −5. Quindi l’equazione in forma canonica é
√ √
2 3+ 5 2 3+ 5 2
x + y + z − 5 = 0.
2 2

Esercizio 136. Classificare la quadrica xy + yz + xz − 2x + 1 = 0 ed i suoi punti.


Determinare inoltre una forma canonica della quadrica.

Svolgimento: La matrice associata alla quadrica é


396 15 Le superfici quadriche.
1 1
 
−1
0 2 2
 1 0 0 12
A= 2 .
 1 1
2 0 2 0
−1 0 0 1

Il determinante della matrice é pari a |A| = 21 , per cui la superficie é una quadrica
generale. Determiniamo gli autovalori della sottomatrice A44 :
−λ 1 1

1 2 21 3 3 1
2 −λ 2 = 0 ⇒ λ − 4 λ + 4 = 0.

1 1
2 −λ

2

La radici del precedente polinomio caratteristico sono quindi {−1, 21 , 12 }, per cui gli
autovalori sono tra loro discordi in segno.
Dalle informazioni ottenute, possiamo concludere che la superficie é un iperboloide
iperbolico.
Una forma canonica dell’equazione di tale quadrica é 12 x2 + 12 y2 − z2 + a044 = 0, con
matrice associata 1 
2 0 0 0
0 1 0 0 
A0 =  2
 0 0 −1 0  .

0 0 0 a044
Il determinante della matrice é un invariante ortogonale per la quadrica, per cui
|A0 | = 21 , ovvero
1 1 1
− · · a044 =
2 2 2
0
da cui a44 = −2. Quindi l’equazione in forma canonica é

1 2 1 2 2
x + y − z − 2 = 0.
2 2
Intersechiamo ora con il piano π di equazione x − y = 0. La conica ottenuta viene
espressa tramite il sistema

xy + yz + xz − 2x + 1 = 0
γ:
x−y = 0
ovvero
x2 + 2xz − 2x + 1 = 0

γ: .
x−y = 0
In particolare, la prima equazione x2 + 2xz − 2x + 1 = 0 esprime una quadrica Q0 , la
cui proiezione su π é costituita dalla stessa γ.
É sufficiente quindi classificare l’intersezione tra Q0 e π. La matrice associata a Q0 é
15.10 Esercizi svolti. 397
 
1 01 −1
 0 00 0 
B=
 
1 00 0 
−1 0 0 1

per cui Q0 é un cilindro, il cui vertice V ha coordinate (0, 1, 0, 0).


Ricordiamo che, fissato un cilindro, l’intersezione con un qualsiasi piano dello spa-
zio genera una conica la cui tipologia (parabola, iperbole, ellisse) é sempre la mede-
sima, cioé dipende dal cilindro preso in considerazione e non dal piano col quale lo
si interseca. In particolare, la tipologia di tale conica é conforme alla conica gene-
ratrice del cilindro. Fanno eccezione i piani contenenti il vertice del cilindro, le cui
sezioni sono sempre parabole ridotte.
Scegliamo di intersecare Q0 col piano improprio e classificare la conica riducibile
che scaturisce:
 2 
x1 + 2x1 x3 = 0 x1 (x1 + 2x3 ) = 0
Q0 ∩ π ∞ : ⇒ .
x4 = 0 x4 = 0

Quindi Q0 ∩ π∞ = r1 ∪ r2 , dove
 
x1 = 0 x1 + 2x3 = 0
r1 : r2 : .
x4 = 0 x4 = 0

In particolare Q0 ∩ π∞ é un’iperbole ridotta, per cui la conica generatrice del cilindro


é un’iperbole: sono tutte iperboli le intersezioni del cilindro con ogni piano dello
spazio (ad eccezione dei piani contenenti il vertice del cilindro, le cui sezioni sono
parabole ridotte).
Concludiamo quindi che anche γ é un’iperbole.
Esercizio 137. Determinare una forma canonica della quadrica Q di equazione x2 +
y2 + z2 + 2xy − 2x = 0. Determinare la conica intersezione tra la quadrica ed il piano
di equazione x = 0; ripetere l’esercizio nel caso in cui il piano abbia equazione
x + y − z = 0.

Svolgimento: La matrice associata alla quadrica é


 
1 1 0 −1
 1 10 0 
A=  0 0 1 0 .

−1 0 0 0

Il determinante della matrice é pari a |A| = −1, per cui la superficie é una quadrica
generale. Inoltre il complemento algebrico A44 é nullo, quindi la superficie é un pa-
raboloide ellittico.
Gli autovalori della sottomatrice A44 sono {0, 1, 2}. Una forma canonica dell’e-
quazione di un siffatto paraboloide ellittico é x2 + 2y2 + 2a034 z = 0, con matrice
associata
398 15 Le superfici quadriche.
 
1 0 0 0
0
0 2 0 0 
A = .
a034 

0 0 0
0 0 a034 0
Il determinante della matrice é un invariante ortogonale per la quadrica, per cui
|A0 | = −1, ovvero
−2a02
34 = −1

da cui a034 = ± √12 . Quindi l’equazione in forma canonica é

2
x2 + 2y2 + √ z = 0
2
od anche
2
x2 + 2y2 − √ z = 0.
2
Intersechiamo Q con il piano π di equazione x = 0. La conica ottenuta viene espressa
tramite il sistema  2
x + y2 + z2 + 2xy − 2x = 0
γ:
x=0
ovvero
y2 + z2

γ: .
x=0
In particolare, la prima equazione x2 + 2xz − 2x + 1 = 0 rappresenta una quadrica
Q0 che ri riduce nell’unione dei due piani complessi coniugati di equazioni rispetti-
vamente y + iz = 0 e y − iz = 0. La conica γ é quindi una ellisse ridotta nell’unione
delle due rette complesse coniugate
 
y + iz = 0 y − iz = 0
x=0 x=0

ed il piano π : x = 0 é tangente alla quadrica Q.


Intersechiamo ora Q con il piano σ di equazione x + y − z = 0. La conica ottenuta
viene espressa tramite il sistema
 2
x + y2 + z2 + 2xy − 2x = 0
γ:
z = x+y
ovvero
x2 + y2 + 2xy − x = 0

γ: .
z = x+y
In particolare, la prima equazione x2 + y2 + 2xy − x = 0 rappresenta un cilindro Q00
la cui matrice associata é
15.10 Esercizi svolti. 399

1 0 − 12
 
1
00
 1 10 0 
A = 0

00 0 
− 12 00 0
ed il cui vertice ha coordinate (0, 0, 1, 0). Anche in questo caso, possiamo clas-
sificare la conica Q ∪ σ , analizzando quale sia la conica impropria del cilindro
Q00 :  2
x1 + x22 + 2x1 x2 = 0 (x1 + x2 )2 = 0

00
Q ∩ π∞ : ⇒ .
x4 = 0 x4 = 0
Quindi Q00 ∩ π∞ = r1 ∪ r1 , dove

x1 + x2 = 0
r1 :
x4 = 0

ovvero Q00 ∩ π∞ é una parabola (ridotta). La conica generatrice del cilindro Q00 é
quindi una parabola, e tale é anche γ = Q ∩ σ = Q00 ∩ σ .
Esercizio 138. Determinare una forma canonica della quadrica Q di equazione

x2 + 4xy + y2 + 3z2 + 4x + 4y + 1 = 0.

Determinare la conica intersezione tra la quadrica ed il piano di equazione y = 1;


ripetere l’esercizio nel caso in cui il piano abbia equazione y − z = 0.

Svolgimento: La matrice associata alla quadrica é


 
1202
2 1 0 2
A=  0 0 3 0 .

2201

Il determinante della matrice é nullo ed il rango della matrice é pari a 3: la superfi-


cie é una quadrica specializzata. Inoltre il complemento algebrico A44 é non nullo,
quindi la superficie é un cono.
Gli autovalori della sottomatrice A44 sono {−1, 3, 3}. Una forma canonica dell’e-
quazione di un siffatto cono (reale) é 3x2 + 3y2 − z2 = 0
Intersechiamo Q con il piano π di equazione y = 1. La conica ottenuta viene espressa
tramite il sistema
 2
x + 4xy + y2 + 3z2 + 4x + 4y + 1 = 0
γ:
y=1
ovvero
x2 + 3z2 + 8x + 6 = 0

γ: .
y=1
In particolare, la prima equazione x2 + 3z2 + 8x + 6 = 0 rappresenta un cilindro Q0
400 15 Le superfici quadriche.

la cui matrice associata é  


1 0 0 4
0 0 0 0
A0 = 


0 0 3 0
4 0 0 6
ed il cui vertice ha coordinate (0, 1, 0, 0). Classifichiamo la conica Q ∪ π, analizzan-
do quale sia la conica impropria del cilindro Q0 :
 2 √ √
x1 + 3x32 = 0

0 (x1 + i 3x3 ) · (x1 + i 3x3 ) = 0
Q ∩ π∞ : ⇒ .
x4 = 0 x4 = 0

Quindi Q0 ∩ π∞ = r1 ∪ r2 , dove
 √  √
x1 + i 3x3 = 0 x1 − i 3x3 = 0
r1 : r2 :
x4 = 0 x4 = 0

ovvero Q00 ∩ π∞ é una ellisse (ridotta in due rette complesse coniugate).


La conica generatrice del cilindro Q0 é quindi una ellisse, e tale é anche γ = Q ∩ π =
Q0 ∩ π.
Intersechiamo ora Q con il piano σ di equazione y − z = 0. La conica ottenuta viene
espressa tramite il sistema
 2
x + 4xy + y2 + 3z2 + 4x + 4y + 1 = 0
γ2 :
z=y
ovvero
x2 + 4xy + 4y2 + 4x + 4y + 1 = 0

γ2 : .
z=y
In particolare, la prima equazione x2 + 4xy + 4y2 + 4x + 4y + 1 = 0 esprime un
cilindro Q00 la cui matrice associata é
 
1202
2 4 0 2
A00 =  
0 0 0 0
2201

ed il cui vertice ha coordinate (0, 0, 1, 0). Anche in questo caso, possiamo classifi-
care la conica γ2 = Q ∪ σ , analizzando quale sia la conica impropria del cilindro
Q00 :  2
x1 + 4x1 x2 + 4x22 = 0 (x1 + 2x2 )2 = 0

Q00 ∩ π∞ : ⇒ .
x4 = 0 x4 = 0
Quindi Q00 ∩ π∞ = r1 ∪ r1 , dove

x1 + 2x2 = 0
r1 :
x4 = 0
15.10 Esercizi svolti. 401

ovvero Q00 ∩ π∞ é una parabola (ridotta). La conica generatrice del cilindro Q00 é
quindi una parabola, e tale é anche γ2 = Q ∩ σ = Q00 ∩ σ .
Esercizio 139. Determinare la sfera passante per il punto (1, 2, 1) e per la circonfe-
renza γ : x2 + y2 + z2 − 2x − 3 = 4x − 11 = 0.

Svolgimento: Determiniamo dapprima l’equazione che rappresenta l’insieme in-


finito di sfere che hanno in comune fra loro tutti e soli i punti della circonferenza
(oltre che i punti del cerchio assoluto). Essa si ottiene a partire dalle equazioni di 2
di tali sfere, oppure anche (ed é il nostro caso) dall’equazione di una di tali sfere e
da quella del piano su cui giace la circonferenza. Tale insieme di sfere é detto fascio
di sfere secanti in γ:

Fγ : x2 + y2 + z2 − 2x − 3 + λ (4x − 11) = 0 λ ∈ R.

Imponendo il passaggio per il punto (1, 2, 1), otterremo l’unica sfera del fascio
passante per tale punto:
1
1 + 4 + 1 − 2 − 3 + λ (4 − 11) = 0 ⇒ λ = .
7
1
Per λ = 7 in Fγ otteniamo la sfera di equazione

7x2 + 7y2 + 7z2 − 10x − 32 = 0.

Esercizio 140. Determinare la sfera tangente al piano π : x − y + z = 0 nel punto


P = (3, 1, 2) e passante per il punto (1, 2, 0).

Svolgimento: Determiniamo dapprima l’equazione che rappresenta l’insieme infi-


nito di sfere che hanno in comune fra loro il solo punto P = (3, 1, 2) (oltre che i punti
del cerchio assoluto). Essa si ottiene a partire dalle equazioni di 2 di tali sfere, oppu-
re anche (ed é il nostro caso) dall’equazione di una di tali sfere e da quella del piano
tangente in (3, 1, 2) a tutte le sfere dell’insieme. Tale insieme di sfere é detto fascio
di sfere tangenti a π in P. Per determinare l’equazione del fascio necessitiamo allo-
ra dell’equazione di una sfera ad esso appartenente. Osserviamo inizialmente che i
centri di ciascuna delle sfere del fascio sono i punti della retta s ortogonale al piano
π e passante per P: 
 x = t +3
s : y = −t + 1
z = t + 2.

Scegliamo una qualsiasi punto di s, ovviamente differente dal punto P. Per t = −3


avremo il punto C di coordinate (0, 4, −1). Una sfera avente centro in√C e tangente
al√piano π nel punto P, deve avere raggio pari alla distanza δ (C, P) = 9 + 9 + 9 =
3 3. Per cui, l’equazione di tale sfera é
402 15 Le superfici quadriche.

x2 + (y − 4)2 + (z + 1)2 = 27.

L’equazione del fascio di sfere tangenti a π nel punto P é quindi la seguente:

F : x2 + (y − 4)2 + (z + 1)2 − 27 + λ (x − y + z) = 0.

Imponendo il passaggio per il punto di coordinate (1, 2, 0) avremo

1 + 4 + 1 − 27 + λ (1 − 2) = 0 ⇒ λ = −21.

Sostituendo tale valore di λ nell’equazione del fascio F, concludiamo che

x2 + y2 + z2 − 21x + 13y − 19z − 10 = 0

é l’equazione della sfera richiesta.


Riferimenti Bibliografici

1. M. Abate, C. de Fabritiis, Geometria analitica con elementi di Algebra lineare


(McGraw-Hill).
2. S. Axler, Linear algebra done right (Springer).
3. C. Ciliberto, Algebra Lineare (Bollati Boringhieri).
4. S. Greco, P. Valabrega, Lezioni di Geometria (Levrotto Bella).
5. W. Greub, Linear Algebra (Springer).
6. S. Roman, Advanced Linear Algebra (Springer).
7. M. Rosati, Lezioni di Geometria (Libreria Cortina).
8. E. Schlesinger, Algebra lineare e Geometria (Zanichelli).
9. E. Sernesi, Geometria I (Bollati Boringhieri).
10. D. Serre, Matrices, Theory and Applications (Springer).
11. M. Stoka, Corso di Geometria (Cedam).
12. G. Vaccaro, A. Carfagna, L. Piccolella, Lezioni di Geometria e Algebra lineare
(Zanichelli).
13. G. Vaccaro, A. Carfagna, L. Piccolella, Complementi ed esercizi di Geometria ed
Algebra lineare (Zanichelli).

403

Potrebbero piacerti anche